You are on page 1of 356

Mastering

!!!!!!Psychiatry:!

A"core"textbook"for"undergraduates!
!!!!!!!!!!!!!!!!!!!!!!!!!!!!!!!!!!!!!!!!!!!!!!!!
!

Melvyn"WB"Zhang"
Cyrus"SH"Ho"
Roger"CM"Ho"
"
Sanjeev"Sockalingam"
Raed"Hawa"
"

Disclaimer:
"All the MCQs, SAQs and sample OSCEs were written by Dr. Roger Ho only. The objective of this companion book is to
help medical students to revise and understand psychiatry through practice and active learning. This allows medical
students to have a better understanding the exam format and expectations. The authors allow medical students to print
and the authors do not receive any financial incentives. The questions will not be used in the future end of posting tests
or exams."

MCQ exam (Paper 1) Questions


Questions
History, mental state exam and psychopathology!
1. A 40-year-old man admitted to the medical ward for upper gastrointestinal tract bleeding is a
known patient dependent on alcohol. His amylase level is high. During the interview, he
mentions that, I dont have a problem with alcohol. What is the defence mechanism?
A.
B.
C.
D.
E.

Acting out
Denial
Projection
Rationalization
Reaction formation.

2. A 50-year-old woman is referred by her family doctor because she suffers from depressive
disorder. During the interview, she has difficulty in verbalizing her emotions. The
phenomenon is BEST described as:
A.
B.
C.
D.
E.

Ambivalence
Affective flattening
Alexithymia
Alogia
Anhedonia.

3. Visual hallucinations are LEAST likely to be found in which of the following?


!
A.
B.
C.
D.
E.

Alcohol!withdrawal!delirium!!
Central!anticholinergic!delirium!!
Lewy!body!dementia!
Parkinsons!disease!
Schizophrenia.!

4. Which of the following is NOT a common sign or symptom of panic attack?


A.
B.
C.
D.
E.

Hypotension
Palpitation
Shortness of breath
Sweating
Trembling.

Cognitive assessment!
5. Which of the following is NOT a prominent cognitive feature of cortical dementia such as
Alzheimers disease?

!
!
1

A.
B.
C.
D.
E.

Amnesia
Anhedonia
Apathy
Agnosia
Apraxia.

Psychiatric epidemiology
6.

In the community, the MOST common psychiatric diagnosis over age 65 is:

A. Alzheimers disease
B. Late-onset schizophrenia
C. Mood disorder
D. Delirium
E. Vascular dementia.

Psychiatric aetiology, diagnosis and classification

7. Which!of!the!following!is!LEAST!LIKELY!to!be!a!risk!factor!for!delusional!disorder?!!
!
A.
B.
C.
D.
E.

Anxiety!
Increased!age!!
Immigration!
Sensory!impairment!!
Social!isolation.!

!
8. A!female!adolescent!was!raised!by!an!abusive!father!when!she!was!young.!Although!he!
has!changed!and!is!no!longer!abusive!towards!her,!she!still!becomes!anxious!as!soon!as!
she!sees!him.!What!is!the!BEST!explanation!based!on!psychological!theory?!
!
A. Classical!conditioning!
B. Operant!conditioning!
C. Free!association!
D. Reaction!formation!
E. Transference.!
!
9. Which of the following factors is the MOST IMPORTANT predictor of the development of PostTraumatic Stress Disorder (PTSD) in a 30-year-old man who has just been involved in a road traffic
accident?

!
!
2

a.
b.
c.
d.
e.

Death of a friend in the accident


Duration of medical leave
His age and gender
History of panic disorder
History of substance abuse.

10. Paul, a 20-year-old man, suffers from schizophrenia. He has a monozygotic twin brother
called Peter. Based on the findings from genetic studies, what is the risk (in %) that Peter will
develop schizophrenia?
A.
B.
C.
D.
E.

17%
27%
37%
47%
57%.

11. The MOST important predisposing factor for Attention Deficit and Hyperactivity Disorder
(ADHD) is:

A.
B.
C.
D.
E.

Adverse social economic status


Development of autism before the onset of ADHD.
Female gender
Prenatal exposure to alcohol and nicotine by mother.
Presence of antisocial personality disorder in father.

12. A 40-year-old widow, said the following, I cant stop thinking about my husband. The pain is
unbearable. Its been eight months and its like it happened yesterday. I still cant believe it. I
cant look at his picture, its too painful. I feel numb. And my friends dont understand. They
still have their husbands. And, I want to kill his doctor. Its his fault my husband died. My life
is completely empty. Its just not fair. Based on her description, what is the MOST correct
diagnosis?
A.
B.
C.
D.
E.

Adjustment disorder
Antisocial personality disorder
Delusional disorder
Intense grief reaction
Schizophrenia.

General adult psychiatry


13.

All!of!the!following!statements!regarding!inpatient!suicide!are!true!EXCEPT:

A. Hanging is a common method used.


B. Inpatient suicide most often occurs during home leave.

C. Male!inpatients!are!at!higher!risk!of!committing!suicide!during!hospitalization.!
D. The!first!week!of!admission!is!a!high!risk!period.!
E. The!risk!for!women!increases!as!they!get!older.!!
!
!
3

14. Which of the following is the MOST important component of maintenance treatment for
schizophrenia?
A.
B.
C.
D.
E.

Cognitive behaviour therapy


Monitored compliance in antipsychotic treatment
Occupational rehabilitation
Psychosocial rehabilitation
Family therapy.

15. Which!of!the!following!statements!regarding!schizophrenia!is!FALSE?!
A. Eye!movement!dysfunction!may!be!a!trait!marker.!
B. Hallucination!is!pathognomonic!for!schizophrenia.!
C. Intelligence!continues!to!deteriorate!with!the!progression!of!the!disorder.!!
D. Post!schizophrenia!depression!occurs!in!25%!of!patients.!
E. Prodromal!signs!and!symptoms!can!be!nonNspecific.!
!
16. Which!of!the!following!is!NOT!a!sign!or!symptom!of!atypical!depression?!!
!
A.
B.
C.
D.
E.

Excessive!guilt!
Increased!appetite!
Interpersonal!rejection!sensitivity!!
Leaden!paralysis!
Mood!reactivity.!

!
17. Which of the following change is LEAST likely to occur in anorexia nervosa, bingepurging type?
a.
b.
c.
d.
e.

Hypoestrogenemia
Hypomagnesemia
Hypokalaemia
Hyperamylasemia
Hyperphosphatemia.

!
18. Which of the following is classified as circadian rhythm sleep disorder?
A.
B.
C.
D.
E.

Delayed sleep phase syndrome


Obstructive sleep apnoea
Kleine Levin syndrome
Late-onset insomnia
Narcolepsy.

19. A!23TyearTold!medical!student!comes!to!the!clinic!with!a!complaint!of!feeling!worried.!!

He!states!that!he!may!fail!his!clinical!exam!in!Medicine!because!he!is!nervous!about!
presenting!a!case!and!performing!a!physical!examination!in!front!of!examiners.!!When!
he!thinks!about!the!exam,!he!feels!nervous.!!He!skipped!a!lot!of!bedside!teaching!when!
!
!
4

he!knew!it!would!be!his!turn!to!present!a!case!in!front!of!his!classmates!and!tutors.!He!
worries!about!criticism!from!his!classmates!and!tutor.!!The!MOST!likely!diagnosis!is:
A. Agoraphobia
B. Generalized anxiety disorder
C. Normal shyness
D. Panic disorder
E. Social phobia.
Old age psychiatry

20. The!MOST!COMMON!cause!of!psychotic!symptoms!in!elderly!outpatients!in!Singapore!is:!
!
A.
B.
C.
D.
E.

Alcohol!abuse!
Anxiety!
Dementia!
Depression!
Delirium.!

21. Which!of!the!following!statements!is!TRUE!about!a!76TyearTold!woman!suffering!from!an!
acute!confusional!state!after!a!total!hip!replacement?!!
!
A.
B.
C.
D.
E.

Anticholinergic!drug!is!the!treatment!of!choice.!
Her!age!and!gender!are!not!a!risk!factor.!
She!should!be!allowed!to!leave!the!ward!against!doctor!advice!should!she!wish!to.!
Fast,!spike!waves!are!often!seen!on!the!EEG.!!
There!is!a!mortality!of!up!to!15%.!!

22. A 65-year-old person is referred for early dementia. Which of the following is classified as
basic activities of daily living?
A.
B.
C.
D.
E.

Bathing
Doing household chores and laundry
Managing medication
Preparing food
Shopping and managing finances.

23. Which of the following drugs/substances is LEAST likely to be abused by old people?
A.
B.
C.
D.
E.

Alcohol
Amphetamine
Analgesics
Anticholinergics
Cough mixture.

24. You!are!the!resident!working!at!the!Accident!and!Emergency!Department.!A!70TyearTold!

!
!
5

woman!with!a!history!of!Alzheimers!disease!presents!to!you!after!a!fall.!!She!had!been!
experiencing! difficulty! sleeping! at! night! for! several! months! and! was! increasingly!
suspicious!of!her!husband!for!harming!her.!!You!have!decided!to!start!an!antipsychotic.!
Which!of!the!following!practices!is!MOST!appropriate?
A. Adjust doses of antipsychotics slowly with long intervals between dose increments.
B. Augmentation with benzodiazepine is highly recommended.
C. Prescribe small doses of several different antipsychotics rather than using one
antipsychotic.
D. Use a long-acting injectable antipsychotic to aid adherence.
E. Use a rapid loading dose of antipsychotic to speed up the response.
Child and adolescent psychiatry

25. Which!of!the!following!is!the!MOST!COMMON!disorder!for!children!and!adolescents!to!be!
referred!to!see!a!child!and!adolescent!psychiatrist?!!
!
A.
B.
C.
D.
E.

ChildNonset!schizophrenia!
Conduct!disorder!!
Down!syndrome!
Separation!anxiety!disorder!
Somatization!disorder.!

26. An 8-year-old boy presents with stereotypies, pronoun reversal, word substitution, social
problems, and echolalia, the most likely diagnosis is:!
!
A.
B.
C.
D.
E.

Autism!
Attention!deficit!and!hyperactivity!disorder!
Conduct!disorder!
Retts!syndrome!
Separation!anxiety!disorder.!

!
27. A 14-year-old adolescent continues to be significantly depressed despite actively
participating in psychotherapy for 3 months. Which of the following is the best treatment
approach?
A.
B.
C.
D.
E.

Change psychotherapy approach


Start amitriptyline
Start electroconvulsive therapy
Start fluoxetine
Start olanzapine.

28. Which of the listed disorders is the MOST common co-morbidity with ADHD in children?
A. Autism
B. Learning disorders in mathematics
C. Learning disorders in expressive language
D. Oppositional defiant disorder
E. Gender identity disorder of childhood.

!
!
6

Substance abuse
29. Based on clinical severity, men from which of the following ethnicities are MOST vulnerable
to alcohol dependence in Singapore?
A. Chinese
B. Eurasian
C. Indian
D. Malay
E. Other ethnic groups.

30. The!aspartate!aminotransferase!(AST)!/alanine!aminotransferase!(ALT)!ratio!in!alcohol!hepatitis!
is:!
!
1.
2.
3.
4.
5.

<!0.5!
0.6!!0.9!
1!!1.4!
1.5!!1.9!
>!2.!

31. A 20-year-old man is suspected of acute stimulant intoxication. His symptoms are
MOST LIKELY TO resemble which of the following conditions?
A.
B.
C.
D.
E.

Alcohol intoxication
Antisocial personality disorder
Mania
Obsessive-compulsive disorder
Panic disorder.

32. A 30-year-old man is dependent on 3 mg alprazolam (Xanax) on a daily basis. He


wants to stop his dependence on alprazolam. Which of the following is the BEST
management?
A.
B.
C.
D.
E.

Change to 15 mg diazepam and then taper off gradually


Change to 100mg hydroxyzine and then taper off gradually
Change to 400mg lithium and then taper off gradually
Change to 10 mg olanzapine and then taper off gradually
Change to 200mg quetiapine and then taper off gradually.

33. A 30-year-old man drinks alcohol and develops flushing and tachycardia. He took disulfiram 2
hours ago. Accumulation of which of the following is accountable for the above
phenomenon?

!
!
7

A.
B.
C.
D.
E.

Acetaldehyde
Acetylcholine
Adrenaline
Alanine
Aspartic acid.

Psychopharmacology
34. Which of the following tricyclic antidepressants is MOST effective in the treatment of
obsessive compulsive disorder?
A.
B.
C.
D.
E.

Amitriptyline
Clomipramine
Desipramine
Doxepin
Imipramine.

35. A depressed patient does not want to take medication on a daily basis. The selective
serotonin reuptake inhibitor (SSRI) with the LONGEST half-life is?
A.
B.
C.
D.
E.

Escitalporam
Fluoxetine
Fluvoxamine
Paroxetine
Sertraline.

36. The antidepressant, mirtazapine is less likely to cause nausea because of its effects on which
of the follow receptors?
A.
B.
C.
D.
E.

Serotonin 5HT1 receptors


Serotonin 5HT2 receptors
Serotonin 5HT3 receptors
Serotonin 5HT4 receptors
Serotonin 5HT5 receptors.

37. Acetylcholinesterase inhibitors (AChEIs) are BEST conceptualized as:


A.
B.
C.
D.
E.

Contraindicated in the treatment of Lewy body dementia.


Only indicated for severe stages of Alzheimer's disease.
Predominately associated with the side effect of metabolic syndrome.
Improving cognitive function significantly from baseline.
Stabilizing cognition, activities of daily living, and behavioural function.

38. Which of the following is FALSE regarding the first generation antipsychotics

!
!
8

(FGAs)?
A. FGAs have high D2 receptor blocking effects.
B. FGAs have increased risk of extrapyramidal side effects as compared with the second
generation antipsychotics.
C. FGAs have increased risk of tardive dyskinesia as compared with the second generation
antipsychotics.
D. FGAs have increased risk of causing metabolic syndrome as compared with the second
generation antipsychotics.
E. FGAs have proven efficacy as pharmacological treatment for schizophrenia.
39. All of the following are true of a patient on risperidone 6mg daily who gets parkinsonism side
effects, EXCEPT:
A.
B.
C.
D.
E.

75% occupancy at D2 receptors is associated with parkinsonism.


This patient is at higher risk for developing secondary negative symptoms.
The dose is above the antipsychotic threshold in this case.
Raising the dose of risperidone leads to a paradoxical reduction in parkinsonism.
Anticholinergic drug can reduce parkinsonism in this patient.

40. Which of the following herbs has been MOST commonly used by patients to treat
mild to moderate depression in developed countries?
!

A.
B.
C.
D.
E.

Gingko!
Ginseng!
Kava!kava!
Passion!flower!
St.!Johns!Wort.!

41. A!40TyearTold!female!patient!with!schizophrenia!has!been!treated!with!haloperidol!and!

trifluoperazine!for!many!years!in!the!past.!!Currently,!she!exhibits!a!nonTrhythmical!
hyperkinetic!movement!disorder!of!the!lips,!jaw!and!tongue.!!The!movement!disorder!
MOST!likely!consistent!with!this!finding!is:!
A.
B.
C.
D.
E.

Acute dystonia
Akathisia
Aphasia
Pseudoparkinsonism
Tardive dyskinesia.

42. A 35-year-old accountant presents to a psychiatrist 8 months after a motor vehicle


accident. He has difficulty sleeping because he has frequent nightmares about the
accident. He has not been able to drive since the accident, and his wife usually
drives for him. Even then, he finds it very difficult to be in a car such as a taxi, after
!
!
9

panicking if another car is near them on the road. Which of the following
medications would be the MOST appropriate for him?
A.
B.
C.
D.
E.

Amitriptyline
Clonazepam
Paroxetine
Propranolol
Quetiapine.

Psychotherapy

43. A!66TyearTold!man!who!worked!as!an!executive!accountant!previously.!He!has!retired!
recently!and!presents!with!a!major!depressive!disorder.!!What!is!the!best!choice!of!
psychotherapy?!
!
A.
B.
C.
D.
E.

Biofeedback!
Eye!movement!desensitization!and!reprocessing!
Hypnosis!
Interpersonal!psychotherapy!
Systemic!desensitization.!

44. A 40-year-old man with obsessive-compulsive disorder presents for treatment. He would
prefer not using a medication due to possible side effects. Which of the following
psychological interventions would be the treatment of choice for his disorder?
A.
B.
C.
D.
E.

Exposure and response prevention


Hypnotherapy
Interpersonal psychotherapy
Psychoanalysis
Supportive psychotherapy.

45. Effective psychotherapy techniques for Borderline Personality Disorder include all of
the following features EXCEPT:
A. A treatment contract should be established in the beginning phase of
psychotherapy.
B. Adverse effects of self-laceration are identified.
C. Focus of treatment is to establish connection between actions and feelings.
D. Therapist is quiet, distant and does not get too involved.
E. Therapist pays careful attention to his or her own feelings in order to manage
countertransference.

Ethics and laws

46. A woman with a history of depression that responded well to antidepressant


medication is now depressed again. She does not want to take medications due to
!
!
10

concerns about side effects and risks. She would like to try psychotherapy instead.
Her husband disagrees with her. Which of the following ethical principles is MOST
relevant in this situation?
A. Autonomy
B. Beneficence
C. Fiduciary duty
D. Non-maleficence
E. Justice.
Liaison and neuropsychiatry

47. You!are!the!resident!working!in!the!oncology!ward.!Which!of!the!following!factors!is!
MOST!predictive!of!high!suicide!risk!in!a!palliative!cancer!patient?!
!
A.
B.
C.
D.
E.

Alopecia!associated!with!chemotherapy!
Low!energy!level!
Poor!appetite!
Financial!difficulty!
Hopelessness.!

!
48. A 40-year-old woman presents with chronic headache and blurred vision. She wants
symptomatic relief but is not concerned about the underlying condition. Which of the
following psychiatric diagnoses is LEAST relevant?

!
A.
B.
C.
D.
E.

Conversion!disorder!
Moderate!depressive!episode!with!somatic!complaints!
Hypochondriasis!
Pain!disorder!
Somatization!disorder.!

!
!
!
MCQ exam (Paper 1) Answer Key
Questions

Answers!

History, mental state exam and


psychopathology!

49. A 40-year-old man admitted to the


medical ward for upper gastrointestinal
tract bleeding is a known patient
dependent on alcohol. His amylase level
is high. During the interview, he
mentions that, I dont have a problem

!
!
11

The answer is B.

Explanation: He exhibits denial and denies the


reality that he is dependent on alcohol and

with alcohol. What is the defence


mechanism?
F.
G.
H.
I.
J.

Acting out
Denial
Projection
Rationalization
Reaction formation.

Year: 2013.

50. A 50-year-old woman is referred by her


family doctor because she suffers from
depressive
disorder.
During
the
interview,
she
has
difficulty
in
verbalizing
her
emotions.
The
phenomenon is BEST described as:
F.
G.
H.
I.
J.

leading to medical complication.

Ambivalence
Affective flattening
Alexithymia
Alogia
Anhedonia.

The!answer!is!C.!
!
Explanation:!This!patient!has!difficulty!to!
express!her!emotion!and!this!phenomenon!is!
known!as!alexithymia.!!
!
!
Year:!2013!
The!answer!is!E.!

51. Visual hallucinations are LEAST


likely to be found in which of the
following?

!
Explanation:!!The!other!options!A!to!D!are!
associated!with!visual!hallucination.!Patients!
suffering!from!Parkinsons!disease!may!
present!with!visual!hallucinations!as!a!result!
of!side!effect!of!madopar.!As!most!patients!
suffering!from!schizophrenia!in!Singapore!do!
not!use!illicit!drugs,!visual!hallucinations!are!
relatively!less!common.!

!
F.
G.
H.
I.
J.

Alcohol!withdrawal!delirium!!
Central!anticholinergic!delirium!!
Lewy!body!dementia!
Parkinsons!disease!
Schizophrenia.!

!
!
Year:!2013!
!
The!answer!is!A.!

52. Which of the following is NOT a


common sign or symptom of panic
attack?
F.

!
Explanation:!!As!a!result!of!sympathetic!

Hypotension

!
!
12

G.
H.
I.
J.

drive,!the!patient!may!experience!
hypertension!rather!than!hypotension.!!

Palpitation
Shortness of breath
Sweating
Trembling.

!
!
Year:!2013.!

Cognitive assessment!

53. Which of the following is NOT a


prominent cognitive feature of cortical
dementia such as Alzheimers disease?
F.
G.
H.
I.
J.

The!answer!is!B.!
!
Explanation:!Anhedonia!(loss!of!interest)!
and!depression!are!more!common!in!
subcortical!dementia.!The!other!4!As:!
amnesia,!apathy,!agnosia!and!apraxia!are!
common!in!cortical!dementia.!

Amnesia
Anhedonia
Apathy
Agnosia
Apraxia.

!
Year:!2013!
!
This!is!an!advanced!level!question.!
!

Psychiatric epidemiology
54. In the community, the MOST common
psychiatric diagnosis over age 65 is:

The answer is C.

A. Alzheimers disease

Explanation: The National Institute of Mental


Health's Epidemiologic Catchment Area (ECA)
program has found that the most common mental
disorder of old age is depressive disorder.

B. Late-onset schizophrenia
C. Mood disorder
D. Delirium
E. Vascular dementia.

Year: 2013

Psychiatric aetiology, diagnosis and


classification

55. Which!of!the!following!is!LEAST!
LIKELY!to!be!a!risk!factor!for!

The!answer!is!A.!

!
!
13

delusional!disorder?!!

!
F.
G.
H.
I.
J.

Explanation:!The!risk!of!delusional!disorder!
increases!with!age.!Hearing!loss!is!a!risk!
factor.!Low!socioeconomic!status!and!severe!
stress!are!also!risk!factors.!

Anxiety!
Increased!age!!
Immigration!
Sensory!impairment!!
Social!isolation.!

!
Year:!2013.!
!
56. A!female!adolescent!was!raised!by!an!
abusive!father!when!she!was!young.!
Although!he!has!changed!and!is!no!
longer!abusive!towards!her,!she!still!
becomes!anxious!as!soon!as!she!sees!
him.!What!is!the!BEST!explanation!
based!on!psychological!theory?!
!
F. Classical!conditioning!
G. Operant!conditioning!
H. Free!association!
I. Reaction!formation!
J. Transference.!
!

The!answer!is!A.!
!
Explanation:!The!father!is!a!conditioned!
stimuli!and!abuse!is!an!unconditioned!
stimuli.!The!fear!is!the!conditioned!response.!
Even!without!the!abuse,!the!patient!is!fearful!
of!the!father.!This!is!known!as!classical!
conditioning.!
!
!
Year:!2013.!
!
This!is!an!advancedN!level!question.!
!
!

57. Which of the following factors is the MOST


IMPORTANT predictor of the development
of Post-Traumatic Stress Disorder (PTSD) in
a 30-year-old man who has just been
involved in a road traffic accident?
f.
g.
h.
i.

The!answer!is!A.!
!
Explanation:!Severity of event or accident is best
predictor. Death of a friend in the accident indicates
severe accident.

Death of a friend in the accident


Duration of medical leave
His age and gender
History of panic disorder

!
!
14

j.

History of substance abuse.


Year: 2013.!

58. Paul, a 20-year-old man, suffers from


schizophrenia. He has a monozygotic
twin brother called Peter. Based on the
findings from genetic studies, what is
the risk (in %) that Peter will develop
schizophrenia?
F.17%
G. 27%
H. 37%
I. 47%
J. 57%.

The answer is D.

Explanation: Based on the genetic studies


focusing on familial transmission of
schizophrenia, the risk is 47% for monozygotic
twins of schizophrenia patients developing
schizophrenia.

Risk of schizophrenia in Specific Populations


Population
General Population
Nontwin sibling of a
schizophrenic patient
Child with one schizophrenic
parent
Dizygotic twin of a schizophrenic
patient
Child of two schizophrenic
parents
Monozygotic twin of a
schizophrenic patient

Risk (%)
1
8
12
12
40
47

Year: 2013.

The!answer!is!D.!

59. The MOST important predisposing


factor for Attention Deficit and
Hyperactivity Disorder (ADHD) is:

!
Explanation:!Foetal!alcohol!syndrome!and!
antenatal!exposure!to!nicotine!is!the!most!
important!predisposing!factor!for!ADHD.!
Option!A!is!nonNspecific.!Male!gender,!rather!
than!female!gender!is!more!common!for!
ADHD.!Option!E!is!an!important!predisposing!
factor!for!conduct!disorder.!!

F. Adverse social economic status


G. Development of autism before the
onset of ADHD.
H. Female gender
I. Prenatal exposure to alcohol and
nicotine by mother.
J. Presence of antisocial personality
disorder in father.

!
!
15

Year: 2013.!
60. A 40-year-old widow, said the following,
I cant stop thinking about my husband.
The pain is unbearable. Its been eight
months and its like it happened
yesterday. I still cant believe it. I cant
look at his picture, its too painful. I feel
numb.
And
my
friends
dont
understand. They still have their
husbands. And, I want to kill his doctor.
Its his fault my husband died. My life is
completely empty. Its just not fair.
Based on her description, what is the
MOST correct diagnosis?

The answer is D.

F.
G.
H.
I.
J.

Year: 2013.

Adjustment disorder
Antisocial personality disorder
Delusional disorder
Intense grief reaction
Schizophrenia.

which persists for longer than 6 months


after the loss, traumatic distress, sense of
disbelief, anger and bitterness, distressing,
intrusive thoughts related to the death,
avoidance of reminders of deceased and
separation distress.

General adult psychiatry


61.

Explanation: She suffers from intense grief

All! of! the! following! statements! The!answer!is!E.!


regarding! inpatient! suicide! are! true!
!
EXCEPT:

F. Hanging is a common method used.


G. Inpatient suicide most often occurs during
home leave.

Explanation:!Option!A,!B,!C,!D!are!correct.!
Hanging!in!bathroom!is!common!and!
H. Male!inpatients!are!at!higher!risk!of!
bathroom!needs!to!be!specially!designed.!It!
committing!suicide!during!
is!important!for!nurses!or!ward!staff!to!give!
hospitalization.!
I. The!first!week!of!admission!is!a!high!risk! patients!or!family!a!call!during!home!leave!or!
immediately!after!discharge!as!these!are!
period.!
J. The!risk!for!women!increases!as!they!get! high!risk!periods.!Option!D!is!incorrect.!The!
older.!!
risk!for!women!plateaus!or!decreases!with!
age.!The!risk!for!men!shows!two!peaks:!
young!age!and!old!age.!
!
!
Year:!2013.!
62. Which of the following is the MOST
important component of maintenance
treatment for schizophrenia?

The answer is B.

Explanation: Without maintenance


pharmacological treatment, 60-70% of
schizophrenia patients relapse within 1 year,

F. Cognitive behaviour therapy


G. Monitored compliance in antipsychotic

!
!
16

treatment
H. Occupational rehabilitation
I. Psychosocial rehabilitation
J. Family therapy.

and almost 90% relapse in 2 years.


Maintenance on antipsychotic therapy is the
single most important factor in preventing
rehospitalisation.

Year: 2013.

63. Which!of!the!following!statements!
regarding!schizophrenia!is!FALSE?!
F. Eye!movement!dysfunction!may!be!a!
trait!marker.!
G. Hallucination!is!pathognomonic!for!
schizophrenia.!
H. Intelligence!continues!to!deteriorate!
with!the!progression!of!the!disorder.!!
I. Post!schizophrenia!depression!occurs!in!
25%!of!patients.!
J. Prodromal!signs!and!symptoms!can!be!
nonNspecific.!

The!answer!is!B.!

64. Which!of!the!following!is!NOT!a!sign!
or!symptom!of!atypical!depression?!!

The!answer!is!A.!

F.
G.
H.
I.
J.

Explanation:!Excessive!guilt!is!part!of!
melancholia.!

!
Excessive!guilt!
Increased!appetite!
Interpersonal!rejection!sensitivity!!
Leaden!paralysis!
Mood!reactivity.!

!
Explanation:!Hallucination!can!occur!in!other!
psychiatric!disorders.!Eye!movement!
dysfunction!occurs!in!50N85%!of!
schizophrenia!patients.!!!
!
!
Year:!2013.!

!
!

Year:!2013.!
Answer!is!E.!

65. Which of the following change is


LEAST likely to occur in anorexia
nervosa, binge-purging type?
f.
g.
h.
i.
j.

Hypoestrogenemia
Hypomagnesemia
Hypokalaemia
Hyperamylasemia
Hyperphosphatemia.

Explanation:!It!should!be!hypophosphatemia!
and!hypocalcaemia.!As!bingeNpuruging!type!
is!associated!with!recurrent!vomiting,!it!will!
lead!to!hypokalaemia!and!inflammation!in!
salivary!gland,!i.e.!hyperamylasemia.!
Anorexia!nervosa!in!general!is!associated!
with!hypoestrogenemia!and!!

!
!
17

hypomagnesemia.!
!
!
Year:!2013!
!
This!is!an!advanced!!level!question.!
The!answer!is!A.!

66. Which of the following is classified as


circadian rhythm sleep disorder?
F.
G.
H.
I.
J.

!
Explanation:!Delayed!sleep!phase!syndrome!
is!classified!as!circadian!rhythm!sleep!
disorder.!

Delayed sleep phase syndrome


Obstructive sleep apnoea
Kleine Levin syndrome
Late-onset insomnia
Narcolepsy.

!
!
Year:!2013!

67. A!23TyearTold!medical!student!comes!

to!the!clinic!with!a!complaint!of!
feeling!worried.!!He!states!that!he!
may!fail!his!clinical!exam!in!Medicine!
because!he!is!nervous!about!
presenting!a!case!and!performing!a!
physical!examination!in!front!of!
examiners.!!When!he!thinks!about!the!
exam,!he!feels!nervous.!!He!skipped!a!
lot!of!bedside!teaching!when!he!knew!
it!would!be!his!turn!to!present!a!case!
in!front!of!his!classmates!and!tutors.!
He!worries!about!criticism!from!his!
classmates!and!tutor.!!The!MOST!
likely!diagnosis!is:
F. Agoraphobia
G. Generalized anxiety disorder
H. Normal shyness
I. Panic disorder
J. Social phobia.

The!answer!is!E.!
!
Explanation:!He!suffers!from!social!
phobia!because!he!develops!marked!fear!
which!is!brought!by!being!the!focus!of!
attention.!
!
!
Year:!2013!

Old age psychiatry

68. The!MOST!COMMON!cause!of!
psychotic!symptoms!in!elderly!
outpatients!in!Singapore!is:!

The!answer!is!C.!
!

!
!
18

!
F.
G.
H.
I.
J.

Explanation:!Dementia!is!the!most!common!
cause!for!psychotic!symptoms!in!elderly!
outpatients.!

Alcohol!abuse!
Anxiety!
Dementia!
Depression!
Delirium.!

!
!
Year:!2013.!

69. Which!of!the!following!statements!is!
TRUE!about!a!76TyearTold!woman!
suffering!from!an!acute!confusional!
state!after!a!total!hip!replacement?!!
!

The!answer!is!E.!
!
Explanation:!!In!elderly,!the!mortality!of!
delirium!and!acute!confusional!state!is!
between!6!to!18%.!Generalized!slowing!is!
the!most!common!finding!on!EEG.!Her!age!is!
a!risk!factor.!

F. Anticholinergic!drug!is!the!treatment!of!
choice.!
G. Her!age!and!gender!are!not!a!risk!factor.!
H. She!should!be!allowed!to!leave!the!ward!
against!doctor!advice!should!she!wish!to.!
!
I. Fast,!spike!waves!are!often!seen!on!the!
EEG.!!
!
J. There!is!a!mortality!of!up!to!15%.!!

Year:!2013!
The!answer!is!A.!

70. A 65-year-old person is referred for


early dementia. Which of the following
is classified as basic activities of daily
living?
F.
G.
H.
I.
J.

!
Explanation:!A!is!considered!to!be!basic!
activities!of!daily!living!while!the!others!are!
classified!as!instrumental!activities!of!daily!
living.!

Bathing
Doing household chores and laundry
Managing medication
Preparing food
Shopping and managing finances.

!
!
Year:!2013!

!
!
19

71. Which of the following


drugs/substances is LEAST likely to be
abused by old people?
F.
G.
H.
I.
J.

The!answer!is!B.!
!
Explanation:!Old!people!are!most!likely!to!
misuse!alcohol!or!prescribed!drugs!than!
street/illicit!drugs!such!as!stimulant.!

Alcohol
Amphetamine
Analgesics
Anticholinergics
Cough mixture.

!
!
Year:!2013.!

72. You! are! the! resident! working! at! the! The!answer!is!A.!

Accident!and!Emergency!Department.!
A! 70TyearTold! woman! with! a! history!
of! Alzheimers! disease! presents! to!
you! after! a! fall.! ! She! had! been!
experiencing! difficulty! sleeping! at!
night! for! several! months! and! was!
increasingly! suspicious! of! her!
husband! for! harming! her.! ! You! have!
decided! to! start! an! antipsychotic.!
Which! of! the! following! practices! is!
MOST!appropriate?
F. Adjust doses of antipsychotics
slowly with long intervals
between dose increments.
G. Augmentation with
benzodiazepine is highly
recommended.
H. Prescribe small doses of
several different antipsychotics
rather than using one
antipsychotic.
I. Use a long-acting injectable
antipsychotic to aid adherence.
J. Use a rapid loading dose of
antipsychotic to speed up the
response.

!
Explanation:!Elderly!with!dementia!are!
sensitive!to!side!effects.!As!a!result,!the!
doctor!should!adjust!the!dose!slowly!with!
long!intervals!between!increments.!Routine!
prescription!of!benzodiazepine!to!elderly!
with!dementia!is!not!recommended!as!it!will!
lead!to!confusion.!
!
!
Year:!2013.!

Child and adolescent psychiatry

73. Which!of!the!following!is!the!MOST!
COMMON!disorder!for!children!and!
adolescents!to!be!referred!to!see!a!
child!and!adolescent!psychiatrist?!!

The!answer!is!B.!
!
Explanation:! Children! with! externalizing!
disorders! (e.g.! conduct! disorder)! are! more!
likely! to! be! referred! and! treated.! 10%! of!

!
F. ChildNonset!schizophrenia!
!
!
20

G.
H.
I.
J.

Conduct!disorder!!
Down!syndrome!
Separation!anxiety!disorder!
Somatization!disorder.!

children! and! adolescents! seeing! a!


psychiatrist! have! conduct! problem.!
Separation! anxiety! disorder! and! Down!
Syndrome!may!not!need!to!see!a!psychiatrist.!
0.1%! have! psychotic! disorder! and! 1%! have!
somatization!disorder.!
!
!
Year:!2013!

74. An 8-year-old boy presents with

The!answer!is!A.!

stereotypies, pronoun reversal, word


substitution, social problems, and
echolalia, the most likely diagnosis is:!

Explanation:!Repetitive!movement,!
communication!problem!and!social!problems!
indicate!this!boy!suffers!from!autism.!

F. Autism!
G. Attention!deficit!and!hyperactivity!
disorder!
H. Conduct!disorder!
I. Retts!syndrome!
J. Separation!anxiety!disorder.!

!
!

Year:!2013.!
75. A 14-year-old adolescent continues to
be significantly depressed despite
actively participating in psychotherapy
for 3 months. Which of the following is
the best treatment approach?
F.
G.
H.
I.
J.

Change psychotherapy approach


Start amitriptyline
Start electroconvulsive therapy
Start fluoxetine
Start olanzapine.

The!answer!is!D.!
!
Explanation:!Starting!an!antidepressant!as!
an!adjunct!to!psychotherapy!after!the!
adolescent!has!failed!to!respond!to!
psychotherapy!alone!is!a!standard!approach!
to!the!treatment!of!adolescent!depression.!!
!
!
Year:!2013!
!

76. Which of the listed disorders is the


MOST common co-morbidity with ADHD
in children?
F. Autism
G. Learning disorders in mathematics

!
!
21

The!answer!is!D.!
!
Explanation:!sleep!disturbance,!conduct!

disorder,!oppositional!defiant!disorder,!
depression,!anxiety!and!substance!abuse!are!
common!comorbidity!with!ADHD.!

H. Learning disorders in expressive


language
I. Oppositional defiant disorder
J. Gender identity disorder of
childhood.

!
!
Year:!2013.!
!
!

Substance abuse

The!answer!is!C.!

77. Based on clinical severity, men from


which of the following ethnicities are
MOST vulnerable to alcohol
dependence in Singapore?

B. Eurasian

Explanation:!Indian!men!seem!to!be!most!
vulnerable!to!alcohol!dependence!in!
Singapore.!Malay!men!have!the!lowest!risk.!

C. Indian

D. Malay
E. Other ethnic groups.

A. Chinese

Year:!2013.!
78. The!aspartate!aminotransferase!(AST)!
/alanine!aminotransferase!(ALT)!ratio!in!
alcohol!hepatitis!is:!
!
6.
7.
8.
9.
10.

The!answer!is!E.!
!
Explanation:!When!greater!than!1.0!but!less!
than!2.0,!it!is!likely!to!be!associated!with!
cirrhosis.!If!<1,!it!indicates!viral!hepatitis.!

<!0.5!
0.6!!0.9!
1!!1.4!
1.5!!1.9!
>!2.!

!
Year:!2013.!!
!
This!is!an!advanced!level!question.!!
79. A 20-year-old man is suspected of
acute stimulant intoxication. His
symptoms are MOST LIKELY TO
resemble which of the following

The answer is C.

!
!
22

conditions?
F.
G.
H.
I.
J.

The syndrome picture of acute cocaine or


amphetamine intoxication can include
hyperawareness, hypersexuality,
hypervigilance, agitation, paranoia, and
delusions. It resembles mania or psychosis.

Alcohol intoxication
Antisocial personality disorder
Mania
Obsessive-compulsive disorder
Panic disorder.

Year: 2013.
80. A 30-year-old man is dependent on 3
mg alprazolam (Xanax) on a daily
basis. He wants to stop his
dependence on alprazolam. Which
of the following is the BEST
management?
F. Change to 15 mg diazepam and
then taper off gradually
G. Change to 100mg hydroxyzine and
then taper off gradually
H. Change to 400mg lithium and then
taper off gradually
I. Change to 10 mg olanzapine and
then taper off gradually
J. Change to 200mg quetiapine and
then taper off gradually.

81. A 30-year-old man drinks alcohol and


develops flushing and tachycardia. He
took disulfiram 2 hours ago.
Accumulation of which of the following
is accountable for the above
phenomenon?
F.

Acetaldehyde

!
!
23

Answer: A

Explanation: Diazepam is the longest


acting benzodiazepine and this will prevent
withdrawal symptoms such as withdrawal
fit.

Year: 2013

The!answer!is!A.!
!
Explanation:!The!mechanism!of!actions!of!
disulfiram!is!to!inhibit!aldehyde!
dehydrogenase!and!lead!to!acetaldehyde!

G.
H.
I.
J.

accumulation.!

Acetylcholine
Adrenaline
Alanine
Aspartic acid.

!
!
Year:!2013!
!
This!is!an!advancedNlevel!question.!
!

Psychopharmacology
82. Which of the following tricyclic
antidepressants is MOST effective in
the treatment of obsessive compulsive
disorder?
F.
G.
H.
I.
J.

The!answer!is!B.!
!
Explanation:!Among!all!TCAs,!clomipramine!
is!most!potent!is!blocking!5HT!reuptake.!

Amitriptyline
Clomipramine
Desipramine
Doxepin
Imipramine.

!
!
Year:!2013!
The!answer!is!B.!

83. A depressed patient does not want to


take medication on a daily basis. The
selective serotonin reuptake inhibitor
(SSRI) with the LONGEST half-life is?
F.
G.
H.
I.
J.

!
The!have!life!of!fluoxetine!is!longer!than!72!
hours!and!patient!can!take!the!medication!
every!other!day.!

Escitalporam
Fluoxetine
Fluvoxamine
Paroxetine
Sertraline.

!
!
Year:!2013!

84. The antidepressant, mirtazapine is less


likely to cause nausea because of its
effects on which of the follow
receptors?

The!answer!is!C.!
!
Explanation:!Its!effect!on!5HT3 receptors is

F.
G.
H.
I.
J.

less likely to cause nausea side effect as


compared to SSRIs.

Serotonin 5HT1 receptors


Serotonin 5HT2 receptors
Serotonin 5HT3 receptors
Serotonin 5HT4 receptors
Serotonin 5HT5 receptors.

!
!
24

Year: 2013.

This is an advanced level question.

!
85. Acetylcholinesterase inhibitors
(AChEIs) are BEST conceptualized
as:

The answer is E.

F. Contraindicated in the treatment of


Lewy body dementia.
G. Only indicated for severe stages of
Alzheimer's disease.
H. Predominately associated with the side
effect of metabolic syndrome.
I. Improving cognitive function
significantly from baseline.
J. Stabilizing cognition, activities of daily
living, and behavioural function.

Explanation: AChEIs are best


conceptualized as drugs that stabilize
cognition, activities of daily living, and
behavioural function and slow clinical
deterioration in Alzheimer's disease but not
significant improvement from baseline.
AChEI e.g. rivastigmine can be used for
dementia with Lewy bodies.
Gastrointestinal side effects e.g. nausea,
vomiting and diarrhoea are the most
common adverse effects.

Year: 2013.
The!answer!is!D.!

86. Which of the following is FALSE


regarding the first generation
antipsychotics (FGAs)?

F. FGAs have high D2 receptor blocking


effects.
G. FGAs have increased risk of
extrapyramidal side effects as
compared with the second generation
antipsychotics.
H. FGAs have increased risk of tardive
dyskinesia as compared with the
second generation antipsychotics.
I. FGAs have increased risk of causing
metabolic syndrome as compared with
the second generation antipsychotics.
J. FGAs have proven efficacy as
pharmacological treatment for

!
!
25

Explanation:!The!second!generation!
antipsychotics!have!higher!risk!for!metabolic!
syndrome!as!compared!to!the!FGAs.!
!
!
Year:!2013.!

schizophrenia.
87. All of the following are true of a patient
on risperidone 6mg daily who gets
parkinsonism side effects, EXCEPT:

The!answer!is!D.!

A. 75% occupancy at D2 receptors is


associated with parkinsonism.
B. This patient is at higher risk for developing
secondary negative symptoms.
C. The dose is above the antipsychotic
threshold in this case.
D. Raising the dose of risperidone leads to a
paradoxical reduction in parkinsonism.
E. Anticholinergic drug can reduce
parkinsonism in this patient.

Explanation:!Increase!the!dose!will!worsen!
parkinsonism.!

88. Which of the following herbs has


been MOST commonly used by
patients to treat mild to moderate
depression in developed countries?

The!answer!is!E.!

F.
G.
H.
I.
J.

Gingko!
Ginseng!
Kava!kava!
Passion!flower!
St.!Johns!Wort.!

!
!
Year:!2013.!

!
Explanation:!St!Johns!Wort!is!the!most!
commonly!used!herb!to!treat!depression!in!
western!countries.!It!shares!similar!
properties!as!other!SSRIs!such!as!increase!in!
bleeding.!
!

!
Year:!2013.!
89. A!40TyearTold!female!patient!with!

The!answer!is!E.!

schizophrenia!has!been!treated!with!
haloperidol!and!trifluoperazine!for!
many!years!in!the!past.!!Currently,!she!
exhibits!a!nonTrhythmical!
hyperkinetic!movement!disorder!of!
the!lips,!jaw!and!tongue.!!The!
movement!disorder!MOST!likely!
consistent!with!this!finding!is:!
F.
G.
H.
I.
J.

!
Explanation:!Tardive!dyskinesia!is!
characterised!by!lip!smacking,!chewing!and!
fly!catching!tongue!protrusion!after!longN
term!treatment!of!antipsychotics.!
!

Acute dystonia
Akathisia
Aphasia
Pseudoparkinsonism
Tardive dyskinesia.

!
Year:!2013.!!

!
!
26

90. A 35-year-old accountant presents


to a psychiatrist 8 months after a
motor vehicle accident. He has
difficulty sleeping because he has
frequent nightmares about the
accident. He has not been able to
drive since the accident, and his
wife usually drives for him. Even
then, he finds it very difficult to be in
a car such as a taxi, after panicking
if another car is near them on the
road. Which of the following
medications would be the MOST
appropriate for him?
F.
G.
H.
I.
J.

The!answer!is!C.!
!
Explanation:!SSRIs!are!generally!most!
appropriate!medication!of!choice!for!post!
traumatic!stress!disorder.!Examples!
include!paroxetine,!sertraline!and!
fluoxetine.!
!
!
Year:!2013!

Amitriptyline
Clonazepam
Paroxetine
Propranolol
Quetiapine.

Psychotherapy

91. A!66TyearTold!man!who!worked!as!an!
executive!accountant!previously.!He!
has!retired!recently!and!presents!
with!a!major!depressive!disorder.!!
What!is!the!best!choice!of!
psychotherapy?!
!
F. Biofeedback!
G. Eye!movement!desensitization!and!
reprocessing!
H. Hypnosis!
I. Interpersonal!psychotherapy!
J. Systemic!desensitization.!

The!answer!is!D.!
!
Explanation:!IPT!and!CBT!are!the!evidenceN
based!psychotherapy!indicated!for!major!
depressive!disorder.!IPT!can!help!him!to!
work!on!role!transition,!grief!and!loss.!
!
!
Year:!2013!

92. A 40-year-old man with obsessivecompulsive disorder presents for


treatment. He would prefer not using a
medication due to possible side effects.
Which of the following psychological
interventions would be the treatment of
choice for his disorder?
F. Exposure and response prevention
G. Hypnotherapy

!
!
27

The!answer!is!A.!
!
Explanation:!Exposure!with!response!
prevention!(ERP)!is!part!of!CBT,!the!
psychotherapeutic!treatment!of!choice!for!
obsessiveNcompulsive!disorder.!In!ERP,!

patients!are!exposed!to!the!feared!stimuli!
and!obsessions!while!rituals!that!typically!
serve!to!reduce!anxiety!are!prevented.!!

H. Interpersonal psychotherapy
I. Psychoanalysis
J. Supportive psychotherapy.

!
!
Year:!2013!
93. Effective psychotherapy techniques
for Borderline Personality Disorder
include all of the following features
EXCEPT:
F. A treatment contract should be
established in the beginning
phase of psychotherapy.
G. Adverse effects of selflaceration are identified.
H. Focus of treatment is to
establish connection between
actions and feelings.
I. Therapist is quiet, distant and
does not get too involved.
J. Therapist pays careful attention
to his or her own feelings in
order to manage
countertransference.

The answer is D.

Explanation: Distant therapist may have


difficulty to establish trust and therapeutic
alliance with patient suffering from
borderline personality disorder.

Year: 2013.

Ethics and laws

94. A woman with a history of


depression that responded well to
antidepressant medication is now
depressed again. She does not want
to take medications due to concerns
about side effects and risks. She
would like to try psychotherapy
instead.
Her husband disagrees
with her. Which of the following
ethical principles is MOST relevant
in this situation?
F. Autonomy
G. Beneficence
H. Fiduciary duty
I. Non-maleficence
J. Justice.

!
!
28

Answer:!The!answer!is!A.!
!
Explanation:!Autonomy!refers!to!the!
obligation!of!a!doctor!to!respect!patients!
right!to!make!her!own!choice!of!treatment!
(i.e.!psychotherapy!instead!of!medication!in!
this!case).!
!
Fiduciary!duty!refers!to!the!duty!that!a!
doctor!should!act!as!the!best!interest!of!the!
patient.!In!this!scenario,!it!is!less!relevant!as!
compared!to!autonomy.!

!
!
Year:!2013!
!
!

Liaison and neuropsychiatry

95. You!are!the!resident!working!in!the!
oncology!ward.!Which!of!the!
following!factors!is!MOST!predictive!
of!high!suicide!risk!in!a!palliative!
cancer!patient?!
!
F.
G.
H.
I.
J.

Alopecia!associated!with!chemotherapy!
Low!energy!level!
Poor!appetite!
Financial!difficulty!
Hopelessness.!

The!answer!is!E.!
!
Explanation:!!Hopelessness!or!cognitive!
symptoms!are!most!predictive!of!high!
suicide!risk!in!a!palliative!cancer!patient.!
!
!
Year:!2013.!

96. A 40-year-old woman presents with


chronic headache and blurred vision.
She wants symptomatic relief but is not
concerned about the underlying
condition. Which of the following
psychiatric diagnoses is LEAST
relevant?

!
F. Conversion!disorder!
G. Moderate!depressive!episode!with!
somatic!complaints!
H. Hypochondriasis!
I. Pain!disorder!
J. Somatization!disorder.!

Answer!is!C.!
!
Explanation:!Patients!suffering!from!
hypochondriasis!are!more!concerned!about!
the!diagnosis!rather!than!symptomatic!relief.!
The!other!options!are!associated!with!
concern!of!symptoms!as!stated!in!this!case.!
!
!

Year:!2013.!
!

!
!
!
!
!
!
!
29

MCQ!exam!(Paper!2):!Questions!
History, mental state exam and psychopathology!

1. Which!of!the!following!is!the!BEST!example!of!inattention?!
A. The!patient!interrupts!the!conversation!to!ask!when!he!will!be!discharged.!
B. The!patient!is!oriented!and!aware!of!his!recent!medical!problems!but!falls!asleep!during!the!
conversation.!
C. The!patient!suddenly!bursts!into!tears!when!you!are!discussing!his!recent!amputation.!
D. The!patient!watches!a!fly!buzzing!on!the!ceiling!while!you!are!discussing!the!prognosis!for!
his!lung!cancer,!then!falls!asleep.!
E. The!patient!cannot!remember!what!you!have!told!him!3!minutes!ago.!Then!he!tries!to!make!
up!the!answer.!
!
2. A!30TyearTold!man!firmly!believes!that!the!alien!has!put!an!implant!in!his!body!and!he!
feels!there!is!a!pushing!sensation!on!his!aorta.!Which!types!of!hallucinations!BEST!
suited!his!description?!
!
A.
B.
C.
D.
E.

Auditory!
Cenesthetic!
Gustatory!
Kinesthetic!!
Visual.!

!
3. Perseveration!is!MOST!commonly!seen!in!which!of!the!following!disorders?!!!
!
A.
B.
C.
D.
E.

Autism!
Attention!deficit!and!hyperactivity!disorder!
Bipolar!disorder!!
Frontal!lobe!dementia!!
Obsessive!compulsive!disorder.!

!
4. A patient presents with persistence, perfectionism and body image distortion. Which of the
following disorders is MOST likely to be associated with the above clinical features?
A. Anorexia nervosa
B. Borderline personality disorder
C. Delusional disorder
D. Hypochondriasis
E. Somatization disorder.
Cognitive assessment!
5. Which of the following statement is FALSE about Mini-mental state examination (MMSE)?
A. A total score of 5 indicates severe dementia.

!
!
30

B. Assessment of orientation to time and place is part of the MMSE.


C. The MMSE can be administered in Chinese or English.
D. The MMSE can be used to monitor treatment progress after initiation of acetylcholinesterase
inhibitors (AChEIs).
E. The MMSE is the gold standard in establishing the diagnosis of dementia.
Psychiatric epidemiology

6. Which!of!the!following!settings!have!the!HIGHEST!prevalence!of!delirium?!
!
A.
B.
C.
D.
E.

Accident!and!emergency!department!
Cardiac!surgery!ward!
General!surgery!ward!
Hospice!with!palliative!advanced!cancer!patients!
Nursing!home!looking!after!dementia!patients.!

!!
7. The worldwide point prevalence of schizophrenia is BEST estimated as:

A.
B.
C.
D.
E.

1%
2.5%
5%
7.5%
10%.

Psychiatric aetiology, diagnosis and classification


8. Advanced paternal age is a well- established risk factor for which of the following
psychiatry illnesses?
A.
B.
C.
D.
E.

Alcoholism
Major depressive disorder
Obsessive compulsive disorder
Generalized anxiety disorder
Schizophrenia.

9. Which of the following social factors is MOST LIKELY to be associated with


relapse of schizophrenia after hospitalization?
A. Discrimination
B. Financial problems
C. Homelessness
D. Increased expressed emotion
E. Isolation and absence of family member.
10. Individuals!with!which!one!of!the!following!genes!have!the!HIGHEST!risk!of!

!
!
31

developing!Alzheimers!disease?
A.
B.
C.
D.
E.

Homozygous apolipoproteins!(APO) E2/E2


Homozygous APO E4/E4
Homozygous APO E6/E6
Heterozygous APO E2/E4
Heterozygous APO E2/E6.

General adult psychiatry

11. A!40TyearTold!man!suffered!from!a!heart!attack!under!general!anaesthesia.!The!
doctors!in!the!operation!theatre!successfully!resuscitated!him.!After!recovery,!he!
wants!to!sue!the!hospital!for!causing!him!postTtraumatic!stress!disorder!(PTSD).!He!
claims!the!resuscitation!process!was!!!extremely!traumatic.!Which!of!the!following!is!
the!MOST!IMPORTANT!feature!to!exclude!the!diagnosis!of!PTSD?!!
!
A.
B.
C.
D.

He does not have past history of PTSD.


He has hidden agenda of suing the hospital and tries to seek compensation.
He was under general anaesthesia and he was not conscious during resuscitation.
The process of resuscitation is not considered to be traumatic in general and doctor
resuscitated him in his best interest.
E. He did not seek treatment from a psychiatrist after the operation.
12. Which of the following schizophrenia patients has the HIGHEST suicide risk?

!
A.!An!adolescent!with!prodromal!symptoms!and!vague!paranoid!idea.!
B.!An!elderly!retired!man!with!late!onset!schizophrenia.!
C.!A!middleNaged!man!with!negative!symptoms.!
D.!A!middleNaged!woman!who!suffers!from!simple!schizophrenia.!
E.!A!young!male!university!student!who!once!had!high!expectations.!
!
13. Which of the following is NOT a recognized alteration of sleep disturbance associated with
major depressive disorder?
A.
B.
C.
D.
E.

Difficulty in falling asleep


Increase in nocturnal awakening
Increase in density of Rapid Eye Movement (REM) sleep
Increase in the latency of Rapid Eye Movement (REM) sleep
Reduction of total sleep time.

14. A 40-year-old woman is staying with her god-brother. She was rejected by her family and

!
!
32

she felt helpless in the past. She needs her god-brother to make decision for her and her
god-brother asks her to be the housekeeper in return. One day, her god-brother needs to
go to Vietnam for 1 week and she feels extremely uncomfortable. Which of the following
personality traits BEST describes this person?
A.
B.
C.
D.
E.

Borderline personality
Dependent personality
Histrionic personality
Schizoid personality
Schizotypal personality.

15. A 30-year-old man comes to see you and he worries about premature ejaculation. The
MOST appropriate time frame to meet the diagnosis of premature ejaculation is:
A. Ejaculation occurs within 60 seconds following vaginal penetration.
B. Ejaculation occurs within 90 seconds following vaginal penetration.
C. Ejaculation occurs within 120 seconds following vaginal penetration.
D. Ejaculation occurs within 150 seconds following vaginal penetration.
E. Ejaculation occurs within 180 seconds following vaginal penetration.
16. A!30TyearTold!man!suffered!from!eight!episodes!of!mood!disturbances!within!a!single!

year.!These!mood!episodes!met!the!diagnostic!criteria!for!major!depression,!mania!
and!hypomania.!These!episodes!were!separated!by!remission.!Which!of!the!following!
is!the!MOST!likely!diagnosis?
!
A.
B.
C.
D.
E.

Cyclothymic!disorder!
Emotionally!unstable!personality!disorder!
Mixed!affective!disorder!
Rapid!cycling!bipolar!disorder!
Schizoaffective!disorder.!

Old age psychiatry


17. Which!of!the!following!is!NOT!a!predisposing!factor!for!depression!in!the!elderly?
A.
B.
C.
D.
E.

Cerebrovascular accident
Poly-pharmacy
Looking after a spouse with chronic illness
Male gender
Widowhood.

18. Dementia!in!general!is!BEST!described!as:!!
!
A.
B.
C.
D.

Behavioural!disturbance!
Global!impairment!
Impaired!memory!for!personal!events!(e.g.!marriage,!past!occupation)!
Personality!change!

!
!
33

E. Progressive!deterioration.!
19.

A!65TyearTold!man!presents!with!memory!loss,!which!of!the!following!is!the!
LEAST!important!indication!for!a!computerized!tomography!(CT)!scan!for!his!brain:

A.
B.
C.
D.
E.

A sudden decrease in cognitive function over a one-month period

Gait abnormalities
His age

Urinary incontinence
Use of anticoagulants.

Child and adolescent psychiatry

20. Which!of!the!following!is!THE!MOST!IMPORTANT!predictor!of!bipolar!disorder!in!
adolescent!depression?!
!
A.
B.
C.
D.
E.

Diurnal!variation!of!mood!
Hypomanic!symptoms!after!taking!antidepressants!
Hypersomnia!
Increased!weight!
Multiple!suicide!attempts.!

21. The!MOST!COMMON!reason!for!children!with!autism!are!brought!to!medical!attention!
by!their!parents!is: !
!
A.
B.
C.
D.
E.

Language!delays!!
Lack!of!toilet!control!
Odd!play!
School!difficulties!
Stereotyped!behaviour.!

22. Which of the following statements is FALSE regarding separation anxiety disorder?
A.
B.
C.
D.
E.

It involves recurrent distress when separate from attachment figures.


Separation anxiety disorder does not occur in adolescents.
Separation anxiety disorder is associated with school refusal.
The child has difficulty falling asleep at night.
The child may complain of somatic symptoms.

23. Which of the following symptoms is MOST LIKELY to persist when a young person suffers
from Attention Deficit and Hyperactivity Disorder (ADHD) becoming an adult?
A. Accident risk
B. Hyperactivity

!
!
34

C. Impulsivity
D. Inattention
E. Learning difficulties.
24. A baby is born with low birth weight, microcephaly, small eyes, upturned nose and a smooth,
undeveloped philtrum. He fails to thrive and develops seizures. Which of the following was he MOST
likely exposed to during pregnancy?
A. Alcohol
B. Cocaine
C. LSD
D. Methamphetamine
E. Inhalants.
Substance abuse

25. The!term!which!BEST!describes!!an!altered!physiological!state!and!neuroT!adaptation!
caused!by!repeated!administration!of!a!drug!is:!
!
A.
B.
C.
D.
E.

Addiction!
Dependence!
Misuse!
Withdrawal!
Tolerance.!
!

26. While!driving,!a!40TyearTold!man!with!no!previous!history!refused!to!stop!for!the!
traffic!police.!He!was!subsequently!brought!into!the!accident!and!emergency!
department!(AED)!by!them.!You!are!the!resident!working!in!the!AED.!!When!you!
assess!him,!he!states!that!he!was!hearing!voices!in!clear!consciousness.!He!admits!to!a!
history!of!alcohol!use.!What!is!the!most!appropriate!diagnosis?!
!
A.
B.
C.
D.
E.

Alcoholic!hallucinosis!!
Delirium!tremens!
Korsakoff!psychosis!
Schizophrenia!
Wernickes!encephalopathy.!

!
27. Which of the following BEST describes the mechanism of action with naltrexone in the treatment of
alcohol dependence?
A.
B.
C.
D.
E.

Naltrexone acts on the benzodiazepine receptor and prevents alcohol withdrawal.


Naltrexone blocks opioid receptors to decrease alcohol craving.
Naltrexone inhibits aldehyde dehydrogenase to decrease alcohol craving.
Naltrexone is a GABA agonist and decreases alcohol craving.
Naltrexone is a glutamate antagonist and decreases alcohol craving.

!
!
35

28.
A.
B.
C.
D.
E.

Which of the following complications is LEAST likely to occur in cocaine abusers?


Arrhythmia
Hyperthermia
Myocardial infarction
Phlebitis
Seizure.

29.
a.
b.
c.
d.
e.

The primary neurotransmitter involved with Lysergic!acid!diethylamide!(LSD)!is:


Acetylcholine
Dopamine

NAminobutyric!acid!
Norepinephrine
Serotonin.

Psychopharmacology
30. Monoamine oxidase inhibitor (MAOI) is LEAST useful in treating which of the following
disorders?
A.
B.
C.
D.
E.

Atypical depression
Obsessive compulsive disorder
Panic disorder
Severe depressive disorder not responding to selective serotonin reuptake inhibitor
Severe depressive disorder with hyponatraemia

31. Which! of! the! following! antidepressants! is! LEAST! likely! to! contribute! to! the!
development!of!serotonin!syndrome?!!$
!
A.
B.
C.
D.
E.

Bupropion!
Fluoxetine!!
Moclobemide!
Paroxetine!
Venlafaxine.!

32. Which of the following antipsychotics exhibit a novel mechanism as a partial antagonist?
A.
B.
C.
D.
E.

Aripiprazole
Clozapine
Olanzapine
Quetiapine
Ziprasidone.

33. Regarding!lamotrigine,!which!of!the!following!statements!is!FALSE?!!!
A. It!acts!at!voltageNsensitive!sodium!channels.!
!
!
36

B.
C.
D.
E.

It!inhibits!the!release!of!excitatory!amino!acid!neurotransmitters.!
It!is!not!effective!in!the!treatment!of!bipolar!depression.!!
Its!use!can!lead!to!toxic!epidermal!necrolysis.!
There!is!no!need!to!monitor!lamotrigine!blood!levels!during!treatment.!

!
34. Anticholinergic side effects include all of the following EXCEPT:

A.
B.
C.
D.
E.

Bradycardia
Constipation
Dry mouth
Exacerbation of open-angle glaucoma
Urinary retention.

35. In a schizophrenia patient without history of asthma, the BEST treatment for chronic
akathisia is:
A.
B.
C.
D.
E.

Alprazolam
Benzhexol
Clonazepam
Propranolol
Vitamin E.

36. Which of the following psychotropic medications is MOST EFFECTIVE in preventing


relapse in rapid cycling bipolar disorder?
A.
B.
C.
D.
E.

Haloperidol
Lithium
Olanzapine
Valproate
Risperidone.

Psychotherapy

37. In! conducting! psychotherapy! with! individuals! who! have! experienced! a! traumatic!
event! and! suffer! from! postTtraumatic! stress! disorder! (PTSD),! the! following! are! all!
recommended!techniques!EXCEPT:!!
!
A.
B.
C.
D.

Encouraging!avoidance!of!emotion!!
Examining!feelings!of!guilt!
Examining!the!patients!response!to!the!trauma!
Offering!consolation!

!
!
37

E. Overcoming!avoidance!or!phobia!related!to!the!trauma.!
38. A 30-year-old woman with panic disorder does not respond to an initial treatment with an
selective serotonin reuptake inhibitor (SSRI). Which of the following treatment is
considered the best approach?
A.
B.
C.
D.
E.

Benzodiazepine
Cognitive behaviour therapy
Serotonin-noradrenaline reuptake inhibitor
Olanzapine
Psychodynamic psychotherapy.

39. Which of the following statements is INCORRECT about Interpersonal therapy?


!
A.
B.
C.
D.
E.

Has!been!shown!to!be!efficacious!in!adult!outpatients!suffering!from!depression.!
Is!more!efficacious!than!psychodynamic!psychotherapy!
Works!as!quickly!as!antidepressant!medication.!
Works!best!in!mild!to!moderate!depression.!
Works!best!in!people!facing!problems!in!role!transition.!

!
40. A! 23TyearTold! student! is! seen! in! the! University! counselling! centre! because! of!

relationship!problems.!!She!has!had!one!serious!relationship!with!her!boyfriend!that!
lasted!about!six!months.!!She!terminated!the!relationship!because!she!discovered!that!
her!boyfriend!was!seeing!someone!else.!!During!psychotherapy,!she!mentions!that!All!
men!just!cant!be!trusted.!!Which!of!the!following!BEST!describes!this!thinking!error?
A.
B.
C.
D.
E.

Arbitrary!inference!
Dichotomous!thinking!
Magnification!
Overgeneralization!
Selective!abstraction.!

Ethics and laws

41. All! medical! students! are! required! to! study! psychiatry! so! that! they! are! qualified! to!
assess! and! manage! common! psychiatric! disorders! after! graduation.! The! compulsory!
psychiatric!training!is!BEST!described!by!which!of!the!following!ethical!principles?!!!
!
A.
B.
C.
D.
E.

Autonomy!
Beneficence!
Morality!
Justice!
Fiduciary!duty.!

Liaison and neuropsychiatry

!
!
38

42. You! are! a! medical! resident.! A! 40TyearTold! schizophrenia! patient! was! admitted! as! a!
result! of! polydipsia.! Which! of! the! following! statements! regarding! polydipsia! in!
schizophrenia!is!NOT!true?!!
!
A.
B.
C.
D.

It!is!associated!with!low!urine!osmolality.!
It!is!associated!with!high!sodium!in!urine.!
It!is!associated!with!low!serum!sodium.!!
Patients!should!be!investigated!for!Syndrome!of!Inappropriate!Secretion!of!AntiNDiuretic!
Hormone!(SIADH).!!
E. Polydipsia!occurs!in!between!5!to!20%!of!patients!suffering!from!chronic!schizophrenia.!!
!
!
43. A!40TyearTold!British!man!is!admitted!to!the!medical!ward.!He!tries!to!run!away!from!
the!ward.!He!is!very!aggressive!and!the!security!guards!have!difficulty!to!stop!him.!He!
claims!that!he!is!currently!working!in!the!office.!The!medical!records!show!that!he!has!
been!drinking!half!of!a!bottle!of!whisky!every!night.!You!need!to!order!a!medication!to!
sedate! him.! He! refuses! to! take! any! oral! medication.! ! Which! of! the! following!
medications!is!the!BEST!alternative!in!Singapore?!
!
A.
B.
C.
D.
E.
!

Intramuscular!diazepam!
Intramuscular!lorazepam!
Intramuscular!olanzapine!
Intramuscular!quetiapine!
Intramuscular!risperidone.!

44. You! are! the! resident! working! for! the! liver! transplant! team.! The! MOST! common!
psychiatric! disorder! in! patients! with! liver! cancer! in! the! week! before! and! after!
transplantation!is:!
!
A.
B.
C.
D.
E.

Adjustment!disorder!
Delirium!
Generalized!anxiety!disorder!!
Major!depressive!disorder!
Somatoform!pain!disorder.!
!

45. You!are!the!resident!working!in!the!obstetrics!department.!Which!of!the!following!is!
the!BEST!treatment!strategies!to!treat!hyperemesis!during!pregnancy?!
!
A.
B.
C.
D.
E.

Benzodiazepines!e.g.!alprazolam!
Relaxation!therapy!
Psychoanalysis!
Selective!serotonin!reuptake!inhibitor!e.g.!fluvoxamine!
Stimulus!deprivation.!

!
!
39

!
MCQ!exam!(Paper!2):!Answer!Key!
History, mental state exam and psychopathology!

46. Which!of!the!following!is!the!BEST!example!of! The!best!answer!is!D.!!


inattention?!
A. Impulsive!interruptions!may!
F. The!patient!interrupts!the!conversation!to!ask!
or!may!not!indicate!
when!he!will!be!discharged.!
inattentiveness.!
G. The!patient!is!oriented!and!aware!of!his!recent!
B. Falling!asleep!may!indicate!
medical!problems!but!falls!asleep!during!the!
inattentiveness,!but!further!
conversation.!
information!would!be!
H. The!patient!suddenly!bursts!into!tears!when!you!
needed!to!rule!out!other!
are!discussing!his!recent!amputation.!
explanations!such!as!recent!
I. The!patient!watches!a!fly!buzzing!on!the!ceiling!
administration!of!a!sedating!
while!you!are!discussing!the!prognosis!for!his!
medication.!!
lung!cancer,!then!falls!asleep.!
C. Sudden!bursts!of!affect!have!
J. The!patient!cannot!remember!what!you!have!
a!significant!differential!
told!him!3!minutes!ago.!Then!he!tries!to!make!up!
beyond!inattention.!!
the!answer.!
D. This!patient!seems!
distracted!despite!discussion!
of!an!issue!of!vital!personal!
importance!to!him.!!In!the!
context!also!of!apparent!
drowsiness,!the!clinician!
should!suspect!the!presence!
of!delirium.!
E. The!patient!may!suffer!from!
dementia!and!cannot!
register!the!information.!
!
!
!
Year:!2013.!
47. A!30TyearTold!man!firmly!believes!that!the!
alien!has!put!an!implant!in!his!body!and!he!
feels!there!is!a!pushing!sensation!on!his!
aorta.!Which!types!of!hallucinations!BEST!
suited!his!description?!

!
F.
G.
H.
I.

Auditory!
Cenesthetic!
Gustatory!
Kinesthetic!!

!
!
40

The!answer!is!B.!
!
Explanation:!Cenesthetic!
hallucinations!are!unfounded!
sensations!of!altered!states!in!bodily!
organs.!!Examples!of!cenesthetic!
hallucinations!are!a!burning!
sensation!in!the!brain,!a!pushing!
sensation!in!the!blood!vessels,!and!a!

J.

Visual.!

cutting!sensation!in!the!bone!
marrow.!!Bodily!distortion!may!also!
occur.!
!

Year:!2013.!
!
This!is!an!advancedNlevel!question.!
!
48. Perseveration!is!MOST!commonly!seen!in!
which!of!the!following!disorders?!!!

The!answer!is!D.!

F.
G.
H.
I.
J.

Explanation:!Perseveration!is!a!
persisting!response!to!a!previous!
stimulus!after!a!new!stimulus!has!
been!presented;!it!is!often!
associated!with!frontal!lobe!lesion.!

!
Autism!
Attention!deficit!and!hyperactivity!disorder!
Bipolar!disorder!!
Frontal!lobe!dementia!!
Obsessive!compulsive!disorder.!

!
Year:!2013!
!
49. A patient presents with persistence,
perfectionism and body image distortion. Which
of the following disorders is MOST likely to be
associated with the above clinical features?
F.
G.
H.
I.
J.

The!answer!is!A.!
!
Explanation:!Persistence!and!
perfectionism!are!associated!with!
obsessiveNcompulsive!personality!
traits!and!disorders,!along!with!
body!image!distortion!occurring!in!
many!patients!with!anorexia!
nervosa.!!

Anorexia nervosa
Borderline personality disorder
Delusional disorder
Hypochondriasis
Somatization disorder.

!
!
Year:!2013!
!

!
!
41

Cognitive assessment!

50. Which of the following statement is FALSE


about Mini-mental state examination (MMSE)?

The!answer!is!E.!
!

F. A total score of 5 indicates severe dementia.


G. Assessment of orientation to time and place is part
of the MMSE.
H. The MMSE can be administered in Chinese or
English.
I. The MMSE can be used to monitor treatment
progress after initiation of acetylcholinesterase
inhibitors (AChEIs).
J. The MMSE is the gold standard in establishing the
diagnosis of dementia.

Explanation:!MMSE!is!a!screening!
test!and!there!is!no!gold!standard!to!
establish!the!diagnosis!of!dementia!
in!clinical!practice.!Clinical!
assessment!by!a!psychiatrist!or!
geriatrician!is!more!important!than!
MMSE!in!establishing!diagnosis!of!
dementia.!
!
!
Year:!2013!
!

Psychiatric epidemiology

51. Which!of!the!following!settings!have!the!
HIGHEST!prevalence!of!delirium?!
!
F.
G.
H.
I.
J.

Accident!and!emergency!department!
Cardiac!surgery!ward!
General!surgery!ward!
Hospice!with!palliative!advanced!cancer!patients!
Nursing!home!looking!after!dementia!patients.!

!!
52. The worldwide point prevalence of
schizophrenia is BEST estimated as:

The!answer!is!D.!!
!
Explanation:!General!surgical!
patients:!!10N15%;!Cardiac!surgery!
patients:!!30%;!Hip!fractures:!!50%;!
Age!>65!admitted!to!ICU:!!70%;!
Palliative!advanced!cancer!patients:!!
88%.!!
!
!
Year:!2013.!
The!answer!is!A.!
!

F.
G.
H.
I.
J.

Explanation:!The!worldwide!point!
prevalence!is!1%!and!Singapore!is!
0.6%.!

1%
2.5%
5%
7.5%
10%.

!
!
Year:!2013!

!
!
42

Psychiatric aetiology, diagnosis and classification


53. Advanced paternal age is a well- established
risk factor for which of the following psychiatry
illnesses?
F.
G.
H.
I.
J.

!
The!answer!is!E.!
!
Explanation:!Advanced!paternal!age!
is!a!risk!factor!for!schizophrenia.!

Alcoholism
Major depressive disorder
Obsessive compulsive disorder
Generalized anxiety disorder
Schizophrenia.

!
!
Year:!2013.!

54. Which of the following social factors is


MOST LIKELY to be associated with relapse
of schizophrenia after hospitalization?
A. Discrimination
B. Financial problems
C. Homelessness
D. Increased expressed emotion
E. Isolation and absence of family
member.

The answer is D.

High levels of expressed emotion


(EE) in the families of patients with
schizophrenia predicted relapse
following hospital discharge.
Psychoeducational family therapy
aims at helping the family reduce
the factors that constitute
expressed emotion. The therapist
also educates the family about
schizophrenia and the need to
continue antipsychotic medication
indefinitely.

!
Year:!2013.
55. Individuals!with!which!one!of!the!following!

genes!have!the!HIGHEST!risk!of!developing!
Alzheimers!disease?
F.
G.
H.
I.
J.

Homozygous apolipoproteins!(APO) E2/E2


Homozygous APO E4/E4
Homozygous APO E6/E6
Heterozygous APO E2/E4
Heterozygous APO E2/E6.

!
!
43

The!answer!is!B.!
!
Explanation:!Homozygous!APO!
E4/E4!increases!the!risk!of!
developing!Alzhimers!disease!by!8!
times.!

!
!
Year:!2013.!
!
!

General adult psychiatry

56. A!40TyearTold!man!suffered!from!a!heart!
attack!under!general!anaesthesia.!The!
doctors!in!the!operation!theatre!successfully!
resuscitated!him.!After!recovery,!he!wants!to!
sue!the!hospital!for!causing!him!postT
traumatic!stress!disorder!(PTSD).!He!claims!
the!resuscitation!process!was!!!extremely!
traumatic.!Which!of!the!following!is!the!MOST!
IMPORTANT!feature!to!exclude!the!diagnosis!
of!PTSD?!!
!
F. He does not have past history of PTSD.
G. He has hidden agenda of suing the hospital
and tries to seek compensation.
H. He was under general anaesthesia and he was
not conscious during resuscitation.
I. The process of resuscitation is not considered
to be traumatic in general and doctor
resuscitated him in his best interest.
J. He did not seek treatment from a psychiatrist
after the operation.

The!answer!is!D.!
!
Explanation:!In!the!new!diagnostic!
criteria,!it!is!possible!for!a!PTSD!
patient!not!to!directly!witness!a!
traumatic!event!and!learn!about!it!
from!others!after!the!accident.!The!
most!crucial!feature!is!whether!
resuscitation!itself!is!not!a!traumatic!
process!and!!it!is!necessary!for!
doctors!to!save!patients!life!by!
resuscitation.!
!
!
Year:!2013!
!
This!is!an!advanced!!level!question.!

57. Which of the following schizophrenia patients


has the HIGHEST suicide risk?

!
A.!An!adolescent!with!prodromal!symptoms!and!vague!
paranoid!idea.!
B.!An!elderly!retired!man!with!late!onset!schizophrenia.!
C.!A!middleNaged!man!with!negative!symptoms.!
D.!A!middleNaged!woman!who!suffers!from!simple!
schizophrenia.!
E.!A!young!male!university!student!who!once!had!high!

!
!
44

The!answer!is!E.!
!
Explanation:!The!profile!of!the!
patient!at!greatest!risk!is!a!young!
man!who!once!had!high!
expectations,!declined!from!a!higher!
level!of!functioning,!realizes!that!his!
dreams!are!not!likely!to!come!true,!
and!has!lost!faith!in!the!effectiveness!
of!treatment.!

expectations.!

!
Year:!2013.!!!
58. Which of the following is NOT a recognized
alteration of sleep disturbance associated with
major depressive disorder?
F. Difficulty in falling asleep
G. Increase in nocturnal awakening
H. Increase in density of Rapid Eye Movement (REM)
sleep
I. Increase in the latency of Rapid Eye Movement
(REM) sleep
J. Reduction of total sleep time.

The!answer!is!D.!
!
Explanation:!It!should!be!reduction!
of!latency!of!REM!sleep.!
!
!
Year:!2013.!
!
This!is!an!advanced!level!question.!

59. A 40-year-old woman is staying with her godbrother. She was rejected by her family and she
felt helpless in the past. She needs her godbrother to make decision for her and her godbrother asks her to be the housekeeper in
return. One day, her god-brother needs to go to
Vietnam for 1 week and she feels extremely
uncomfortable. Which of the following
personality traits BEST describes this person?
F.
G.
H.
I.
J.

Borderline personality
Dependent personality
Histrionic personality
Schizoid personality
Schizotypal personality.

The!answer!is!B.!
!
Explanation:!She!is!best!described!to!
suffer!from!dependent!personality!
because!she!cannot!make!decision!
on!her!own,!uncomfortable!when!
left!alone!and!subordination!to!her!
godNbrothers!need!(e.g.!to!be!a!
housekeeper).!
!
!
Year:!2013!

60. A 30-year-old man comes to see you and he


worries about premature ejaculation. The MOST
appropriate time frame to meet the diagnosis of
premature ejaculation is:
F. Ejaculation occurs within 60 seconds following
vaginal penetration.
G. Ejaculation occurs within 90 seconds following
vaginal penetration.
H. Ejaculation occurs within 120 seconds following
vaginal penetration.

!
!
45

The!answer!is!A.!
!
Explanation:!Premature!ejaculation!
is!defined!as!ejaculation!occurs!
within!60!second!following!vaginal!
penetration!and!the!patient!wishes!

I.
J.

Ejaculation occurs within 150 seconds following


vaginal penetration.
Ejaculation occurs within 180 seconds following
vaginal penetration.

it.!
!
!
Year:!2013!

61. A!30TyearTold!man!suffered!from!eight!

episodes!of!mood!disturbances!within!a!
single!year.!These!mood!episodes!met!the!
diagnostic!criteria!for!major!depression,!
mania!and!hypomania.!These!episodes!were!
separated!by!remission.!Which!of!the!
following!is!the!MOST!likely!diagnosis?
!
F.
G.
H.
I.
J.

Cyclothymic!disorder!
Emotionally!unstable!personality!disorder!
Mixed!affective!disorder!
Rapid!cycling!bipolar!disorder!
Schizoaffective!disorder.!

!
Explanation:!This!patient!has!more!
than!4!episodes!of!mania!and!
depressive!disorder!in!1!year.!He!is!
too!severe!for!cyclothymic!disorder.!
!
!
Year:!2013!
!

Old age psychiatry


62. Which!of!the!following!is!NOT!a!predisposing!

factor!for!depression!in!the!elderly?
F.
G.
H.
I.
J.

The!answer!is!D.!

The!answer!is!D.!
!

Cerebrovascular accident
Poly-pharmacy
Looking after a spouse with chronic illness
Male gender
Widowhood.

Explanation:!Women!have!longer!
life!span!compared!to!men.!As!a!
result,!women!are!more!likely!to!be!
a!widow!and!correspond!to!option!E.!
!
!
Year:!2013.!
!

63. Dementia!in!general!is!BEST!described!as:!!
!

The!answer!is!E.!
!

F. Behavioural!disturbance!
G. Global!impairment!
H. Impaired!memory!for!personal!events!(e.g.!
marriage,!past!occupation)!
I. Personality!change!
!
!
46

Explanation:!
dementia!
is!
a!
progressive! ! deterioration! of!
cognition,!including!memory!and/or!

J.

Progressive!deterioration.!

one! of! four! other! cognitive!


symptoms!
(aphasia,!
apraxia,!
agnosia,!
impaired!
executive!
functioning)! but! it! may! not! be! a!
global! impairment.! Impairment!
memory! for! recent! events! is! more!
common!than!personal!events.!
!
!
Year:!2013.!

64.

A!65TyearTold!man!presents!with!
memory!loss,!which!of!the!following!is!the!
LEAST!important!indication!for!a!
computerized!tomography!(CT)!scan!for!his!
brain:

F. A sudden decrease in cognitive function over a onemonth period


G. Gait abnormalities
H. His age
I. Urinary incontinence
J. Use of anticoagulants.

The!answer!is!C.!
!
Explanation:!!The!other!factors!
suggest!an!organic!cause.!
!
!
Year:!2013.!

Child and adolescent psychiatry

65. Which!of!the!following!is!THE!MOST!
IMPORTANT!predictor!of!bipolar!disorder!in!
adolescent!depression?!
!

The!answer!is!B.!
!
Explanation:!!Antidepressant!is!the!
most!common!cause!of!hypomania!
and!hypomania!is!part!of!the!bipolar!
disorder.!

F. Diurnal!variation!of!mood!
G. Hypomanic!symptoms!after!taking!
antidepressants!
H. Hypersomnia!
I. Increased!weight!
J. Multiple!suicide!attempts.!

!
!
Year:!2013.!

66. The!MOST!COMMON!reason!for!children!with!
autism!are!brought!to!medical!attention!by!
their!parents!is: !

!
!
47

The!answer!is!A.!
!

!
F.
G.
H.
I.
J.

Explanation:!Language!delay!is!the!
most!obvious!sign!and!most!
disturbing!to!parents!when!they!
discover!their!children!are!lacking!
behind!in!language!development!
compared!to!other!children.!

Language!delays!!
Lack!of!toilet!control!
Odd!play!
School!difficulties!
Stereotyped!behaviour.!

!
!
Year:!2013.!
67. Which of the following statements is FALSE
regarding separation anxiety disorder?

The!answer!is!B.!
!

F.

It involves recurrent distress when separate from


attachment figures.
G. Separation anxiety disorder does not occur in
adolescents.
H. Separation anxiety disorder is associated with school
refusal.
I. The child has difficulty falling asleep at night.
J. The child may complain of somatic symptoms.

Explanation:!Separation anxiety disorder

68. Which of the following symptoms is MOST


LIKELY to persist when a young person suffers
from Attention Deficit and Hyperactivity
Disorder (ADHD) becoming an adult?

F.
G.
H.
I.
J.

is more common in children and it can


occur in adolescents.

!
!
Year:!2013.!

The!answer!is!D.!
!

Accident risk
Hyperactivity
Impulsivity
Inattention
Learning difficulties.

Answer:!Although!symptoms!of!
hyperactivity!and!impulsivity!often!
improve!as!the!child!grows!older,!
inattentive!symptoms!are!likely!to!
persist.!
!
!
Year:!2013!

69. A baby is born with low birth weight, microcephaly,


small eyes, upturned nose and a smooth, undeveloped
philtrum. He fails to thrive and develops seizures.
Which of the following was he MOST likely exposed
to during pregnancy?

!
!
48

The!answer!is!A.!
!
Explanation:!This!is!a!classical!

description!of!the!facial!features!of!
foetal!alcohol!syndrome!

F. Alcohol
G. Cocaine
H. LSD
I. Methamphetamine
J. Inhalants.

!
!
Year:!2013.!
!

Substance abuse

70. The!term!which!BEST!describes!!an!altered!
physiological!state!and!neuroT!adaptation!
caused!by!repeated!administration!of!a!drug!
is:!
!
F.
G.
H.
I.
J.

Addiction!
Dependence!
Misuse!
Withdrawal!
Tolerance.!
!

71. While!driving,!a!40TyearTold!man!with!no!
previous!history!refused!to!stop!for!the!traffic!
police.!He!was!subsequently!brought!into!the!
accident!and!emergency!department!(AED)!
by!them.!You!are!the!resident!working!in!the!
AED.!!When!you!assess!him,!he!states!that!he!
was!hearing!voices!in!clear!consciousness.!He!
admits!to!a!history!of!alcohol!use.!What!is!the!
most!appropriate!diagnosis?!
!
F.
G.
H.
I.
J.

Alcoholic!hallucinosis!!
Delirium!tremens!
Korsakoff!psychosis!
Schizophrenia!
Wernickes!encephalopathy.!

The!answer!is!B.!
!
Explanation:!The!term!dependence!
indicates!physiological!changes.!
Misuse!refers!to!use!of!any!drug,!
usually!by!selfNadministration,!in!a!
manner!that!deviates!from!approved!
social!or!medical!patterns.!Addiction!
is!nonNspecific!and!ignores!the!
concept!of!substance!dependence!is!
a!medical!condition.!!
!
!
Year:!2013.!
The!answer!is!A.!
!
Explanation:!!Auditory!hallucinosis!
is!usually!composed!of!auditory!
hallucination.!Sensorium!is!clear!and!
it!is!different!from!delirium!tremens.!
There!are!not!enough!symptoms!to!
conclude!that!he!suffers!from!
schizophrenia.!
!
Year:!2013.!

!
72. Which of the following BEST describes the
mechanism of action with naltrexone in the treatment
of alcohol dependence?

!
!
49

The!answer!is!B.!
!

F.

Naltrexone acts on the benzodiazepine receptor and


prevents alcohol withdrawal.
G. Naltrexone blocks opioid receptors to decrease alcohol
craving.
H. Naltrexone inhibits aldehyde dehydrogenase to decrease
alcohol craving.
I. Naltrexone is a GABA agonist and decreases alcohol
craving.
J. Naltrexone is a glutamate antagonist and decreases
alcohol craving.

Explanation:!Option!A!refers!to!
benzodiazepine.!Option!C!refers!to!
the!action!of!disulfiram.!Option!D!
and!E!refer!to!the!action!of!
acamprosate.!!
!
!
Year:!2013.!

73. Which of the following complications is LEAST


likely to occur in cocaine abusers?
F. Arrhythmia
G. Hyperthermia
H. Myocardial infarction
I. Phlebitis
J. Seizure.

The!answer!is!D.!
!
Explanation:!Cocaine!is!least!likely!
to!be!injected!via!IV!route.!As!a!
result,!phlebitis!is!relatively!less!
likely!to!occur!in!patients!misusing!
cocaine.!
!
!
Year:!2013!
!
This!is!an!advancedNlevel!question.!
!!

74. The primary neurotransmitter involved with


Lysergic!acid!diethylamide!(LSD)!is:
f. Acetylcholine
g. Dopamine
h. NAminobutyric!acid!
i. Norepinephrine
j. Serotonin.

The!answer!is!E.!
!
Explanation:!LSD!increases!the!level!
of!serotonin!and!overdose!of!LSD!is!
associated!with!serotonin!
syndrome.!
!
!
Year:!2013.!

!
!
50

!
!

Psychopharmacology
75. Monoamine oxidase inhibitor (MAOI) is LEAST
useful in treating which of the following
disorders?
F.
G.
H.
I.
J.

Atypical depression
Obsessive compulsive disorder
Panic disorder
Severe depressive disorder not responding to
selective serotonin reuptake inhibitor
Severe depressive disorder with hyponatraemia

The!answer!is!B.!
!
Explanation:!MAOI!is!least!useful!to!
treat!OCD.!
!
!
Year:!2013.!
!
This!is!an!advancedNlevel!question.!

76. Which! of! the! following! antidepressants! is! The!answer!is!A.!


LEAST! likely! to! contribute! to! the!
!
development!of!serotonin!syndrome?!!$
!
F.
G.
H.
I.
J.

Explanation:!Bupoprion!works!on!
dopamine!and!norepinephrine.!As!a!
result,!it!is!least!likely!to!cause!
serotonin!syndrome.!

Bupropion!
Fluoxetine!!
Moclobemide!
Paroxetine!
Venlafaxine.!

!
!
Year:!2013.!
!
This!is!an!advancedNlevel!question.!
!

77. Which of the following antipsychotics exhibit a


novel mechanism as a partial antagonist?

The!answer!is!A.!
!

F.
G.
H.
I.
J.

Aripiprazole
Clozapine
Olanzapine
Quetiapine
Ziprasidone.

Explanation:!Aripiprazole!exhibits!a!
novel!mechanism!by!definition!
while!other!antipsychotics!listed!do!
not!have!this!property.!

!
!
51

!
!
Year:!2013.!
78. Regarding!lamotrigine,!which!of!the!following!
statements!is!FALSE?!!!
F. It!acts!at!voltageNsensitive!sodium!channels.!
G. It!inhibits!the!release!of!excitatory!amino!acid!
neurotransmitters.!
H. It!is!not!effective!in!the!treatment!of!bipolar!
depression.!!
I. Its!use!can!lead!to!toxic!epidermal!necrolysis.!
J. There!is!no!need!to!monitor!lamotrigine!blood!
levels!during!treatment.!
!

The!answer!is!C.!
!
Explanation:!DoubleNblind,!placeboN
controlled!trials!have!shown!that!it!
has!acute!and!prophylactic!antiN
depressant!effects.!It!can!lead!to!
severe!dermatological!condition!
such!as!Steven!Johnson!syndrome/!
toxic!epidermal!necrolysis.!In!
Singapore,!there!is!no!blood!test!to!
check!its!level.!
!
!
Year:!2013!
!
This!is!an!advancedNlevel!question.!
!

79. Anticholinergic side effects include all of

the following EXCEPT:


F.
G.
H.
I.
J.

Bradycardia
Constipation
Dry mouth
Exacerbation of open-angle glaucoma
Urinary retention.

The!answer!is!A.!
!
Explanation:!Anticholinergic!side!
effect!include!tachycardia.!
!
!
Year:!2013.!

80. In a schizophrenia patient without history of


asthma, the BEST treatment for chronic
akathisia is:

!
!
52

The!answer!is!D.!
!

F.
G.
H.
I.
J.

Explanation:!Propranolol!(beta!
blocker)!is!the!best!treatment!in!this!
patient!without!history!of!asthma!as!
it!does!not!cause!dependence!as!in!
Option!A!and!C.!Beta!blockers!can!
reduce!restlessness!and!tremor.!!
Option!B!is!more!useful!for!
pseudoparkinsonism.!

Alprazolam
Benzhexol
Clonazepam
Propranolol
Vitamin E.

!
!
Year:!2013.!
81. Which
of
the
following
psychotropic
medications is MOST EFFECTIVE in preventing
relapse in rapid cycling bipolar disorder?
F.
G.
H.
I.
J.

The!answer!is!D.!
!
Explanation:!Lithium!is!less!effective!
in!preventing!relapse!in!rapid!
cycling!bipolar!disorder.!Valproate!
and!carbamazepine!are!preferred!
over!lithium.!

Haloperidol
Lithium
Olanzapine
Valproate
Risperidone.

!
!
2013!
!

Psychotherapy

82. In!conducting!psychotherapy!with!individuals! The!answer!is!A.!


who!have!experienced!a!traumatic!event!and!
suffer! from! postTtraumatic! stress! disorder! !
(PTSD),! the! following! are! all! recommended!
Explanation:!A!is!the!answer.!Affect!
techniques!EXCEPT:!!
or!emotion!should!not!be!avoided!in!
psychotherapy.!
F. Encouraging!avoidance!of!emotion!!
G. Examining!feelings!of!guilt!
!
H. Examining!the!patients!response!to!the!trauma!
I. Offering!consolation!
!
J. Overcoming!avoidance!or!phobia!related!to!the!
trauma.!
Year!2013.!

!
!
53

83. A 30-year-old woman with panic disorder does


not respond to an initial treatment with an
selective serotonin reuptake inhibitor (SSRI).
Which of the following treatment is considered
the best approach?
F.
G.
H.
I.
J.

Benzodiazepine
Cognitive behaviour therapy
Serotonin-noradrenaline reuptake inhibitor
Olanzapine
Psychodynamic psychotherapy.

The!answer!is!B.!
!
Explanation:!The!effect!of!CBT!is!as!
efficacious!as!SSRI.!
!
!
Year:!2013.!

84. Which of the following statements


INCORRECT about Interpersonal therapy?

is

The!answer!is!C.!
!

F.

Has!been!shown!to!be!efficacious!in!adult!outpatients!
suffering!from!depression.!
G. Is!more!efficacious!than!psychodynamic!
psychotherapy!
H. Works!as!quickly!as!antidepressant!medication.!
I. Works!best!in!mild!to!moderate!depression.!
J. Works!best!in!people!facing!problems!in!role!
transition.!
!

Explanation:!Interpersonal!therapy!
is!slower!than!the!effect!of!
antidepressant!because!the!
therapist!needs!time!to!assess!the!
patient!and!formulate!strategies!(e.g.!
interpersonal!inventory!and!role!
play).!
!
!
!
Year:!2013.!

85. A! 23TyearTold! student! is! seen! in! the! The!answer!is!D.!

University! counselling! centre! because! of!


relationship! problems.! ! She! has! had! one!
serious! relationship! with! her! boyfriend! that!
lasted! about! six! months.! ! She! terminated! the!
relationship! because! she! discovered! that! her!
boyfriend! was! seeing! someone! else.! ! During!
psychotherapy,! she! mentions! that! All! men!
just! cant! be! trusted.! ! Which! of! the! following!
BEST!describes!this!thinking!error?
F.
G.
H.
I.
J.

Arbitrary!inference!
Dichotomous!thinking!
Magnification!
Overgeneralization!
Selective!abstraction.!

!
Explanation:!This!patient!
demonstrates!overgenearlization!
because!she!believes!in!and!follows!
a!general!rule!on!the!basis!of!limited!
examples.!
!
!
Year:!2013.!

!
!
54

Ethics and laws

86. All! medical! students! are! required! to! study!


psychiatry!so!that!they!are!qualified!to!assess!
and! manage! common! psychiatric! disorders!
after!graduation.!The!compulsory!psychiatric!
training! is! BEST! described! by! which! of! the!
following!ethical!principles?!!!
!
F.
G.
H.
I.
J.

Autonomy!
Beneficence!
Morality!
Justice!
Fiduciary!duty.!

The!answer!is!B.!
!
Explanation:!With!competent!
psychiatric!knowledge,!the!future!
doctors!can!provide!benefits!to!
patients!when!making!decision!on!
diagnosis!and!management.!The!
other!concept!that!is!relevant!is!
dereliction.!Fiduciary!duty!refers!to!
the!duty!that!the!doctor!must!act!in!
the!patients!best!interest!and!it!is!
not!as!relevant!as!beneficence!in!this!
case.!!
!
!
Year:!2013!
!
!

Liaison and neuropsychiatry

87. You! are! a! medical! resident.! A! 40TyearTold! The!answer!is!B.!


schizophrenia! patient! was! admitted! as! a!
result! of! polydipsia.! Which! of! the! following! !
statements!
regarding!
polydipsia!
in!
Explanation:!Psychogenic!
schizophrenia!is!NOT!true?!!
polydispsia!is!a!common!condition!
for!chronic!schizophrenia.!The!
F. It!is!associated!with!low!urine!osmolality.!
estimates!of!prevalence!range!from!
G. It!is!associated!with!high!sodium!in!urine.!
5!to!20%.!The!patient!drinks!excess!
H. It!is!associated!with!low!serum!sodium.!!
I. Patients!should!be!investigated!for!Syndrome!of! water!and!results!in!diluted!urine.!
The!body!tries!to!preserve!sodium!
Inappropriate!Secretion!of!AntiNDiuretic!
and!results!in!low!urine!sodium.!The!
Hormone!(SIADH).!!
J. Polydipsia!occurs!in!between!5!to!20%!of!
serum!is!diluted.!SIADH!should!be!
patients!suffering!from!chronic!schizophrenia.!!
considered!for!DDX.!
!
!
!

!
!
55

Year:!2013.!
!
This!is!an!advancedNlevel!question.!
88. A! 40TyearTold! British! man! is! admitted! to! the!
medical! ward.! He! tries! to! run! away! from! the!
ward.! He! is! very! aggressive! and! the! security!
guards! have! difficulty! to! stop! him.! He! claims!
that!he!is!currently!working!in!the!office.!The!
medical! records! show! that! he! has! been!
drinking! half! of! a! bottle! of! whisky! every!
night.! You! need! to! order! a! medication! to!
sedate! him.! He! refuses! to! take! any! oral!
medication.! ! Which! of! the! following!
medications! is! the! BEST! alternative! in!
Singapore?!
!
F.
G.
H.
I.
J.
!

Intramuscular!diazepam!
Intramuscular!lorazepam!
Intramuscular!olanzapine!
Intramuscular!quetiapine!
Intramuscular!risperidone.!

The!answer!is!B.!
!
Explanation:!He!suffers!from!
delirium!tremens.!Intramuscular!
diazepam!has!poor!absorption.!
Option!C!and!D!are!not!available!in!
Singapore.!Option!E!is!a!depot!
antipsychotics!and!it!is!not!useful!in!
acute!setting.!
!
!
Year:!2013!
!
!
!
!

89. You! are! the! resident! working! for! the! liver!


transplant! team.! The! MOST! common!
psychiatric! disorder! in! patients! with! liver!
cancer! in! the! week! before! and! after!
transplantation!is:!
!
F.
G.
H.
I.
J.

Adjustment!disorder!
Delirium!
Generalized!anxiety!disorder!!
Major!depressive!disorder!
Somatoform!pain!disorder.!
!

The!answer!is!B.!
!
Explanation:!Delirium!is!the!most!
common!disorder!and!it!can!be!
caused!by!systemic!infection,!side!
effect!of!immunosuppressant,!
impairment!of!liver!function,!hepatic!
encephalopathy.!
!
!
Year:!2013.!

90. You!are!the!resident!working!in!the!obstetrics! The!answer!is!B.!

!
!
56

department.! Which! of! the! following! is! the! !


BEST! treatment! strategies! to! treat! Explanation:!Benzodiazepine!may!
hyperemesis!during!pregnancy?!
cause!cleft!palate!in!the!first!
trimester.!Psychoanalysis!is!not!for!
symptomatic!relief.!SSRI!like!
F. Benzodiazepines!e.g.!alprazolam!
fluvoxamine!is!associated!with!
G. Relaxation!therapy!
nausea!side!effect.!Hyperemesis!is!
H. Psychoanalysis!
I. Selective! serotonin! reuptake! inhibitor! e.g.! not!associated!with!stimulus.!
fluvoxamine!
Relaxation!therapy!is!the!best!
J. Stimulus!deprivation.!
treatment.!!
!
!
Year:!2013.!

!
!
!
!
!
!
!
!
!
!
!
!
!
MCQ!exam!(Paper!3)!:!Questions!
History and mental state exam!

1. A!40TyearTold!woman!is!brought!by!her!husband!brought!to!see!you!as!a!result!of!
abnormal!behaviour.!During!the!interview,!she!laughs!and!cries!with!a!very!short!
period!of!time.!In!your!record,!her!affect!is!BEST!described!as:!
!
!
57

!
A.
B.
C.
D.
E.

Apathy!
Blunted!affect!
Euthymic!affect!
Labile!affect!
Flat!affect.!
Cognitive assessment!
2. Which of the following is the presentation of cortical dementia?
A. A 40-year-old man presents with memory loss and writhing!movements.!His!father!suffered!

from!the!same!condition.!
B. A!50NyearNold!man!presents!with!change!in!personality,!poor!judgment,!disinhibition!and!
perservation.!He!made!a!lot!of!mistakes!in!Lurias!hand!test.!
C. A 60-year-old woman presents with memory loss and depression after a cerebrovascular
accident.!
D. A 70-year-old man presents with memory loss, rigidity, slowness!of!movement!and!difficulty!

with!walking!and!gait.!
E. A 75-year-old woman presents with visual hallucination. The GP gave her haloperidol and she
develops severe extrapyramidal side effects.!
Psychiatric epidemiology
3. After the age of 65, the prevalence of dementia increases by how many times every 5
years?
A. 2 times
B. 4 times
C. 6 times
D. 8 times
E. 10 times.
Psychiatric aetiology, diagnosis and classification
4. An 18-year-old woman presents with questions about her risk of developing
schizophrenia. She is worried because her 23-year-old brother has recently been
diagnosed with schizophrenia and required hospitalization for psychosis. There are no
other family members who have a history of schizophrenia. Having a sibling with
schizophrenia increases her risk of developing schizophrenia by?
A.
B.
C.
D.
E.

2 times
4 times
6 times
8 times
10 times.

5. Which of the following neuroanatomical structures is the critical region for fear
conditioning?
A. Amgydala

!
!
58

B. Basal nucleus of Meynert


C. Mesocortical region
D. Nucleus accumbens
E. Raphe nucleus.

6. The most likely neuroanatomical substrate of Tourette's syndrome is


A. The thalamus.
B. The pituitary.
C. The prefrontal cortex.
D. The basal ganglia.
E. The amygdala.

Psychopathology

7. Agoraphobia!without!panic!disorder!can!be!described!as:!!!
!
A.
B.
C.
D.
E.

A!fear!of!enclosed!spaces!!
A!fear!of!speaking!in!public!
A!fear!of!height!
Avoidance!of!authority!figures!
Worrying!excessively!about!physical!health.!

8. Uncomplicated!bereavement!is!NOT!characterized!by:!!
A. Anger!
B. Depressive!mood!for!one!month!
C. Feelings!of!worthlessness!!
D. Sensations!of!seeing!or!hearing!the!deceased!
E. Transient!guilt.!
9. The common clinical symptoms for pre-menstrual dysphoric disorder includes
all of the following EXCEPT:
a. Bloating
b. Difficulty concentrating
c. Guilt
!
!
59

d. Sleep disturbance
e. Lethargy and fatigue

10. A!40TyearTold!airTconditioner!technician!is!referred!to!you!because!of!change!
in!behaviour!and!deterioration!in!functioning.!When!he!feels the gust of cold
air coming from the air-conditioner, he interprets the cold air as poison gas
from Mars. This phenomenon is BEST described as:!
A.
B.
C.
D.
E.

Delusional memory
Delusional perception
Tactile hallucination
Primary delusion
Secondary delusion.

11. Which of the following behaviour is NOT considered to be purging?


A.
B.
C.
D.
E.

Self-induced vomiting
Excessive exercise
Excessive use of laxative
Excessive use of diuretic
Excessive use of enema.

General adult psychiatry

12. In!schizophrenia,!which!of!the!following!factors!is!LEAST!Likely!to!restrict!full!
rehabilitation!potential?!
!
A.
B.
C.
D.
E.

Florid!delusions!and!hallucinations!!
Lack!of!pleasure!from!social!intervention!and!physical!activity!
Loss!of!interest!in!rehabilitation!
Poor!motivation!
Restricted!affect.!

!
13. Somnambulism!(Sleepwalking!disorder)!occurs!in!
!
A.
B.
C.
D.
E.

Active!sleep!
Earliest!phase!of!sleep!
Paradoxical!sleep!
Rapid!eye!movement!(REM)!sleep!
SlowNwave!sleep.!

!
!
60

!
!
14. Which!of!the!following!is!a!poor!prognostic!factor!for!schizophrenia?!
!
A. Family!history!of!mood!disorder!
B. Female!gender!
C. Late!onset!
D. Precipitating!factor!
E. Presence!of!neurological!soft!sign.!
!
!

!
15. All of the following medical disorders are more common in schizophrenia patients as
compared to the general population EXCEPT?
A.
B.
C.
D.
E.

Chronic obstructive pulmonary disease


Diabetes mellitus
Human immunodeficiency virus infection
Metabolic syndrome
Rheumatoid arthritis.

16. A!40TyearTold!executive!is!afraid!of!flying!by!aeroplane.!She!has!this!problem!since!young.!!As!
a!result,!she!has!to!quit!her!job!because!it!requires!frequent!travelling.!What!is!the!MOST!
LIKELY!diagnosis?!
!
A.!Agoraphobia!
B.!Generalized!anxiety!disorder!
C.!Panic!disorder!
D.!Social!phobia!
E.!Specific!phobia.!
!
Old age psychiatry

17.
A.
B.
C.
D.
E.

Which!is!the!MOST!preferred!antidepressant!in!elderly?!
Amitriptyline!
Imipramine!
Moclobemide!
Sertraline!
Venlafaxine.!

!
!
61

!
18. Which!of!the!following!statements!is!TRUE!regarding!risk!factors!for!Alzheimers!
disease? !
!
A.
B.
C.
D.

APO!E2!genotype!increases!the!risk!of!Alzheimers!disease.!
APO!E4!genotype!increases!the!risk!of!Alzheimers!disease.!
APO!E6!genotype!increases!the!risk!of!Alzheimers!disease.!!
Male!gender!is!a!risk!factor!!
E. Smoking!is!protective!against!Alzheimers!disease.
19. Which!of!the!following!is!the!MOST!common!type!of!hallucination!in!people!suffering!
from!dementia? !
A. Functional!
B. Gustatory!
C. Olfactory!
D. Tactile!
E. Visual.!!!
!
20. Late-onset psychotic disorder such as schizophrenia or delusional disorder is associated
with:
A.
B.
C.
D.
E.

Less paranoia
More likely to have sensory deficits
More loosening of associations
More thought disorders
More violent behaviour.

21. Which of the following statements about sleep disorders in old people is FALSE?
A.
B.
C.
D.
E.

Excessing napping is a common cause of insomnia.


Insomnia is common in old people in Singapore.
Insomnia is associated with increased irks of depression.
If there is a need for benzodiazepine, diazepam is the preferred sedative for old people.
The first line treatment is sleep hygiene.

22. An 80-year-old man presents with terminal insomnia, weight loss, decreased
appetite and pervasive anhedonia. Physical examination, laboratory
investigations and imaging reveal normal findings. The MOST specific symptom
to his likely diagnosis is:
A.
B.
C.
D.

Anxiety
Decreased concentration
Decreased energy
Psychomotor changes

!
!
62

E. Suicidal ideation.
Child and adolescent psychiatry

23. Which!of!the!following!childhood!psychiatric!disorders!has!the!EARLIEST!mean!age!
of!onset?!
!
A.
B.
C.
D.
E.

Attention!deficit!hyperactivity!disorder!
Conduct!disorder!
Oppositional!defiant!disorder!
PostNtraumatic!stress!disorder!
Secondary!encopresis.!!

24. Which!of!the!following!is!MOST!CORRECT!about!Aspergers!disorder?!!!
!
A.
B.
C.
D.
E.

They!are!good!at!sport.!
They!have!a!normal!IQ.!
They!have!impaired!nonNverbal!communication.!
They!have!delay!in!speech.!
They!have!high!intelligence!in!specific!areas.!

25. A 15-year-old girl presents with fatigue and dehydration. On physical


examination she is noted to have swollen salivary glands and calluses on her
knuckles. She is also hypokalemic. These findings are MOST consistent with
which of the following diagnoses?
A. Anorexia nervosa
B. Bulimia nervosa
C. Chronic fatigue syndrome
D. Obsessive-compulsive disorder
E. Rumination disorder
26. Which!of!the!following!antidepressants!has!the!GREATEST!body!of!evidence!for!
efficacy!in!treating!major!depressive!disorder!in!children!and!adolescents?
A. Bupoprion
B. Fluvoxamine
C. Fluoxetine
D. Paroxetine
E. Sertraline.
Substance abuse

27. A 55-year-old man who has been drinking Chinese wine every night says, I dont
think it is a problem. Which Prochaskas and Diclementes stage of change
BEST describes his current status?
A. Action
B. Contemplation
C. Precontemplation
!
!
63

D. Maintenance
E. Relapse.
28. A!23TyearTold!medical!student!is!brought!to!the!Accident!and!Emergency!
Department!by!his!parents.!He!admits!to!having!recently!used!a!recreational!drug!
during!his!overseas!elective!in!the!United!States.!He!returned!two!days!ago.!He!is!
suspicious!and!sensitive!to!what!others!are!saying.!Physical!examination!reveals!
dilated!pupils!and!electrocardiogram!(ECG)!shows!cardiac!arrhythmias.!Which!drug!
is!MOST!likely!to!be!responsible?!!
!
a.
b.
c.
d.
e.

Alcohol!
Cannabis!
Cocaine!!
Mescaline!
Heroin.!
!

29. A 24-year-old man presents to the Accident and Emergency Department with
fever, hypotension, agitation, tachycardia and odour of alcohol on his breath. His
friend with him stated that he was prescribed a medication by a psychiatrist 4
weeks ago. The most likely medication responsible is:
A.
B.
C.
D.
E.

Diazepam
Disulfiram
Fluoxetine
Haloperidol
Moclobemide.

30. A 40-year-old heroin addict reports continued carving. He is on methadone100mg daily.


His methadone dose is increased to 110mg daily. After one week, he has a syncope and
is taken to the Accident and Emergency Department. The most likely finding to explain
his syncope is:
A. Electroencephalogram (EEG) shows evidence of seizure activity.
B. Liver function tests reveal marked transaminitis.
C. Prolonged!QTc!interval
D. Ultrasound Doppler shows deep vein thrombosis.
E. Urine toxicology screen is positive for opioid.

31. A!50TyearTold!with!history!of!major!depressive!disorder!and!ischaemic!heart!
disease.!He!has!not!received!any!psychiatric!treatment.!Which!of!the!following!
medications!is!the!BEST!option!to!help!him!to!quit!smoking?!
!
A. Bupropion!
B. Naltrexone!
!
!
64

C. Nicotine!replacement!treatment!
D. Sertraline!
E. Varenicline.!
Psychopharmacology
32. The WEAKEST evidence for efficacy for Post-traumatic Stress Disorder (PTSD) is for
which class of pharmacological agents?
a. Amitriptyline
b. Diazepam
c. Fluoxetine
d. Fluvoxamine
e. Paroxetine.

33.
A.
B.
C.
D.
E.

Regarding!escitalopram,!all!of!the!following!are!true!EXCEPT: !
Absorption!is!not!affected!by!food.!
It!has!better!tolerability!than!other!SSRIs.!
It!shows!linear!pharmacokinetics!at!clinically!relevant!doses.!!
It!causes!a!lot!of!drug!interaction.!!
Peak!plasma!levels!occur!2N4!hours!after!a!single!dose.!
!

34. The MOST common side effect of selective serotonin reuptake inhibitor (SSRI) is:
A.
B.
C.
D.
E.

Acne
Erectile dysfunction
Gastrointestinal disturbances
Metabolic syndrome
Tinnitus.

35. Which of the following antipsychotic drugs is MOST LIKELY to be associated with
adverse haematological side effects?
A.
B.
C.
D.
E.

Clozapine
Olanzapine
Quetiapine
Risperidone
Ziprasidone.

36. Which!of!the!following!is!the!LEAST!common!side!effects!of!acetylcholinesterase!
inhibitors?!!!
!
A. Dizziness!
B. Nausea!
C. Nightmares!
!
!
65

D. Seizures!
E. Tachycardia.!
!
37.
A.
B.
C.
D.
E.

Priapism is MOST likely to be associated with which of the following medications?


Bupoprion
Duloxetine
Moclobemide
Phenelzine
Trazodone.

38. A patient presents with involuntary frowning, blinking grimacing, and


choreoathetoid movements of the upper extremities after several years of
antipsychotic medication treatment. Which one of the following is the MOST
likely diagnosis?
A.
B.
C.
D.
E.

Akinesia
Akathisia
Acute dystonia
Parkinsonism
Tardive dyskinesia

Psychotherapy

39. At!the!one!year!followTup!of!panic!disorder,!which!of!the!following!treatments!
results!with!the!BEST!outcome!and!the!LEAST!functional!impairment?!!
!
A.
B.
C.
D.
E.

Benzodiazepines!
BetaNblockers!
Brief!dynamic!psychotherapy!
Cognitive!behaviour!therapy!$
Supportive!psychotherapy.!

40. A 20-year-old with patient suffers from agoraphobia and she has phobia of using
MRT train. Which is the BEST psychological treatment for this patient?
A. No exposure to MRT train and encourage alternative transportation mode
B. Short exposure to MRT train with coping avoidance
C. Short exposure to MRT train without coping avoidance
D. Graded exposure to MRT train with coping avoidance
E. Graded exposure to MRT train without coping avoidance.

!
!
66

41. Defining!highTrisk!situations,!covert!antecedents,!and!stimulus!control!techniques!
A.
B.
C.
D.
E.

are!a!focus!of!which!of!the!following!therapeutic!modalities?
Brief!psychodynamic!psychotherapy
Cognitive!behaviour!therapy
Supportive!therapy
Interpersonal!therapy
Relapse!prevention!therapy.

42. A!security!guard!attacks!a!government!building,!killing!3!individuals!and!

significantly!injuring!another!10!before!he!takes!his!own!life.!!Psychologists!from!a!
general!hospital!are!dispatched!to!the!scene!to!provide!immediate!psychological!
debriefing!to!civil!servants!working!in!the!building!to!prevent!postTtraumatic!stress!
disorder!(PTSD)!symptoms.!!Using!an!evidenceTbased!framework,!what!is!the!most!
likely!impact!of!this!intervention!on!direct!survivors?
!
A. It!will!be!ineffective!or!harmful!to!the!direct!survivors.
B. It!will!decrease!the!probability!of!developing!PTSD!symptoms.
C. It!will!decrease!the!severity!of!PTSD!symptoms.
D. It!will!decrease!the!time!period!for!recovery!from!PTSD!symptoms.
E. It!will!delay!the!onset!of!PTSD!symptoms.
Ethics and laws
43. When!managing!a!chronically!suicidal!patient,!respecting!the!patients!preference!to!

remain!at!home!and!not!be!admitted!to!the!psychiatric!unit!for!additional!care!
reflects!which!of!the!following!ethical!principles?
A.
B.
C.
D.
E.

Autonomy
Beneficence
NonNmaleficence
Egalitarianism
Fiduciary!duty.!

Liaison and neuropsychiatry

44. Features of neuroleptic malignant syndrome include all of the following


EXCEPT?
a. Diaphoresis
b. Elevated temperature
c. Extrapyramidal side-effects
d. Labile blood pressure
e. Flaccidity.
45. You are working as a resident in the weight management programme. A 40-year-old
schizophrenia patient is referred to you for weight management issue. Which of the
following medications has the BEST evidence as an adjunct for reducing weight gain

!
!
67

A.
B.
C.
D.
E.

associated with the second generation antipsychotics:


Fluoxetine
Metformin
Sibutramine
Simvastatin
Topiramate.

46. In medically ill patients who suffer from psychiatric illness, which of the following
pharmacological properties is MOST appropriate?
A.
B.
C.
D.
E.

Long half-life psychotropic drug.


Psychotropic drugs with active metabolites.
Psychotropic drug with wide therapeutic index.
Psychotropic drug which inhibits metabolic enzymes.
Psychotropic drug which induces metabolic enzymes.

47. A!47TyearTold!woman!is!brought!for!a!psychiatric!evaluation!to!assess!recent!

A.
B.
C.
D.
E.

changes!in!her!behaviour.!!Over!the!past!six!months,!she!has!become!increasingly!
disinhibited!and!impulsive!in!her!behaviour.!!Physical!examination!reveals!mild!
dysarthria,!dysphagia,!and!drooling.!!Slit!lamp!examination!of!the!eyes!indicates!the!
presence!of!Kayser!Fleischer!rings.!!Which!of!the!following!BEST!describes!the!
genetic!basis!for!her!illness?
Autosomal!recessive!
CoNdominant!
Mitochondrial!
Polygenetic!
XNlinked!dominant.!

!
!
MCQ!exam!(Paper!3)!
!
History and mental state exam!

48. A!40TyearTold!woman!is!brought!by!her!
husband!brought!to!see!you!as!a!result!of!
abnormal!behaviour.!During!the!
interview,!she!laughs!and!cries!with!a!
very!short!period!of!time.!In!your!record,!
her!affect!is!BEST!described!as:!
!
F.
G.
H.
I.

Apathy!
Blunted!affect!
Euthymic!affect!
Labile!affect!

The!answer!is!D.!
!
Explanation:!Labile!affect!refers!to!rapid!
fluctuation!of!affect!(e.g.!from!tearfulness!and!
laughter!in!a!10!!minute!interview,!seen!in!
patients!with!bipolar!disorder).!
!

!
!
68

J.

Flat!affect.!

!
Year:!2013.!
!

Cognitive assessment!

49. Which of the following is the presentation


of cortical dementia?
F. A 40-year-old man presents with memory loss
and writhing!movements.!His!father!

The!answer!is!B.!
!

Explanation:!Option!B!refers!to!frontal!lobe!
suffered!from!the!same!condition.!
dementia!which!is!cortical!dementia.!
G. A!50NyearNold!man!presents!with!change!in!
personality,!poor!judgment,!disinhibition!
!
and!perservation.!He!made!a!lot!of!mistakes!
in!Lurias!hand!test.!
Option!A,!C,!D!and!E!are!considered!to!be!
H. A 60-year-old woman presents with memory
subcortical!dementia.!
I.

loss and depression after a cerebrovascular


accident.!
A 70-year-old man presents with memory loss, !
rigidity, slowness!of!movement!and!difficulty!

Option!A!=!Hungtingtons disease

with!walking!and!gait.!
J.

A 75-year-old woman presents with visual


hallucination. The GP gave her haloperidol
and she develops severe extrapyramidal side
effects.!

Option C = Vascular dementia!

Option!D!=!Parkinsons!disease!
Option!E!=!Lewy!body!dementia!
!
!
Year:!2013.!
!
This!is!an!advancedNlevel!question.!
!

Psychiatric epidemiology
50. After the age of 65, the prevalence of
dementia increases by how many times
every 5 years?
F. 2 times
G. 4 times
H. 6 times
I. 8 times
J. 10 times.
Psychiatric aetiology, diagnosis and classification

!
!
69

The!answer!is!A.!
!
!
Year:!2013!
!

51. An 18-year-old woman presents with


questions about her risk of developing
schizophrenia. She is worried because her
23-year-old brother has recently been
diagnosed with schizophrenia and required
hospitalization for psychosis. There are no
other family members who have a history
of schizophrenia. Having a sibling with
schizophrenia increases her risk of
developing schizophrenia by?
F.
G.
H.
I.
J.

The!answer!is!E!
!
Explanation:!The!risk!of!general!population!is!
1%.!The!risk!of!a!sibling!is!10%.!Hence,!the!
risk!is!increased!by!10!times.!
!
!

2 times
4 times
6 times
8 times
10 times.

Year:!2013.!

52. Which of the following neuroanatomical


structures is the critical region for fear
conditioning?
A. Amgydala

The!answer!is!A.!
!
Explanation:!Option!B!for!Alzheimers!
disease,!Option!C!for!schizophrenia,!Option!D!
for!addiction!and!Option!E!for!depression.!

B. Basal nucleus of Meynert


C. Mesocortical region

D. Nucleus accumbens

E. Raphe nucleus.

Year:!2013.!
53. The most likely neuroanatomical
substrate of Tourette's syndrome is
A. The thalamus.

The answer is D.

B. The pituitary.

Explanation: Lesions in basal ganglia can


cause motor tic and compulsive behaviour.

C. The prefrontal cortex.


D. The basal ganglia.
E. The amygdala.

Year: 2013

This is an advanced-level question.

!
!
70

Psychopathology

54. Agoraphobia!without!panic!disorder!can!
be!described!as:!!!

Answer!is!A.!

F.
G.
H.
I.
J.

Explanation:!A!fear!of!enclosed!space!is!one!of!
the!most!common!complaints!among!patients!
in!Singapore!(e.g.!crowded!MRT!trains).!
Option!B!is!social!phobia.!

!
A!fear!of!enclosed!spaces!!
A!fear!of!speaking!in!public!
A!fear!of!height!
Avoidance!of!authority!figures!
Worrying!excessively!about!physical!health.!

!
!
Year:!2013.!
55. Uncomplicated!bereavement!is!NOT!
characterized!by:!!
F. Anger!
G. Depressive!mood!for!one!month!
H. Feelings!of!worthlessness!!
I. Sensations!of!seeing!or!hearing!the!
deceased!
J. Transient!guilt.!

The!answer!is!C.!
!
Indicators!that!bereavement!may!have!
progressed!into!depression!include:!
2. symptoms!still!present!two!months!
after!the!loss!
3. guilt!about!things!other!than!actions!
taken!or!not!taken!by!the!survivor!at!
the!time!of!death!
4. thoughts!of!death!other!than!the!
survivor!feeling!that!he!or!she!would!
be!better!off!dead!or!should!have!died!
with!the!deceased!person!
5. morbid!preoccupation!with!
worthlessness!
6. marked!psychomotor!retardation!
7. prolonged!and!marked!functional!
impairment!
8. hallucinatory!experiences!other!than!
thinking!that!he!or!she!hears!the!voice!
of,!or!transiently!sees!the!image!of,!
the!deceased!person!
!

56. The common clinical symptoms for


pre-menstrual dysphoric disorder
includes all of the following EXCEPT:
f. Bloating
g. Difficulty concentrating
h. Guilt
!
!
71

The answer is C.

Explanation: Guilt is not considered to be a


common clinical symptom of premenstrual

i.
j.

Sleep disturbance
Lethargy and fatigue

dysmorphic disorder.

Year: 2013
The!answer!is!B.!
57. A!40TyearTold!airTconditioner!
technician!is!referred!to!you!because!
!
of!change!in!behaviour!and!
deterioration!in!functioning.!When!
Explanation:!As!this!person!works!as!an!airN
he!feels the gust of cold air coming
from the air-conditioner, he
interprets the cold air as poison gas
from Mars. This phenomenon is BEST
described as:!
A.
B.
C.
D.
E.

Delusional memory
Delusional perception
Tactile hallucination
Primary delusion
Secondary delusion.

conditioner!technician,!he!is!used!to!feel!gust!
of!cold!air!from!air!conditioner.!This!is!a!
normal!perception.!He!adds!a!delusional!
interpretation!and!claims!that!the!cold!air!is!a!
poison!gas!from!another!planet.!This!is!
illogical!and!impossible.!As!a!result,!there!is!
delusional!interpretation!of!a!real!perception.!
This!phenomenon!is!known!as!delusional!
perception.!
!
!
Year:!2013!
!

58. Which of the following behaviour is


NOT considered to be purging?

The answer is B.

F.
G.
H.
I.
J.

Explanation: In medicine, purging is defined


as to cause evacuation of the
gastrointestinal or urinary system. Option B
is not related to the gastrointestinal or
urinary system.

Self-induced vomiting
Excessive exercise
Excessive use of laxative
Excessive use of diuretic
Excessive use of enema.

!
Year:!2013!

!
!
72

General adult psychiatry

59. In!schizophrenia,!which!of!the!following!
factors!is!LEAST!Likely!to!restrict!full!
rehabilitation!potential?!
!
F. Florid!delusions!and!hallucinations!!
G. Lack!of!pleasure!from!social!intervention!
and!physical!activity!
H. Loss!of!interest!in!rehabilitation!
I. Poor!motivation!
J. Restricted!affect.!
!

The!answer!is!A.!
!
Explanation:!Option!A!is!positive!symptom!of!
schizophrenia!and!is!least!likely!to!restrict!full!
rehabilitation!potential!as!compared!to!
negative!symptoms.!
!
!!
!
Year:!2013!
!

60. Somnambulism!(Sleepwalking!disorder)!
occurs!in!

The!answer!is!E.!

F.
G.
H.
I.
J.

Explanation:!Parasomnia!is!divided!into!slowN
wave!sleep!disorders!(e.g.!sleep!terrors,!
sleepwalking!disorder)!and!rapid!eye!
movement!sleep!disorders!(e.g.!nightmare!
disorder,!REM!behaviour!disorder!such!as!
kicking!in!the!middle!of!the!night).!!

!
Active!sleep!
Earliest!phase!of!sleep!
Paradoxical!sleep!
Rapid!eye!movement!(REM)!sleep!
SlowNwave!sleep.!

!
!

!
Somnabulism!or!sleepwalking!disorder!is!
associated!with!slow!wave!sleep.!Because!
sleepwalking!disorder!arises!from!SWS!!
sleep,!the!patient!is!difficult!to!be!!
awaken!during!sleepwalking.!
!
!
Year:!2013.!
61. Which!of!the!following!is!a!poor!
prognostic!factor!for!schizophrenia?!

!
!
73

The!answer!is!E.!

!
F.
G.
H.
I.
J.
!

Family!history!of!mood!disorder!
Female!gender!
Late!onset!
Precipitating!factor!
Presence!of!neurological!soft!sign.!

Explanation:!Option!E!suggests!that!the!aetiology!of!
schizophrenia!is!biological!and!resulted!from!a!neurological!
lesion.!Hence,!the!prognosis!poor.!Good!prognostic!and!poor!
prognostic!factors!are!summarized!as!follows:!
!

Good!Prognosis!
Late!onset!

Obvious!precipitating!factors!No!precipitating!factors!
Acute!onset!

!
!

Good!premorbid!function!!
Married! !

Poor!Prognosis!
!
Young!onset!

Insidious!onset!
Poor!premorbid!function!

!!!!!!!!!!!!!!!!!!!!Single,!divorced,!or!widowed!

Family!history!of!mood!disorders!Family!history!of!
schizophrenia!
!
Good!support!systems!

Poor!support!systems!

Positive!symptoms! !

Negative!symptoms!

!!

!
Year:!2013.!

!
62. All of the following medical disorders are
more common in schizophrenia patients as
compared to the general population
EXCEPT?
F.
G.
H.
I.
J.

The!answer!is!E.!
!
Explanation:!The!risk!for!schizophrenia!
patients!developing!RA!is!oneNthird!the!risk!of!
general!population.!The!reason!is!due!to!
reduction!in!immune!responses!in!
schizophrenia!patients.!

Chronic obstructive pulmonary disease


Diabetes mellitus
Human immunodeficiency virus infection
Metabolic syndrome
Rheumatoid arthritis.

!
!
Year:!2013.!
63. A!40TyearTold!executive!is!afraid!of!flying!by! The!answer!is!E.!
aeroplane.!She!has!this!problem!since!young.!!
As!a!result,!she!has!to!quit!her!job!because!it! !
requires!frequent!travelling.!What!is!the!
MOST!LIKELY!diagnosis?!

!
!
74

Explanation:!Fear!of!shadow!is!specific!phobia.!

A.!Agoraphobia!

B.!Generalized!anxiety!disorder!

C.!Panic!disorder!

Year:!2013.!

D.!Social!phobia!
E.!Specific!phobia.!
!

Old age psychiatry

64. Which!is!the!MOST!preferred!
antidepressant!in!elderly?!
F. Amitriptyline!
G. Imipramine!
H. Moclobemide!
I. Sertraline!
J. Venlafaxine.!
!

The!answer!is!D.!
!
Explanation:!Sertraline,!a!SSRI,!is!the!most!
preferred!antidepressants!as!compared!to!
imipramine!and!amitriptyline!which!are!TCA.!
Venlafaxine!is!associated!with!doseNrelated!
hypertension.!
!
!
Year:!2013!
!
This!is!an!advancedNlevel!question.!
!

65. Which!of!the!following!statements!is!
TRUE!regarding!risk!factors!for!
Alzheimers!disease? !
!
F. APO!E2!genotype!increases!the!risk!of!
Alzheimers!disease.!
G. APO!E4!genotype!increases!the!risk!of!
Alzheimers!disease.!
H. APO!E6!genotype!increases!the!risk!of!
Alzheimers!disease.!!
I. Male!gender!is!a!risk!factor!!
J. Smoking!is!protective!against!Alzheimers!
!
!
75

The!answer!is!B.!
!
Explanation:!Studies!have!suggested!that!
APOE4!genotype!increases!risk!of!Alzheimers!
disease,!and!especially!in!women.!APOE2!
genotype!is!protective.!Female!gender!is!a!
risk!factor!and!smoking!is!not!protective.!
!

disease.

!
Year:!2013!

66. Which!of!the!following!is!the!MOST!
common!type!of!hallucination!in!people!
suffering!from!dementia? !
F. Functional!
G. Gustatory!
H. Olfactory!
I. Tactile!
J. Visual.!!!
!

The!answer!is!E.!
!
Explanation:!Visual!hallucinations!are!the!
most!common!type!of!hallucination!in!
dementia.!It!is!because!one!type!of!dementia,!
lewy!body!dementia!is!associated!with!visual!
hallucinations.!
!
!
Year:!2013!

67. Late-onset psychotic disorder such as


schizophrenia or delusional disorder is
associated with:
F.
G.
H.
I.
J.

The!answer!is!B.!
!
Explanation:!LateNonset!psychotic!disorder!is!
more!likely!to!be!associated!with!sensory!
deficit.!

Less paranoia
More likely to have sensory deficits
More loosening of associations
More thought disorders
More violent behaviour.

!
!
Year:!2013.!

68. Which of the following statements about


sleep disorders in old people is FALSE?

The!answer!is!D.!
!

F. Excessing napping is a common cause of


insomnia.
G. Insomnia is common in old people in
Singapore.
H. Insomnia is associated with increased irks of
depression.
I. If there is a need for benzodiazepine,
diazepam is the preferred sedative for old
people.
J. The first line treatment is sleep hygiene.

Explanation:!Diazepam!is!longNacting!and!
cause!oversedation!or!hangNover!effect.!This!
will!increase!fall!risk!among!the!elderly.!

69. An 80-year-old man


terminal
insomnia,

The answer is E.

presents with
weight
loss,

!
!
76

!
!
Year:!2013!

decreased appetite and pervasive


anhedonia.
Physical
examination,
laboratory investigations and imaging
reveal normal findings. The MOST
specific symptom to his likely
diagnosis is:
F.
G.
H.
I.
J.

Anxiety
Decreased concentration
Decreased energy
Psychomotor changes
Suicidal ideation.

Year: 2013.

Child and adolescent psychiatry

70. Which!of!the!following!childhood!
psychiatric!disorders!has!the!EARLIEST!
mean!age!of!onset?!
!
F.
G.
H.
I.
J.

Explanation: This elderly man suffers from


depressive disorder based on the history
provided. Hence, suicidal ideation is the
most specific symptom.

Attention!deficit!hyperactivity!disorder!
Conduct!disorder!
Oppositional!defiant!disorder!
PostNtraumatic!stress!disorder!
Secondary!encopresis.!!

The!answer!is!E.!
!
Explanation:!Encopresis:!4!years!by!
definition.!ADHD:!12!years,!CD:!10!years,!
ODD:!8!years,!PTSD:!can!be!childhood!or!
adult.!
!
!
Year:!2013!
!

71. Which!of!the!following!is!MOST!
CORRECT!about!Aspergers!disorder?!!!

The!answer!is!C.!

F. They!are!good!at!sport.!
G. They!have!a!normal!IQ.!
H. They!have!impaired!nonNverbal!
communication.!
I. They!have!delay!in!speech.!
J. They!have!high!intelligence!in!specific!
areas.!

Explanation:!They!have!better!verbal!
communications!as!compared!to!autism!and!
no!delay!in!speech.!They!may!have!lower!IQ.!
Some!have!high!intelligence!in!specific!areas!
but!not!all.!

!
!
Year:!2013.!

!
!
77

72. A 15-year-old girl presents with fatigue


and dehydration. On physical
examination she is noted to have
swollen salivary glands and calluses
on her knuckles. She is also
hypokalemic. These findings are
MOST consistent with which of the
following diagnoses?
F. Anorexia nervosa
G. Bulimia nervosa
H. Chronic fatigue syndrome
I. Obsessive-compulsive disorder
J. Rumination disorder

The!answer!is!B.!
!
Explanation:!This!patient!suffers!from!bulimia!
nervosa.!She!has!selfNinduced!vomiting!and!
the!gastric!acid!causes!inflammation!in!the!
salivary!glands.!She!uses!her!fingers!to!induce!
vomiting!and!leave!calluses!on!her!knuckles,!
which!is!known!as!Russells!sign.!Repeated!
selfNinduced!vomiting!is!associated!with!
hypokaelemia.!
!
!
Year:!2013.!
!

73. Which!of!the!following!antidepressants!

F.
G.
H.
I.
J.

has!the!GREATEST!body!of!evidence!for!
efficacy!in!treating!major!depressive!
disorder!in!children!and!adolescents?
Bupoprion
Fluvoxamine
Fluoxetine
Paroxetine
Sertraline.

The!answer!C!
!
Explanation:!Fluoxetine!and!escitalopram!
have!the!greatest!body!of!evidence!for!
efficacy!in!treating!major!depressive!disorder!
in!children!and!adolescents.!
!
!
Year:!2013!
!

Substance abuse

74. A 55-year-old man who has been


drinking Chinese wine every night
says, I dont think it is a problem.
Which Prochaskas and Diclementes
stage of change BEST describes his
current status?
F. Action
G. Contemplation
H. Precontemplation
I. Maintenance
J. Relapse.

!
!
78

The answer is C.

Explanation: He is unaware that his alcohol


dependence is a problem.

Year: 2013.

75. A!23TyearTold!medical!student!is!brought!
to!the!Accident!and!Emergency!
Department!by!his!parents.!He!admits!to!
having!recently!used!a!recreational!drug!
during!his!overseas!elective!in!the!
United!States.!He!returned!two!days!ago.!
He!is!suspicious!and!sensitive!to!what!
others!are!saying.!Physical!examination!
reveals!dilated!pupils!and!
electrocardiogram!(ECG)!shows!cardiac!
arrhythmias.!Which!drug!is!MOST!likely!
to!be!responsible?!!
!
f.
g.
h.
i.
j.

!
Explanation:!Dilated!pupil,!arrhythmia,!
nausea!and!suspiciousness!are!common!signs!
associated!with!cocaine!intoxication.!
!
!
Year:!2013.!
!

Alcohol!
Cannabis!
Cocaine!!
Mescaline!
Heroin.!
!

This!is!an!advancedNlevel!question.!

76. A 24-year-old man presents to the


Accident and Emergency Department
with fever, hypotension, agitation,
tachycardia and odour of alcohol on
his breath. His friend with him stated
that he was prescribed a medication by
a psychiatrist 4 weeks ago. The most
likely medication responsible is:
F.
G.
H.
I.
J.

The!answer!is!C.!

Diazepam
Disulfiram
Fluoxetine
Haloperidol
Moclobemide.

The!answer!is!B.!
!
Explanation:!Disulfiram!blocks!this!reaction!
at!the!acetaldehyde!dehydrogenase.!After!
alcohol!intake!under!the!influence!of!
disulfiram,!the!concentration!of!acetaldehyde!
in!the!blood!may!be!five!to!10!times!higher!
than!that!found!during!metabolism!of!the!
same!amount!of!alcohol!alone.!As!
acetaldehyde!is!one!of!the!major!causes!of!the!
symptoms!experienced!by!this!man.!
!
!
Year:!2013.!

77. A 40-year-old heroin addict reports


continued carving. He is on
methadone100mg daily. His methadone
dose is increased to 110mg daily. After one
week, he has a syncope and is taken to the
Accident and Emergency Department. The
most likely finding to explain his syncope
is:

!
!
79

The!answer!is!C.!
!
Explanation:!Methodone!is!associated!with!
prolonged!QTc.!This!occurs!after!an!increase!
in!the!dose!of!methadone.!This!may!lead!to!

F. Electroencephalogram (EEG) shows evidence


of seizure activity.
G. Liver function tests reveal marked

transaminitis.
H. Prolonged!QTc!interval
I.
J.

Ultrasound Doppler shows deep vein


thrombosis.
Urine toxicology screen is positive for opioid.

cardiac!arrhythmia!and!results!in!syncope.!
!
!
Year:!2013!
!
This!is!an!advanced!level!question.!

78. A!50TyearTold!with!history!of!major!
depressive!disorder!and!ischaemic!heart!
disease.!He!has!not!received!any!
psychiatric!treatment.!Which!of!the!
following!medications!is!the!BEST!option!
to!help!him!to!quit!smoking?!
!
F.
G.
H.
I.
J.

Bupropion!
Naltrexone!
Nicotine!replacement!treatment!
Sertraline!
Varenicline.!

The!answer!is!A.!
!
Explanation:!Bupoprion!is!the!best!option!in!
this!case!as!it!can!overcome!depression!and!
smoking.!Option!C!and!E!can!tackle!smoking!
but!not!depression.!
!
!
Year:!2013.!
!

Psychopharmacology
79. The WEAKEST evidence for efficacy for
Post-traumatic Stress Disorder (PTSD) is
for which class of pharmacological
agents?
f. Amitriptyline
g. Diazepam
h. Fluoxetine
i. Fluvoxamine
j. Paroxetine.

The!answer!is!B.!
!
Explanation:!Diazepam,!a!long!term!
benzodiazepine!may!cause!dependence!and!
does!not!help!to!relieve!anxiety!symptoms!in!
long!run!as!it!may!cause!withdrawal.!
!
!
Year:!2013.!
!

80. Regarding!escitalopram,!all!of!the!
following!are!true!EXCEPT: !
F. Absorption!is!not!affected!by!food.!
G. It!has!better!tolerability!than!other!SSRIs.!

!
!
80

The!answer!is!D.!
!

H. It!shows!linear!pharmacokinetics!at!
clinically!relevant!doses.!!
I. It!causes!a!lot!of!drug!interaction.!!
J. Peak!plasma!levels!occur!2N4!hours!after!a!
single!dose.!
!

Explanation:!Escitalopram!causes!the!least!
drug!interaction!among!all!SSRIs.!Sertralines!!
absorption!is!significantly!affected!by!food.!
Escitalopram!is!better!tolerated!than!other!
SSRIs.!Fluoxetine!and!paroxetine!are!the!only!
SSRIs!that!do!not!show!linear!
pharmacokinetics!at!clinically!relevant!doses.!
!
!
Year:!2013!

81. The MOST common side effect of selective


serotonin reuptake inhibitor (SSRI) is:

The!answer!is!C.!
!

F.
G.
H.
I.
J.

Acne
Erectile dysfunction
Gastrointestinal disturbances
Metabolic syndrome
Tinnitus.

Explanation:!GI!disturbances!such!as!nausea!
and!vomiting!are!the!most!common!side!
effects.!
!
!
Year:!2013.!

82. Which of the following antipsychotic drugs


is MOST LIKELY to be associated with
adverse haematological side effects?
F.
G.
H.
I.
J.

Clozapine
Olanzapine
Quetiapine
Risperidone
Ziprasidone.

The!answer!is!A.!
!
Explanation:!Clozapine!is!the!second!
generation!antipsychotics!which!is!most!
likely!to!be!associated!with!agranulocytosis.!
!
!
Year:!2013!

83. Which!of!the!following!is!the!LEAST!
common!side!effects!of!
acetylcholinesterase!inhibitors?!!!

The!answer!is!E.!
!

Explanation:!!The!AChEI!causes!bradycardia!
as!a!result!of!increased!vagal!tone.!

F. Dizziness!
G. Nausea!
H. Nightmares!

!
!
81

I.
J.

Seizures!
Tachycardia.!

!
Year:!2013.!

!
This!is!an!advancedNlevel!question!
84. Priapism is MOST likely to be associated with
which of the following medications?
F. Bupoprion
G. Duloxetine
H. Moclobemide
I. Phenelzine
J. Trazodone.

The!answer!is!E.!
!
Explanation:!Trazodone!is!a!serotonin!
antagonist!and!reuptake!inhibitor.!It!is!good!
for!depression!and!insomnia.!It!is!available!in!
SGH!pharmacy!but!not!NUH!pharmacy.!It!is!
associated!with!priapism!in!men.!!
!
!
Year:!2013!
!
This!is!an!advancedNlevel!question.!
!

85. A patient presents with involuntary


frowning, blinking grimacing, and
choreoathetoid movements of the
upper extremities after several years of
antipsychotic medication treatment.
Which one of the following is the MOST
likely diagnosis?
F.
G.
H.
I.
J.

Akinesia
Akathisia
Acute dystonia
Parkinsonism
Tardive dyskinesia

The!answer!is!E.!
!
Explanation:!This!patient!develops!
involuntary!movements!after!taking!
medication!for!several!years.!The!best!
description!is!tardive!dyskinesia.!
!
!
Year:!2013.!
!

Psychotherapy

86. At!the!one!year!followTup!of!panic!
disorder,!which!of!the!following!
treatments!results!with!the!BEST!
outcome!and!the!LEAST!functional!

The!answer!is!D.!
!

!
!
82

impairment?!!

Explanation:!CBT!has!better!outcome!than!
brief!dynamic!psychotherapy!and!supportive!
psychotherapy.!Benzodiazepines!and!betaN
blockers!only!provide!symptomatic!relief.!

!
F.
G.
H.
I.
J.

Benzodiazepines!
BetaNblockers!
Brief!dynamic!psychotherapy!
Cognitive!behaviour!therapy!$
Supportive!psychotherapy.!

!
!
Year:!2013.!

87. A 20-year-old with patient suffers from


agoraphobia and she has phobia of
using MRT train. Which is the BEST
psychological treatment for this
patient?
A. No exposure to MRT train and encourage
alternative transportation mode

The answer is E.

Explanation: The main principle of treatment


is to not have any form of avoidance but to
extinguish anxiety in response to the feared
stimulus, i.e. the MRT train.

B. Short exposure to MRT train with coping


avoidance
C. Short exposure to MRT train without coping
avoidance

Year: 2013.

D. Graded exposure to MRT train with coping


avoidance
E. Graded exposure to MRT train without coping
avoidance.

88. Defining!highTrisk!situations,!covert!

F.
G.
H.
I.
J.

The!answer!is!B!

antecedents,!and!stimulus!control!
techniques!are!a!focus!of!which!of!the!
following!therapeutic!modalities?
Brief!psychodynamic!psychotherapy
Cognitive!behaviour!therapy
Supportive!therapy
Interpersonal!therapy
Relapse!prevention!therapy.

!
Explanation:!Defining!high!risk!situations,!
covert!antecedents!and!stimulus!control!
techniques!are!part!of!the!dysfunctional!
thought!diary!used!in!CBT.!
!
!
Year:!2013.!

!
!
83

!
89. A!security!guard!attacks!a!government!

building,!killing!3!individuals!and!
significantly!injuring!another!10!before!
he!takes!his!own!life.!!Psychologists!from!
a!general!hospital!are!dispatched!to!the!
scene!to!provide!immediate!
psychological!debriefing!to!civil!servants!
working!in!the!building!to!prevent!postT
traumatic!stress!disorder!(PTSD)!
symptoms.!!Using!an!evidenceTbased!
framework,!what!is!the!most!likely!
impact!of!this!intervention!on!direct!
survivors?
!
F. It!will!be!ineffective!or!harmful!to!the!
direct!survivors.
G. It!will!decrease!the!probability!of!
developing!PTSD!symptoms.
H. It!will!decrease!the!severity!of!PTSD!
symptoms.
I. It!will!decrease!the!time!period!for!
recovery!from!PTSD!symptoms.
J. It!will!delay!the!onset!of!PTSD!
symptoms.

The!answer!A.!
!
Explanation:!Debriefing!is!shown!to!be!
harmful!to!individuals!who!are!the!witness!of!
a!traumatic!event.!The!individuals!should!be!
given!access!to!psychological!help!and!they!
can!consult!a!psychologist!on!a!voluntary!
basis.!!
!
!
Year:!2013!
!
This!is!an!advancedNlevel!question.!!

Ethics and laws


90. When!managing!a!chronically!suicidal!

The!answer!is!A.!
patient,!respecting!the!patients!
preference!to!remain!at!home!and!not!be! !
admitted!to!the!psychiatric!unit!for!
Explanation:!As!this!!question!emphasizes!on!
additional!care!reflects!which!of!the!
following!ethical!principles?
respecting!the!patients!preference,!the!best!
answer!is!respecting!patients!autonomy.!

F.
G.
H.
I.
J.

Autonomy
Beneficence
NonNmaleficence
Egalitarianism
Fiduciary!duty.!

!
!
Year:!2013.!
!

Liaison and neuropsychiatry

91. Features of neuroleptic malignant


syndrome include all of the following
EXCEPT?
f. Diaphoresis
g. Elevated temperature
!
!
84

The answer is E.

Explanation: It should be rigidity and not

h. Extrapyramidal side-effects
i. Labile blood pressure
j. Flaccidity.

flaccidity.

Year: 2013.
92. You are working as a resident in the weight
management programme. A 40-year-old
schizophrenia patient is referred to you for
weight management issue. Which of the
following medications has the BEST
evidence as an adjunct for reducing weight
gain associated with the second
generation antipsychotics:
F. Fluoxetine
G. Metformin
H. Sibutramine
I. Simvastatin
J. Topiramate.

The answer is B.

Explanation: Meformin has the best evidence in


reducing weight gain associated with the second
generation antipsychotics.

Year 2013

This is an advanced-level question.


93. In medically ill patients who suffer from
psychiatric illness, which of the following
pharmacological properties is MOST
appropriate?
F.
G.
H.
I.
J.

Long half-life psychotropic drug.


Psychotropic drugs with active metabolites.
Psychotropic drug with wide therapeutic index.
Psychotropic drug which inhibits metabolic
enzymes.
Psychotropic drug which induces metabolic
enzymes.

94. A!47TyearTold!woman!is!brought!for!a!

psychiatric!evaluation!to!assess!recent!
changes!in!her!behaviour.!!Over!the!past!
six!months,!she!has!become!increasingly!
disinhibited!and!impulsive!in!her!
behaviour.!!Physical!examination!
reveals!mild!dysarthria,!dysphagia,!and!
drooling.!!Slit!lamp!examination!of!the!
eyes!indicates!the!presence!of!Kayser!
Fleischer!rings.!!Which!of!the!following!
BEST!describes!the!genetic!basis!for!her!
illness?
!
!
85

The answer is C.

Explanation: Narrow therapeutic index like


lithium is dangerous to medically ill patients.

Year 2013

The!answer!is!A.!
!
Explanation:!This!patient!suffers!from!
Wilsons!disease!which!is!autosomal!
recessive.!
!
!

F.
G.
H.
I.
J.

Autosomal!recessive!
CoNdominant!
Mitochondrial!
Polygenetic!
XNlinked!dominant.!

Year:!2013.!
!
This!is!an!advancedNlevel!question.!

!
!
!
!
!
!
!
!
!
!
!
!
!
!
!
!
!
!
!
!
MCQ exam (Paper 4): Questions
History and mental state exam
1. You are a medical resident. The nurse informs you that an 80-year-old man who
was admitted for cellulitis seems to be depressed and mentions about passive
!
!
86

suicidal thought. When you assess him, he sees ghost in the ward and expresses
paranoid ideation against nursing staff. Which of the following assessment is
MOST important to establish the diagnosis?
A.
B.
C.
D.
E.

Assess orientation
Assess judgement
Assess recognition
Assess registration and short-term recall
Assess somatic symptoms.

Cognitive assessment
2. Which of the following cognitive tasks is NOT an assessment of short term or
long term memory?
A. Ask the patient to name the current Prime Minister of Singapore.
B. Ask the patient to name as many animals as possible that can be found in the
Singapore Zoo.
C. Ask the patient to tell you his or her address and later, you check the answer with
patients medical record.
D. Inform the patient 3 objects (e.g. Apple, Newspaper and Train) and ask the patient to
name the 3 objects immediately.
E. Inform the patient 3 objects (e.g. Apple, Newspaper and Train) and ask the patient to
name the 3 objects after 5 minutes.
Psychiatric, epidemiology, etiology, diagnosis and classification
3. A 20-year-old man with an identical twin is diagnosed with major depressive
disorder. His twin brother asks you to comment on the chance he will develop
depressive disorder. Which of the following is the CORRECT response to his twin
brother?
A. Major depressive disorder is a strongly inheritable disease. You will definitely
develop it as well.
B. The likelihood of you developing depressive disorder is higher than other siblings.
C. The likelihood of you developing depressive disorder is same as non-identical twins.
D. There is only a slightly increased risk that you will become depressed as compared to
the general population.
E. You do not need to worry because environmental factor is an important factor to
determine whether you will develop depressive disorder.

4. A 35-year-old woman suffers from low mood, low energy, poor sleep, poor
appetite and recurrent suicidal thoughts. She firmly believes that she deserves
the death sentence for minor mistakes made in the past. Which of the following
statements is FALSE?

!
!
87

A. For severe depressive episode with psychotic features, psychotic symptoms usually
occur after manifestation of depressive symptoms.
B. Her psychotic symptoms are considered to be mood congruent.
C. Based on the case scenario, there is enough evidence to suggest that she suffers
from schizoaffective disorder.
D. The prescription of antipsychotic drug should be considered.
E. Electroconvulsive therapy (ECT) is a recognised treatment for his condition.
5. Which of the following disorders has been shown to have the GREATEST degree
of heritability?
A. Alcoholism
B. Attention-deficit/hyperactivity disorder (ADHD)
C. Autism
D. Major depressive disorder
E. Schizophrenia.
6. Which of the following is the MOST common cause of insomnia among
psychiatric patients in Singapore?
A.
B.
C.
D.
E.

Sleep apnea
Shift work
Stimulant use
Underlying psychiatric illness
Use of sleeping pills.

7. Regarding the age of onset in schizophrenia, which of the following is CORRECT?


A. European women have earlier age of onset than Asian women.
B. Current research findings remain inconclusive about the age of onset in
schizophrenia.
C. There is no difference in the age of onset between men and women.
D. Men usually have earlier age of onset as compared to women.
E. Women usually have earlier age of onset as compared to men.
Psychopathology
8. Agoraphobia without panic attack is BEST referred to:
A.
B.
C.
D.
E.

Concern about physical appearance


Constant worried about physical health
Fear of blood and needle
Fear of collapse in an enclosed space
Fear of authority figures.

!
!
88

9. Which of the following is MOST suggestive of the diagnosis of schizophrenia?


A. Acute onset of psychosis
B. Apathy
C. Cognitive impairment
D. Hearing own thoughts
E. Persistent deterioration of personality.

General adult psychiatry


10. Peter is 27-year-old and lives with his parents. He has been employed as a
delivery man for most of the time since leaving school, but has recently left his
job as a salesman. He has never taken any illicit drugs. His parents state that in
the last three weeks, he has been extremely active, requiring less sleep and not
appearing tired, being over-talkative and disinhibited and on occasions quite
irritable. He claimed to have invented a machine for curing cancer and wished to
go to the U.S. to sell it. When stopped by his parents, he became violent, and they
called the police. Which of the following diagnosis is MOST appropriate for this
patient?
A.
B.
C.
D.
E.

Bipolar I disorder with manic features


Bipolar II disorder with hypomanic features
Cyclothymia
Mixed affective disorder
Schizoaffective disorder.

11. Which of the following personality disorders is LEAST likely to be considered as a


differential diagnosis to schizophrenia?
A.
B.
C.
D.
E.

Borderline personality disorder


Obsessive compulsive personality disorder
Paranoid personality disorder
Schizotypal personality disorder
Schizoid personality disorder.

12. Which of the following is LEAST likely to be found in research in the relationship
between depressive disorder and hormones?
A. Blunted adrenocorticotropic hormone (ACTH) response to the corticotropin-releasing
hormone (CRH)
B. Blunted thyroid stimulating hormone (TSH) response to thyrotropin-releasing hormone
(TRH)
C. Cortisol suppression with dexamethasone
!
!
89

D. There is an increased cortisol.


E. There is an increased adrenal sensitivity to ACTH.
13. Which of the following disorders is LEAST likely to be considered as a differential
diagnosis of a 35-year-old driver develops nightmare, flashback and
hypervigilance after a road traffic accident?
A. Factitious disorder
B. Generalized anxiety disorder
C. Major depressive disorder
D. Obsessive compulsive disorder
E. Phobic disorder.
14. Rapid eye movement (REM) sleep is associated with all of the following EXCEPT:
A.
B.
C.
D.
E.

Increased parasympathetic activity


Increased cerebral blood flow
Increased complexity of dreams
Maximal loss of muscle tone
Transient runs of conjugate eye movements.

15. A 17-year-old male comes to see you because he is experiencing tremendous


stress in his new job. He has finished his O level examination and waiting to
start the National Service in 6 months. He got an offer to work in a shoe shop
which specializes in selling female shoes. He realizes that he feels sexually
aroused by female shoes. He spends a lot of time fantasizing about female shoes.
He stole multiple pairs of shoes from the shop and used them for masturbation.
He used to be a normal person. He was a hardworking student whose main
interests were chemistry and physics. What is the MOST likely diagnosis?
A.
B.
C.
D.
E.

Adjustment disorder
Fetishism
Kleptomania
Sadomasochism
Voyeurism.

16. Which of the following psychiatric disorders is MOST common among patients
with bulimia nervosa?
A.
B.
C.
D.
E.

Alcohol use disorder


Major depressive disorder
Obsessive compulsive disorder
Panic disorder
Intermittent explosive disorder.

Old age psychiatry

!
!
90

17. Manic episodes in old people are associated with:


A.
B.
C.
D.
E.

Less euphoria
Less mixed presentation with depression
Less paranoid delusions
More hyperactivity
More flight of ideas.

18. All of the following statements are true regarding post-operative delirium
EXCEPT:
A.
B.
C.
D.
E.

Delirium cannot be diagnosed if all laboratory results are normal.


It can occur in an 80-year-old man after gastrectomy.
It can occur in a 16-year-old girl after renal transplant.
Haloperidol is one of the treatments of choice.
Physical restraint should be avoided if possible.

19. A 70-year-old man with a history of drinking Chinese wine, hepatitis, chronic renal
failure and hypertension was brought by his wife to the hospital for treatment of
an acute cellulitis. He was noted to have tachycardia in the Accident and
Emergency Department. You are the on-call medical resident and this man was
admitted to the medical ward. What is the MOST appropriate first approach?
A. Assess short term recall to rule out dementia on delirium.
B. Obtain history from collaterals whether he has in fact been drinking Chinese wine
recently.
C. Put him on propranolol 10mg three times per day to control tachycardia.
D. Start diazepam 5mg three times per day to prevent delirium tremens.
E. Start lorazepam 1mg three times per day to prevent delirium tremens.

20. A 70-year-old woman with severe depressive symptoms and strong suicidal
ideation which is not responded to amitriptyline 150mg daily and fluvoxamine
200mg daily. Each medication was tried for 6 months with good adherence. She
has good past health. Her children consult you for further management. The
MOST appropriate treatment which you would recommend is:
A. Cognitive behaviour therapy

!
!
91

B. Electroconvulsive therapy
C. Donepezil
D. Olanzapine
E. Repetitive transcranial magnetic stimulation.

Child and adolescent psychiatry and intellectual disability


21. You are helping a mother to develop behavioural program to deal with her son
who has oppositional defiant disorder. Which of the following is the MOST
significant component of this program?
A.
B.
C.
D.
E.

Extinction
Positive reinforcement
Punishment
School suspension
Strict parenting.

22. Which of the following factors are LEAST likely to be the aetiological factors in
attention deficit and hyperactivity disorder (ADHD)?
A. Antenatal exposure to alcohol
B. Antenatal exposure to nicotine
C. Delivery complications
D. High birth weight
E. Traumatic brain injury during infancy and early childhood.
23. You are a general practitioner. A 40-year-old woman consults you as a result of
high!blood!pressure.!General!inspection!shows!ptosis,!a!broad!neck!and!indistinct!
hairline.!Her!chest!appears!to!be!broad.!Auscultation!reveals!cardiac!murmurs.!She!is!
single!and!stays!with!her!parents.!She!completed!her!education!in!the!Institute!of!
Technological!Education!(ITE)!and!works!as!a!store!room!officer.!If!chromosome!
analysis!is!performed,!the!MOST!likely!finding!is:
!
A. XO syndrome
B. XXY syndrome
C. XXXY syndrome
D. XYY syndrome
E. YO syndrome.
24. You are a paediatric resident. A 7-year-old boy is suspected to suffer from autism.
You referred this boy to see an education psychologist for assessment. The
!
!
92

education psychologist sends a report to you and you need to explain the
findings to her mother. His strength is MOST likely found in which of the following
areas?
A.
B.
C.
D.
E.

Abstract thinking
Block design
Explain similarities
Oral presentation skills
Verbal concept formation

Substance abuse
25. Which of the following neuroanatomical structures is MOST implicated in
substance misuse and dependence?
A.
B.
C.
D.
E.

Amygdala
Basal nucleus of Meynert
Nucleus accumbens
Hippocampus
Raphe nucleus.

26. A 50-year-old man is admitted to the hepatobillary ward as a result of pancreatitis.


You are the resident on call. His wife tells you that she has seen him intoxicated
with alcohol in the past few days. The patient denies. Which of the following is the
BEST method available in a general hospital in Singapore to confirm that this
patient has been drinking recently?
A.
B.
C.
D.
E.

Carbohydrate-deficient transferrin (CDT)


Gamma-glutamyl transferase (GGT)
Mean corpuscular volume (MCV)
Serum alcohol level
Serum amylase.

27. A common and safe pharmacological treatment for reducing relapse in alcohol
dependence has direct actions upon which receptors?
A.
B.
C.
D.
E.

Benzodiazapine
Cannabinoid
Dopamine
-Aminobutyric acid (GABA)
Opioid.

!
!
93

28. Which of the following is LEAST consistent with the objective of motivational
interviewing?
A.
B.
C.
D.
E.

Allow the patient give their inputs without interruption.


Establishing a collaborative patient-therapist relationship.
Identifying appropriate reinforcements.
Providing empirical advice.
Usage of open-ended questions.

Psychopharmacology
29. Which of the following antidepressants is LEAST likely to have sexual sideeffects?
A.
B.
C.
D.
E.

Amitriptyline
Moclobemide
Mirtazapine
Fluoxetine
Venlafaxine.

30. The benzodiazepine with the LONGEST half-life is:


A. Clonazepam
B. Diazepam
C. Flurazepam
D. Lorazepam
E. Triazolam.
31. Which of the following is NOT an acetylcholinesterase inhibitor?
A. Buprenorphine
B. Donepezil
C. Galantamine
D. Rivastigmine
E. Tacrine.
32. Which of the following regarding side effects of risperidone is FALSE?
A. Akathisia is common with high dose of risperidone.
B. Prolactin elevation, probably greater than that seen with other second generation
antipsychotics.
C. Insomnia, headache and nausea are common.
D. Rhinitis is a possible side effect.
E. The risk for weight gain and the metabolic syndrome is higher than olanzapine.

!
!
94

33. Tricyclic antidepressants should be avoided with all of the following EXCEPT
A. Patients hospitalized for severe melancholic depression
B. Recent myocardial infarction
C. Right bundle branch block
D. Untreated glaucoma
E. Urinary retention.
34. Which of the following is CORRECT regarding galactorrhoea induced by
antipsychotic drugs?
A.
B.
C.
D.
E.

Antipsychotic drugs modulate the hypothalamic function and lead to galactorrhoea.


Antipsychotic drugs cause pituitary adenoma and galactorrhoea.
Bromocriptine can treat galactorrhoea induced by antipsychotic drugs.
Galactorrhea is caused by antipsychotics acting directly on the breast tissue.
Galactorrhoea is caused by the concurrent use of anticholinergic medication.

35. Which of the following drugs is LEAST likely to increase lithium toxicity?
A.
B.
C.
D.
E.

Angiotensin-converting enzyme (ACE) inhibitors

Calcium channel blockers


Non-steroidal anti-inflammatory drugs (NSAIDS)

Sodium valproate
Thiazide diuretics.

36. Which of the following is FALSE about clozapine?


A.
B.
C.
D.
E.

It can cause agranulocytosis.


It has high affinity for D2 receptors.
It has low risk of extrapyramidal side effects.
It is associated with metabolic syndrome.
It is good for treatment resistant schizophrenia.

37. A 30-year-old woman with 3 episodes of major depression in the past 5 years
responds to fluoxetine 40mg every morning. Her last episode was 6 months ago.
In order to minimize the risk of relapse, which of the following treatment
strategies is MOST effective?
A.
B.
C.
D.

Continue fluoxetine 40mg every morning for five years.


Continue fluoxetine 40 mg for six months and then stop.
Provide intermittent maintenance electroconvulsive therapy.
Stop fluoxetine and restart it at the first sign of relapse.

!
!
95

E. Stop fluoxetine and start cognitive behaviour therapy.


Psychotherapy
38. Which of the following treatment strategies has the MOST evidence for treating
obsessive compulsive disorder?
A. Fluoxetine and brief dynamic psychotherapy
B. Fluoxetine and exposure and response prevention
C. Fluoxetine and eye movement desensitization and reprocessing
D. Fluoxetine and hypnotherapy
E. Fluoxetine and interpersonal psychotherapy.
39. You have to study anaesthesia, emergency medicine, obstetrics and
gynaecology, otolaryngology, ophthalmology and psychiatry in less than two
weeks. Your first thought is, I will never be able to study all six subjects in such
a short time! The upcoming examination is very stressful. You ignore the fact
that you were successful in the previous professional MBBS examinations and
you could handle multiple subjects in one examination in the past. From a
cognitive therapy perspective, what is the BEST description of this type of
thinking?
A.
B.
C.
D.
E.

Catastrophic thinking
Magnification
Minimization
Personalization
Selective abstraction.

Ethics and laws


40. In Singapore, a person who is disturbed and aggressive, with poor insight into his
illness, refusing treatment and threatening family members can be admitted
involuntarily under the:
A.
B.
C.
D.
E.

Advanced Care Directive


Mental Health (Care and Treatment) Act
Mental Capacity Act
Mental Health Act
Psychiatric Disorders and Treatment Act.

Liaison and neuropsychiatry


41. Which of the following medical conditions is LEAST likely to present with panic
attacks?

!
!
96

A. Asthma
B. Cushing syndrome
C. Insulinoma
D. Phaeochromocytoma
E. Thyrotoxicosis.

42. A 50-year-old man suffering from schizophrenia and he has been taken
haloperidol for the past 20 years. His QTc is 550 ms. The medical resident wants
to find the potential medical complication if he continues to take haloperidol.
Which of the following complications is LEAST LIKELY:
A.
B.
C.
D.
E.

Myocardial infarction
Palpitation
Ventricular fibrillation
Sudden cardiac death
Torsade de pointes.

43. A 40-year-old schizophrenia patient presents with high urine volume, low urine
osmolality, low serum sodium and low urine sodium. Which of the following is the
MOST likely diagnosis?
A. Diabetes mellitus
B. Nephrogenic diabetes insipidus
C. Psychogenic polydipsia
D. Simple hyponatremia
E. Syndrome of inappropriate antidiuretic hormone secretion.

44. You are a resident working in the gynaecology ward. A 35-year-old woman was
admitted for ovarian cystectomy. She suffers from major depressive disorder and
insomnia. She consults a psychiatrist at the Institute of Mental Health (IMH). She
is prescribed with venlafaxine, zolpidem, lorazepam and hydroxyzine. She is due
for discharge today and she wants to go home. She expresses concern that she
cannot see her psychiatrist in the coming weeks due to pain associated with the
operation. She requests to obtain 3-month supply of her psychiatric medications
from you. She also requests a pain killer called tramadol. She threatens that if you
do not give her medication, she will commit suicide. What is the BEST approach
to handle this situation?
!
!
97

A. Do not give her any psychotropic medication and tramadol. Discharge her the same
day.
B. Negotiate with her for shorter duration of psychotropic medications and tramadol.
Obtain earlier appointment at IMH. Discharge her today.
C. Postpone discharge and use this to motivate her not to request for more
psychotropic medications.
D. Give her 3-month supply of psychotropic medications and tramadol. Discharge her
today.
E. Send her to IMH for assessment because she threatens that she will commit suicide.
MCQ exam (Paper 4)
History and mental state exam
45. You are a medical resident. The nurse informs
you that an 80-year-old man who was admitted
for cellulitis seems to be depressed and
mentions about passive suicidal thought.
When you assess him, he sees ghost in the
ward and expresses paranoid ideation against
nursing staff. Which of the following
assessment is MOST important to establish
the diagnosis?
F.
G.
H.
I.
J.

Assess orientation
Assess judgement
Assess recognition
Assess registration and short-term recall
Assess somatic symptoms.

The answer is A.

Explanation: This man may suffer


from delirium and acute
confusional state. It is most
important to assess orientation.

Assessing somatic symptoms is


not useful as it is caused by
medical conditions.

Year: 2013.

Cognitive assessment
46. Which of the following cognitive tasks is NOT
an assessment of short term or long term
memory?
F. Ask the patient to name the current Prime
Minister of Singapore.
G. Ask the patient to name as many animals as
possible that can be found in the Singapore
Zoo.
H. Ask the patient to tell you his or her address
and later, you check the answer with patients
!
!
98

The answer is B.

Explanation: Option B refers to


verbal fluency which is part of the
frontal lobe assessment. Option D
refers to registration and it is an
important component before
assessing the short-term recall
(Option E). To be fair to the

medical record.
I. Inform the patient 3 objects (e.g. Apple,
Newspaper and Train) and ask the patient to
name the 3 objects immediately.
J. Inform the patient 3 objects (e.g. Apple,
Newspaper and Train) and ask the patient to
name the 3 objects after 5 minutes.

patient, the patient should be given


multiple attempts to register the 3
items before testing short-term
recall in Option E.

Year: 2013.

Psychiatric, epidemiology, etiology, diagnosis and


classification
47. A 20-year-old man with an identical twin is
diagnosed with major depressive disorder.
His twin brother asks you to comment on the
chance he will develop depressive disorder.
Which of the following is the CORRECT
response to his twin brother?
F. Major depressive disorder is a strongly
inheritable disease. You will definitely develop
it as well.
G. The likelihood of you developing depressive
disorder is higher than other siblings.
H. The likelihood of you developing depressive
disorder is same as non-identical twins.
I. There is only a slightly increased risk that you
will become depressed as compared to the
general population.
J. You do not need to worry because
environmental factor is an important factor to
determine whether you will develop depressive
disorder.

The answer is B.

Explanation:

Family studies show that a person


has 40-70% chance to develop
depressive episode if a first degree
relative suffer from depressive
episode.
Twin studies show that the
concordance rate for monozygotic
twins is 40 50% and for dizygotic
twins is 20%.

Year: 2013.

48. A 35-year-old woman suffers from low mood,


low energy, poor sleep, poor appetite and
recurrent suicidal thoughts. She firmly
believes that she deserves the death sentence
for minor mistakes made in the past. Which of
the following statements is FALSE?
F. For severe depressive episode with psychotic
features, psychotic symptoms usually occur
after manifestation of depressive symptoms.
G. Her psychotic symptoms are considered to be
!
!
99

The answer is C.

Explanation: This man suffers from


severe depressive disorder with
psychotic features. Option A, B, D
and E are correct.

mood congruent.
H. Based on the case scenario, there is enough
evidence to suggest that she suffers from
schizoaffective disorder.
I. The prescription of antipsychotic drug should
be considered.
J. Electroconvulsive therapy (ECT) is a
recognised treatment for his condition.

There is not enough evidence to


suggest the diagnosis of
schizoaffective disorder because
her symptoms do not suggest that
she suffers from schizophrenia.

Year: 2013.
49. Which of the following disorders has been
shown to have the GREATEST degree of
heritability?
F. Alcoholism
G. Attention-deficit/hyperactivity disorder (ADHD)
H. Autism
I. Major depressive disorder
J. Schizophrenia.

The answer is C.

Explanation:
Heritability of autism to be more
than 90%. 90% of the differences
between autistic and non-autistic
individuals is due to genetic
effects.

Year: 2013.

50. Which of the following is the MOST common


cause of insomnia among psychiatric patients
in Singapore?
F.
G.
H.
I.
J.

The answer is D.

Explanation: Underlying psychiatric


disorder is the most common
cause of insomnia among
psychiatric patients. Students
should observe this during their
clinical attachment. Depressive
disorder, bipolar disorder and
generalized anxiety disorder are
associated with insomnia.

Sleep apnea
Shift work
Stimulant use
Underlying psychiatric illness
Use of sleeping pills.

51. Regarding the age of onset in schizophrenia,


which of the following is CORRECT?

!
!
100

The answer is D.

F. European women have earlier age of onset


than Asian women.
G. Current research findings remain
inconclusive about the age of onset in
schizophrenia.
H. There is no difference in the age of onset
between men and women.
I. Men usually have earlier age of onset as
compared to women.
J. Women usually have earlier age of onset as
compared to men.

Explanation: Current research


findings in schizophrenia show that
men have earlier onset as
compared to women.

Psychopathology
52. Agoraphobia without panic attack is BEST
referred to:
F.
G.
H.
I.
J.

Concern about physical appearance


Constant worried about physical health
Fear of blood and needle
Fear of collapse in an enclosed space
Fear of authority figures.

The answer is D.

Explanation: Patients suffering


from agoraphobia has fear that
they cannot escape from the
enclosed space.

Year: 2013.
53. Which of the following is MOST suggestive of
the diagnosis of schizophrenia?
F. Acute onset of psychosis
G. Apathy
H. Cognitive impairment
I. Hearing own thoughts
J. Persistent deterioration of personality.

The answer is D.

Explanation: Hearing own thoughts


or echo de la pense only occurs in
schizophrenia and is most
suggestive of such diagnosis.

Option A can occur in brief


psychosis or acute/transient
psychosis, not necessarily
schizophrenia.

Option B, D and E occur in both


schizophrenia and dementia.

!
!
101

Year: 2013.
General adult psychiatry
54. Peter is 27-year-old and lives with his parents.
He has been employed as a delivery man for
most of the time since leaving school, but has
recently left his job as a salesman. He has
never taken any illicit drugs. His parents state
that in the last three weeks, he has been
extremely active, requiring less sleep and not
appearing tired, being over-talkative and
disinhibited and on occasions quite
irritable. He claimed to have invented a
machine for curing cancer and wished to go to
the U.S. to sell it. When stopped by his
parents, he became violent, and they called
the police. Which of the following diagnosis is
MOST appropriate for this patient?
F.
G.
H.
I.
J.

Bipolar I disorder with manic features


Bipolar II disorder with hypomanic features
Cyclothymia
Mixed affective disorder
Schizoaffective disorder.

55. Which of the following personality disorders


is LEAST likely to be considered as a
differential diagnosis to schizophrenia?
F.
G.
H.
I.
J.

Borderline personality disorder


Obsessive compulsive personality disorder
Paranoid personality disorder
Schizotypal personality disorder
Schizoid personality disorder.

The answer is A.

Explanation: Peter develops


grandiose delusion because he
does not have the capacity to
make a machine to cure cancer
based on his background. The
most appropriate diagnosis is
bipolar disorder with manic
features.

Year: 2013.

The answer is B.

Explanation: Obsessive
compulsive personality disorder is
least likely to be associated with
psychotic features.

Year: 2013
56. Which of the following is LEAST likely to be
found in research in the relationship between
depressive disorder and hormones?
F. Blunted adrenocorticotropic hormone (ACTH)
response to the corticotropin-releasing
hormone (CRH)
G. Blunted thyroid stimulating hormone (TSH)
response to thyrotropin-releasing hormone
!
!
102

The answer is C.

Explanation: There is no
suppression of cortisol with
dexamethasone in research finding
in depressive disorder. Option A
indicates the failure of negative

(TRH)
H. Cortisol suppression with dexamethasone
I. There is an increased cortisol.
J. There is an increased adrenal sensitivity to
ACTH.

feedback and causes an increase


in cortisol (Option D and E). Option
B is associated with low T3 or T4
level.

Year: 2013.

This is an advanced-level
question.
57. Which of the following disorders is LEAST
likely to be considered as a differential
diagnosis of a 35-year-old driver develops
nightmare, flashback and hypervigilance after
a road traffic accident?
F. Factitious disorder
G. Generalized anxiety disorder
H. Major depressive disorder
I. Obsessive compulsive disorder
J. Phobic disorder.

The answer is D.

Explanation: OCD is least likely to


share symptoms with posttraumatic stress disorder (PTSD).
As a result, OCD is least like to be
a DDX of PTSD.

Year: 2013.
58. Rapid eye movement (REM) sleep is
associated with all of the following EXCEPT:
F.
G.
H.
I.
J.

Increased parasympathetic activity


Increased cerebral blood flow
Increased complexity of dreams
Maximal loss of muscle tone
Transient runs of conjugate eye
movements.

The answer is A.

Explanation: REM sleep is


associated with an increase in
sympathetic activity.

Year: 2013.

This is an advanced level question.


59. A 17-year-old male comes to see you because
he is experiencing tremendous stress in his
new job. He has finished his O level
examination and waiting to start the National
Service in 6 months. He got an offer to work in
a shoe shop which specializes in selling
female shoes. He realizes that he feels
!
!
103

The answer is B.

Explanation:

sexually aroused by female shoes. He spends


a lot of time fantasizing about female shoes.
He stole multiple pairs of shoes from the shop
and used them for masturbation. He used to
be a normal person. He was a hardworking
student whose main interests were chemistry
and physics. What is the MOST likely
diagnosis?
F.
G.
H.
I.
J.

Adjustment disorder
Fetishism
Kleptomania
Sadomasochism
Voyeurism.

This man suffers from fetishism


because he derives sexual
stimulation from an inanimate
object (i.e. female shoes) and his
fetishism causes social and
occupational dysfunction because
he spends inordinate time
fantasizing about female shoes.

He stole shoes to satisfy his


fetishism and this is not a case of
kleptomania.

Year: 2013.

60. Which of the following psychiatric disorders is


MOST common among patients with bulimia
nervosa?
F.
G.
H.
I.
J.

The answer is B.

Explanation: Major depressive


disorder is the most common
psychiatric comorbidity in bulimia
nervosa (around 50- 85%)

Alcohol use disorder


Major depressive disorder
Obsessive compulsive disorder
Panic disorder
Intermittent explosive disorder.

Year: 2013.

Old age psychiatry


61. Manic episodes in old people are associated
with:
F.
G.
H.
I.
J.

Less euphoria
Less mixed presentation with depression
Less paranoid delusions
More hyperactivity
More flight of ideas.

!
!
104

The answer is A.

Explanation: Mania in old people is


associated with less euphoria, less
hyperactivity, less flight of ideas
but more mixed presentation and
more paranoid ideation.

Year: 2013.

This is an advanced level question.


The answer is A.
62. All of the following statements are true
regarding post-operative delirium EXCEPT:
F. Delirium cannot be diagnosed if all laboratory
results are normal.
G. It can occur in an 80-year-old man after
gastrectomy.
H. It can occur in a 16-year-old girl after renal
transplant.
I. Haloperidol is one of the treatments of choice.
J. Physical restraint should be avoided if possible.

Explanation: 30% of patients


suffering from delirium may have
normal laboratory results and no
abnormality was found. Delirium
can occur in young people who
undergo a complicated operation
with immunosuppressant.

Year: 2013.
63. A 70-year-old man with a history of drinking
Chinese wine, hepatitis, chronic renal failure
and hypertension was brought by his wife to
the hospital for treatment of an acute cellulitis.
He was noted to have tachycardia in the
Accident and Emergency Department. You are
the on-call medical resident and this man was
admitted to the medical ward. What is the
MOST appropriate first approach?
F. Assess short term recall to rule out
dementia on delirium.
G. Obtain history from collaterals whether he
has in fact been drinking Chinese wine
recently.
H. Put him on propranolol 10mg three times
per day to control tachycardia.
I. Start diazepam 5mg three times per day to
prevent delirium tremens.
J. Start lorazepam 1mg three times per day to
prevent delirium tremens.

The answer is B.

Explanation:
This patient is already at a high
risk for delirium based on his age
and medical comorbidities,
including chronic renal failure.

Without adequate collateral


history, providing a high dose
regimen of potentially unnecessary
benzodiazepine puts the patient at
risk of worsened delirium. His
elevated heart rate may reflect
agitation or pain due to acute
cellulitis.

Beta blockers most often mask the


sympathetic outflow signs of
!
!
105

withdrawal of alcohol and the


prescription of propranolol is not
necessary.

Year: 2013.
64. A 70-year-old woman with severe depressive
symptoms and strong suicidal ideation which
is not responded to amitriptyline 150mg daily
and fluvoxamine 200mg daily. Each
medication was tried for 6 months with good
adherence. She has good past health. Her
children consult you for further management.
The MOST appropriate treatment which you
would recommend is:

The answer is B.

Explanation: This elderly woman


suffers from treatment resistant
depression and electroconvulsive
therapy is the treatment of choice
to treat suicidal ideation.

A. Cognitive behaviour therapy


B. Electroconvulsive therapy

Old age is not a contraindication


for electroconvulsive therapy.

C. Donepezil
D. Olanzapine
E. Repetitive transcranial magnetic stimulation.

Year: 2013.

Child and adolescent psychiatry and intellectual


disability
65. You are helping a mother to develop
behavioural program to deal with her son who
has oppositional defiant disorder. Which of
the following is the MOST significant
component of this program?
F.
G.
H.
I.
J.

Extinction
Positive reinforcement
Punishment
School suspension
Strict parenting.

The answer is B.

Explanation: Positive
reinforcement of promoting
desirable behaviours through
rewards is the most significant
component of the program.

Year: 2013.
66. Which of the following factors are LEAST
likely to be the aetiological factors in attention
!
!
106

The answer is D.

deficit and hyperactivity disorder (ADHD)?


A.
B.
C.
D.
E.

Antenatal exposure to alcohol


Antenatal exposure to nicotine
Delivery complications
High birth weight
Traumatic brain injury during infancy
and early childhood.

67. You are a general practitioner. A 40-year-old


woman consults you as a result of high!blood!
pressure.!General!inspection!shows!ptosis,!a!
broad!neck!and!indistinct!hairline.!Her!chest!
appears!to!be!broad.!Auscultation!reveals!
cardiac!murmurs.!She!is!single!and!stays!with!
her!parents.!She!completed!her!education!in!the!
Institute!of!Technological!Education!(ITE)!and!
works!as!a!store!room!officer.!If!chromosome!
analysis!is!performed,!the!MOST!likely!finding!
is:
!
A. XO syndrome
B. XXY syndrome
C. XXXY syndrome
D. XYY syndrome
E. YO syndrome.

Explanation: Low birth weight, not


high birth weight is associated with
the risk of developing ADHD.

Year: 2013.

The!answer!is!A.!
!
Explanation:!She!suffers!from!
Turners!syndrome!(XO!syndrome)!
as!evidenced!by!webbed!neck!(a!
broad!neck!and!a!low!or!indistinct!
hairline),!coarctation!of!aorta!
(cardiac!murmurs,!hypertension)!
and!normal!/borderline!intelligence.!
She!may!have!infertility.!Her!single!
status!cannot!confirm!the!infertility!
but!it!remains!a!possibility.!
!
Year: 2013.

This is an advanced-level
question.
68. You are a paediatric resident. A 7-year-old boy
is suspected to suffer from autism. You
referred this boy to see an education
psychologist for assessment. The education
psychologist sends a report to you and you
need to explain the findings to her mother. His
strength is MOST likely found in which of the
following areas?
A.
B.
C.
D.

Abstract thinking
Block design
Explain similarities
Oral presentation skills

The answer is B.

Explanation: In autism,
performance IQ (Block design) is
higher than verbal IQ.

Year: 2013.

!
!
107

E. Verbal concept formation


Substance abuse
69. Which of the following neuroanatomical
structures is MOST implicated in substance
misuse and dependence?
F.
G.
H.
I.
J.

Explanation: Substance abuse and


dependence produce additive
effects on dopamine release in the
nucleus accumbens.

Amygdala
Basal nucleus of Meynert
Nucleus accumbens
Hippocampus
Raphe nucleus.

Year: 2013

70. A 50-year-old man is admitted to the


hepatobillary ward as a result of pancreatitis.
You are the resident on call. His wife tells you
that she has seen him intoxicated with alcohol
in the past few days. The patient denies.
Which of the following is the BEST method
available in a general hospital in Singapore to
confirm that this patient has been drinking
recently?
F.
G.
H.
I.
J.

Carbohydrate-deficient transferrin (CDT)


Gamma-glutamyl transferase (GGT)
Mean corpuscular volume (MCV)
Serum alcohol level
Serum amylase.

71. A common and safe pharmacological


treatment for reducing relapse in alcohol
dependence has direct actions upon which
receptors?
F.
G.
H.
I.
J.

The answer is C.

Benzodiazapine
Cannabinoid
Dopamine
-Aminobutyric acid (GABA)
Opioid.

The answer is B.

Explanation: GGT is an early


indicator of alcohol relapse,
SENSITIVITY is 40-60% and
SPECIFICITY is 80%. 4 drinks per
day for the past 2 weeks will
increase GGT. CDT is a very
sensitive and specific test but it is
not readily available in a general
hospital in Singapore. The cost
CDT is much higher than GGT.
Year: 2013
The answer is E.

Explanation: This question refers


to naltrexone, a common and safe
pharmacological treatment for
reducing relapse in alcohol
dependence.

Year: 2013

!
!
108

72. Which of the following is LEAST consistent


with the objective of motivational
interviewing?
F. Allow the patient give their inputs without
interruption.
G. Establishing a collaborative patient-therapist
relationship.
H. Identifying appropriate reinforcements.
I. Providing empirical advice.
J. Usage of open-ended questions.

The answer is D.

Explanation: Motivational
interviewing emphasizes on
individuals patients needs and
avoid empirical advice.

Year: 2013

Psychopharmacology
73. Which of the following antidepressants is
LEAST likely to have sexual side-effects?
F.
G.
H.
I.
J.

Amitriptyline
Moclobemide
Mirtazapine
Fluoxetine
Venlafaxine.

The answer is C.

Explanation: Mirtazapine and


bupoprion have relatively low risk
of causing sexual side effects as
compared to other
antidepressants.

Year: 2013
74. The benzodiazepine with the LONGEST halflife is:
F. Clonazepam
G. Diazepam
H. Flurazepam
I. Lorazepam
J. Triazolam.

The answer is B.

Explanation: Diazepam has the


longest half life which lasts for
more than one day.

Year: 2013
75. Which of the following is NOT an
acetylcholinesterase inhibitor?
F. Buprenorphine
G. Donepezil
H. Galantamine
I. Rivastigmine
J. Tacrine.

The answer is A.

Explanation: Buprenorphine is a
partial opioid agonist which is used
in the treatment of opioid
dependence but this medication

!
!
109

was banned in Singapore due to


diversion and misuse.

Year: 2013.

76. Which of the following regarding side effects


of risperidone is FALSE?
F. Akathisia is common with high dose of
risperidone.
G. Prolactin elevation, probably greater than that
seen with other second generation
antipsychotics.
H. Insomnia, headache and nausea are common.
I. Rhinitis is a possible side effect.
J. The risk for weight gain and the metabolic
syndrome is higher than olanzapine.

77. Tricyclic antidepressants should be avoided


with all of the following EXCEPT
F. Patients hospitalized for severe melancholic
depression
G. Recent myocardial infarction
H. Right bundle branch block
I. Untreated glaucoma
J. Urinary retention.

The answer is E.

Explanation: The risk for weight


gain and metabolic syndrome is
lower than olanzapine.

Year: 2013.

The answer is A.

Explanation: TCA may be helpful


in severe depression. The other
are relative contraindications.

Year: 2013.
78. Which of the following is CORRECT regarding
galactorrhoea induced by antipsychotic
drugs?
F. Antipsychotic drugs modulate the hypothalamic
function and lead to galactorrhoea.
G. Antipsychotic drugs cause pituitary adenoma
and galactorrhoea.
H. Bromocriptine can treat galactorrhoea induced
by antipsychotic drugs.
I. Galactorrhea is caused by antipsychotics acting
!
!
110

The answer is C.

Explanation: Galactorrhoea can be


treated with bromocriptine.

Galactorrhoea is due to a direct


effect on the anterior pituitary by
dopamine secreting

directly on the breast tissue.


J. Galactorrhoea is caused by the concurrent use
of anticholinergic medication.

tuberoinfundibular neurons, which


normally inhibit prolactin release.

Antipsychotic drugs do not cause


pituitary adenoma.

Year: 2013

This is an advanced-level
question.
79. Which of the following drugs is LEAST likely
to increase lithium toxicity?
F.
G.
H.
I.
J.

The answer is D.

Angiotensin-converting enzyme (ACE) inhibitors

Calcium channel blockers


Non-steroidal anti-inflammatory drugs (NSAIDS)

Sodium valproate
Thiazide diuretics.

Co-administration of valproate and


lithium have no effect on the steadystate kinetics of lithium.

Year: 2013

80. Which of the following is FALSE about


clozapine?
F.
G.
H.
I.
J.

It can cause agranulocytosis.


It has high affinity for D2 receptors.
It has low risk of extrapyramidal side effects.
It is associated with metabolic syndrome.
It is good for treatment resistant schizophrenia.

The answer is B.

Explanation: It has low affinity for


D2 receptors.

Year: 2013

81. A 30-year-old woman with 3 episodes of major


depression in the past 5 years responds to
fluoxetine 40mg every morning. Her last
episode was 6 months ago. In order to
minimize the risk of relapse, which of the
following treatment strategies is MOST
effective?
!
!
111

The answer is A.

Explanation: This patient has 3


episodes of major depression in
the past 5 years and the number of

F. Continue fluoxetine 40mg every morning for


five years.
G. Continue fluoxetine 40 mg for six months and
then stop.
H. Provide intermittent maintenance
electroconvulsive therapy.
I. Stop fluoxetine and restart it at the first sign of
relapse.
J. Stop fluoxetine and start cognitive behaviour
therapy.

episodes is considered to be
frequent.

It is dangerous to reduce the dose


or stop antidepressant soon.

Year: 2013

Psychotherapy
82. Which of the following treatment strategies
has the MOST evidence for treating obsessive
compulsive disorder?
A. Fluoxetine and brief dynamic
psychotherapy
B. Fluoxetine and exposure and response
prevention
C. Fluoxetine and eye movement
desensitization and reprocessing
D. Fluoxetine and hypnotherapy
E. Fluoxetine and interpersonal
psychotherapy.

The answer is B.

Explanation: Exposure and


response prevention is part of the
cognitive behaviour therapy and
has the most evidence for treating
obsessive compulsive disorder.

Year: 2013

83. You have to study anaesthesia, emergency


medicine, obstetrics and gynaecology,
otolaryngology, ophthalmology and
psychiatry in less than two weeks. Your first
thought is, I will never be able to study all six
subjects in such a short time! The upcoming
examination is very stressful. You ignore the
fact that you were successful in the previous
professional MBBS examinations and you
could handle multiple subjects in one
examination in the past. From a cognitive
therapy perspective, what is the BEST
description of this type of thinking?
F. Catastrophic thinking
!
!
112

The answer is E.

Explanation: This student


demonstrate selective abstraction:
focus on current negative aspect
but ignore the positive aspects in
the past.

Year: 2013

G.
H.
I.
J.

Magnification
Minimization
Personalization
Selective abstraction.

Ethics and laws


84. In Singapore, a person who is disturbed and
aggressive, with poor insight into his illness,
refusing treatment and threatening family
members can be admitted involuntarily under
the:
F.
G.
H.
I.
J.

Advanced Care Directive


Mental Health (Care and Treatment) Act
Mental Capacity Act
Mental Health Act
Psychiatric Disorders and Treatment Act.

The answer is B.

Explanation: Mental Health (Care


and Treatment) Act was passed in
2008 and this is the most correct
answer.

Year: 2013.

Liaison and neuropsychiatry


85. Which of the following medical conditions is
LEAST likely to present with panic attacks?

The answer is B.

Explanation: Cushing syndrome is


more likely to be associated with
depression rather than panic
attacks.

A. Asthma
B. Cushing syndrome
C. Insulinoma
D. Phaeochromocytoma

Year: 2013.

E. Thyrotoxicosis.

86. A 50-year-old man suffering from


schizophrenia and he has been taken
haloperidol for the past 20 years. His QTc is
550 ms. The medical resident wants to find the
potential medical complication if he continues
to take haloperidol. Which of the following
complications is LEAST LIKELY:
F. Myocardial infarction
G. Palpitation
H. Ventricular fibrillation
!
!
113

The answer is A.

Explanation: A prolonged QTc


mainly affects the ventricles,
leading to ventricular fibrillation,
torsade de pointes and sudden
cardiac death. Palpitation is a
recognized complication.
Myocardial infarction is unlikely to

I. Sudden cardiac death


J. Torsade de pointes.

be associated with prolonged QTc.

Year: 2013.

This is an advanced-level
question.
87. A 40-year-old schizophrenia patient presents
with high urine volume, low urine osmolality,
low serum sodium and low urine sodium.
Which of the following is the MOST likely
diagnosis?
A. Diabetes mellitus
B. Nephrogenic diabetes insipidus
C. Psychogenic polydipsia

The answer is C.

Explanation: He suffers from


psychogenic polydipsia because
there is a dilution in urine volume,
urine osmolality, serum sodium
and urine sodium due to fluid
overload and a result of excessive
drinking.

D. Simple hyponatremia
E. Syndrome of inappropriate antidiuretic hormone
secretion.

88. You are a resident working in the gynaecology


ward. A 35-year-old woman was admitted for
ovarian cystectomy. She suffers from major
depressive disorder and insomnia. She
consults a psychiatrist at the Institute of
Mental Health (IMH). She is prescribed with
venlafaxine,
zolpidem,
lorazepam
and
hydroxyzine. She is due for discharge today
and she wants to go home. She expresses
concern that she cannot see her psychiatrist
in the coming weeks due to pain associated
with the operation. She requests to obtain 3month supply of her psychiatric medications
from you. She also requests a pain killer
called tramadol. She threatens that if you do
not give her medication, she will commit
suicide. What is the BEST approach to handle
this situation?
F. Do not give her any psychotropic medication
!
!
114

Year: 2013.

The answer is B.

Explanation: It is reasonable that


patient may not be able to see her
psychiatrist shortly after operation.
Her suicide risk may not be
genuine in this case because it is
after a condition (if no medication
is given).

In real situation, the best answer is


to assess her suicide risk by
yourself or your hospital
psychiatrist. This option is not
available and you need to choose

and tramadol. Discharge her the same day.


G. Negotiate with her for shorter duration of
psychotropic medications and tramadol.
Obtain earlier appointment at IMH. Discharge
her today.
H. Postpone discharge and use this to motivate
her not to request for more psychotropic
medications.
I. Give her 3-month supply of psychotropic
medications and tramadol. Discharge her
today.
J. Send her to IMH for assessment because she
threatens that she will commit suicide.

the next best answer.

Option A may cause a relapse of


her psychiatric illness by not giving
her any psychotropic medications.
Option C will intensify her
disappointment and postpone
discharge does not offer solution.

Option D: 3-month supply


medication is too long and it may
pose risk to patients.

Option E will affect therapeutic


alliance and it is important to
further assess her suicide risk. She
mentions about suicidal thought as
a sign of anger.

Year: 2013

This is an advanced-level
question.

!
!
Paper!5:!Questions!
History, mental state exam and psychopathology!

1. A! 50TyearTold! woman! presents! with! severe! depressive! episode! with! psychotic!


features.!Which!of!the!following!delusions!is!LEAST!likely!to!occur!in!this!patient?!
!
A. Deserved!punishment!
!
!
115

B.
C.
D.
E.

Guilt!
Incurable!illnesses!
Jealousy!
SelfNdepreciation.!

2. A!30TyearTold!prisoner!gives!approximate!answers!to!questions.!For!example,!if!it!
is!Thursday,!he!will!say!it!is!Friday!and!claims!a!dog!has!three!legs.!This patient is
MOST likely suffering from:
A.
B.
C.
D.
E.

Capgras syndrome
Ekboms syndrome
Fregolis syndrome
Gansers syndrome
Othellos syndrome.

3. A psychiatric patient suddenly realises that he is an interpreter for deaf people and tries
to interpret others speech in sign language. He demonstrates strange repetitive
movements. His signs appear to come in threes or fours, occasionally swinging his
shoulders, as if he is conveying a message. Which of the following is the BEST term to
describe his movements?

!
A.
B.
C.
D.
E.

Ambitendency!
Mannerism!
Negativism!
Stereotypies!
Waxy!flexibility.!

!
!
Cognitive assessment!
4. A 33-year-old man with childhood developmental delay and epilepsy received special
education and now works as a cleaner. He has worked in a tofu factory for three years.
He can only perform simple routine work in the factory. His supervisor reports his work
performance is slow and poor. His family says he cannot live independently and
demonstrated delay in achievement in self-care since young. How would you rate the
level of mental retardation?
A.
B.
C.
D.
E.

Very mild mental retardation


Mild mental retardation
Moderate mental retardation
Severe mental retardation
Profound mental retardation.

Psychiatric epidemiology
5. A 65-year-old retired teacher is concerned about dementia and has assessed online
resources for information. She asks which type of dementia is more common in Asians
as compared to Caucasians. Your answer is:

!
!
116

A. Alzheimers disease
B. Lewy body dementia
C. Pseudodementia
D. Vascular dementia
E. Fronto-temporal lobe dementia.
Psychiatric aetiology, diagnosis and classification

6. A 20-year-old national serviceman was diagnosed to suffer from adjustment


disorder with depressed mood. He read the internet and found another disorder
known as dysthymic disorder. He wants to know the differences between these
two disorders. The following statements about differences between adjustment
disorder with depressed mood and dysthymia are true EXCEPT:
K. Adjustment disorder with depressed mood results from a known stressor, whereas
dysthymic disorder does not.
L. Adjustment disorder with depressed mood shows a genetic pattern, whereas
dysthymic disorder does not.
M. The duration of symptoms is shorter for adjustment disorder with depressed mood
than it is for dysthymic disorder.
N. The co-existence of a major depressive disorder with dysthymic disorder is called
double depression. There is no term for a major depressive disorder complicating
adjustment disorder with depressed mood.
O. Treatment is usually shorter and does not require the use of medication for
adjustment disorder with depressed mood as compared to dysthymic disorder.
7. You are a family doctor. A male and a female schizophrenia patient who stay in the
community have decided to get married and have a child. What is the risk for their child
to develop schizophrenia in the future?
A.
B.
C.
D.
E.

16%
26%
36%
46%
56%.

8. You!are!an!oncology!resident.!!A!60TyearTold!oncology!patient!developed!severe!nausea!as!a!
sideTeffect!during!chemotherapy.!However,!after!completing!treatment!she!continues!to!
experience!nausea!whenever!she!returns!to!the!hospital!for!followTup.!Which!of!the!
following!psychological!theories!BEST!describes!her!experience?
A. Biofeedback
B. Classical conditioning
C. Cognitive learning

!
!
117

D. Operant conditioning
E. Social learning.
General adult psychiatry
9. A 25-year-old woman experiences episodes of depersonalization, hyperventilation,
dizziness, and diaphoresis lasting for 15 minutes whenever she arrives at work. She
has been overwhelmed by work and conflicts with her supervisor for the past 3 months.
There are no such episodes at home or outside the office. What is the MOST likely
diagnosis?

!
A.!Acute!stress!disorder!
B.!Generalized!anxiety!disorder!
C.!Panic!disorder!!
D.!PostNtraumatic!stress!disorder!
E.!Somatization!disorder.!

10. You are an orthopaedic resident managing a 24-year-old motorcyclist who was involved
in a road traffic accident 3 days ago in which he sustained multiple fractures and his
pillion rider died. He is irritable, sleeps poorly and has nightmares and refuses to talk
about the accident. The MOST likely psychiatric diagnosis is:

!
A.!Acute!stress!disorder!!
B.!Adjustment!disorder!with!anxiety!
C.!Depressive!disorder!
D.!Generalized!anxiety!disorder!
E.!PostNtraumatic!stress!disorder.!

11. Based on the current research findings, which of the following statements is CORRECT
regarding the prognosis for a 35-year-old Indonesian man with schizophrenia who lives
in a rural village and treated by risperidone?
A.
B.
C.
D.
E.

He has a worse prognosis as compared to patients staying in an urban city.


His prognosis is poor because he will seek treatment from traditional healer in the village.
He has a better prognosis as compared to patients staying in an urban city.
His prognosis is the same as patients staying in in an urban city
His prognosis is not related to the urban development of the society where he resides.

!
!
118

!
12. You!are!a!resident!working!in!the!Accident!and!Emergency!Department.!!A!39TyearTold!
English!man!was!married!to!a!Chinese!Singaporean!is!brought!to!the!Emergency!
Department!after!he!attempted!to!hang!himself.!He!is!actively!suicidal!and!has!marital!
problems.!He!has!not!spoken!to!his!wife!for!three!days.!The!hospital!does!not!have!a!
psychiatric!ward!nor!a!stayTin!psychiatric!team.!What!is!the!MOST!appropriate!
management!plan?!
A. Admit Peter to the medical ward of your hospital.
B. Discharge Peter from Emergency Department with follow-up in the polyclinic to reduce the
stigma.
C. Discharge Peter from Emergency Department with an early follow-up in the psychiatric
department.
D. Refer the couple for marital counselling by on-call medical social worker.
E. Transfer Peter to IMH for assessment and consider admission under Mental Health (Care and
Treatment) Act.
13. A 40-year-old single man consulting you in a general practice clinic reveals that since
adolescence he enjoys observing naked people and those involved in sexual activities
as it helps him achieve sexual arousal. The psychiatric condition associated with this
behaviour is:
A. Exhibitionism
B. Fetishism
C. Sadomasochism
D. Transvestism
E. Voyeurism.

14. A 65-year-old retired man consulting you in a general practice clinic reveals that he has
marital problems. His wife complains that he is very fuzzy about cooking. He does not
eat outside nowadays. If he goes to a high class restaurant, he expects that the steak
must be properly done and he is entitled to change the steak if it is poorly done. Now
he finds most steak restaurants in Singapore are not up to his standard. He claims it is
better to cook by himself. If the food is not up to his standard, he has no one to blame.
He likes to go to high-class club in town. He claims to be a special member in the club
due to his unlimited success and donation. Which of the following personality traits
BEST describes this man?
A.
B.
C.
D.
E.

Antisocial
Avoidant
Borderline
Histrionic
Narcissistic.

!
!
119

15. A!30TyearTold!executive!complains!that!he!cannot!fall!asleep!at!night.!!Two!months!ago,!his!
life! was! interrupted! by! a! work! trip! to! the! United! States.! ! After! returning! to! Singapore,! he!
can!only!falls!asleep!at!2!to!3!a.m.!and!he!cannot!wake!up!on!time!the!next!morning.!!This!
leads!to!negative!impact!on!his!work.!!What!is!the!MOST!likely!diagnosis?!!
A. Circadian rhythm sleep disorder advanced sleep phase
B. Circadian rhythm sleep disorder delayed sleep phase
C. Excessive daytime sleepiness
D. Kleine-Levin syndrome
E. Random eye movement sleep disorder.
Old age psychiatry

16. A!65TyearTold!lady!with!bipolar!disorder!and!severe!functional!decline!presents!with!threeT
day!history!of!acute!agitation,!suicidal!ideation!and!refusing!to!eat!and!drink.!Her!MiniT
Mental!Status!Examination!(MMSE)!score!was!29/30.!auditory!hallucinations.!She!has!not!
responded!to!sodium!valproate!and!olanzapine.!!The!MOST!appropriate!treatment!at!this!
stage!is:!!!

A.
B.
C.
D.
E.

Acetylcholinesterase!inhibitor!
Augmentation!with!antidepressant!
Augmentation!with!anxiolytics!
Change!valproate!to!lithium!
Electroconvulsive!therapy.!

17. You!are!public!health!doctor!and!work!in!the!Ministry!of!Health.!The!Ministry!
wants!to!identify!protective!factors!against!the!development!of!Alzheimers!
disease.!Which!of!the!following!is!least!associated!with!the!development!of!
Alzheimers!disease?!!!
A. Consuming!red!meat!on!a!frequent!basis!
B. Female!gender!
C. High!education!level!!
D. High!homocysteine!level!
E. Never!married.!

18. You!are!a!member!of!the!Hospital!Quality!Improvement!Committee.!The!hospital!wants!to!
improve! the! management! of! delirium.! Which! of! the! following! statements! regarding!
delirium!is!FALSE?!
A. Around!30%!of!the!elderly!admitted!to!the!medical!ward!will!develop!delirium!during!
hospitalisation.!
B. Delirium!is!often!missed!or!overlooked!by!health!care!professionals!in!the!elderly.!
C. Delirium!occurs!in!10%!of!children!undergoing!day!surgery.!
D. Delirium!usually!clears!within!3!days!of!correcting!underlying!medical!cause.!
E. The!incidence!of!delirium!increases!with!age.!
19. You are a geriatric doctor. The manager of a nursing home wants to consult you the
behavior which is most likely correlated with delusions in Alzheimers disease. Your
answer is:
A. Aggression and agitation
B. Disrobing (i.e.state of wearing of no clothes)

!
!
120

C. Shadowing
D. Suicide
E. Wandering.

20. You!are!a!paediatric!resident.!You!are!concerned!about!the!mental!health!of!a!50T
yearTold! mother! whose! 12TyearTold! daughter! died! of! leukaemia.! You! are! not!
certain!whether!she!suffers!from!normal!grief!or!pathological!grief.!Normal!grief!is!
NOT!characterized!by:!!
!
A.
B.
C.
D.
E.

Denial!
Recurrent!suicidal!thoughts!!
Searching!for!the!deceased!
Sensations!of!seeing!or!hearing!the!deceased!
Transient!guilt.!

21. You are a geriatric resident reviewing a 75-year-old woman with Alzheimers disease.
Her family is keen for her to take acetylcholinesterase inhibitor. Which of the following
medical conditions is MOST contraindicated?
A.
B.
C.
D.
E.

Anaemia
Asthma
Diabetes
Hyperthyroidism
Rheumatoid arthritis.

Child and adolescent psychiatry

22. Which of the following is the COMMONEST cardiac complication in a 14-yearold adolescent suffering from anorexia nervosa on admission?
A.
B.
C.
D.
E.

Atrial fibrillation and cardiac arrhythmia.


Bradycardia and hypertension
Bradycardia and hypotension
Tachycardia and hypotension
Tachycardia and hypertension.

Substance abuse

23. A! 30TyearTold! man! admitted! to! the! medical! ward! has! no! access! to! the! substance!
which! he! used! prior! to! admission.! He! complains! of! nausea! and! muscle! pain.!
Physical!examination!reveals!pupillary!dilation.!Which!of!the!following!substances!
is!MOST!likely!to!cause!the!above!withdrawal!symptoms?!
!
A. Alcohol!
!
!
121

B. Cocaine!
C. Midazolam!
D. Nictoine!
E. Opiate.
24. A female student has returned from Amsterdam after an exchange programme and
admits using cocaine for 1 year. She developed psychotic symptoms. Which of the
following is the MOST IMPORTANT risk factor for the development of psychosis while
using cocaine?

A.
B.
C.
D.
E.

An!elevated!body!mass!index!
Combination!with!benzodiazepine!

Being!a!first!time!user!of!cocaine!
Being!female!
NonNintravenous!use.!

25. A!30TyearTold!man!who!is!dependent!on!alcohol!enquires!of!his!family!physician!if!
disulfiram!will!help!him!to!reduce!his!alcohol!intake!while!he!still!drinks!in!a!controlled!
manner.!!Which!of!the!following!recommendations!is!CORRECT?!

A.
B.
C.
D.
E.

!
Disulfiram!is!recommended!because!it!has!more!evidence!than!other!medications!in!
helping!patients!to!quit!alcohol.!
Disulfiram!is!recommended!because!it!will!discourage!him!from!drinking!alcohol.!
Disulfiram!is!recommended!because!he!has!motivation!to!reduce!his!drinking.!
Disulfiram!is!not!recommended!because!it!may!cause!severe!side!effects!when!it!mixes!
with!alcohol.!
Disulfiram!is!not!recommended!because!it!is!indicated!for!opioid!dependence.!

!
Psychopharmacology

26. A!50TyearTold!man!with!a!major!depressive!disorder!did!not!respond!well!to!an!adequate!
trial!of!sertraline!200mg!(a!selective!serotonin!reuptake!inhibitor).!!Neither!did!he!fully!
respond!to!a!subsequent!8Tweek!trial!of!venlafaxine!150mg!(serotonin!noradrenaline!
reuptake!inhibitor).!!Which!of!the!following!is!NOT!RECOMMENDED!for!this!man?!!!

A.
B.
C.
D.
E.

!
Augment!SSRI!with!lithium!(a!mood!stabilizer)!
Augment!SSRI!with!phenelzine!(a!monoaxmine!oxidase!inhibitor)!
Consider!electroconvulsive!therapy!
Consider!psychotherapy!
Increase!the!dose!of!venlafaxine.!!

27. Bupoprion differs from SSRI in which of the following ways?


F.

Bupoprion has a higher incidence of sexual side effects and a lower incidence of

!
!
122

sedation and weight gain.


G. Bupoprion has a lower incidence of sexual side effects and a lower incidence of
sedation and weight gain.
H. Bupoprion has a lower incidence of sexual side effects and higher incidence of
sedation and weight gain
I. Bupoprion has lower incidence of sexual side effects and weight gain and a higher
incidence of sedation.
J. Bupoprion has a higher incidence of sexual side effects and sedation and a lower
incidence of weight gain.
28. Besides treatment resistant schizophrenia, another appropriate indication for clozapine
is?
A.
B.
C.
D.
E.

Severe agranulocytosis
Severe embolism
Severe hypersalivation
Severe metabolic syndrome
Severe tardive dyskinesia.

29. A!50TyearTold!cardiac!patient!suffers!from!depression!and!the!psychiatrist!
prescribed!fluoxetine.!Which!of!the!following!statements!about!fluoxetine!is!TRUE?!
!
A. !Fluoxetine!decreases!warfarin!levels.!
B.!!!!!Fluoxetine!increases!metabolism!of!warfarin.!
C. !!!!!Fluoxetine!increases!bruising!while!patient!is!taking!
!!!!!!!!!!!!!!!!warfarin.!
D. He!has!no!increased!risk!of!gastrointestinal!bleeding!with!fluoxetine.!
E. !!!!He!should!not!change!to!paroxetine!because!paroxetine!decreases!anticoagulant!effect!of!
warfarin.!
!
!
30. Which of the following hypnotic agents causes the LEAST daytime sedation is:
A.
B.
C.
D.
E.

Diphenhydramine
Diazepam
Lorazepam
Zopiclone
Zolpidem.

31. Which of the following medications has the most convincing evidence for reducing
suicidal behavior in bipolar patients?
A. Clozapine
B. Lamotrigine

!
!
123

C. Lithium
D. Olanzapine
E. Sodium valproate.
32. An otherwise healthy 50-year-old male patient receiving clozapine 300mg at night for
treatment resistant schizophrenia suddenly dies. The MOST likely cause of death is?
A. Myocarditis
B. Neuroleptic malignant syndrome
C. Pneumonia
D. Serotonin syndrome
E. Tardive dyskinesia.

33. A 28-year-old woman has a borderline personality disorder, primarily manifested by an


inability to control her urges to hurt herself. She frequently cuts her wrists when
frustrated. Which of the following medications has been found to be MOST helpful and
safest in curbing her self-harm behaviour?
A.
B.
C.
D.
E.

Aripiprazole
Clonazepam
Fluoxetine
Lithium
Olanzapine.

Psychotherapy
34. A 30-year-old woman suffers from obsessive compulsive disorder and washes her hand
50 times a day. Which of the following techniques is the MOST essential in the
psychological treatment of her compulsive hand washing behaviour?
A.
B.
C.
D.
E.

Cognitive restructuring
Exposure and response prevention
Social skill training
Systemic desensitization
Thought stopping.

35. A!25TyearTold!woman!with!a!panic!disorder!seeks!an!opinion!whether!pharmacotherapy!or!
psychotherapy! is! the! best! treatment! option! for! her.! Which! of! the! following! statements! is!
CORRECT?
A. Adding alprazolam on p.r.n. basis will make exposure therapy ineffective.
B. Adding alprazolam in the first week will lead to dependence and does not improve patients
outcome.
C. Antidepressants such as selective serotonin reuptake inhibitors are not useful to treat panic
disorder.
D. Relaxation therapy is more effective than cognitive behaviour therapy in treating panic

!
!
124

disorder.
E. Psychotherapy such as cognitive behaviour therapy requires more motivation than taking
medication.
36. A!70TyearTold!man!lodged!a!complaint!against!the!hospital.!!His!wife!died!suddenly!one!
year!ago.!!She!suffered!from!diabetes!and!had!frequent!episodes!hypoglycaemia.!!The!
husband!gave!her!higher!than!prescribed!dose!of!hypoglycaemic!agent!because!he!thought!
this!could!control!her!diabetes.!!His!children!disagreed!with!his!management!but!he!
refused!to!listen.!!The!coroners!report!stated!that!his!wife!died!of!hypoglycaemia.!!The!70T
yearTold!man!is!angry!with!the!endocrinologist!and!thinks!that!it!was!his!fault!to!prescribe!
the!wrong!hypoglycamic!agent!leading!to!his!wifes!death.!!He!demanded!a!written!apology!
from!the!endocrinologist.!!Which!of!the!following!defence!mechanisms!is!demonstrated!by!
this!man?
A. Altruism
B. Displacement
C. Reaction formation
D. Splitting
E. Undoing.
37. For post-traumatic stress disorder, which of the following interventions is LEAST
helpful to patients?

A.
B.
C.
D.
E.

Cognitive behaviour therapy


Eye movement desensitization and reprocessing
Immediate group debriefing
Relaxation training
Supportive psychotherapy

38. You are a resident in plastic surgery. A 24-year-old woman believes her nose is
malformed and always checks her nose in the mirror. She was seen by multiple
doctors and they confirmed that her nose is normal. Which of the following
psychological treatment is the treatment of choice for this patient?
A.
B.
C.
D.
E.

Exposure and response prevention


Family therapy
Problem solving therapy
Psychodynamic psychotherapy
Supportive psychotherapy.

Ethics and laws

39. A! 40TyearTold! woman! complained! of! headache,! backache,! nausea,! numbness! and! visual!
problems.! ! She! consulted! a! neurologist! whom! she! trusted! to! be! a! good! doctor.! ! The!
neurologist! diagnosed! the! patient! suffering! from! migraine! and! somatisation! disorder.!!
When! the! patient! requested! for! Magnetic! Resonance! Imaging! (MRI),! the! neurologist!
claimed!that!the!patient!had!exaggerated!her!somatic!symptoms!and!insisted!that!the!MRI!

!
!
125

scan! should! be! reserved! for! more! severe! patients.! ! Six! months! later,! the! patient! died! of!
brain! tumour.! ! In! addition! to! negligence,! which! of! the! following! ethical! principles! was!
violated!by!the!neurologist?!

A.
B.
C.
D.
E.

!
Autonomy!
Confidentiality!
Fiduciary!duty!
NonNmaleficience!
Justice.!

40. 40-year-old man arrested for shoplifting is referred for a psychiatric assessment after
he tells the judge he suffers from kleptomania. Which of the following criteria is MOST
important in establishing the diagnosis of kleptomania?
A. Acts reus.
B. Patient informs you that he was diagnosed with kleptomania by a private psychiatrist before
but cannot provide further clinical details.
C. Patient informs you that he cannot control his impulse to steal and he does not want to steal.
D. Patient informs you that he has started stealing since Primary 6.
E. Past history of criminal record of stealing.
41. A medical student has applied for a 3-month research elective and he follows a
professor to conduct research. The professor asks him to conduct a cross-sectional
questionnaire study to assess mood symptoms of cancer patients. He is expected to
recruit 300 patients in 3 months. This project was approved by the ethics committee.
He attempts to obtain consent from an inpatient who has received chemotherapy to
participate in this study. Patient refuses because he feels very lethargic. Thirty minutes
later, he asks the patient to re-consider participating in this study again. The patient
lodges a complaint to the professor about this medical student. Which ethical
principles has the medical student violated?

A.
B.
C.
D.
E.

Autonomy!
Beneficence!
Confidentiality!
NonNmaleficence!
Justice.!

Liaison and neuropsychiatry


42. A 50-year-old man who is dependent on alcohol is admitted for observation after a fall when
intoxicated. The ward team is concerned about complications associated with alcohol withdrawal.
Which of the following statements is INCORRECT?
A. Delirium tremens usually occur 6-12 hours after the last drink.
B. Tremors usually occur 6-12 hours after the last drink.
C. Anxiety usually occurs 12-18 hours after the last drink.
D. Convulsion usually occurs 12-18 hours after the last drink.
E. Sleep disturbance usually occurs 12-18 hours after the last drink.

!
!
126

43. You are a resident in general surgery. A 50-year-old man was admitted last night
after he cut his head, arms and hands during a suicide attempt. He is married
with a 10-year-old child. He works in the field of informational technology.
According to his wife, his elder brother died of gastric cancer 6 months ago. He
believed that he is genetically vulnerable for gastric cancer. He saw multiple
gastroenterologists and gastric surgeons. He went through multiple endoscopies
and biopsies with normal findings. Despite normal findings, he still worries that
he has undiagnosed gastric cancer. He argued with his wife tonight and wanted to
commit suicide. What of the following is the MOST appropriate psychiatric
diagnosis?
A.
B.
C.
D.
E.

Acute stress reaction/marital problems


Depression/abnormal grief
Generalised anxiety disorder / phobia
Hypochondriasis/illness anxiety
Schizophrenia/psychosis.

44. You have become a consultant physician. A resident called you and informed you

that the parameter of a 20-year-old male psychiatric patient over the phone, His
temperature is 41 degree Celsius. His heart rate is 105 per minutes and blood
pressure fluctuates from 120/70 to 180/100. His breathing is normal and chest is
clear. His abdomen is soft. His creatinine kinase is 800 IU/L (normal <100 IU/L)
and leukocyte count is 10 x109/L (normal range: 1.5-5 x109/L). His limbs are very
rigid and he demonstrates difficulty in swallowing. He was given intravenous
injection of haloperidol 20mg three days ago. What is the MOST likely
diagnosis?
A.
B.
C.
D.
E.

Catatonia
Meningitis
Neuroleptic malignant syndrome
Serotonin syndrome
Status epilepticus.

45. You are a doctor working in the polyclinic. A 40-year-old man suffers from
bipolar disorder and he takes lithium on a daily basis. His blood pressure is
170/100mmHg. Which of the following medications is the BEST to lower his blood
pressure?
A.
B.
C.
D.
E.

Angiotensin-converting-enzyme inhibitor
Alpha agonist
Benzodiazapine
Beta blocker
Thiazide diuretic.

46. In medically ill patients with psychiatric illness, which pharmacological properties of a

!
!
127

psychotropic drug is MOST appropriate?


A.
B.
C.
D.
E.

Long half-life psychotropic drug.


Psychotropic drugs with active metabolites.
Psychotropic drug with wide therapeutic index.
Psychotropic drug which inhibits metabolic enzymes.
Psychotropic drug which induces metabolic enzymes.

!
2014!First!Rotation!MCQ!
History, mental state exam and psychopathology!

47. A! 50TyearTold! woman! presents! with! severe! The!answer!is!D.!


depressive! episode! with! psychotic! features.!
Which! of! the! following! delusions! is! LEAST! !
likely!to!occur!in!this!patient?!
Explanation:!Delusion!of!jealousy!is!
F. Deserved!punishment!
the!least!mood!congruent!as!
G. Guilt!
compared!to!other!options.!
H. Incurable!illnesses!
I. Jealousy!
!
J. SelfNdepreciation.!
Year:!2014!
!
48. A! 30TyearTold! prisoner! gives! approximate! The!answer!is!D.!
answers! to! questions.! For! example,! if! it! is!
Thursday,!he!will!say!it!is!Friday!and!claims!a! !
dog! has! three! legs.! This patient is MOST
Explanation:!The!patient!provides!
likely suffering from:
an!approximate!answer!which!is!a!
clinical!feature!of!Gansers!
F. Capgras syndrome
syndrome.!
G. Ekboms syndrome
H. Fregolis syndrome
I. Gansers syndrome
J. Othellos syndrome.

!
Option!A!and!C!are!misidentification!
syndrome.!Option!B!refers!to!
delusion!of!infestation.!Option!E!
refers!to!morbid!jealousy.!
!
Year:2014!

!
!
128

49. A psychiatric patient suddenly realises that he is


an interpreter for deaf people and tries to
interpret others speech in sign language. He
demonstrates strange repetitive movements.
His signs appear to come in threes or fours,
occasionally swinging his shoulders, as if he is
conveying a message. Which of the following is
the BEST term to describe his movements?

!
F.
G.
H.
I.
J.

Ambitendency!
Mannerism!
Negativism!
Stereotypies!
Waxy!flexibility.!

The!answer!is!B.!
!
Explanation:!Mannerism!is!a!goal!!
directed!repetitive!movement!(e.g.!a!
speaker!tries!to!move!his!hands!
repetitively!to!convey!his!
messages).!
!
This!mans!movements!seemed!to!
be!goal!directed!because!he!
attempted!to!show!sign!language!
but!not!understandable!to!deaf!
people.!

!
!

!
!
Year!2014!
Cognitive assessment!

50. A 33-year-old man with childhood developmental


delay and epilepsy received special education
and now works as a cleaner. He has worked in a
tofu factory for three years. He can only perform
simple routine work in the factory. His
supervisor reports his work performance is slow
and poor. His family says he cannot live
independently and demonstrated delay in
achievement in self-care since young. How
would you rate the level of mental retardation?
F.
G.
H.
I.
J.

Answer:!C!
!
Explanation:! This! case! is! more!
severe! than! mild! retardation! which!
demonstrates! more! competency! in!
selfNcare!and!independence.!
People! with! severe! or! profound!
mental! retardation! are! not! able! to!
work!in!the!factory.!

Very mild mental retardation


Mild mental retardation
Moderate mental retardation
Severe mental retardation
Profound mental retardation.

!
!
Year!2014!
!

Psychiatric epidemiology
51. A 65-year-old retired teacher is concerned about
dementia and has assessed online resources for
information. She asks which type of dementia is

!
!
129

The!answer!is!D.!

more common in Asians as compared to


Caucasians. Your answer is:
F.
G.
H.
I.
J.

!
Explanation:!Vascular!dementia!
(VaD)!is!found!to!be!more!common!
than!Alzheimers!disease!(AD)!in!
some!Asian!countries.!In!a!study!
conducted!by!National!
Neuroscience!Institute!(Singapore),!
53%!suffered!from!VaD!versus!47%!
from!AD.!The!ethnic!variation!is!due!
to!genetic!variation,!differences!in!
vascular!risk!factors!and!lifestyles.!!

Alzheimers disease
Lewy body dementia
Pseudodementia
Vascular dementia
Fronto-temporal lobe dementia.

!
Year:!2014!
Psychiatric aetiology, diagnosis and classification

52. A 20-year-old national serviceman was


diagnosed to suffer from adjustment
disorder with depressed mood. He read the
internet and found another disorder known
as dysthymic disorder. He wants to know
the
differences
between
these
two
disorders. The following statements about
differences between adjustment disorder with
depressed mood and dysthymia are true
EXCEPT:

P. Adjustment disorder with depressed mood


results from a known stressor, whereas
dysthymic disorder does not.
Q. Adjustment disorder with depressed mood
shows a genetic pattern, whereas
dysthymic disorder does not.
R. The duration of symptoms is shorter for
adjustment disorder with depressed mood
than it is for dysthymic disorder.
S. The co-existence of a major depressive
disorder with dysthymic disorder is called
double depression. There is no term for a
major depressive disorder complicating
adjustment disorder with depressed mood.
T. Treatment is usually shorter and does not
require the use of medication for adjustment
disorder with depressed mood as compared
to dysthymic disorder.

!
!
130

!
The answer is B.

Explanation:
Option A is correct and relevant for
this case.
Option B is incorrect because
dysthymic disorder is more likely
to show a genetic pattern.
Option C is true because
adjustment disorder begins within
three months of a stressor, and
ends within six months of the
stressor ending. Dysthymic
disorder has a duration of two
years in adults and one year in
children.
Option D is correct
Option E is correct.

Year: 2014

53. You are a family doctor. A male and a female


schizophrenia patient who stay in the
community have decided to get married and
have a child. What is the risk for their child to
develop schizophrenia in the future?

The correct answer is D.

Prevalence of Schizophrenia in
Specific Populations

F.16%
G. 26%
H. 36%
I. 46%
J. 56%.

Population
General Population
Non-twin sibling of a
schizophrenic patient
Child with one
schizophrenic parent
Dizygotic twin of a
schizophrenic patient
Child of two
schizophrenic parents
Monozygotic twin of a
schizophrenic patient
!

54. You!are!an!oncology!resident.!!A!60TyearTold!
oncology!patient!developed!severe!nausea!as!a!
sideTeffect!during!chemotherapy.!However,!after!
completing!treatment!she!continues!to!experience!
nausea!whenever!she!returns!to!the!hospital!for!
followTup.!Which!of!the!following!psychological!
theories!BEST!describes!her!experience?
F.
G.
H.
I.
J.

Biofeedback
Classical conditioning
Cognitive learning
Operant conditioning
Social learning.

Prevalence
(%)

1
9
13
14
46
46

Year: 2014

The!answer!is!B.!
Explanation:! ! This! phenomenon! is!
known! as! classical! conditioning.!
Chemotherapy! is! an! unconditional!
stimulus! (UCS)! and! the! hospital!
building! is! conditional! stimulus!
(CS).! The! conditional! (CR)! and!
unconditional! response! (UCR)!
include! nausea.! The! pairing! of! CS!
and! UCR! will! persist! even! UCS! (i.e.!
chemotherapy)!disappears.!
!
Year:!2014!
!

General adult psychiatry


55. A 25-year-old woman experiences episodes of
depersonalization, hyperventilation, dizziness,
and diaphoresis lasting for 15 minutes whenever
she arrives at work. She has been overwhelmed
by work and conflicts with her supervisor for the
past 3 months. There are no such episodes at
home or outside the office. What is the MOST

!
!
131

The!answer!is!C.!
!
Explanation:!the!history!and!
symptoms!suggest!the!diagnosis!of!

panic!disorder!because!the!anxiety!
symptoms!are!episodic!with!a!clear!
trigger.!

likely diagnosis?

!
A.!Acute!stress!disorder!

B.!Generalized!anxiety!disorder!

Year:!2014.!

C.!Panic!disorder!!
D.!PostNtraumatic!stress!disorder!
E.!Somatization!disorder.!

56. You are an orthopaedic resident managing a 24year-old motorcyclist who was involved in a
road traffic accident 3 days ago in which he
sustained multiple fractures and his pillion rider
died. He is irritable, sleeps poorly and has
nightmares and refuses to talk about the
accident. The MOST likely psychiatric diagnosis
is:

!
A.!Acute!stress!disorder!!

The!answer!is!A.!!
!
Explanation:!The!duration!of!
symptoms!is!only!2!days.!It!is!too!
early!to!conclude!whether!he!will!
develop!PostNTraumatic!Stress!
Disorder!(PTSD).!
!

B.!Adjustment!disorder!with!anxiety!

Year:!2014!

C.!Depressive!disorder!
D.!Generalized!anxiety!disorder!
E.!PostNtraumatic!stress!disorder.!

57. Based on the current research findings, which of


the following statements is CORRECT regarding
the prognosis for a 35-year-old Indonesian man
with schizophrenia who lives in a rural village
and treated by risperidone?
F. He has a worse prognosis as compared to
patients staying in an urban city.
G. His prognosis is poor because he will seek
treatment from traditional healer in the village.
H. He has a better prognosis as compared to patients
staying in an urban city.
I. His prognosis is the same as patients staying in in
an urban city
J. His prognosis is not related to the urban
development of the society where he resides.

!
!
132

Answer!is!C!
!
Explanation:! ! Schizophrenia! patients!
residing! in! the! rural! area! have! a! more!
favourable! prognosis! as! compared! to!
patients! staying! in! urban! or!
industrialized! areas.! In! rural! areas,!
there! is! less! likelihood! of! social!
isolation! and! a! greater! degree! of! social!
support.! ! There! is! more! emphasis! on!
social! and! familial! bonds! that! protect!
patients!
from!
social!
isolation.!
Furthermore,!it!is!easier!for!patients!to!

cope! with! workNrelated! tasks! in!


agricultural! or! village! settings.! There! is!
less!stigma!in!rural!areas.!!

!
Year:!2014.

!
58. You!are!a!resident!working!in!the!Accident!and!
Emergency!Department.!!A!39TyearTold!English!
man!was!married!to!a!Chinese!Singaporean!is!
brought!to!the!Emergency!Department!after!he!
attempted!to!hang!himself.!He!is!actively!suicidal!
and!has!marital!problems.!He!has!not!spoken!to!
his!wife!for!three!days.!The!hospital!does!not!have!
a!psychiatric!ward!nor!a!stayTin!psychiatric!team.!
What!is!the!MOST!appropriate!management!plan?!
F. Admit Peter to the medical ward of your hospital.
G. Discharge Peter from Emergency Department with
follow-up in the polyclinic to reduce the stigma.
H. Discharge Peter from Emergency Department with
an early follow-up in the psychiatric department.
I. Refer the couple for marital counselling by on-call
medical social worker.
J. Transfer Peter to IMH for assessment and consider
admission under Mental Health (Care and
Treatment) Act.

The!answer!is!E.!

59. A 40-year-old single man consulting you in a


general practice clinic reveals that since
adolescence he enjoys observing naked people
and those involved in sexual activities as it
helps him achieve sexual arousal. The
psychiatric condition associated with this
behaviour is:

The!answer!is!E.!

A. Exhibitionism
B. Fetishism
C. Sadomasochism
D. Transvestism

!
Explanation:!Hanging!is!considered!
to!be!a!dangerous!suicide!method!
and!this!patient!has!high!suicide!
risk.!As!he!is!not!forthcoming!and!
does!not!suffer!from!any!medical!or!
surgical!complication,!sending!him!
to!IMH!for!assessment!under!the!
Mental!Health!Act!is!the!safest!
option.!
!
Year:!2014!

!
Explanation:!Voyeurism!refers!to!
the!presence!of!recurrent!and!
intense!sexual!arousal!from!
observing!an!unsuspecting!person!
who!is!naked,!in!the!process!of!
disrobing!or!engaging!in!sexual!
activity!over!a!total!duration!of!6!
months.!
!

E. Voyeurism.

Year:!2014!!
60. A 65-year-old retired man consulting you in a
general practice clinic reveals that he has

!
!
133

The!answer!is!E.!

marital problems. His wife complains that he is


very fuzzy about cooking. He does not eat
outside nowadays. If he goes to a high class
restaurant, he expects that the steak must be
properly done and he is entitled to change the
steak if it is poorly done. Now he finds most
steak restaurants in Singapore are not up to his
standard. He claims it is better to cook by
himself. If the food is not up to his standard, he
has no one to blame. He likes to go to highclass club in town. He claims to be a special
member in the club due to his unlimited success
and donation.
Which of the following
personality traits BEST describes this man?

!
Explanation:!
!
He!
exhibits!
narcissistic! personality! trait.! He!
demonstrates!
of!
sense!
of!
entitlement!and!holds!belief!that!he!
is! better! than! the! others! (e.g.! chef!
from! all! steak! restaurants).! He!
preoccupies!with!unlimited!success.!
!
Year:!2014!

F.
G.
H.
I.
J.

Antisocial
Avoidant
Borderline
Histrionic
Narcissistic.

61. A! 30TyearTold! executive! complains! that! he! cannot!


fall! asleep! at! night.! ! Two! months! ago,! his! life! was!
interrupted! by! a! work! trip! to! the! United! States.!!
After! returning! to! Singapore,! he! can! only! falls!
asleep! at! 2! to! 3! a.m.! and! he! cannot! wake! up! on!
time! the! next! morning.! ! This! leads! to! negative!
impact! on! his! work.! ! What! is! the! MOST! likely!
diagnosis?!!
F. Circadian rhythm sleep disorder advanced sleep
phase
G. Circadian rhythm sleep disorder delayed sleep
phase
H. Excessive daytime sleepiness
I. Kleine-Levin syndrome
J. Random eye movement sleep disorder.

Answer is B.

Explanation: This patient exhibits


circadian rhythm sleep disorder delayed sleep phase because he
can only sleep after 2 to 3 a.m.
The advanced phase sleep
disorder is associated with early
onset of sleep and early morning
wakening.

Year: 2014
!

Old age psychiatry

62. A!65TyearTold!lady!with!bipolar!disorder!and!
severe!functional!decline!presents!with!threeTday!
history!of!acute!agitation,!suicidal!ideation!and!
refusing!to!eat!and!drink.!Her!MiniTMental!Status!
Examination!(MMSE)!score!was!29/30.!auditory!
hallucinations.!She!has!not!responded!to!sodium!
valproate!and!olanzapine.!!The!MOST!appropriate!
treatment!at!this!stage!is:!!!

F. Acetylcholinesterase!inhibitor!
G. Augmentation!with!antidepressant!
H. Augmentation!with!anxiolytics!
!
!
134

The!answer!is!E.!
!
Explanation:!ECT!seems!to!be!the!
best!treatment!because!she!is!
severely!depressed!with!psychotic!
features!and!suicidal!ideation.!Her!
MMSE!is!close!to!full!score!and!

I.
J.

Change!valproate!to!lithium!
Electroconvulsive!therapy.!

potential!cognitive!impairment!
should!not!be!a!relative!
contraindication!for!ECT.!On!the!
other!hand,!her!renal!impairment!is!
a!contraindication!for!lithium.!
!
Year:!2014!

63. You!are!public!health!doctor!and!work!in!the!
Ministry!of!Health.!The!Ministry!wants!to!
identify!protective!factors!against!the!
development!of!Alzheimers!disease.!Which!of!
the!following!is!least!associated!with!the!
development!of!Alzheimers!disease?!!!
F. Consuming!red!meat!on!a!frequent!basis!
G. Female!gender!
H. High!education!level!!
I. High!homocysteine!level!
J. Never!married.!

The!answer!is!C.!
!
Explanation:!
Risk!factors!for!Alzheimers!disease!
include!old!age,!family!history,!
ApoE4!genotype,!female!gender,!
hypertension,!diabetes,!never!
married,!low!education!level,!head!
trauma,!high!homocysteine!levels.!
!
Protective!factors!include!high!
levels!of!education,!APOE2,!
consuming!fish,!antiN
inflammatories,!statins$,$
bilingualism.!
!
Year:!2014!

64. You! are! a! member! of! the! Hospital! Quality! The!answer!is!D.!


Improvement! Committee.! The! hospital! wants! to!
improve! the! management! of! delirium.! Which! of! !
the! following! statements! regarding! delirium! is!
FALSE?!
Explanation:!The!typical!course!of!
delirium!last!for!10N12!days.!It!is!too!
early!to!expect!the!majority!of!delirious!
F. Around!30%!of!the!elderly!admitted!to!the!medical!
patients!recover!by!Day!3.!
ward!will!develop!delirium!during!hospitalisation.!
G. Delirium!is!often!missed!or!overlooked!by!health!care!
!
professionals!in!the!elderly.!
H. Delirium!occurs!in!10%!of!children!undergoing!day!
Year:!2014!
surgery.!
I. Delirium!usually!clears!within!3!days!of!correcting!
!
underlying!medical!cause.!

!
!
135

J.

The!incidence!of!delirium!increases!with!age.!

!
65. You are a geriatric doctor. The manager of a
nursing home wants to consult you the behavior
which is most likely correlated with delusions in
Alzheimers disease. Your answer is:
F.
G.
H.
I.
J.

Aggression and agitation


Disrobing (i.e.state of wearing of no clothes)
Shadowing
Suicide
Wandering.

The!answer!is!A.!
!
Explanation:!Patients!with!severe!
Alzheimers!disease!often!suffer!
from!delusion!of!theft!and!this!
correlates!with!aggression!and!
agitation!to!family!members!and!
domestic!helpers.!
!
Year:!2014!

!
66. You! are! a! paediatric! resident.! You! are!
concerned! about! the! mental! health! of! a! 50T
yearTold! mother! whose! 12TyearTold! daughter!
died! of! leukaemia.! You! are! not! certain!
whether! she! suffers! from! normal! grief! or!
pathological! grief.! Normal! grief! is! NOT!
characterized!by:!!
!
F.
G.
H.
I.
J.

Denial!
Recurrent!suicidal!thoughts!!
Searching!for!the!deceased!
Sensations!of!seeing!or!hearing!the!deceased!
Transient!guilt.!

The!answer!is!B.!
!
Explanation:!Recurrent!suicidal!
thoughts!are!considered!to!be!
pathological!grief.!
!
Year:!2014!

67. You are a geriatric resident reviewing a 75-yearold woman with Alzheimers disease. Her family
is keen for her to take acetylcholinesterase
inhibitor. Which of the following medical
conditions is MOST contraindicated?
F.
G.
H.
I.
J.

Anaemia
Asthma
Diabetes
Hyperthyroidism
Rheumatoid arthritis.

The!answer!is!B.!
!
Explanation:!Acetylcholinesterase!
inhibitors!increase!cholinergic!
functions!and!may!cause!spasm!in!
smooth!muscle!or!bronchospasm.!
As!a!result,!it!may!worsen!asthma.!
!

!
!
136

Year:!2014!

!
!

Child and adolescent psychiatry

68. Which of the following is the COMMONEST


cardiac complication in a 14-year-old
adolescent suffering from anorexia nervosa
on admission?
F.
G.
H.
I.
J.

Atrial fibrillation and cardiac arrhythmia.


Bradycardia and hypertension
Bradycardia and hypotension
Tachycardia and hypotension
Tachycardia and hypertension.

The!answer!is!C.!!
!
Explanation:!Bradycardia!and!
hypotension!are!the!most!common!
cardiac!complications!when!
patients!suffering!from!anorexia!
nervosa!are!admitted!to!the!ward.!
!
Year:!2014!

!
!

Substance abuse

69. A! 30TyearTold! man! admitted! to! the! medical!


ward!has!no!access!to!the!substance!which!he!
used! prior! to! admission.! He! complains! of!
nausea!
and!
muscle!
pain.!
Physical!
examination!reveals!pupillary!dilation.!Which!
of! the! following! substances! is! MOST! likely! to!
cause!the!above!withdrawal!symptoms?!
!
F. Alcohol!
G. Cocaine!
H. Midazolam!
I. Nictoine!
J. Opiate.

The!answer!is!E.!
!
Explanation:!Opiate!withdrawal!
leads!to!mood!changes,!
gastrointestinal!disturbances!(e.g.!
nausea,!vomiting),!muscular!aches,!
lacrimation!and!pupillary!dilation.!
!
Year:!2014!

!
70. A female student has returned from Amsterdam
after an exchange programme and admits using
cocaine for 1 year. She developed psychotic
symptoms. Which of the following is the MOST
IMPORTANT risk factor for the development of
psychosis while using cocaine?

F. An!elevated!body!mass!index!
G. Combination!with!benzodiazepine!
H. Being!a!first!time!user!of!cocaine!
!
!
137

The!answer!is!C.!
!
Explanation:!The!risk!factors!for!
psychosis!in!cocaine!users!include:!
male!gender,!intravenous!users,!
first!time!users,!longer!duration!and!
greater!amount!of!cocaine!use,!low!

I.
J.

Being!female!
NonNintravenous!use.!

BMI.!
!
Year:!2014!

!
This!is!an!advanced!level!question.!
71. A!30TyearTold!man!who!is!dependent!on!alcohol!

enquires!of!his!family!physician!if!disulfiram!will!
help!him!to!reduce!his!alcohol!intake!while!he!still!
drinks!in!a!controlled!manner.!!Which!of!the!
following!recommendations!is!CORRECT?!

!
F. Disulfiram!is!recommended!because!it!has!more!
evidence!than!other!medications!in!helping!
patients!to!quit!alcohol.!
G. Disulfiram!is!recommended!because!it!will!
discourage!him!from!drinking!alcohol.!
H. Disulfiram!is!recommended!because!he!has!
motivation!to!reduce!his!drinking.!
I. Disulfiram!is!not!recommended!because!it!may!
cause!severe!side!effects!when!it!mixes!with!
alcohol.!
J. Disulfiram!is!not!recommended!because!it!is!
indicated!for!opioid!dependence.!

The!answer!is!D.!
!
Explanation:!!This!patient!does!not!
express!motivation!to!quit!drinking!
or!remain!abstinent!with!alcohol.!He!
wants!to!reduce!alcohol!intake!by!
controlled!drinking.!It!is!a!
dangerous!situation!if!he!drinks!and!
takes!disulfiram!at!the!same!time.!
!
!
Year:!2014!

!
!

Psychopharmacology

72. A!50TyearTold!man!with!a!major!depressive!
disorder!did!not!respond!well!to!an!adequate!trial!
of!sertraline!200mg!(a!selective!serotonin!
reuptake!inhibitor).!!Neither!did!he!fully!respond!
to!a!subsequent!8Tweek!trial!of!venlafaxine!150mg!
(serotonin!noradrenaline!reuptake!inhibitor).!!
Which!of!the!following!is!NOT!RECOMMENDED!for!
this!man?!!!

!
F. Augment!SSRI!with!lithium!(a!mood!stabilizer)!
G. Augment!SSRI!with!phenelzine!(a!monoaxmine!
oxidase!inhibitor)!
H. Consider!electroconvulsive!therapy!
I. Consider!psychotherapy!
J. Increase!the!dose!of!venlafaxine.!!

The!answer!is!B.!
!
Explanation:!Option!B!may!lead!to!
serotonin!syndrome.!
!
Year:!2014.!

The answer is B.
73. Bupoprion differs from SSRI in which of the

!
!
138

following ways?
K.

Bupoprion has a higher incidence of sexual


side effects and a lower incidence of
sedation and weight gain.
L. Bupoprion has a lower incidence of sexual
side effects and a lower incidence of
sedation and weight gain.
M. Bupoprion has a lower incidence of sexual
side effects and higher incidence of
sedation and weight gain
N. Bupoprion has lower incidence of sexual
side effects and weight gain and a higher
incidence of sedation.
O. Bupoprion has a higher incidence of sexual
side effects and sedation and a lower
incidence of weight gain.
74. Besides treatment resistant schizophrenia,
another appropriate indication for clozapine is?

Explanation: Bupoprion has a lower


incidence of sexual side effects (not
working on serotonin receptors) and a
lower incidence of sedation and
weight gain (as an energetic
antidepressant).

!
Year:!2014.!

The!answer!is!E.!
!

F.
G.
H.
I.
J.

Severe agranulocytosis
Severe embolism
Severe hypersalivation
Severe metabolic syndrome
Severe tardive dyskinesia.

Explanation:!Severe!tardive!
dyskinesia!is!an!indication!for!
clozapine.!Option!A,!B,!C!and!D!are!
side!effects!associated!with!
clozapine.!

!
Year:!2014.!
75. A!50TyearTold!cardiac!patient!suffers!from!
depression!and!the!psychiatrist!prescribed!
fluoxetine.!Which!of!the!following!statements!
about!fluoxetine!is!TRUE?!
!
F. !Fluoxetine!decreases!warfarin!levels.!
G.!!!!!Fluoxetine!increases!metabolism!of!warfarin.!
H. !!!!!Fluoxetine!increases!bruising!while!patient!is!
taking!
!!!!!!!!!!!!!!!!warfarin.!
I. He!has!no!increased!risk!of!gastrointestinal!
bleeding!with!fluoxetine.!
J. !!!!He!should!not!change!to!paroxetine!because!
paroxetine!decreases!anticoagulant!effect!of!
warfarin.!
!

The!answer!is!C.!
!
Explanation:!Fluoxetine!has!been!
reported!to!produce!bleeding!and!
bruising!in!some!individuals.!SSRIs!
as!a!class!are!believed!to!inhibit!
platelet!aggregation,!which!may!
underlie!this!effect.!The!ability!of!
SSRIs!to!reduce!platelet!aggregation!
may!be!an!important!intervention!
for!patients!with!occlusive!coronary!
and!cerebrovascular!artery!disease!
and!deserves!study.!
!

!
!
139

In!addition,!warfarin!is!metabolized!
via!CYP!3A4!and!fluoxetine!inhibits!
CYP!3A4!and!increases!the!levels!of!
warfarin.!
!
Year:!2014.!
!
This!is!an!advanced!level!question.!
76. Which of the following hypnotic agents causes
the LEAST daytime sedation is:

The!answer!is!E.!
!

F.
G.
H.
I.
J.

Diphenhydramine
Diazepam
Lorazepam
Zopiclone
Zolpidem.

Explanation:!The!hypnotic!drug!
with!the!least!half!life!have!least!day!
time!sedation.!
!
Diphenhydramine!(halfNlife):!9N12!
hours!
Diazepam:!20N100!hours!
Lorazepam:!9N16!hours!
Zopiclone:!6!hours!
Zolpidem!2N3!hours.!
!

Year:!2014.!

This!is!an!advanced!level!question.!
77. Which of the following medications has the most
convincing evidence for reducing suicidal
behavior in bipolar patients?
F.
G.
H.
I.
J.

The!answer!is!C.!
!
Explanation:!Lithium!reduces!
suicidal!ideation!in!bipolar!patients.!

Clozapine
Lamotrigine
Lithium
Olanzapine
Sodium valproate.

!
!
140

Year:!2014.!
!
78. An otherwise healthy 50-year-old male patient
receiving clozapine 300mg at night for treatment
resistant schizophrenia suddenly dies. The
MOST likely cause of death is?

The!answer!is!A.!
!

B. Neuroleptic malignant syndrome

Explanation:!Myocarditis!occur!in!1!
in!1300!patients!and!causes!death!
by!loss!of!right!ventricular!function.!

C. Pneumonia

Year:!2014.!

D. Serotonin syndrome

A. Myocarditis

E. Tardive dyskinesia.

79. A 28-year-old woman has a borderline


personality disorder, primarily manifested by an
inability to control her urges to hurt herself. She
frequently cuts her wrists when frustrated.
Which of the following medications has been
found to be MOST helpful and safest in curbing
her self-harm behaviour?
F.
G.
H.
I.
J.

Aripiprazole
Clonazepam
Fluoxetine
Lithium
Olanzapine.

The!answer!is!C.!
!
Explanation:!Fluoxetine!is!the!first!
line!treatment!for!impulsivity,!mood!
lability!and!rejection!sensitivity!in!
borderline!personality!disorder.!
!
Year:!2014.!
!
!

Psychotherapy
80. A 30-year-old woman suffers from obsessive
compulsive disorder and washes her hand 50
times a day. Which of the following techniques
is the MOST essential in the psychological
treatment of her compulsive hand washing
behaviour?
F.
G.
H.
I.
J.

Cognitive restructuring
Exposure and response prevention
Social skill training
Systemic desensitization
Thought stopping.

The!answer!is!B.!
!
Explanation:!Exposure!and!
response!prevention!is!most!
important!behavioural!treatment!
for!OCD.!Systemic!desensitization!is!
indicated!for!phobia.!
!

!
!
141

Year:!2014.!
81. A!25TyearTold!woman!with!a!panic!disorder!seeks!
an! opinion! whether! pharmacotherapy! or!
psychotherapy! is! the! best! treatment! option! for!
her.! Which! of! the! following! statements! is!
CORRECT?
F. Adding alprazolam on p.r.n. basis will make
exposure therapy ineffective.
G. Adding alprazolam in the first week will lead to
dependence and does not improve patients
outcome.
H. Antidepressants such as selective serotonin
reuptake inhibitors are not useful to treat panic
disorder.
I. Relaxation therapy is more effective than cognitive
behaviour therapy in treating panic disorder.
J. Psychotherapy such as cognitive behaviour therapy
requires more motivation than taking medication.

The!answer!is!E.!
!
Explanation:!ShortNterm!
benzodiazepine!is!useful!and!does!
not!cause!dependence.!!SSRI!is!
useful!to!treat!panic!disorder.!
Relaxation!therapy!can!be!part!of!
the!behaviour!therapy!of!CBT.!
!
Psychotherapy!requires!more!
motivation!from!patient!as!patient!
needs!to!cooperate!with!
psychologist!and!practise!various!
behavioural!techniques!and!
challenge!cognitive!errors.!!It!
requires!12N16!weekly!or!
fortnightly!sessions.!
!
Year:!2014.!
!!

82. A!70TyearTold!man!lodged!a!complaint!against!the!
hospital.!!His!wife!died!suddenly!one!year!ago.!!She!
suffered!from!diabetes!and!had!frequent!episodes!
hypoglycaemia.!!The!husband!gave!her!higher!than!
prescribed!dose!of!hypoglycaemic!agent!because!
he!thought!this!could!control!her!diabetes.!!His!
children!disagreed!with!his!management!but!he!
refused!to!listen.!!The!coroners!report!stated!that!
his!wife!died!of!hypoglycaemia.!!The!70TyearTold!
man!is!angry!with!the!endocrinologist!and!thinks!
that!it!was!his!fault!to!prescribe!the!wrong!
hypoglycamic!agent!leading!to!his!wifes!death.!!He!
demanded!a!written!apology!from!the!
endocrinologist.!!Which!of!the!following!defence!
mechanisms!is!demonstrated!by!this!man?
F. Altruism
G. Displacement
H. Reaction formation
I. Splitting
J. Undoing.

!
!
142

The!answer!is!B.!
!
Explanation:!!
Displacement!refers!to!transferring!
the!emotional!response!to!a!
particular!person!which!carries!less!
emotional!risk.!!
!
In!this!case,!the!endocrinologist!
bear!less!emotional!risk!than!his!
children!who!are!angry!with!him.!He!
does!not!want!to!blame!himself.!He!
displaced!his!guilt!of!causing!his!

wife!death!to!the!endocrinologist.!
!
Year:!2014.!
!
This!is!an!advanced!level!question.!
83. For post-traumatic stress disorder, which of the
following interventions is LEAST helpful to
patients?

F.
G.
H.
I.
J.

Cognitive behaviour therapy


Eye movement desensitization and reprocessing
Immediate group debriefing
Relaxation training
Supportive psychotherapy

The!answer!is!C.!
!
Explanation:!!Group!debriefing!may!
be!harmful!based!on!recent!
research!findings.!
!
Year:!2014.!
!

84. You are a resident in plastic surgery. A 24year-old woman believes her nose is
malformed and always checks her nose in the
mirror. She was seen by multiple doctors and
they confirmed that her nose is normal.
Which of the following psychological
treatment is the treatment of choice for this
patient?
F.
G.
H.
I.
J.

Exposure and response prevention


Family therapy
Problem solving therapy
Psychodynamic psychotherapy
Supportive psychotherapy.

The!answer!is!A.!
!
Explanation:!The!current!guidelines!
recommend!offering!cognitive!
behaviour!therapy!(including!
exposure!response!prevention)!to!
people!suffering!from!body!
dysmorphic!disorder.!Exposure!and!
response!prevention!is!part!of!the!
cognitive!behaviour!therapy.!
!
Year:!2014!

Ethics and laws

85. A! 40TyearTold! woman! complained! of! headache,! The!answer!is!C!

backache,!nausea,!numbness!and!visual!problems.!!
She! consulted! a! neurologist! whom! she! trusted! to! !
be! a! good! doctor.! ! The! neurologist! diagnosed! the!
patient! suffering! from! migraine! and! somatisation! Explanation:!Fidiciary!duty!refers!to!
disorder.! ! When! the! patient! requested! for!

!
!
143

Magnetic! Resonance! Imaging! (MRI),! the!


neurologist! claimed! that! the! patient! had!
exaggerated! her! somatic! symptoms! and! insisted!
that! the! MRI! scan! should! be! reserved! for! more!
severe!patients.!!Six!months!later,!the!patient!died!
of!brain!tumour.!!In!addition!to!negligence,!which!
of!the!following!ethical!principles!was!violated!by!
the!neurologist?!

!
F. Autonomy!
G. Confidentiality!
H. Fiduciary!duty!
I. NonNmaleficience!
J. Justice.!

the!duty!that!a!doctor!(i.e.!
neurologist)!must!act!in!the!
patients!best!interest.!The!
neurologist!is!in!a!legal!contract!
with!the!patient!to!provide!best!
care.!A!fiduciary!duty!exists!when!
the!late!patient!places!confidence!in!
the!doctor!and!relied!upon!him!to!
exercise!his!expertise!in!diagnosing!
and!managing!her!illness.!
In!this!case,!the!neurologist!failed!to!
offer!earlier!MRI!scan!despite!
patients!request.!!He!wrongly!
attributed!the!neurological!
symptoms!to!somatisation!disorder!
without!thorough!investigation.!
!
Year:!2014.!
!
This!is!an!advanced!level!question.!

86. 40-year-old man arrested for shoplifting is


referred for a psychiatric assessment after he
tells the judge he suffers from kleptomania.
Which of the following criteria is MOST
important in establishing the diagnosis of
kleptomania?

The answer is C.

F. Acts reus.
G. Patient informs you that he was diagnosed with
kleptomania by a private psychiatrist before but
cannot provide further clinical details.
H. Patient informs you that he cannot control his
impulse to steal and he does not want to steal.
I. Patient informs you that he has started stealing
since Primary 6.
J. Past history of criminal record of stealing.

Option Act reus refers to a criminal act


which involves bodily movement
whether voluntary or involuntary. In
this case, it refers to the act of stealing
and its presence suggests that he
indeed
committed
shoplifting.
Similarly, Option D and E suggest that
he had mostly likely committed a
crime as he has tendency to steal and
is not a honest person.

Explanation:

Option B is a common reason


provided by patients who committed

!
!
144

shopkifting but we need further clinical


details to establish the diagnosis of
kleptomania.

Year:!2014.!
!
87. A medical student has applied for a 3-month
research elective and he follows a professor to
conduct research. The professor asks him to
conduct a cross-sectional questionnaire study
to assess mood symptoms of cancer patients.
He is expected to recruit 300 patients in 3
months. This project was approved by the
ethics committee. He attempts to obtain consent
from
an
inpatient
who
has
received
chemotherapy to participate in this study.
Patient refuses because he feels very lethargic.
Thirty minutes later, he asks the patient to reconsider participating in this study again. The
patient lodges a complaint to the professor
about this medical student.
Which ethical
principles has the medical student violated?

F.
G.
H.
I.
J.

Autonomy!
Beneficence!
Confidentiality!
NonNmaleficence!
Justice.!

The!answer!is!A.!
!
Explanation:!Autonomy!refers!to!the!
obligation! of! a! doctor! to! respect! a!
patients! rights! to! make! his! or! her!
own! choice! in! treatment! or!
participation! in! research! in!
accordance!with!his!or!her!beliefs!or!
preferences.!The!patient!has!refused!
to! participate! and! the! decision!
should!be!respected!without!further!
asking.!
!
Year:!2014.!
!
!

Liaison and neuropsychiatry


88. A 50-year-old man who is dependent on alcohol is
admitted for observation after a fall when intoxicated.
The ward team is concerned about complications
associated with alcohol withdrawal. Which of the
following statements is INCORRECT?
F. Delirium tremens usually occur 6-12 hours after
the last drink.
G. Tremors usually occur 6-12 hours after the last
drink.
H. Anxiety usually occurs 12-18 hours after the last
drink.
I. Convulsion usually occurs 12-18 hours after the last
drink.
J. Sleep disturbance usually occurs 12-18 hours after
the last drink.

!
!
145

The!answer!is!A.!
!
Explanation:!Delirium!tremens!
usually!occur!3N4!days!after!the!last!
drink.!
!
Year:!2014.!
!

89. You are a resident in general surgery. A 50year-old man was admitted last night after he
cut his head, arms and hands during a
suicide attempt. He is married with a 10year-old child. He works in the field of
informational technology. According to his
wife, his elder brother died of gastric cancer
6 months ago. He believed that he is
genetically vulnerable for gastric cancer. He
saw multiple gastroenterologists and gastric
surgeons. He went through multiple
endoscopies and biopsies with normal
findings. Despite normal findings, he still
worries that he has undiagnosed gastric
cancer. He argued with his wife tonight and
wanted to commit suicide. What of the
following is the MOST appropriate
psychiatric diagnosis?
F.
G.
H.
I.
J.

Acute stress reaction/marital problems


Depression/abnormal grief
Generalised anxiety disorder / phobia
Hypochondriasis/illness anxiety
Schizophrenia/psychosis.

90. You have become a consultant physician. A

resident called you and informed you that the


parameter of a 20-year-old male psychiatric
patient over the phone, His temperature is
41 degree Celsius. His heart rate is 105 per
minutes and blood pressure fluctuates from
120/70 to 180/100. His breathing is normal
and chest is clear. His abdomen is soft. His
creatinine kinase is 800 IU/L (normal <100
IU/L) and leukocyte count is 10 x109/L
(normal range: 1.5-5 x109/L). His limbs are
very rigid and he demonstrates difficulty in
swallowing. He was given intravenous
injection of haloperidol 20mg three days
ago. What is the MOST likely diagnosis?

!
The!answer!is!D.!
!
Explanation:!This!patient!suffers!
from!hypochondriasis!or!illness!
anxiety!disorder!because!he!worries!
(not!firmly!believes!as!in!delusion)!
that!he!suffers!from!gastric!cancer!
but!not!confirmed!by!multiple!
investigations.!
!
There!are!not!enough!symptoms!to!
support!the!diagnosis!of!
schizophrenia!and!abnormal!grief.!
!
Year:!2014.!

The!answer!is!C.!
!
Explanation:!This!young!man!suffers!
from!neuroleptic!malignant!
syndrome!as!evidenced!by!high!
fever,!muscle!rigidity,!tachycardia,!
labile!blood!pressure,!leucocytosis!
and!laboratory!evidence!of!muscle!
injury!(i.e.!increase!in!creatinine!
kinase!levels)!after!intravenous!
antipsychotic!injection!three!days!
ago.!
!

F. Catatonia
G. Meningitis
!
!
146

H. Neuroleptic malignant syndrome


I. Serotonin syndrome
J. Status epilepticus.

91. You are a doctor working in the polyclinic. A


40-year-old man suffers from bipolar
disorder and he takes lithium on a daily
basis. His blood pressure is 170/100mmHg.
Which of the following medications is the
BEST to lower his blood pressure?

The!answer!is!D.!

F.
G.
H.
I.
J.

Year:!2014!

Angiotensin-converting-enzyme inhibitor
Alpha agonist
Benzodiazapine
Beta blocker
Thiazide diuretic.

!
Explanation:!Thiazide!diuretics,!ACE!
inhibitors!and!NSAIDS!increase!
lithium!level!and!may!cause!lithium!
toxicity.!
!
Option!B!and!C!do!not!reduce!blood!
pressure.!
!
The!best!answer!is!Option!D.!
!
Year!2014.!!
!
This!is!an!advanced!level!question!

92. In medically ill patients with psychiatric illness,


which pharmacological properties of a
psychotropic drug is MOST appropriate?
F.
G.
H.
I.
J.

Long half-life psychotropic drug.


Psychotropic drugs with active metabolites.
Psychotropic drug with wide therapeutic index.
Psychotropic drug which inhibits metabolic
enzymes.
Psychotropic drug which induces metabolic
enzymes.

The answer is C.

Explanation: Narrow therapeutic


index like lithium is dangerous to
medically ill patients.

Year 2014

!
!

!
!
147

!
!
!
Paper 6 - Questions
Psychopathology
1. A 50-year-old woman claims that unfamiliar people whom she met on the street were
her husband. She believes that her husband put on a disguise. What is the diagnosis?
A.
B.
C.
D.
E.

Capgras syndrome
Charles de Bonnet syndrome
DeClerambaults syndrome
Fregoli syndrome
Ganser syndrome.

2. A 50-year-old woman hears her late mothers voice when she falls asleep. She cannot
recall the content of her mothers speech. This phenomenon is known as:
A.
B.
C.
D.
E.

Auditory illusion
Hypnopompic hallucination
Hypnagogic hallucination
Pseudohallucination
Third-person auditory hallucination.

3. You are seeing a 70-year-old man who suffers from Parkinsons disease. He
demonstrates a dulled emotional tone and seems to be indifferent to jokes. His family
describes him as being emotionally detached. Which of the following terms BEST
describes his psychopathology?
A.
B.
C.
D.
E.

Alexithymia
Alogia
Anhedonia
Apathy
Attentional deficit.

4. A 50-year-old Singaporean Chinese man firmly believes that he needs to flee from
Singapore to Russia for his personal safety. He has developed this belief after reading a
newspaper article about Edward Snowden who released national security materials of
the United States (US). This man firmly believes that the newspaper mentions his name
and describes about his role in the leak of US national secrets. Which of the following
terms BEST describe his experience?
A. Delusion of erotomania
B. Delusion of grandiosity

!
!
148

C. Delusion of guilt
D. Delusion of jealousy
E. Delusion of reference.
Cognitive assessment
5. You are administering the Mini-Mental State Examination (MMSE) to a 70-year-old
woman who did not receive any formal education. She has difficulty to complete the full
version of MMSE. Which of the following tasks is MOST useful to assess her attention?

A.
B.
C.
D.
E.

Constructing a double pentagon


Naming the months from December in backward direction
Orientation to time and place
Three-item registration and recall
Three-stage command.

6. The Montreal Cognitive Assessment (MOCA) is different from the Mini-Mental State
Examination. Which of the following neuroanatomical structures is specifically
assessed in the MOCA but not the MMSE?
A.
B.
C.
D.
E.

Frontal lobe
Hippocampus
Occipital lobe
Parietal lobe
Temporal lobe.

Psychiatric epidemiology / Aetiology


7. After 65 years of age, the rate of Alzheimers disease changes approximately by how
many times every 5 years increase in age?
A. 0.5 time
B. 1 time
C. 2 times
D. 3 times
E. 4 times.
8. Lesions in which of the following neuroanatomical areas is the MOST vulnerable to
amnestic syndrome?
A.
B.
C.
D.
E.

Basal ganglia
Cerebellum
Hippocampus
Nucleus accumbens
Raphe nucleus.

!
!
149

General adult psychiatry / Psychiatric diagnosis


9. Which of the following psychiatric diagnoses is MOST likely to contribute to inpatient
suicide?
A.
B.
C.
D.
E.

Borderline personality disorder


Major depressive disorder
Generalised anxiety disorder
Hypochondriasis/ illness anxiety disorder
Schizophrenia.

10. Which of the following features is MOST characteristic of Cotards syndrome?


A.
B.
C.
D.
E.

Delusional belief that familiar people have been replaced by their imposters
Delusional belief that the spouse is unfaithful
Delusional belief that strangers have taken on the psychological identity of a familiar person
Delusional belief that a person of higher status falls in love with the patient
Nihilistic and hypochondriacal delusions found in psychotic depression.

11. A 23-year-old man, recently promoted to a middle management position, presents to


your clinic with complaints of acute episodes of anxiety associated with heart
palpitations, dyspnoea, shaking, detachment from his body, and fear of losing control.
These experiences only occur during oral presentation in front of the upper
management. Which of the following diagnoses is MOST likely?
A.
B.
C.
D.
E.

Generalized anxiety disorder


Panic disorder
Panic disorder with agoraphobia
Post-traumatic stress disorder
Social phobia.

12. A 40-year-old woman, Ms. Tan has witnessed a road traffic accident. Four hours ago,
an elderly man was knocked down by a taxi and suffered from severe head injury. Ms.
Tan was walking along the street and witnessed the whole accident. She did not suffer
from any injury. The police officer wants to interview Ms. Tan to obtain more
information about the accident. She cannot recall any details related to the accident.
Which of the following is the MOST appropriate explanation for her amnesia?
A.
B.
C.
D.
E.

Acute and transient psychosis


Conversion disorder
Dissociative state
Factitious disorder
Underlying dementia.

13. You are a medical officer working in the Army Camp. A 19-year-old man is recently
enlisted for National Service and his parents inform you that he walks around his flat
during sleep. Which of the following recommendations is LEAST appropriate?

!
!
150

A.
B.
C.
D.
E.

Safety measures
Sleep hygiene
Stay out of camp at night
Stimulant
Supportive psychotherapy.

14. A 25-year-old man believes that he suffers from adult ADHD. Which of the following
criteria is compulsory to establish such diagnosis?
A.
B.
C.
D.
E.

Absence of inattention symptoms in adulthood


History of ADHD symptoms before the age of 12
History of conduct disorder
Presence of hyperactivity symptoms in adulthood
History of substance abuse.

15. All of the following findings are found in anorexia nervosa EXCEPT:
A.
B.
C.
D.
E.

Decreased bone mineral density


Decreased growth hormone
Decreased gonadotropins
Decreased potassium
Decreased heart rate.

Old age psychiatry


16. You are a family doctor and a 70-year-old man consults you for annual medical
assessment to certify that he is fit to drive. Which of the following is LEAST relevant to
driving?
A.
B.
C.
D.
E.

Assess his visual acuity by Snellens Chart


Assess his hearing by whispering to his ears
Inquire his driving practice and road safety records
Examine his elbow and knee joints
Measure his blood pressure.

17. Which of the following is TRUE about suicide in old people?


A. Old people with terminal illness have low suicide risk because they foresee that they will die
soon.
B. Old people often attempt suicide as a means to manipulate other people.
C. Old people frequently communicate their intent prior to committing suicide.
D. Old people generally attempt self-harm rather than suicide.
E. Suicide attempts are more lethal in old people as compared to young people.
18. Which of the following statements is FALSE about delirium in old people?
A. Could be an initial presentation of underlying dementia

!
!
151

B. Onset could be insidious


C. Rarely results in full resolution of symptoms in a short period of time
D. Un-medicated old people are at higher risk to develop delirium as compared to old people
treated by multiple medications.
E. Usually persists for weeks or months in old people hospitalized for medical or surgical
reasons.

19. The daughter of an 80-year-old woman with history of bipolar disorder calls you for
advice. Her mother assaulted one helper and the other helper is terrified. They tried to
stop her from leaving the house to buy things. Which of the following is MOST
appropriate management strategy?
A.
B.
C.
D.
E.

Admission to the psychiatry ward


Employ a part-time nurse to look after her at home
Increase the dose of antipsychotics and review in 1 week
Increase the dose of benzodiazepine and review in1 week
Increase the dose of mood stabilizer and review in 1 week.

Child and adolescent psychiatry


20. A 4-year-old boys grandmother died four months ago. The grandmother was one of the
childs primary caregivers. The child still speaks to the grandmother. When he walks, he
still holds her hands up as though holding hands with his grandmother. Which of the
following is CORRECT?
A.
B.
C.
D.
E.

This child fully understands the concept of death.


This child suffers from major depression.
This child is psychotic.
The child suffers from adjustment disorder with depressed mood.
This reaction is within normal limits for a child of this age.

21. Which of the following is the MOST established psychopharmacological treatment for
children and adolescents suffering from autism?
A.
B.
C.
D.
E.

Actetylcholinesterase inhibitor
Antipsychotic drug
Benzodiazepine
Omega-3 supplement
Stimulant.

Substance abuse
22. You are a resident working at the Accident and Emergency Department. A 21-year-old
man is brought in by police and he is disorientated to time, place and person. He needs
to urinate very often and seems to have urinary incontinence. A package of capsules is
found in his pocket and suspected to be illicit drugs. Which of the following drugs is
MOST likely to cause the above symptoms?

!
!
152

A.
B.
C.
D.
E.

Amphetamine
Cannabis
Cocaine
Ketamine
Phencyclidine.

23. Hypertension associated with cocaine misuse is BEST treated by:

A.
B.
C.
D.
E.

AngiotensinNconvertingNenzyme!inhibitors
Beta-blockers
Benzodiazapine
Loop diuretics
Thiazide diuretics.

24. Which of the following addiction has the HIGHEST suicide risk?
A, Gambling
B. Internet
C. Sex
D. Steroid addiction in young athlete
E. Stimulant.

25. You are a general practitioner (GP). A 20-year-old university student wants you to prescribe
methylphenidate (ritalin). The student believes that ritalin can enhance his examination
performance and he has been obtaining Ritalin prescribed from another GP. He does not
have history of attention deficit and hyperactivity disorder (ADHD). Which of the following
advices is MOST APPROPRIATE?
A. Advise him to take methylphenidate on the days of examination and stops immediately after
the examination.
B. Advise him to stop taking methylphenidate because it is a controlled drug and not indicated in
his situation.
C. Methylphenidate can enhance his attention if he has family history of ADHD.
D. Methylphenidate can enhance his attention if he has past history of conduct disorder.
E. There is evidence to suggest that methylphenidate can enhance his attention and improve his
examination performance.
Psychopharmacology
26. A 30-year-old man suffers from autism and epilepsy. He has been very aggressive. The
psychiatrist prescribes sodium valproate 1300mg nocte to control his aggression and
epilepsy. His mother concludes the sodium valproate is not effective. The first step of

!
!
153

management should be:


A.
B.
C.
D.
E.
27.

Check the serum sodium valproate level


Increase the dose of sodium valproate
Increase the dose of sodium valproate and add an antipsychotic drug
Refer the patient to see a psychologist
Switch to another antipsychotic drug.
Which of the following side effects is more common in tadalafil as compared to other
phosphodiesterase type 5 inhibitors such as vardenafil and sildenafil?

A.
B.
C.
D.
E.
28.

Dyspepsia
Flushing
Headache
Impaired vision
Priapism.
A 30-year-old man suffers from schizophrenia. He presents a list of drugs which he took
in the past. Which of the following is NOT a second generation antipsychotic drug?
a. Clozapine
b. Flupenthixol
c. Olanzapine
d. Quetiapine
e. Risperidone.

29. A 30-year-old man suffers from insomnia and he sleeps well after taking diazepam. He
worries that he lacks a particular neurotransmitter in the brain and diazepam can
enhance the actions of this neurotransmitter. Which of the following neurotransmitters
is MOST relevant In this case?
A.
B.
C.
D.
E.
30.

Acetylcholine
Dopamine
-Aminobutyric acid
Noradrenaline
Serotonin.
Mrs. Tan is a 65-year-old woman who lost her husband three years ago. She is still
angry with him for not taking better care of his health. He was cremated and she keeps
his ashes in the urn. She is unsure what to do with the ashes. She coped well after her
husbands death, until her son moved to Australia a few months ago. She has long
standing headaches, but these have become worse over the past six months. She also
has new onset of somatic symptoms as well as anxiety symptoms which are
overwhelming and include a fear of dying. Her depression has been partially treated
with fluoxetine 40mg per day, but her anxiety symptoms persist and she feels restless.
Which of the following actions is LEAST appropriate?

A.
B.
C.
D.
E.

Add alprazolam to her treatment for 2 weeks


Discontinue fluoxetine and start other antidepressant drug such as paroxetine
Increase the dose of fluoxetine to 80mg per day
Order a thyroid function test
Start psychotherapy with a focus on grief.

!
!
154

31. Which of the following antidepressants cause MOST SEDATION after potentiation with
benzodiazepine?
A.
B.
C.
D.
E.
32.

Agomelatine
Escitalopram
Fluvoxamine
Mirtazapine
Venlafaxine.
Which of the following factors limits the use of clozapine in treatment-resistant
schizophrenia?

A.
B.
C.
D.
E.

A low incidence of seizures


The lack of efficacy against positive symptoms
The lack of efficacy against negative symptoms
The lack of efficacy against cognitive symptoms
The occurrence of agranulocytosis.

33. Which of the following medications is MOST likely to exacerbate psoriasis?


A.
B.
C.
D.
E.

Amitriptyline
Haloperidol
Lithium
Risperidone
Sodium valproate.

34. Which of the following is CORRECT regarding repetitive transcranial magnetic


stimulation (rTMS)?
A.
B.
C.
D.
E.

rTMS is administered under general anaesthesia.


rTMS is more efficacious than electroconvulsive therapy (ECT).
rTMS induces seizures with less energy as compared to ECT.
rTMS is a new imaging modality in psychiatry.
rTMS may be useful in the treatment of depression.

35. The MOST robust effect of lamotrigine is found in which of the following?
a.

Bipolar depression

b.

Manic episode

c.

Mixed episodes

d.

Hypomanic episode

e.

Cyclothymia.

!
!
155

36. A 30-year-old woman complains of lack of sexual drive after taking paroxetine. She
wants to change her antidepressant. Which of the following antidepressant is the BEST
option to avoid sexual dysfunction?
A.
B.
C.
D.
E.
37.

Amitriptyline
Reboxetine
Mirtazapine
Duloxetine
Venlafaxine.

A.
B.
C.
D.
E.

Early morning
Late morning
Early afternoon
Early evening
Past mid-night.

Agomelatine,!a!new!antidepressant!!is!BEST!given!in!which!of!the!following!parts!of!
the!day?

Psychotherapy!
38. A! 24TyearTold! depressed! woman! said,! People! in! my! office! try! to! make! my! life!
difficult! and! then! deem! themselves! as! ultimate! smart! to! know! about! me.! I! have!
resigned! as! I! am! uncomfortable! with! the! office! settings.! I! am! aware! that! I! am!
getting! verbally! aggressive! towards! other! people! whenever! they! provoke! me.! I!
want! to! see! a! psychotherapist! because! I! want! to! deal! with! these! people! before! I!
lose! my! selfTidentity.! Which! of! the! following! psychotherapies! is! MOST!
appropriate?!
!!
A. Cognitive!behaviour!therapy!
B. Grief!therapy!
C. Interpersonal!therapy!
D. Relaxation!exercise!
E. Supportive!psychotherapy!
39. You are a family doctor. A 30-year-old woman suffers from schizophrenia. Although she
is free from first rank symptoms, she is affected by weight gain, negative symptoms of
schizophrenia and no interest in sex. Her husband has high sexual drive and he is not
happy with their sexual life. He requests for anti-androgen to reduce his sexual drive.
What is BEST management strategy? !
A.
B.
C.
D.
E.

Cognitive behaviour therapy


Eye movement desensitization and processing
Hypnotic therapy
Prescribe anti-androgen
Sensate focus therapy.

!
40. A 40-year-old man suffers from post-traumatic stress disorder (PTSD) and has history
of misusing cannabis. He claims that the government should legalise the sales of
cannabis because cannabis can reduce the PTSD symptoms and helps him to relax.

!
!
156

What is his defence mechanism?


A.
B.
C.
D.
E.

Denial
Projection
Rationalisation
Repression
Sublimation.

Ethics and laws


41. The Mental Health (Care and Treatment) Act in Singapore can be applied at which of the
following hospital(s) for involuntary admission?
a.
b.
c.
d.
e.

Changi General Hospital (CGH) only


Institute of Mental Health (IMH) only
National University Hospital (NUH) only
IMH, NUH and CGH
IMH, NUH, CGH, Singapore General Hospital (SGH) and Tan Tock Seng Hospital
(TTSH).

42. A 21-year-old woman suffering from body dysmorphic disorder consulted a doctor for
Botulinum toxin (botox) treatment. She was concerned of the smoothness of her face
and spent $10,000 for multiple botox injection. The botox injection caused paralysis of
facial muscles. Botox injection is not indicated for women younger than 30 years and
body dysmorphic disorder. Which of the following ethical principles was violated by
this doctor?

K.
L.
M.
N.
O.

Autonomy!
Capacity!
Confidentiality!
NonNmaleficence!
Justice.!

43. A 25-year-old man was arrested for murder. According to the police, he shows no
remorse towards his act. He firmly believes that killing one person will not lead to death
sentence. He needs to kill several people to get death sentence. He always carries a
chopper and wants to attack innocent people. If he is released from the prison, he will
kill more people, including his parents. Which of the following diagnoses is MOST
appropriate?
A. Biopathy
B. Egopathy
C. Psychopathy
D. Sociopathy
E. Superegopathy.
Consultation liaison psychiatry
44. You are a resident working in the Children Emergency Department. A 5-year-old boy is
presented for recurrent urinary tract infection and confirmed by raised white blood cell
counts in urine. According to his mother, the boy likes to insert foreign body into his
penis via the urethra meatus. After taking a thorough history and conducting physical

!
!
157

examination, you cannot gather any evidence the boy could insert foreign body into his
penis. It is not anatomically and physiologically feasible for the boy to do so. You
cannot exclude the possibility that it could have been done by his mother. History also
reveals that the parents have marital problems and his father is suspected to visit
commercial sex workers frequently. She is very keen to admit her son for further
invasive investigations. What is the MOST likely diagnosis?
A.
B.
C.
D.
E.

Conversion disorder induced by the mother


Hypochondriasis (or illness anxiety) induced by the mother
Malingering induced by the mother
Mnchausen syndrome by proxy induced by the mother
Somatisation disorder induced by the mother.

45. Which!of!the!following!questionnaires!would!be!MOST!useful!in!identifying!
symptoms!of!depression!and!anxiety!in!patients!suffering!from!chronic!medical!
illness?!!!
!
A.
B.
C.
D.
E.

The!Beck!Anxiety!and!Depression!Inventory!
The!General!Health!Questionnaire!
The!Hamilton!Anxiety!and!Depression!Rating!Scale!
The!Hospital!Anxiety!and!Depression!Scale!
The!StateNTrait!Anxiety!Inventory.!

46. You are a resident working in the Accident and Emergency Department (AED). A 26year-old woman gave birth one month ago. She is a single mother and broke up with
her boyfriend. She has poor social support and no one helps her to look after the baby.
She has been depressed for 1 month and wanted to jump down from her flat two weeks
ago. She was referred to the womans mental health service. She was referred to a case
manager but no medication was prescribed. She breastfeeds her baby. Today, she
wants to jump again. The mother and her baby are bought to the AED by the case
manager. Which of the following is the BEST management?
A. Admit the mother and her baby to the hospital. The mother will go to the psychiatric ward and
the baby will go to paediatric ward. She will benefit from antidepressant and psychological
intervention.
B. Admit the mother and her baby to the hospital. The mother will go to the psychiatric ward and
the baby will go to paediatric ward. She should receive psychological intervention only
because antidepressants are dangerous for breastfeeding.
C. Discharge the mother and her baby from the AED because she suffers from adjustment
disorder. Her risk is low.
D. Discharge the mother and her baby from the AED because she receives good care from case
manager.
E. Send both mother and her baby to the Institute of Mental Health for compulsory admission.
47. What of the following psychiatric side effects is MOST common in Hepatitis C patients
receiving interferon treatment?
A.
B.
C.
D.
E.

Anxiety
Cognitive impairment
Confusion
Depression
Hallucination.

!
!
158

48. The hospital wants to develop the best method to detect and identify delirium in
medically ill patients. Which of the following is the BEST strategy to detect and identify
delirium?

A. Administer Delirium Rating Scale to all patients older than 65 years to screen for delirium.
B. Looking for abnormalities on computerized topography (CT) brain scans for all patients.
C. Looking for abnormalities on magnetic resonance imaging (MRI) brain scans for suspected
patients.
D. Monitor signs of delirium on those patients with laboratory abnormalities.
E. Screen for history of delirium and monitor those patients with history of delirium only.

!
Rotation 2 - 2014
Psychopathology
7. A 50-year-old woman claims that unfamiliar
people whom she met on the street were her
husband. She believes that her husband put
on a disguise. What is the diagnosis?
F.
G.
H.
I.
J.

Capgras syndrome
Charles de Bonnet syndrome
DeClerambaults syndrome
Fregoli syndrome
Ganser syndrome.

Explanation: Fregoli syndrome is a


delusional mis-indentification syndrome
when unfamiliar people are identified as
a familiar person.

Year: 2014

8. A 50-year-old woman hears her late mothers


voice when she falls asleep. She cannot recall
the content of her mothers speech. This
phenomenon is known as:
F.
G.
H.
I.
J.

The answer is D.

The answer is C.

Explanation: Hypnogogic hallucination


occurs when a person falls asleep and
conscious level fluctuates considerably in
different stages of sleep.

Auditory illusion
Hypnopompic hallucination
Hypnagogic hallucination
Pseudohallucination
Third-person auditory hallucination.

Year: 2014.
9. You are seeing a 70-year-old man who suffers
from Parkinsons disease. He demonstrates a
dulled emotional tone and seems to be
indifferent to jokes. His family describes him
as being emotionally detached. Which of the
following
terms
BEST
describes
his
psychopathology?
F. Alexithymia

!
!
159

The answer is D.

Explanation: Apathy refers to detachment


from feeling or a dulled emotional tone.
The person conveys a sense of
indolence and indifference to what
normally causes excitement or interest.

G.
H.
I.
J.

Alogia
Anhedonia
Apathy
Attentional deficit.

Detachment is one of the aspects of


apathy.

Anhedonia is defined as the inability to


experience pleasure from activities
usually found enjoyable which is less
evident in this case.

Year: 2014.
10. A 50-year-old Singaporean Chinese man
firmly believes that he needs to flee from
Singapore to Russia for his personal safety.
He has developed this belief after reading a
newspaper article about Edward Snowden
who released national security materials of
the United States (US).
This man firmly
believes that the newspaper mentions his
name and describes about his role in the leak
of US national secrets. Which of the following
terms BEST describe his experience?
F.
G.
H.
I.
J.

Delusion of erotomania
Delusion of grandiosity
Delusion of guilt
Delusion of jealousy
Delusion of reference.

The answer is E.

Explanation: This man firmly believes


that the newspaper article referred to him
and this is impossible because the
person involved is a Caucasian with US
citizenship. This is known as delusion of
reference.

Year: 2014.

Cognitive assessment
11. You are administering the Mini-Mental State
Examination (MMSE) to a 70-year-old woman
who did not receive any formal education. She
has difficulty to complete the full version of
MMSE. Which of the following tasks is MOST
useful to assess her attention?

F. Constructing a double pentagon


G. Naming the months from December in backward
direction
H. Orientation to time and place
I. Three-item registration and recall
J. Three-stage command.

The answer is B.

Explanation: Option B is the most useful


task to assess her attention because she
may not be able to perform serial 7 due
to low level of education. Option A
assesses constructional apraxia. Option
C assesses orientation. Option D
assesses registration and short term
memory. Option E assess working
memory.

Year: 2014.

!
!
160

12. The Montreal Cognitive Assessment (MOCA)


is different from the Mini-Mental State
Examination.
Which
of
the
following
neuroanatomical structures is specifically
assessed in the MOCA but not the MMSE?
F.
G.
H.
I.
J.

Frontal lobe
Hippocampus
Occipital lobe
Parietal lobe
Temporal lobe.

The answer is A.

Explanation: MOCA tests the frontal lobe


function by the trail making task. This
task assesses alternating sequencing
(number and alphabet) and rule out
perservation.

Year: 2014.

This is an advanced-level question.


Psychiatric epidemiology / Aetiology
49. After 65 years of age, the rate of Alzheimers
disease changes approximately by how many
times every 5 years increase in age?
F. 0.5 time
G. 1 time
H. 2 times
I. 3 times
J. 4 times.

The answer is C.

Explanation: the rate of Alzheimers


disease doubles approximately every 5
years of age.

Year: 2014
50. Lesions in which of the following
neuroanatomical areas is the MOST
vulnerable to amnestic syndrome?
F.
G.
H.
I.
J.

The answer is C.

Explanation: Damage to hippocampus is


most vulnerable to amnestic syndrome.

Basal ganglia
Cerebellum
Hippocampus
Nucleus accumbens
Raphe nucleus.

Year: 2014
General adult psychiatry / Psychiatric diagnosis
51. Which of the following psychiatric diagnoses
is MOST likely to contribute to inpatient
suicide?

The answer is B.

Explanation: Major depressive disorder is


most likely to contribute to inpatient

F. Borderline personality disorder

!
!
161

G.
H.
I.
J.

Major depressive disorder


Generalised anxiety disorder
Hypochondriasis/ illness anxiety disorder
Schizophrenia.

suicide among all the options.

Year: 2014.

52. Which of the following features is MOST


characteristic of Cotards syndrome?

The answer is E.

F. Delusional belief that familiar people have been


replaced by their imposters
G. Delusional belief that the spouse is unfaithful
H. Delusional belief that strangers have taken on the
psychological identity of a familiar person
I. Delusional belief that a person of higher status
falls in love with the patient
J. Nihilistic and hypochondriacal delusions found in
psychotic depression.

Explanation: Cotards syndrome is a


nihilistic delusion in which a person
believes that their possessions, friends,
or parts of their own body do not exist or
are about to not exist.

53. A 23-year-old man, recently promoted to a


middle management position, presents to
your clinic with complaints of acute episodes
of anxiety associated with heart palpitations,
dyspnoea, shaking, detachment from his
body, and fear of losing control. These
experiences
only
occur
during
oral
presentation
in
front
of
the
upper
management. Which of the following
diagnoses is MOST likely?

The answer is E.

F.
G.
H.
I.
J.

Year: 2014.

Explanation: The diagnosis is social


phobia but not panic disorder because he
demonstrates those symptoms during the
presentation in front of the management.

Year: 2014

Generalized anxiety disorder


Panic disorder
Panic disorder with agoraphobia
Post-traumatic stress disorder
Social phobia.

54. A 40-year-old woman, Ms. Tan has witnessed


a road traffic accident. Four hours ago, an
elderly man was knocked down by a taxi and
suffered from severe head injury. Ms. Tan was
walking along the street and witnessed the
whole accident. She did not suffer from any
injury. The police officer wants to interview
Ms. Tan to obtain more information about the
accident. She cannot recall any details related
to the accident. Which of the following is the
MOST appropriate explanation for her
amnesia?

!
!
162

The answer is C.

Explanation: This woman suffers from


acute stress reaction after witnessing the
road traffic accident. She develops
dissociative amnesia and not able to
provide details related to the accident.

Year: 2014
F.
G.
H.
I.
J.

Acute and transient psychosis


Conversion disorder
Dissociative state
Factitious disorder
Underlying dementia.

55. You are a medical officer working in the Army


Camp. A 19-year-old man is recently enlisted
for National Service and his parents inform
you that he walks around his flat during sleep.
Which of the following recommendations is
LEAST appropriate?
F.
G.
H.
I.
J.

Safety measures
Sleep hygiene
Stay out of camp at night
Stimulant
Supportive psychotherapy.

Explanation: Stimulant treatment (e.g.


modanfinil) is used to treat narcolepsy
but not sleep-walking.

Year: 2014

56. A 25-year-old man believes that he suffers


from adult ADHD. Which of the following
criteria is compulsory to establish such
diagnosis?
F.
G.
H.
I.
J.

The answer is D.

Absence of inattention symptoms in adulthood


History of ADHD symptoms before the age of 12
History of conduct disorder
Presence of hyperactivity symptoms in adulthood
History of substance abuse.

The answer is B.

Explanation: History of ADHD symptoms


in childhood is a compulsory diagnostic
criteria for diagnosing adult ADHD
symptoms.

Year: 2014
57. All of the following findings are found in
anorexia nervosa EXCEPT:
F.
G.
H.
I.
J.

Decreased bone mineral density


Decreased growth hormone
Decreased gonadotropins
Decreased potassium
Decreased heart rate.

The answer is B.

Explanation: Growth hormone is


increased but not reduced in anorexia
nervosa.

Year: 2014
Old age psychiatry
58. You are a family doctor and a 70-year-old man
consults you for annual medical assessment
to certify that he is fit to drive. Which of the
following is LEAST relevant to driving?

The answer is E.

Explanation: Option A, B, C and D

!
!
163

F. Assess his visual acuity by Snellens Chart


G. Assess his hearing by whispering to his ears
H. Inquire his driving practice and road safety
records
I. Examine his elbow and knee joints
J. Measure his blood pressure.

assess driving and body systems that are


directly involved in driving. Blood
pressure is not directly involved in driving
and less likely to influence driving
performance.

Year: 2014
59. Which of the following is TRUE about suicide
in old people?
F. Old people with terminal illness have low suicide
risk because they foresee that they will die soon.
G. Old people often attempt suicide as a means to
manipulate other people.
H. Old people frequently communicate their intent
prior to committing suicide.
I. Old people generally attempt self-harm rather
than suicide.
J. Suicide attempts are more lethal in old people as
compared to young people.

The answer is E.

60. Which of the following statements is FALSE


about delirium in old people?

The answer is D.

F. Could be an initial presentation of underlying


dementia
G. Onset could be insidious
H. Rarely results in full resolution of symptoms in a
short period of time
I. Un-medicated old people are at higher risk to
develop delirium as compared to old people
treated by multiple medications.
J. Usually persists for weeks or months in old
people hospitalized for medical or surgical
reasons.

Explanation: Old people who take


multiple medications (or polypharmacy)
are at higher risk to develop delirium.

61. The daughter of an 80-year-old woman with


history of bipolar disorder calls you for
advice. Her mother assaulted one helper and
the other helper is terrified. They tried to stop
her from leaving the house to buy things.
Which of the following is MOST appropriate
management strategy?

The answer is A.

F. Admission to the psychiatry ward


G. Employ a part-time nurse to look after her at
home
H. Increase the dose of antipsychotics and review in
1 week

!
!
164

Explanation: Suicide in elderly is


common and they are determined to die.
Doctors should take this seriously and
advise for hospitalization.

Year: 2014

Year: 2014

Explanation: As part of the risk


management, this elderly lady should be
admitted to the psychiatry ward for
assessment. Besides further assessment
of bipolar disorder, the doctors need to
assess other medical causes of violent
behaviour (e.g. delirium or alcohol
withdrawal)

I.
J.

Increase the dose of benzodiazepine and review


in1 week
Increase the dose of mood stabilizer and review
in 1 week.

Year: 2014

Child and adolescent psychiatry


62. A 4-year-old boys grandmother died four
months ago. The grandmother was one of the
childs primary caregivers. The child still
speaks to the grandmother. When he walks,
he still holds her hands up as though holding
hands with his grandmother. Which of the
following is CORRECT?
F.
G.
H.
I.
J.

This child fully understands the concept of death.


This child suffers from major depression.
This child is psychotic.
The child suffers from adjustment disorder with
depressed mood.
This reaction is within normal limits for a child of
this age.

The answer is E.

Explanation: This is a normal grief


reaction. Child at this age may not
understand the concept of death. He may
believe that his grandmother will return
one day.

Year: 2014

This is an advanced-level question


63. Which of the following is the MOST
established psychopharmacological treatment
for children and adolescents suffering from
autism?
F.
G.
H.
I.
J.

The answer is B.

Explanation: Antipsychotic drug e.g.


risperidone is the most established
psychopharmacological drug to treat
young people with autism.

Actetylcholinesterase inhibitor
Antipsychotic drug
Benzodiazepine
Omega-3 supplement
Stimulant.

Year: 2014.
Substance abuse
64. You are a resident working at the Accident
and Emergency Department. A 21-year-old
man is brought in by police and he is
disorientated to time, place and person. He
needs to urinate very often and seems to have
urinary incontinence. A package of capsules
is found in his pocket and suspected to be
illicit drugs. Which of the following drugs is
MOST likely to cause the above symptoms?
F. Amphetamine
G. Cannabis
H. Cocaine

!
!
165

The answer is D.

Explanation:
Ketamine-induced
ulcerative cystitis which includes urge
incontinence,
decreased
bladder
compliance, decreased bladder volume,
detrusor
overactivity,
and
painful
haematuria.

I.
J.

Ketamine
Phencyclidine.

Year: 2014.

65. Hypertension associated with cocaine misuse


is BEST treated by:

F.
G.
H.
I.
J.

AngiotensinNconvertingNenzyme!inhibitors
Beta-blockers
Benzodiazapine
Loop diuretics
Thiazide diuretics.

The answer is C.

Explanation: Benzodiazapine is the


safest and most appropriate option here.
The other option is alpha blocker which is
not available. Beta-blockers will worsen
hypertension by leaving alpha receptors
unopposed. As cocaine increases
noradrenaline levels, diuretics are less
relevant.

Year: 2014.

This is an advanced level question.


66. Which of the following addiction has the
HIGHEST suicide risk?
A, Gambling

The answer is A.

Explanation: The risk for suicide in


gambling is twice the risk compared to
other forms of addiction as a result of
huge financial consequences and
unsettled debts. Other tell-tales signs of
suicide amog gamblers include insomnia,
poor appetite loss of interest to work and
somatic complaints.

B. Internet
C. Sex
D. Steroid addiction in young athlete
E. Stimulant.

Year: 2014.

67. You are a general practitioner (GP). A 20-year-old


university student wants you to prescribe
methylphenidate (ritalin). The student believes
that ritalin can enhance his examination
performance and he has been obtaining Ritalin
prescribed from another GP. He does not have
history of
attention deficit and hyperactivity
disorder (ADHD). Which of the following advices

!
!
166

The answer is B.

Explanation: In Singapore, patients are


more likely to misuse prescribed
medication than illicit substances due to

is MOST APPROPRIATE?
F. Advise him to take methylphenidate on the days
of examination and stops immediately after the
examination.
G. Advise him to stop taking methylphenidate
because it is a controlled drug and not indicated
in his situation.
H. Methylphenidate can enhance his attention if he
has family history of ADHD.
I. Methylphenidate can enhance his attention if he
has past history of conduct disorder.
J. There is evidence to suggest that
methylphenidate can enhance his attention and
improve his examination performance.

tight control of illicit drugs. This student


may have misused methylphenidate as
stimulants and see different doctors to
obtain methylphenidate. He is
recommended to stop taking
methylphenidate and there is no
evidence to suggest that this will help his
examination performance.

Year: 2014.

Psychopharmacology
68. A 30-year-old man suffers from autism and
epilepsy. He has been very aggressive. The
psychiatrist prescribes sodium valproate
1300mg nocte to control his aggression and
epilepsy. His mother concludes the sodium
valproate is not effective. The first step of
management should be:
F. Check the serum sodium valproate level
G. Increase the dose of sodium valproate
H. Increase the dose of sodium valproate and add
an antipsychotic drug
I. Refer the patient to see a psychologist
J. Switch to another antipsychotic drug.

The answer is A.

Explanation: The patient is on a relatively


high dose of sodium valproate. The first
step should check serum level of sodium
valproate because patient may not noncompliant to the medication.

Year: 2014.

This is an advanced level question.


69. Which of the following side effects is more
common in tadalafil as compared to other
phosphodiesterase type 5 inhibitors such as
vardenafil and sildenafil?
F.
G.
H.
I.
J.

The answer is E.

Explanation: Tadalafil has the longest


half-life, 17.5 hours as compared to
vardenafil (4-5 hours) and sildenafil (3-4
hours).

Dyspepsia
Flushing
Headache
Impaired vision
Priapism.

Year: 2014.

This is an advanced level question.

!
!
167

70. A 30-year-old man suffers from schizophrenia.


He presents a list of drugs which he took in
the past. Which of the following is NOT a
second generation antipsychotic drug?
f. Clozapine
g. Flupenthixol
h. Olanzapine
i. Quetiapine
j. Risperidone.

The answer is B.

Explanation: Option B is a firstgeneration antipsychotic drug.

Year: 2014
71. A 30-year-old man suffers from insomnia and
he sleeps well after taking diazepam. He
worries
that
he
lacks
a
particular
neurotransmitter in the brain and diazepam
can
enhance
the
actions
of
this
neurotransmitter. Which of the following
neurotransmitters is MOST relevant In this
case?
F.
G.
H.
I.
J.

Acetylcholine
Dopamine
-Aminobutyric acid
Noradrenaline
Serotonin.

The answer is C.

Explanation: Benzodiazapine drugs bind


to benzodiazepine receptors and this
enhances the binding of -Aminobutyric
acid (GABA) to GABA receptors. This will
lead to influx of chloride ions and cause
sedation.

Year: 2014

72. Mrs. Tan is a 65-year-old woman who lost her


husband three years ago. She is still angry
with him for not taking better care of his
health. He was cremated and she keeps his
ashes in the urn. She is unsure what to do
with the ashes. She coped well after her
husbands death, until her son moved to
Australia a few months ago. She has long
standing headaches, but these have become
worse over the past six months. She also has
new onset of somatic symptoms as well as
anxiety symptoms which are overwhelming
and include a fear of dying. Her depression
has been partially treated with fluoxetine
40mg per day, but her anxiety symptoms
persist and she feels restless. Which of the
following actions is LEAST appropriate?

F. Add alprazolam to her treatment for 2 weeks


G. Discontinue fluoxetine and start other
antidepressant drug such as paroxetine
H. Increase the dose of fluoxetine to 80mg per day
I. Order a thyroid function test
J. Start psychotherapy with a focus on grief.

!
!
168

The answer is C.

Explanation: increase fluoxetine to 80mg


may worsen anxiety and restlessness.

Year: 2014

This is an advanced level question.

73. Which of the following antidepressants cause


MOST SEDATION after potentiation with
benzodiazepine?
F.
G.
H.
I.
J.

The answer is D.

Explanation: Mirtazapine is the most


sedative antidepressant. The sedation
effect will be worsen after potentiation
with benzodiazepine.

Agomelatine
Escitalopram
Fluvoxamine
Mirtazapine
Venlafaxine.

Year: 2014.
74. Which of the following factors limits the use
of clozapine in treatment-resistant
schizophrenia?
F.
G.
H.
I.
J.

A low incidence of seizures


The lack of efficacy against positive symptoms
The lack of efficacy against negative symptoms
The lack of efficacy against cognitive symptoms
The occurrence of agranulocytosis.

The answer is E.

Explanation: 1 to 2% of patients using


clozapine will develop agranulocytosis,
and this is the main factor limiting its use;
risk increases with age, and is higher
among women; the risk is the highest in
the first six months of treatment (hence
the need for weekly blood tests in the
first six months, and then every two
weeks after that).

Year: 2014.
75. Which of the following medications is MOST
likely to exacerbate psoriasis?

The answer is C

F.
G.
H.
I.
J.

Explanation: Lithium has higher risk to


worsen psoriasis as compared to other
psychotropic medications.

Amitriptyline
Haloperidol
Lithium
Risperidone
Sodium valproate.

Year: 2014.

This is an advanced level question.


76. Which of the following is CORRECT regarding
repetitive transcranial magnetic stimulation
(rTMS)?

!
!
169

The answer is E.

F. rTMS is administered under general anaesthesia.


G. rTMS is more efficacious than electroconvulsive
therapy (ECT).
H. rTMS induces seizures with less energy as
compared to ECT.
I. rTMS is a new imaging modality in psychiatry.
J. rTMS may be useful in the treatment of
depression.

Explanation: rTMS does not require


general anaesthesia. rTMS is an
alternative to ECT if patient has medical
or surgical contraindicates to ECT. rTMS
is not intended to induce seizures and it
is not related to imaging.

Year: 2014.

77. The MOST robust effect of lamotrigine is


found in which of the following?
a.

Bipolar depression

b.

Manic episode

c.

Mixed episodes

d.

Hypomanic episode

e.

Cyclothymia.

Explanation: Lamotrigine is effective in


treatment of bipolar depression with
200mg per day.

Year: 2014.

78. A 30-year-old woman complains of lack of


sexual drive after taking paroxetine. She
wants to change her antidepressant. Which of
the following antidepressant is the BEST
option to avoid sexual dysfunction?
F.
G.
H.
I.
J.

The answer is A.

Amitriptyline
Reboxetine
Mirtazapine
Duloxetine
Venlafaxine.

The answer is C.

Explanation: The 5HT2A antagonism of


mirtazapine is associated with lesser
sexual side effects.

Year: 2014.

79. Agomelatine,!a!new!antidepressant!!is!BEST! The!answer!is!D.!

given!in!which!of!the!following!parts!of!the!
!
day?
!
F.
G.
H.
I.

Explanation:! Early! evening! is! the! best!


time!
for!
circadian!
rhythms!
resynchonizing! (1N2h! before! dark!

Early morning
Late morning
Early afternoon
Early evening

!
!
170

J.

phase).!

Past mid-night.

!
Year: 2014.

Psychotherapy!

80. A! 24TyearTold! depressed! woman! said,!


People! in! my! office! try! to! make! my! life!
difficult! and! then! deem! themselves! as!
ultimate! smart! to! know! about! me.! I! have!
resigned! as! I! am! uncomfortable! with! the!
office!settings.!I!am!aware!that!I!am!getting!
verbally! aggressive! towards! other! people!
whenever!they!provoke!me.!I!want!to!see!a!
psychotherapist! because! I! want! to! deal!
with! these! people! before! I! lose! my! selfT
identity.! Which! of! the! following!
psychotherapies!is!MOST!appropriate?!
!!
F.
G.
H.
I.
J.

Cognitive!behaviour!therapy!
Grief!therapy!
Interpersonal!therapy!
Relaxation!exercise!
Supportive!psychotherapy!

The!answer!is!C.!
!
Explanation:! Interpersonal! therapy!
(IPT)! is! indicated! for! depressive!
disorder.! It! helps! the! patient! to!
understand!
the!
communication!
difficulties!
(verbal!
aggression).!
Psychotherapist!can!perform!roleNplay!
to! help! patient! to! improve!
communcation.! IPT! also! helps! patient!
to! deal! with! the! loss! associated! with!
her!selfNidentity!and!resignation.!
!
Year: 2014.

!
This!is!an!advanced!level!question.!
81. You are a family doctor. A 30-year-old woman

suffers from schizophrenia. Although she is


free from first rank symptoms, she is affected
by weight gain, negative symptoms of
schizophrenia and no interest in sex. Her
husband has high sexual drive and he is not
happy with their sexual life. He requests for
anti-androgen to reduce his sexual drive.
What is BEST management strategy? !
F. Cognitive behaviour therapy
G. Eye movement desensitization and
processing
H. Hypnotic therapy
I. Prescribe anti-androgen
J. Sensate focus therapy.

!
!
171

The!answer!is!E.!
!
Explanation:!Sensate!focus!therapy!is!a!
sex!therapy.!The!couple!focus!on!nonN
vaginal!intercourse!and!master!other!
techniques!such!as!communication!
and!mutual!stimulation.!Prescription!
of!antiNandrogen!is!associated!with!
side!effects!such!as!osteoporosis.!The!
other!options!are!not!specific!for!
psychosexual!problems.!

!
Year:!2014!
82. A 40-year-old man suffers from post-traumatic
stress disorder (PTSD) and has history of
misusing cannabis. He claims that the
government should legalise the sales of
cannabis because cannabis can reduce the
PTSD symptoms and helps him to relax. What
is his defence mechanism?
F.
G.
H.
I.
J.

Denial
Projection
Rationalisation
Repression
Sublimation.

!
Explanation:!This!man!justifies!
cannabis!misuse!with!an!explanation!
but!totally!forgets!the!potential!harms!
associated!with!cannabis!misuse.!
!
Year:!2014!

Ethics and laws


83. The Mental Health (Care and Treatment) Act in
Singapore can be applied at which of the
following
hospital(s)
for
involuntary
admission?
f.
g.
h.
i.
j.

The!answer!is!C.!

Changi General Hospital (CGH) only


Institute of Mental Health (IMH) only
National University Hospital (NUH) only
IMH, NUH and CGH
IMH, NUH, CGH, Singapore General
Hospital (SGH) and Tan Tock Seng
Hospital (TTSH).

84. A 21-year-old woman suffering from body


dysmorphic disorder consulted a doctor for
Botulinum toxin (botox) treatment. She was
concerned of the smoothness of her face and
spent $10,000 for multiple botox injection. The
botox injection caused paralysis of facial
muscles. Botox injection is not indicated for
women younger than 30 years and body
dysmorphic disorder. Which of the following
ethical principles was violated by this doctor?

The!answer!is!B.!
!
Explanation:!The!Mental!Health!Act!
can!only!be!applied!at!IMH!for!
involuntary!admission.!!
!
Year:!2014.!
The!answer!is!D.!
!
Explanation:!NonNmaleficence!refers!to!
the!obligation!of!a!doctor!to!avoid!
harm,!causing!muscular!atrophy!by!
botox!injection!in!this!case.!
!

P.
Q.
R.
S.
T.

Autonomy!
Capacity!
Confidentiality!
NonNmaleficence!
Justice.!

Year:!2014.!

!
!
172

85. A 25-year-old man was arrested for murder.


According to the police, he shows no remorse
towards his act. He firmly believes that killing
one person will not lead to death sentence. He
needs to kill several people to get death
sentence. He always carries a chopper and
wants to attack innocent people. If he is
released from the prison, he will kill more
people, including his parents. Which of the
following diagnoses is MOST appropriate?
F.
G.
H.
I.
J.

Biopathy
Egopathy
Psychopathy
Sociopathy
Superegopathy.

The!answer!is!C.!
!
Explanation:!In!forensic!psychiatry,!
psychopath!represents!the!most!
severe!form!of!antisocial!personality!
disorder.!People!with!psychopathy!
demonstrate!extremely!low!level!of!
empathy!and!remorse.!They!have!high!
chance!of!recidivism.!
!
Year:!2014.!
!

Consultation liaison psychiatry


86. You are a resident working in the Children
Emergency Department. A 5-year-old boy is
presented for recurrent urinary tract infection
and confirmed by raised white blood cell
counts in urine. According to his mother, the
boy likes to insert foreign body into his penis
via the urethra meatus. After taking a
thorough history and conducting physical
examination, you cannot gather any evidence
the boy could insert foreign body into his
penis.
It
is
not
anatomically
and
physiologically feasible for the boy to do so.
You cannot exclude the possibility that it
could have been done by his mother. History
also reveals that the parents have marital
problems and his father is suspected to visit
commercial sex workers frequently. She is
very keen to admit her son for further invasive
investigations.
What is the MOST likely
diagnosis?
F. Conversion disorder induced by the mother
G. Hypochondriasis (or illness anxiety) induced by
the mother
H. Malingering induced by the mother
I. Mnchausen syndrome by proxy induced by the
mother
J. Somatisation disorder induced by the mother.

87. Which!of!the!following!questionnaires!
would!be!MOST!useful!in!identifying!
symptoms!of!depression!and!anxiety!in!
patients!suffering!from!chronic!medical!
illness?!!!
!

!
!
173

The!answer!is!D.!
!
Explanation:! Mnchausen syndrome by
proxy occurs when an individual (the
mother) must have presented another
individual (the boy) to others (AED
doctor) as medically ill (recurrent urinary
tract infection) without the intention of
obvious external rewards but invasive
investigation
or
hospitalizations.
Mnchausen syndrome by proxy is
considered to be a form of child abuse.
She is displacing her anger (husband
visiting commercial sex workers) towards
her son.

Year:!2014.!

The!answer!is!D.!
!
Explanation:!The!Hospital!Anxiety!and!
Depression!Scale!is!the!best!scale!

F. The!Beck!Anxiety!and!Depression!Inventory!
G. The!General!Health!Questionnaire!
H. The!Hamilton!Anxiety!and!Depression!Rating!
Scale!
I. The!Hospital!Anxiety!and!Depression!Scale!
J. The!StateNTrait!Anxiety!Inventory.!

because!the!items!are!specially!
constructed!to!enhance!sensitivity!in!
detecting!depression!and!anxiety!
symptoms!in!medical!patients.!The!
Beck!depression!and!anxiety!inventory!
and!Hamilton!Anxiety!and!Depression!
Rating!Scale!are!more!suitable!for!
psychiatric!patients.!
!
Year:!2014.!

88. You are a resident working in the Accident


and Emergency Department (AED). A 26-yearold woman gave birth one month ago. She is a
single mother and broke up with her
boyfriend. She has poor social support and
no one helps her to look after the baby. She
has been depressed for 1 month and wanted
to jump down from her flat two weeks ago.
She was referred to the womans mental
health service. She was referred to a case
manager but no medication was prescribed.
She breastfeeds her baby. Today, she wants
to jump again. The mother and her baby are
bought to the AED by the case manager.
Which of the following is the BEST
management?
F. Admit the mother and her baby to the hospital.
The mother will go to the psychiatric ward and the
baby will go to paediatric ward. She will benefit
from antidepressant and psychological
intervention.
G. Admit the mother and her baby to the hospital.
The mother will go to the psychiatric ward and the
baby will go to paediatric ward. She should
receive psychological intervention only because
antidepressants are dangerous for breastfeeding.
H. Discharge the mother and her baby from the AED
because she suffers from adjustment disorder.
Her risk is low.
I. Discharge the mother and her baby from the AED
because she receives good care from case
manager.
J. Send both mother and her baby to the Institute of
Mental Health for compulsory admission.
89. What of the following psychiatric side effects
is MOST common in Hepatitis C patients
receiving interferon treatment?
F. Anxiety
G. Cognitive impairment

The!answer!is!A.!
!
Explanation:! ! In! this! situation,! both!
mother! and! baby! should! be! admitted!
to! the! hospital! because! no! one! looks!
after! the! baby! at! home.! IMH! does! not!
have!facility!to!look!after!her!baby.!
!
The!psychiatrist!in!the!hospital!should!
consider! starting! antidepressant!
because!she!is!highly!suicidal!and!SSRI!
like!sertraline!is!safe!for!breastfeeding.!
!
Year:!2014.!

The answer is D.

Explanation: Interferon treatment is

!
!
174

H. Confusion
I. Depression
J. Hallucination.

well known to be associated with


depression.

Year: 2014.
90. The hospital wants to develop the best
method to detect and identify delirium in
medically ill patients. Which of the following
is the BEST strategy to detect and identify
delirium?

F. Administer Delirium Rating Scale to all patients


older than 65 years to screen for delirium.
G. Looking for abnormalities on computerized
topography (CT) brain scans for all patients.
H. Looking for abnormalities on magnetic resonance
imaging (MRI) brain scans for suspected patients.
I. Monitor signs of delirium on those patients with
laboratory abnormalities.
J. Screen for history of delirium and monitor those
patients with history of delirium only.

!
!
!
!
!
!
!
!
!
!
!
!
!
Paper!7!N!Questions!

!
!
175

The answer is A.

Explanation: Option A is the most costeffective strategy to detect and identify


delirium. Option D is not the best strategy
because there are elderly patients who
develop delirium without any laboratory
abnormalities. Option E is not the best
strategy because history of delirium may
not predict future episodes of delirium.

Year: 2014.

History, mental state exam, psychopathology and risk assessment!

1. You are a resident working in the accident and emergency department (AED).
A 30-year-old schizophrenia patient is brought in by his mother. He suddenly
heard a voice asking him to use a sword to stab himself and open up his
abdomen. His mother stopped him and bought him to the hospital. He does not
have antisocial behaviour. Which of the following BEST describes his current
risk at the AED?
A. High risk due to the possibility that he may attack other people.
B. High risk due to the possibility of damaging internal organs such as aorta and liver if
he stabs himself.
C. Low risk due to the fact that he has informed his mother and the chance to carry out
action is low.
D. Low risk due to the unlikeliness for him to harm himself as a result of disorganized
behaviour seen in schizophrenia patients.
E. Moderate risk due to possibility of superficial cut on abdomen.
2. You are a resident working in the accident and emergency department. A 30-year-old
bank officer is brought in by her supervisors after she has a failed suicide attempt in
the bank. She was recently transferred out from the head office to another branch.
According to her relatives, the patient claims that the Chief Executive Officer (CEO)
fell in love with her. She spent hours waiting for the CEO after work. She tried to stop
his car from leaving the car park. The CEO has never worked with her and does not
know her personally. Which of the following is the correct diagnosis?

A.
B.
C.
D.
E.

Capgras!syndrome!
Charles!de!Bonnets!syndrome!
De!Clrambault's!syndrome!
Gansers!syndrome!
Othellos!syndrome!

Which of the following is NOT a thought disorder found in patients suffering from
schizophrenia?

A.
B.
C.
D.
E.

Derailment
Echopraxia
Loosening of association
Knights move thinking
Word salad.

Cognitive assessment!
4

Which of the following is NOT a prominent cognitive feature of cortical dementia?

!
!
176

A.
B.
C.
D.
E.

Problems with articulation


Problems with calculation
Problems with drawing a clock face
Problems with expression of emotion
Problems with recognition

A 20-year-old woman complains of low mood because she is bullied in her workplace.
She is a factory worker and her supervisor complains that she is very slow at work.
She stays with her family and is able to look after herself. Her mother describes the
patient as simple-minded. When she was young, she exhibited delay in understanding
and learning language. She kept failing the N level examination despite multiple
attempts. Which of the following BEST describes her current level of intelligence?
A.
B.
C.
D.
E.

Normal intelligence
Mild intellectual disability
Moderate intellectual disability
Severe intellectual disability
Profound intellectual disability.

Psychiatric epidemiology

By 2020, which of the following illnesses is predicted to have higher global


disease burden than depressive disorder?

A.
B.
C.
D.
E.

Anxiety disorder
Dementia
Diabetes
Ischaemic heart disease
Schizophrenia.

Which of the following mood disorders has the HIGHEST rate of co-occurrence with a
substance abuse disorder?

A.
B.
C.
D.
E.

Adjustment disorder
Seasonal affective disorder
Cyclothymia
Bipolar disorder
Dysthymia.

Psychiatric aetiology, diagnosis and classification

8 Which of the following neuroanatomical areas is MOST implicated in


!
!
177

obsessive compulsive disorder?


A.
B.
C.
D.
E.

Amygdala and prefrontal cortex


Frontal lobe and parietal lobe
Hippocampus and locus coeruleus
Nucleus accumbens and prefrontal cortex
Orbito-frontal cortex and basal ganglia

Which of the following is the MOST common cause of secondary hypersomnia?

A.
B.
C.
D.
E.

Consumption of hydroxyzine
Dementia
Mixed anxiety and depression
Obstructive sleep apnea
Shift work.

10 You are a general practitioner. A 33-year-old man complains of stress due to


interpersonal problems and long term unemployment. He claims that he suffers from
depression but does not require antidepressant treatment. He has no history of selfharm or criminal record. He has long-standing problems with his wife. His wife
mentions that he knows every detail about cooking and household chores. She finds
the 3- year marriage miserable and has gone through a very difficult period. His wife
realises that she will never satisfy his expectations. She describes the patient as
being too idealistic and extremely careful. When she cooks, the patient tries to belittle
her by watching her every single movement and interjecting a lot of advice to make a
perfect meal. He often scolds her for spending money and she is afraid of the patient.
She finds him very rigid with no sense of humour. Which of the following is the
correct diagnosis?
A.
B.
C.
D.
E.

Avoidant personality disorder


Borderline personality disorder
Narcissistic personality disorder
Obsessive compulsive personality disorder
Paranoid personality disorder.

11 The mother of a schizophrenia patient wants to consult you the risk of his dizygotic
twin brother in developing schizophrenia. Your answer is:
A. 4%
B. 14%
C. 24%
D. 34%
E. 44%.
General adult psychiatry

12 You are a general practitioner. A 22-year-old patient comes to consult


you because she is stressed. Her father boarded an aeroplane which
was found missing and probably has crashed. She is not certain about
!
!
178

the status of her father and he is not contactable. At this moment, she is
MOST likely to develop which of the following psychiatric conditions?

A.
B.
C.
D.
E.

Anticipatory grief
Compensation neurosis
Dissociative state
Psychosis
Hypochondriasis.

13 A 30-year-old schizophrenia patient drops out from rehabilitation. The


family member wants to find out the underlying reason. Which of the
following factors is LEAST likely to restrict rehabilitation potential?
A.
B.
C.
D.
E.

Residual auditory hallucination


Intellectual disability
Lack of pleasure from social intervention and physical activity
Poor motivation
Restricted affect.

!
14 Which!of!the!following!statements!concerning!generalized!anxiety!disorder!is!
FALSE?!!
!
A.
B.
C.
D.
E.

Alcohol!misuse!is!a!common!comorbidity.!!
Cognitive!behaviour!therapy!is!generally!helpful.!
Excessive!worries!and!anxiety!symptoms!are!episodic.!
ShortNterm!benzodiazepines!can!be!an!effective!treatment!
There!is!a!2!to!1!ratio!of!women!to!men!suffering!from!this!disorder.!

15 Which!of!the!following!is!TRUE!regarding!delusional!disorder?!
!!
A.
B.
C.
D.

Infrequent!hallucinations!totally!rule!out!the!diagnosis!of!delusional!disorder.!
Onset!is!in!adolescence.!!
Point!prevalence!is!3%.!
From!genetic!point!of!view,!there!is!frequent!crossover!to!other!psychotic!disorders!
such!as!schizophrenia.!
E. The!persecutory!subtype!is!the!most!common.!!
!
16 A 45-year-old man consults you because he is stressed over his daughters eating
disorder. His daughter suffers from anorexia nervosa. He is very concerned about her
condition. He wants to seek your reassurance. Which of the following indicates a
GOOD prognostic factor for his daughter?

!
!
179

A.
B.
C.
D.
E.

Early age of onset


Past hospitalisation
Frequent self-induced vomiting rather than food restriction
Very low BMI
Perfectionistic personality.

17 You are a general practitioner. A 25-year-old man with history of Aspergers


syndrome. He has history of violent and compulsive behaviour. After recent increase
in dose of psychotropic medications, his father discovers that he masturbates less
often and seems to develop erectile dysfunction. Which of the following medications
is MOST likely to cause erectile dysfunction?
A.
B.
C.
D.
E.
18

Alprazolam
Bupropion
Mirtazapine
Sertraline
Hydroxyzine
You are a general practitioner. A 30-year-old man comes to the clinic and claims that
he received a diagnosis of bipolar II disorder in the United States. Bipolar II disorder is
best described as:

A.
B.
C.
D.
E.

Dysthymia and cyclothymia


Hypomanic episode and depressive episode
Manic episode only
Manic episode and depressive episode
Seasonal affective disorder

Old age psychiatry


19 Which of the following statements about depressive!disorder!in!old!people!is!TRUE?
A. Depressive disorder in elderly is accompanied by a much lower suicide risk than in younger
adults.
B. Depressive disorder in elderly is less likely to be associated with paranoia as compared to
younger adults.
C. Elderly suffering from depressive disorder takes shorter time to respond as compared to
younger adults
D. Mild depressive disorder is more prevalent in elderly women than men.
E. Prevalence of depressive disorder rises sharply with age in the community.
20 Which!of!the!following!is!MOST!commonly!seen!as!a!normal!change!associated!

with!aging?
A.
B.
C.
D.
E.

Increase!in!acetylcholine!
Increase!in!paranoia!
Increase!in!brain!ventricle!size!!
Increased!electroencephalogram!(EEG)!frequency!
Increased!slowNwave!sleep.!

!
!
180

21 What!is!the!MOST!likely!diagnosis!in!a!75T!yearTold!man!with!declining!cognition,!
visual!hallucinations,!and!parkinsonism!symptoms?!!!
!
A.
B.
C.
D.
E.

Alzheimers!disease!
Dementia!with!Lewy!Bodies!
LateNonset!schizophrenia!
FrontoNtemporal!lobe!dementia!
Vascular!dementia.!

22 Which of the following statements is false with regards to delirium in


the elderly?
A. 30% or more of the elderly admitted to medical ward will develop delirium
during hospitalisation.
B. Antipsychotic drug is the first-line psychiatric treatment for delirium.
C. Delirium is often overlooked by health care professionals in the elderly.
D. Delirium typically resolves within 3 days after correcting the underlying cause.
E. Incidence of delirium increases with age.
Child and adolescent psychiatry
23 If a young boy presents with intellectual disability, autistic features and multiple
maternal relatives are carriers of a genetic condition. Which of the following diagnosis
is MOST likely?
A.
B.
C.
D.
E.

Angleman syndrome
Down syndrome
Prader Willi syndrome
Fragile X syndrome
Williams syndrome

Addiction and Substance abuse


24 You are an orthopaedic resident. A 22-year-old man suffers from cervical spondylosis
and results in severe neck pain. His pain does not respond to analgesics and
becomes very depressed and suicidal. His mother describes him as being obsessed
with online games. He is an active member of several online game groups. He spends
more around 20 hours per day in the internet caf and becomes very agitated when he
is not allowed to play online games. He has history of alcohol dependence. Which of
the following symptoms found in this patient is MOST important in establishing the
diagnosis of internet addiction?

!
!
181

A.
B.
C.
D.
E.
25

Active member of the online group


Agitation when not allowed to play online game
Cervical spondylosis
Depression and suicidal thought
History of alcohol abuse.
Which of the following is NOT a factor which increases the risk of relapse to cocaine use in a 30year-old man who recently became abstinent from cocaine?

A.
B.
C.
D.
E.

Feeling depressed, tired and the need to overeat.


Going to pubs where ice and special K are available.
Seeing a counsellor to talk about the previous use of cocaine.
Seeing a psychiatrist to complain about attention deficit and try to obtain methylphenidate.
Spending time with colleagues who use coke.

26 A 27-year-old woman presents with unplanned pregnancy. She smokes


cannabis 3-4 times per day. Her husband is concerned about the effects of
cannabis on her baby. During the interview, she avoids eye contact and
appears to be very anxious. She claims that she eats and sleeps well. She
thinks cannabis is safe for her foetus because she has a friend who delivered
a healthy baby despite continued use of cannabis during pregnancy. Which
Prochaskas and Diclementes stage of change BEST describes her current
status?
A.
B.
C.
D.
E.

Action
Contemplation
Denial
Precontemplation
Maintenance.

Psychopharmacology

27 Which!of!the!following!neurochemicals!is!metabolized!by!monoamine!oxidase? !
!
A.
B.
C.
D.
E.

Acetylcholine!
BrainNderived!neutrophic!factor!
Histamine!
Glutamate!
Serotonin.!

28 A 30-year-old man takes mirtazapine every night and experiences significant sedation
at 7.5mg/day. Which of the following neurotransmitter receptors are MOST likely to be
responsible for causing this side effect?
A.
B.
C.
D.
E.

Adrenergic receptors
Dopamine receptors
Glutamate receptors
Histamine receptors
Nicotinic receptors.

!
!
182

29 Which of the following psychotropic medications is the MOST likely to


raise the prolactin levels in female psychiatric patients?
A.
B.
C.
D.
E.

Aripiprazole
Clozapine
Lithium
Olanzapine
Risperidone.

30 Which of the following statements regarding quetiapine is LEAST CORRECT?


a.
b.
c.
d.
e.

Quetiapine has anxiolytic effects.


Quetiapine has antidepressant effects
Quetiapine has antipsychotic effects.
Quetiapine has extrapyramidal effects
Quetiapine has sedative effects.

31 Which of the following is NOT a well-established side effect of lithium?


A.
B.
C.
D.
E.

Hepatotoxicity
Hypothyroidism
Nephrotoxicity
Tremor
Weight Gain

32 A 30-year-old man suffers from bipolar disorder. He requests to be treated by


carbamazepine because it works on his friend. Which of the following genetic tests is
required by the Ministry of Health (MOH) before prescription of carbamazepine?
A.
B.
C.
D.
E.

Dopamine transporter gene - D*1502


Human leukocyte antigen gene - B*1502
Integrin alpha M gene I*1502
Mannan-binding lectin gene M*1502
Serotonin transporter gene S*1502.

33 A 25-year-old woman is treated by bupropion for depressive disorder. She gives you a
list of symptoms and attributes to the side effects of bupropion. Which of the
following symptoms is LEAST likely to be side effect of bupropion?
A. Anxiety
B. Nausea
C. Seizure

!
!
183

D. Transient hallucination
E. Weight gain.
34 A 30-year-old schizophrenia patient receives olanzapine 20mg per day. He is a chronic
smoker and agrees to cease smoking in the past one month. After he quitted smoking,
he experiences more side effects associated with 20mg of olanzapine which include
significant sedation and weight gain. Which of the following is MOST likely
explanation?
A.
B.
C.
D.
E.

He is not complaint to olanzapine.


He took an overdose of olanzapine.
After he stopped smoking, the hepatic metabolism of olanzapine reduces.
Nicotine reduces the risk of metabolic syndrome.
This is due to the effect of nicotine patch.

35 Which!of!the!following!is!the!mechanism!of!action!of!memantine,!a!medication!

used!to!slow!cognitive!decline!in!Alzheimers!dementia?
A.
B.
C.
D.
E.

Cholinesterase!inhibitor!
Dopamine!receptor!blocker!
GammaNaminobutyric!acid!receptor!agonist!
NNmethylNDNaspartate!receptor!antagonist!
Serotonergic!reuptake!inhibitor.!

Psychotherapy
36 A 30-year-old woman with panic disorder does not respond to an initial treatment with
a selective serotonin reuptake inhibitor (SSRI). Which of the following treatment is
considered the best approach?
K.
L.
M.
N.
O.

Benzodiazepine
Cognitive behaviour therapy
Hypnotherapy
Olanzapine
Psychodynamic psychotherapy.

37 You are a medical officer working in the army. A 22-year-old national serviceman
comes to see you because he is very angry with his male supervisor and wants to see
a psychologist. He also informs you that he is very angry with his own father. He
passed you an emotional 500-word open letter posted on his Facebook account. In his
letter, the patient mentions that his father has been abusive towards him. He finds his
male supervisor is as abusive as his father and the pattern keeps repeating itself.
Which of the following psychotherapy is MOST appropriate in this case?
A. Cognitive behaviour therapy
B. Problem solving therapy

!
!
184

C. Psychodynamic psychotherapy
D. Reminiscence therapy
E. Validation therapy.
38 A"20%year%old"woman"suffers"from"borderline"personality"disorder."She"works"as"a"

clerk." During" psychotherapy" session," she" discloses" that" she" really" hates" the"
psychologist"and"has"thoughts"of"killing"the"psychologist."Her"conviction"to"kill"the"
psychologist" is" 1" out" of" 10" (1" =very" unlikely," 10" =" very" likely)." Furthermore," she"
does"not"have"a"plan"or"not"sure"when"to"kill"the"psychologist."She"has"no"forensic"
history"and"no"history"of"violence."She"admits"the"idea"of"killing"the"psychologist"is"
her" own" thought." She" has" no" command" hallucination." " Which" of" the" following"
BEST"describes"this"phenomenon?

6
A.
B.
C.
D.
E.

Chronic felling of emptiness


Displacement
Projective identification
Splitting

Transference6

39 A 30-year-old man suffers from disseminated gonococcal infection. The medical team
asks him about unprotected sexual activities. He claims he has one stable sexual
partner and practises safe sex all the time. The medical team thinks that his condition
could due to an underlying autoimmune cause because he has no risky behaviour.
Several days later, he finally admits that he has sexual intercourse with commercial
sex worker once per month. Sometimes, he does not use condom to protective
himself. What is the initial defence mechanism used by this patient?
A.
B.
C.
D.
E.

Denial
Displacement
Projection
Projective identification
Undoing

Ethics and laws


40 You are a general practitioner. A 30-year-old female secretary complains of low mood,
poor appetite, poor sleep and suicidal thought. She is very depressed over the death
of her son who passed away 26 hours after birth. She has history of depression
without any psychotic feature. She underwent a traumatic delivery because an
obstetrician in private practice performed amniocentesis, induction of labour and
administered syntocinon 3 weeks before the expected date of delivery. The patient
later found out that the obstetrician induced labour because the doctor planned to go
for a holiday. Patient did not fully understand about the procedure and did not agree
with induction of labour which resulted in premature birth of her son. The obstetrician
claims that she does not have capacity to make decision due to history of depression.
She felt that the doctor performed the procedure against her will and the death of her
son was avoidable. Which ethical principle was violated by the obstetrician?
A. Autonomy

!
!
185

B.
C.
D.
E.

Beneficence
Confidentiality
Involuntary treatment
Justice.

41 You are a resident posted to the Accident and Emergency Department of


Institute of Mental Health (IMH). A 55-year-old man attended a party in a buffet
restaurant. He seldom drinks alcohol. In the party, he drank a few glasses of
hard liquor and could not find his way to the table. When a female waitress
tried to help him, he was irritable and physically assaulted her. The waitress
suffered from minor injury but the restaurant manager insisted to report to the
police. Subsequently, he was arrested and stayed in the police station for one
night. On the next day, he was sent to IMH for assessment. When you
interview him, he says, Oh gosh, I cant recall the event which occurred last
night. He does not have past psychiatric illness. Which of the following terms
BEST describes his behaviour leading to the arrest?
A. Actus reus
B. Jealousy
C. Automatism
D. Diminished responsibility
E. Mens rea.
Liaison Psychiatry
42 A 20-year-old man was given a high dose of intravenous haloperidol. He
develops high fever and you suspect that he may develop neuroleptic
malignant syndrome (NMS). All of the following clinical features suggest NMS
EXCEPT:
A.
B.
C.
D.
E.

Autonomic instability
Diaphoresis
Decreased catecholamines in urine
Myoglobinuria
Rigidity.

43 An obstetrician refers a 30-year-old woman who presents with


psychosis after delivery to you. She wants to know whether this woman
suffers from postpartum psychosis. You want to find out from the
obstetrician the onset of psychotic symptoms after delivery. Which of
the following days of onset is MOST congruent with the diagnosis of
postpartum psychosis?
A. 1- 6 weeks after delivery
B. 7-12 weeks after delivery
C. 13 -18 weeks after delivery
!
!
186

D. 19 - 24 weeks after delivery


E. 25 31 weeks after delivery.
!

44 A 40-year-old man went to South Korea for skiing and suffered from head
injury. All of the following symptoms are commonly associated with sequelae of
head injury EXCEPT:
A.
B.
C.
D.
E.

Light and noise sensitivity


Headache
Insomnia
Obsession
Word finding difficulty.

45 Which of the following is LEAST likely to be an area of concern in a 30-year-old


woman suffering from body dysmorphic disorder?
A.
B.
C.
D.
E.

Breast
Nose
Genitalia
Hair
Skin.

46 You are a medical resident. A 40-year-old man is admitted to the medical ward

due to altered mental state and requires further medical investigations. Past
records show a history of substance abuse. He sleeps poorly and paces
around the unit, restless and grumpy on the next day. He appears to be
confused and exhibits tachycardia at 106 beats per minutes. His palms and
forehead are sweaty and his tongue is showing a course tremor. When asked,
the patient says he feels anxious. Which of the following is the CORRECT
diagnosis?
A.
B.
C.
D.
E.

Alcohol withdrawal
Amphetamine withdrawal
Cannabis withdrawal
Nicotine withdrawal
Opioid withdrawal.

!
!
Paper!7!N!Questions!
History, mental state exam, psychopathology and risk
assessment!

3. You are a resident working in the accident and The answer is B.


emergency department (AED). A 30-year-old
schizophrenia patient is brought in by his mother.
He suddenly heard a voice asking him to use a sword
to stab himself and open up his abdomen. His
mother stopped him and bought him to the hospital. Explanation: This patient is
!
!
187

F.
G.
H.
I.
J.

He does not have antisocial behaviour. Which of the at high risk due to two
following BEST describes his current risk at the reasons: 1) command
AED?
hallucination 2) the
possibility of damaging the
High risk due to the possibility that he may attack other aorta and liver. He may die
in 30 seconds if aorta is
people.
High risk due to the possibility of damaging internal
damaged.
organs such as aorta and liver if he stabs himself.
Low risk due to the fact that he has informed his mother
and the chance to carry out action is low.
Year: 2014
Low risk due to the unlikeliness for him to harm
himself as a result of disorganized behaviour seen in
schizophrenia patients.
Moderate risk due to possibility of superficial cut on
abdomen.

4. You are a resident working in the accident and


emergency department. A 30-year-old bank officer is
brought in by her supervisors after she has a failed
suicide attempt in the bank. She was recently
transferred out from the head office to another branch.
According to her relatives, the patient claims that the
Chief Executive Officer (CEO) fell in love with her. She
spent hours waiting for the CEO after work. She tried to
stop his car from leaving the car park. The CEO has
never worked with her and does not know her
personally. Which of the following is the correct
diagnosis?

The!answer!is!C.!

F.
G.
H.
I.
J.

Year: 2014

Capgras!syndrome!
Charles!de!Bonnets!syndrome!
De!Clrambault's!syndrome!
Gansers!syndrome!
Othellos!syndrome!

!
Explanation:!The!patient!
suffers!from!delusion!of!
love!for!a!person!with!high!
status!but!has!no!contact!
with!the!patient.!
!

47 Which of the following is NOT a thought disorder found in


patients suffering from schizophrenia?

The answer is B.

F.
G.
H.
I.
J.

Echopraxia is the motor symptom


analogous to echolalia.
Echopraxia refers to the imitation
of movements and gestures of the
person the patient is observing.

Derailment
Echopraxia
Loosening of association
Knights move thinking
Word salad.

!
!
188

Year: 2013.!
Cognitive assessment!

48 Which of the following is NOT a prominent cognitive


feature of cortical dementia?

The!answer!is!A.!
!

F.
G.
H.
I.
J.

Problems with articulation


Problems with calculation
Problems with drawing a clock face
Problems with expression of emotion
Problems with recognition

Explanation:!Articulation!
difficulty!is!associated!with!
subcortical!dementia!which!
is!characterised!by!
neurological!sign.!
!

Year: 2014
!
49 A 20-year-old woman complains of low mood because
she is bullied in her workplace. She is a factory worker
and her supervisor complains that she is very slow at
work. She stays with her family and is able to look after
herself. Her mother describes the patient as simpleminded. When she was young, she exhibited delay in
understanding and learning language. She kept failing
the N level examination despite multiple attempts.
Which of the following BEST describes her current level
of intelligence?
F.
G.
H.
I.
J.

Normal intelligence
Mild intellectual disability
Moderate intellectual disability
Severe intellectual disability
Profound intellectual disability.

The!answer!is!B.!
!
Explanation:!She!has!mild!
intellectual!disability!which!
is!characterised!by!ability!to!
work!in!practical!
occupations!and!ability!to!
look!after!oneself.!
!
Her!slowness!at!work,!
repeated!failing!of!the!N!
level!examination!and!
delayed!in!language!
development!do!not!support!
normal!intelligence.!
!
Year: 2014

!
!

Psychiatric epidemiology

!
!
189

50 By 2020, which of the following illnesses is


predicted to have higher global disease burden
than depressive disorder?
F.
G.
H.
I.
J.

The!answer!is!D.!
!
Explanation:!Based!on!the!
prediction!of!WHO,!
ischaemic!heart!disease!will!
rank!No.1!while!depressive!
disorder!will!rank!No.2!in!
global!disease!burden.!

Anxiety disorder
Dementia
Diabetes
Ischaemic heart disease
Schizophrenia.

!
Year: 2014

!
51 Which of the following mood disorders has the HIGHEST
rate of co-occurrence with a substance abuse disorder?

Answer!is!D.!
!

F.
G.
H.
I.
J.

Adjustment disorder
Seasonal affective disorder
Cyclothymia
Bipolar disorder
Dysthymia.

Explanation:!Bipolar!
disorder!is!the!commonest!
mood!disorder!and!
demonstrates!the!highest!
rate!of!coNoccurrence!with!a!
substance!abuse!disorder!
among!all!the!choices.!
!
Year: 2014

!!
Psychiatric aetiology, diagnosis and classification

52 Which of the following neuroanatomical areas


is MOST implicated in obsessive compulsive
disorder?
F.
G.
H.
I.
J.

Amygdala and prefrontal cortex


Frontal lobe and parietal lobe
Hippocampus and locus coeruleus
Nucleus accumbens and prefrontal cortex
Orbito-frontal cortex and basal ganglia

!
!
190

!
The answer is E.

Explanation: Orbito-frontal
cortex and basal ganglia are
most implicated in compulsive
behaviours associated with
OCD.

Year: 2014

!
53 Which of the following is the MOST common cause of
secondary hypersomnia?

Answer!is!D.!
!

F.
G.
H.
I.
J.

Consumption of hydroxyzine
Dementia
Mixed anxiety and depression
Obstructive sleep apnea
Shift work.

Explanation:!Obstructive!
sleep!apnoea!is!the!most!
common!cause!of!secondary!
hypersomnia!
!
Year: 2014

!
54 You are a general practitioner. A 33-year-old man
complains of stress due to interpersonal problems and
long term unemployment. He claims that he suffers from
depression but does not require antidepressant
treatment. He has no history of self-harm or criminal
record. He has long-standing problems with his wife. His
wife mentions that he knows every detail about cooking
and household chores. She finds the 3- year marriage
miserable and has gone through a very difficult period.
His wife realises that she will never satisfy his
expectations. She describes the patient as being too
idealistic and extremely careful. When she cooks, the
patient tries to belittle her by watching her every single
movement and interjecting a lot of advice to make a
perfect meal. He often scolds her for spending money
and she is afraid of the patient. She finds him very rigid
with no sense of humour. Which of the following is the
correct diagnosis?

The!answer!is!D.!

F.
G.
H.
I.
J.

Avoidant personality disorder


Borderline personality disorder
Narcissistic personality disorder
Obsessive compulsive personality disorder
Paranoid personality disorder.

55 The mother of a schizophrenia patient wants to consult


you the risk of his dizygotic twin brother in developing
schizophrenia. Your answer is:
F.
G.
H.
I.

!
Explanation:!This!patient!is!
characterised!by!being!
perfectionistic,!rigid,!frugal!
and!preoccupation!with!
minor!details.!!!
His!personality!disorder!
resulted!in!long!term!
unemployment,!
interpersonal!problems!and!
marital!problems.!!

Year: 2014
!
The!answer!is!B.!
!
Explanation:!The!risk!for!
another!dizygotic!twin!to!
develop!schizophrenia!is!

4%
14%
24%
34%

!
!
191

J.

14%.!

44%.

Year: 2014
!
!

General adult psychiatry

56 You are a general practitioner. A 22-year-old


patient comes to consult you because she is
stressed. Her father boarded an aeroplane
which was found missing and probably has
crashed. She is not certain about the status of
her father and he is not contactable. At this
moment, she is MOST likely to develop which
of the following psychiatric conditions?

F.
G.
H.
I.
J.

The!answer!is!A.!
!
Explanation:!As!the!plane!has!
crashed,!the!chance!of!
survival!for!her!father!is!low.!
She!will!experience!
anticipatory!grief!during!the!
period!of!uncertainty.!
!

Anticipatory grief
Compensation neurosis
Dissociative state
Psychosis
Hypochondriasis.

Year: 2014
!

57 A 30-year-old schizophrenia patient drops out


from rehabilitation. The family member wants
to find out the underlying reason. Which of the
following factors is LEAST likely to restrict
rehabilitation potential?
F. Residual auditory hallucination
G. Intellectual disability
H. Lack of pleasure from social intervention and
physical activity
I. Poor motivation
J. Restricted affect.
!

The!answer!is!A.!
!
Explanation:!Auditory!
hallucination!is!positive!
symptom!and!responds!to!
antipsychotic!drug.!Other!
options!are!negative!
symptoms!of!schizophrenia!
and!intellectual!disability.!
These!factors!are!less!likely!
to!respond!to!antipsychotic!
treatment.!
!
Year:!2014!

58 Which!of!the!following!statements!concerning!
generalized!anxiety!disorder!is!FALSE?!!
!
!
192

The!answer!is!C.!

!
F.
G.
H.
I.
J.

Alcohol!misuse!is!a!common!comorbidity.!!
Cognitive!behaviour!therapy!is!generally!helpful.!
Excessive!worries!and!anxiety!symptoms!are!episodic.!
ShortNterm!benzodiazepines!can!be!an!effective!
treatment!
There!is!a!2!to!1!ratio!of!women!to!men!suffering!from!
this!disorder.!

Explanation:!Option!C!refers!
to!panic!disorder.!
!
Year:!2014!

59 Which!of!the!following!is!TRUE!regarding!delusional!
disorder?!

The!answer!is!E.!

F. Infrequent!hallucinations!totally!rule!out!the!diagnosis!of!
delusional!disorder.!
G. Onset!is!in!adolescence.!!
H. Point!prevalence!is!3%.!
I. From!genetic!point!of!view,!there!is!frequent!crossover!to!
other!psychotic!disorders!such!as!schizophrenia.!
J. The!persecutory!subtype!is!the!most!common.!!

Explanation:!Delusion!
disorder!is!quite!rare,!from!
0.025!to!0.03%.!The!mean!
age!of!onset!is!40!years.!
Delusional!disorder!does!not!
cross!over!in!family!studies!
with!schizophrenia!or!mood!
disorder.!Olfactory!
hallucinations!in!a!somatic!
delusion!of!body!odour,!may!
occur.!

!!

!
!
Year:!2014.!
60 A 45-year-old man consults you because he is stressed
over his daughters eating disorder. His daughter suffers
from anorexia nervosa. He is very concerned about her
condition. He wants to seek your reassurance. Which of
the following indicates a GOOD prognostic factor for his
daughter?
F.
G.
H.
I.
J.

Early age of onset


Past hospitalisation
Frequent self-induced vomiting rather than food restriction
Very low BMI
Perfectionistic personality.

The!answer!is!A.!
!
Explanation:!Based!on!eating!
disorder!research,!early!age!
of!onset!indicates!good!
prognostic!factor.!The!other!
factors!are!poor!prognostic!
factors.!
!
Year:!2014.!!
!

!
!
193

61 You are a general practitioner. A 25-year-old man with


history of Aspergers syndrome. He has history of
violent and compulsive behaviour. After recent increase
in dose of psychotropic medications, his father
discovers that he masturbates less often and seems to
develop erectile dysfunction. Which of the following
medications is MOST likely to cause erectile
dysfunction?

The!answer!is!D.!

F.
G.
H.
I.
J.

Alprazolam
Bupropion
Mirtazapine
Sertraline
Hydroxyzine

!
Explanation:!Sertraline!is!A!
SSRI!and!most!likely!to!cause!
sexual!dysfunction!among!all!
the!choices.!

Year:!2014.!!
!

62 You are a general practitioner. A 30-year-old man comes


to the clinic and claims that he received a diagnosis of
bipolar II disorder in the United States. Bipolar II
disorder is best described as:
F.Dysthymia and cyclothymia
G. Hypomanic episode and depressive episode
H. Manic episode only
I. Manic episode and depressive episode
J. Seasonal affective disorder

The answer is B.

Explanation: Bipolar I
disorder is characterised
by at least one manic
episode. Bipolar II disorder
is characterised by
hypomanic episode and
depressive episode.

Year: 2013.

Old age psychiatry


63 Which of the following statements about depressive!
disorder!in!old!people!is!TRUE?

The!answer!is!D.!
!

F. Depressive disorder in elderly is accompanied by a much


lower suicide risk than in younger adults.
G. Depressive disorder in elderly is less likely to be associated
with paranoia as compared to younger adults.
H. Elderly suffering from depressive disorder takes shorter time
to respond as compared to younger adults
I. Mild depressive disorder is more prevalent in elderly women
than men.
J. Prevalence of depressive disorder rises sharply with age in
the community.

!
!
194

Explanation:!Depression!in!
elderly!is!accompanied!by!
higher!suicide!risk!and!more!
paranoia.!Elderly!takes!longer!
time!to!respond!as!they!cannot!
tolerate!medications!as!good!
as!young!adults.!Prevalence!of!
depression!in!elderly!is!around!
3%!(10N15%!if!depressive!
symptoms!are!included).!Only!
10%!of!depression!emerges!in!

elderly!and!there!is!no!sharp!
rise!in!prevalence.!
!
Year:!2014.!!
!
64 Which!of!the!following!is!MOST!commonly!seen!as!a!

normal!change!associated!with!aging?
F.
G.
H.
I.
J.

Increase!in!acetylcholine!
Increase!in!paranoia!
Increase!in!brain!ventricle!size!!
Increased!electroencephalogram!(EEG)!frequency!
Increased!slowNwave!sleep.!

The!answer!is!C.!
!
Explanation:!Option!A,!D!and!E!
should!be!reduced.!Increase!in!
paranoia!occurs!in!late!onset!
depression,!bipolar!disorder!
and!schizophrenia!but!not!part!
of!normal!aging.!
!
Year:!2014!

65 What!is!the!MOST!likely!diagnosis!in!a!75T!yearTold!
man!with!declining!cognition,!visual!hallucinations,!
and!parkinsonism!symptoms?!!!
!
F.
G.
H.
I.
J.

The!answer!is!B.!
!
Explanation:!The!classical!
triad!of!dementia!with!Lewy!
Bodies!is!memory!loss,!visual!
hallucination!and!
parkinsonism.!

Alzheimers!disease!
Dementia!with!Lewy!Bodies!
LateNonset!schizophrenia!
FrontoNtemporal!lobe!dementia!
Vascular!dementia.!

!
Year:!2014!

66 Which of the following statements is false with


regards to delirium in the elderly?

The!answer!is!D!

F. 30% or more of the elderly admitted to medical


ward will develop delirium during hospitalisation.
G. Antipsychotic drug is the first-line psychiatric
treatment for delirium.
H. Delirium is often overlooked by health care
professionals in the elderly.
I. Delirium typically resolves within 3 days after

Explanation:!Delirium!will!take!
long!time!to!resolve!and!not!
typically!resolve!in!3!days.!

!
!
195

!
Year:!2014!

correcting the underlying cause.


J. Incidence of delirium increases with age.
!

Child and adolescent psychiatry


67 If a young boy presents with intellectual disability,
autistic features and multiple maternal relatives are
carriers of a genetic condition. Which of the following
diagnosis is MOST likely?
F.
G.
H.
I.
J.

The!answer!is!D.!
!
Explanation:!Males!with!
intellectual!disability,!autistic!
features!and!have!maternal!
relatives!being!carrier!highly!
suggestive!of!fragile!X!
syndrome.!Option!A,!C,!and!E!
are!microdeletion!syndromes!
and!less!likely!to!run!in!
families.!

Angleman syndrome
Down syndrome
Prader Willi syndrome
Fragile X syndrome
Williams syndrome

!
Year:!2014!
!
!

Addiction and Substance abuse


68 You are an orthopaedic resident. A 22-year-old man
suffers from cervical spondylosis and results in severe
neck pain. His pain does not respond to analgesics and
becomes very depressed and suicidal. His mother
describes him as being obsessed with online games. He
is an active member of several online game groups. He
spends more around 20 hours per day in the internet
caf and becomes very agitated when he is not allowed
to play online games. He has history of alcohol
dependence. Which of the following symptoms found in
this patient is MOST important in establishing the
diagnosis of internet addiction?
F.
G.
H.
I.
J.

Active member of the online group


Agitation when not allowed to play online game
Cervical spondylosis
Depression and suicidal thought
History of alcohol abuse.

The!answer!is!B.!
!
Explanation:!This!patient!has!
psychological!withdrawal!(i.e.!
agitation)!when!he!is!not!
allowed!to!play!online!game.!
This!is!common!in!other!forms!
of!substances!abuses!(e.g.!
opioid!dependence)!when!the!
subject!is!denied!access!to!the!
substance.!
!
Year:!2014!
!

!
!
196

!
69 Which of the following is NOT a factor which increases the risk
of relapse to cocaine use in a 30-year-old man who recently
became abstinent from cocaine?
F.
G.
H.
I.
J.

Feeling depressed, tired and the need to overeat.


Going to pubs where ice and special K are available.
Seeing a counsellor to talk about the previous use of cocaine.
Seeing a psychiatrist to complain about attention deficit and try to
obtain methylphenidate.
Spending time with colleagues who use coke.

The!answer!is!C.!
!
Explanation:!Seeing!a!
counsellor!to!talk!about!the!
previous!use!does!not!increase!
the!risk!of!relapse.!Option!A!
refers!to!crash!from!
stimulation.!Option!B,!C,!and!E!
will!increase!the!exposure!to!
stimulants.!
!
Year:!2014!
!

70 A 27-year-old woman presents with unplanned


pregnancy. She smokes cannabis 3-4 times per
day. Her husband is concerned about the effects of
cannabis on her baby. During the interview, she
avoids eye contact and appears to be very anxious.
She claims that she eats and sleeps well. She
thinks cannabis is safe for her foetus because she
has a friend who delivered a healthy baby despite
continued use of cannabis during pregnancy.
Which Prochaskas and Diclementes stage of
change BEST describes her current status?

The!answer!is!D.!

F.
G.
H.
I.
J.

Action
Contemplation
Denial
Precontemplation
Maintenance.

!
Explanation:!She!has!no!
motivation!to!change!with!
impaired!insight!on!the!effect!
of!cannabis!on!the!foetus.!This!
stage!is!known!as!preN
contemplation.!

Year:!2014!
!
!

Psychopharmacology

71 Which!of!the!following!neurochemicals!is!
metabolized!by!monoamine!oxidase? !

The!answer!is!E.!

F.
G.
H.
I.
J.

!
Acetylcholine!
BrainNderived!neutrophic!factor!
Histamine!
Glutamate!
Serotonin.!

Explanation:!This!refers!to!
pharmacodynamic!action!of!

!
!
197

MAOI.!
!
Year:!2014!
72 A 30-year-old man takes mirtazapine every night and
experiences significant sedation at 7.5mg/day. Which of
the following neurotransmitter receptors are MOST likely
to be responsible for causing this side effect?
F.
G.
H.
I.
J.

The answer is D.

Explanation: Besides the action


of mirtazapine on the serotonin
receptors, its action on the
histamine receptors cause
sedation and weght gain.

Adrenergic receptors
Dopamine receptors
Glutamate receptors
Histamine receptors
Nicotinic receptors.

!
Year:!2014!

73 Which of the following psychotropic


medications is the MOST likely to raise the
prolactin levels in female psychiatric patients?
F.
G.
H.
I.
J.

The answer is E.
Explanation: Risperidone is
most potent D2 receptor
blockage among all second
generation antipsychotic
drugs.

Aripiprazole
Clozapine
Lithium
Olanzapine
Risperidone.

Year:!2014

74 Which of the following statements regarding


quetiapine is LEAST CORRECT?
f.
g.
h.
i.
j.

Quetiapine has anxiolytic effects.


Quetiapine has antidepressant effects
Quetiapine has antipsychotic effects.
Quetiapine has extrapyramidal effects
Quetiapine has sedative effects.

The answer is D.

Explanation: Unlike the other


second
generation
antipsychotic
drug,
the
incidence of extrapyramidal
effects with quetiapine is the
same as with placebo.

Year:!2014
!
!
198

75 Which of the following is NOT a well-established side


effect of lithium?

The!answer!is!A.!
!

F.
G.
H.
I.
J.

Hepatotoxicity
Hypothyroidism
Nephrotoxicity
Tremor
Weight Gain

Explanation:!Lithium!is!
excreted!by!kidney.!It!has!
minimal!effects!on!the!liver.!
!
Year:!2014
!

76 A 30-year-old man suffers from bipolar disorder. He


requests to be treated by carbamazepine because it
works on his friend. Which of the following genetic tests
is required by the Ministry of Health (MOH) before
prescription of carbamazepine?
F.
G.
H.
I.
J.

Dopamine transporter gene - D*1502


Human leukocyte antigen gene - B*1502
Integrin alpha M gene I*1502
Mannan-binding lectin gene M*1502
Serotonin transporter gene S*1502.

The!answer!is!B.!
!
Explanation:!!The!MOH!and!
Health!Science!Authority!of!
Singapore!recommends!doctor!
to!check!HLAB*=1502!
genotype!before!prescribing!
carbamazepine.!Those!who!are!
positive!for!this!genotype!may!
have!high!risk!of!developing!
Steven!Johnson!syndrome.!
!
Year:!2014
!

77 A 25-year-old woman is treated by bupropion for


depressive disorder. She gives you a list of symptoms
and attributes to the side effects of bupropion. Which of
the following symptoms is LEAST likely to be side effect
of bupropion?
F.
G.
H.
I.
J.

Anxiety
Nausea
Seizure
Transient hallucination
Weight gain.

The!answer!is!E.!
!
Explanation:!Bupropion!is!
associated!no!weight!gain!and!
mild!degree!of!weight!loss.!
!
Year:!2014
!

!
!
199

78 A 30-year-old schizophrenia patient receives olanzapine


20mg per day. He is a chronic smoker and agrees to
cease smoking in the past one month. After he quitted
smoking, he experiences more side effects associated
with 20mg of olanzapine which include significant
sedation and weight gain. Which of the following is
MOST likely explanation?
F. He is not complaint to olanzapine.
G. He took an overdose of olanzapine.
H. After he stopped smoking, the hepatic metabolism of
olanzapine reduces.
I. Nicotine reduces the risk of metabolic syndrome.
J. This is due to the effect of nicotine patch.

The!answer!is!C.!
!
Explanation:!Nicotine!induces!
the!metabolism!of!olanzapine!
via!cytochrome!P450!1A2!and!
reduces!its!serum!
concentration.!After!he!quits!
smoking,!the!serum!
concentration!of!olanzapine!
has!increased!and!resulted!in!
more!side!effects.!
!
Year:!2014.!
!

79 Which!of!the!following!is!the!mechanism!of!action!of!

F.
G.
H.
I.
J.

The!answer!is!D.!

memantine,!a!medication!used!to!slow!cognitive!
decline!in!Alzheimers!dementia?

Cholinesterase!inhibitor!
Dopamine!receptor!blocker!
GammaNaminobutyric!acid!receptor!agonist!
NNmethylNDNaspartate!receptor!antagonist!
Serotonergic!reuptake!inhibitor.!

Explanation:!Memantine!
works!on!the!NMDA!
receptors!and!reduces!the!
neurotoxicity!caused!by!
glutamate.!
!
Year:!2014.!
!
!

Psychotherapy
80 A 30-year-old woman with panic disorder does not
respond to an initial treatment with a selective serotonin
reuptake inhibitor (SSRI). Which of the following
treatment is considered the best approach?
P.
Q.
R.
S.
T.

The!answer!is!B.!
!
Explanation:!The!effect!of!CBT!
is!as!efficacious!as!SSRI.!

Benzodiazepine
Cognitive behaviour therapy
Hypnotherapy
Olanzapine
Psychodynamic psychotherapy.

!
!

!
!
200

Year:!2014.!
81 You are a medical officer working in the army. A 22-yearold national serviceman comes to see you because he is
very angry with his male supervisor and wants to see a
psychologist. He also informs you that he is very angry
with his own father. He passed you an emotional 500word open letter posted on his Facebook account. In his
letter, the patient mentions that his father has been
abusive towards him. He finds his male supervisor is as
abusive as his father and the pattern keeps repeating
itself. Which of the following psychotherapy is MOST
appropriate in this case?
F.
G.
H.
I.
J.

Cognitive behaviour therapy


Problem solving therapy
Psychodynamic psychotherapy
Reminiscence therapy
Validation therapy.

82

A" 20%year%old" woman" suffers" from" borderline"


personality" disorder." She" works" as" a" clerk." During"
psychotherapy" session," she" discloses" that" she" really"
hates"the"psychologist"and"has"thoughts"of"killing"the"
psychologist."Her"conviction"to"kill"the"psychologist"is"
1" out" of" 10" (1" =very" unlikely," 10" =" very" likely)."
Furthermore," she" does" not" have" a" plan" or" not" sure"
when" to" kill" the" psychologist." She" has" no" forensic"
history" and" no" history" of" violence." She" admits" the"
idea" of" killing" the" psychologist" is" her" own" thought."
She" has" no" command" hallucination." " Which" of" the"
following"BEST"describes"this"phenomenon?

The!answer!is!C!
!
Explanation:!Psychodynamic!
psychotherapy!is!most!
appropriate!in!this!case!
because!this!patient!needs!to!
analyse!the!pattern!of!history!
repeating!itself.!He!may!learn!
not!to!project!his!father!onto!
the!male!supervisor.!!
!
Year:!2014.!
The!answer!is!E.!
!
Explanation:!The!patient!
exhibits!intense!negative!
transference!towards!the!
psychologist.!She!does!not!
have!the!real!intention!to!kill!
the!psychologist!but!she!
mentions!this!due!to!intense!
negative!transference.!!
!

F.
G.
H.
I.
J.

Chronic felling of emptiness


Displacement
Projective identification
Splitting

Year:!2014.!

Transference6

83 A 30-year-old man suffers from disseminated


gonococcal infection. The medical team asks him about
unprotected sexual activities. He claims he has one
stable sexual partner and practises safe sex all the time.
The medical team thinks that his condition could due to
an underlying autoimmune cause because he has no
risky behaviour. Several days later, he finally admits that
he has sexual intercourse with commercial sex worker
once per month. Sometimes, he does not use condom to

!
!
201

The!answer!is!A.!
!
Explanation:!He!denies!the!
threatening!fact!that!he!has!
been!practising!unprotected!
sex!with!commercial!sex!

protective himself.
What is
mechanism used by this patient?
F.
G.
H.
I.
J.

the

initial

workers!and!result!in!
gonorrhoea!infection.!

defence

Denial
Displacement
Projection
Projective identification
Undoing

!
!
Year:!2014!
!

Ethics and laws

The!answer!is!A.!

84 You are a general practitioner. A 30-year-old female


secretary complains of low mood, poor appetite, poor
sleep and suicidal thought. She is very depressed over
the death of her son who passed away 26 hours after
birth. She has history of depression without any
psychotic feature. She underwent a traumatic delivery
because an obstetrician in private practice performed
amniocentesis, induction of labour and administered
syntocinon 3 weeks before the expected date of
delivery. The patient later found out that the obstetrician
induced labour because the doctor planned to go for a
holiday. Patient did not fully understand about the
procedure and did not agree with induction of labour
which resulted in premature birth of her son. The
obstetrician claims that she does not have capacity to
make decision due to history of depression. She felt that
the doctor performed the procedure against her will and
the death of her son was avoidable. Which ethical
principle was violated by the obstetrician?
F.
G.
H.
I.
J.

!
Explanation:!Most!depressed!
patients!have!the!capacity!to!
give!consent.!Based!on!the!
history,!there!is!no!evidence!to!
suggest!that!she!lacks!the!
capacity!to!make!decision!!(e.g.!
no!psychotic!features,!working!
as!a!secretary).!The!
obstetrician!did!not!respect!
patients!autonomy!before!
carrying!out!aminocentesis,!
induction!of!labour!and!
administering!syntocinon.

Autonomy
Beneficence
Confidentiality
Involuntary treatment
Justice.

Year:!2014.!

85 You are a resident posted to the Accident and


Emergency Department of Institute of Mental
Health (IMH). A 55-year-old man attended a party in
a buffet restaurant. He seldom drinks alcohol. In
the party, he drank a few glasses of hard liquor
and could not find his way to the table. When a
female waitress tried to help him, he was irritable
and physically assaulted her. The waitress
suffered from minor injury but the restaurant
manager insisted to report to the police.
Subsequently, he was arrested and stayed in the
police station for one night. On the next day, he
was sent to IMH for assessment. When you
!
!
202

The!answer!is!C.!
!
Explanation:!Automatism!is!a!
legal!term!and!the!person!
commits!a!crime!without!his!or!
her!own!personal!control!or!
awareness!due!to!an!
underlying!medical!condition!
or!substances.!Automatism!can!
be!caused!by!alcohol!

interview him, he says, Oh gosh, I cant recall the


event which occurred last night. He does not have
past psychiatric illness. Which of the following
terms BEST describes his behaviour leading to the
arrest?

!
Year:!2014.!

A. Actus reus
B. Jealousy
C. Automatism
D. Diminished responsibility
E. Mens rea.

Liaison Psychiatry
86 A 20-year-old man was given a high dose of
intravenous haloperidol. He develops high fever
and you suspect that he may develop neuroleptic
malignant syndrome (NMS). All of the following
clinical features suggest NMS EXCEPT:
F.
G.
H.
I.
J.

intoxication,!sleep!walking,!
epilepsy!and!hypoglycaemia.!!

Autonomic instability
Diaphoresis
Decreased catecholamines in urine
Myoglobinuria
Rigidity.

The answer is C.

Explanation: Patients
suffering from NMS usually
present with elevated CK,
ALT, AST, LDH in the serum
and high levels of myoglobin
and protein in the urine

Year:!2014.

87 An obstetrician refers a 30-year-old woman


who presents with psychosis after delivery to
you. She wants to know whether this woman
suffers from postpartum psychosis. You want
to find out from the obstetrician the onset of
psychotic symptoms after delivery. Which of
the following days of onset is MOST congruent
with the diagnosis of postpartum psychosis?

F.
G.
H.
I.
J.

1- 6 weeks after delivery


7-12 weeks after delivery
13 -18 weeks after delivery
19 - 24 weeks after delivery
25 31 weeks after delivery.

The!answer!is!A.!
!
Explanation:!The!onset!is!
usually!between!1!to!6!weeks!
because!the!progresterone!
levels!fall!but!dopamine!levels!
rise!in!the!days!after!child!
birth.!
!
Year:!2014.!

88 A 40-year-old man went to South Korea for skiing The!answer!is!D.!


and suffered from head injury. All of the following
symptoms are commonly associated with sequelae of
!
!
203

F.
G.
H.
I.
J.

head injury EXCEPT:

Light and noise sensitivity


Headache
Insomnia
Obsession
Word finding difficulty.

Explanation:!Obsession!is!not!a!
common!symptom!of!postN
concussion!syndrome.!
!
!
Year:!2014.!

89 Which of the following is LEAST likely to be an area of


concern in a 30-year-old woman suffering from body
dysmorphic disorder?

The!answer!is!C.!

F.
G.
H.
I.
J.

Explanation:!Genitalia!is!least!
likely!to!be!an!area!of!concern!
among!the!other!options.!The!
common!areas!are!listed!as!
follows:!skin!(73%),!hair!
(56%),!nose!(37%),!weight!
(22%),!stomach!(22%),!
breasts/chest/nipples!(21%)!!

Breast
Nose
Genitalia
Hair
Skin.

eyes!(20%)!thighs!(20%)!teeth!
(20%).!
!
Year:!2014.!
90 You are a medical resident. A 40-year-old man is

admitted to the medical ward due to altered mental


state and requires further medical investigations.
Past records show a history of substance abuse.
He sleeps poorly and paces around the unit,
restless and grumpy on the next day. He appears
to be confused and exhibits tachycardia at 106
beats per minutes. His palms and forehead are
sweaty and his tongue is showing a course tremor.
When asked, the patient says he feels anxious.
Which of the following is the CORRECT diagnosis?
F.
G.
H.
I.
J.

The!answer!is!A.!
!
Explanation:!The!patient!
suffers!from!delirium!tremens,!
resulted!from!alcohol!
withdrawal.!
!
Year:!2014.!

Alcohol withdrawal
Amphetamine withdrawal
Cannabis withdrawal
Nicotine withdrawal
Opioid withdrawal.

!
!
!
204

!
!
!
!
!
!
!
!
!
!
!
!

!
!
205

Paper 8- Questions

History, mental state exam, psychopathology, diagnosis


1. You are a resident doctor working at the Institute of Mental Health (IMH). The police
officers bought a 39-year-old man charged with possessing a scheduled weapon
and was sent to the IMH for a psychiatric evaluation. Although he speaks English,
you cannot make out what he is saying because his speech becomes diffuse and
unfocused. Which of the following best describes his psychopathology?
A.
B.
C.
D.
E.

Circumstantiality
Loosening of association
Thought insertion
Thought withdrawal
Tangentiality.

2. You are a resident working at the Accident and Emergency Department (AED). A
35-year-old man was sent to the AED for psychiatric assessment by an ambulance.
He was involved in a road traffic accident. He was a passenger in a taxi. He told the
taxi driver that he was God and threatened the driver to believe that he was the
saviour of the world. The driver found him very irritable and he kept hitting the head
of the driver. The driver stopped the taxi and tried to call the police. The patient
drove the taxi at high speed and crashed the taxi into a lamp post. He was not under
influence of alcohol at the time of accident. You are required to enter a diagnosis
into the AED computerised system. Based on the information provided, which of the
following psychiatric diagnosis is most relevant?
A.
B.
C.
D.
E.

Antisocial personality disorder


Bipolar disorder
Conversion disorder
Dissociative disorder
Schizophrenia.

Cognitive assessment
3. A 30-year-old woman has received 6 sessions of electroconvulsive therapy (ECT).
After the ECT, she complains of cognitive impairment and attributes the following
symptoms as side effects of ECT. Which of the following is most likely caused by
ECT?
A.
B.
C.
D.
E.

Cannot follow the instructions given by her supervisor after she returns to work
Cannot recall the password of her email account
Cannot remember details of her ex-marriage
Difficulty to perform two tasks at one time
Unable to calculate.

!
!
206

4. You are administering the Serial 7 test to a patient. His answers are listed as follows: 100
7 = 90; 90 7 = 77; 77 7 = 70; 70 7 = 63; 63 7 = 50. What is his score?
A.
B.
C.
D.
E.

0 out of 5
1 out of 5
2 out of 5
3 out of 5
4 out of 5.

Psychiatric epidemiology
5. Which of the following is the most common psychiatric co-morbidity of panic
disorder?
A. Alcohol misuse
B. Agoraphobia
C. Avoidant personality disorder
D. Benzodiazapine misuse
E. Generalised anxiety disorder.
Psychiatric aetiology, diagnosis and classification

6. The prevalence of borderline personality disorder in Singapore and worldwide


has increased in the past three decades. The most likely explanation for this
observation is:
A.
B.
C.
D.
E.

Changes in diagnostic criteria


Increase in competition in schools
Increase in divorce rate
Increase in immigration and migration
Increasing urbanization of the population.

7. Which of the following is least likely to be a risk factor for delusional disorder?
K.
L.
M.
N.
O.

Having a religion
Increased age
Immigration
Sensory impairment
Social isolation.

8. The genetic polymorphisms of which of the following neurotransmitter transporter


genes affect the risk of developing depressive disorder?
A. Dopamine
!
!
207

B.
C.
D.
E.
9.

-Aminobutyric acid (GABA)


Glutamate
Norephinephrine
Serotonin.
A 40-year-old patient with schizophrenia presents to the emergency department with
confusion and agitation following an overdose of haloperidol. The patient has a
temperature of 41 degree Celsius, blood pressure of 160/100 mm Hg, and pulse of
120 beats per minute. Physical examination reveals rigidity, hot, wet skin, and
decreased bowel sounds. This patients symptoms, are most likely caused by
blocking of which of the following receptors?

A. Alpha1-adrenergic
B. Dopaminergic
C. Histaminergic
D. Muscurinic cholinergic
E. Nicotinic cholinergic.
General adult psychiatry
10. A 30-year-old man suffers from schizophrenia. He first exhibited first rank symptoms
at the age of 16 and the onset was insidious. He exhibits anhedonia although his
hallucinations are under control by antipsychotic drug. His uncle suffers from
depressive disorder. All of the following are poor prognostic factors except?
A.
B.
C.
D.
E.

Anhedonia
Exhibition of first rank symptoms at the age of 16
Family history of depressive disorder
Male gender
Insidious onset.

11. You are the resident working in the Accident and Emergency Department. A
schizophrenia patient informs you that he has homicidal thought. Which of the
following sign or symptom is least important in predicting homicide?
A.
B.
C.
D.
E.

Command hallucinations
Delusion of reference
History of previous violence
Irresistible urge to attack
Need to defend oneself as are result of persecutory delusion.

12. All of the following are poor prognostic signs for obsessive compulsive disorder
except:
A.
B.
C.
D.

Bizarre compulsions
Childhood onset
Episodic course
Comorbid major depression

!
!
208

E. Succumbing to obsessions and compulsions.


13. You are a general practitioner. A 35-year-old hotel butler came to see you because
his lawyer wants him to be seen by a psychiatrist. He was recently arrested for
installing pinhole camera in the hotel guestrooms. He secretly filmed couples having
sex in the hotel guestrooms. He has filmed more than 100 videos and enjoyed
watching them over and over again. He claims that he has the compulsion to collect
those videos and he feels relieved after the arrest. Which of the following is the most
likely diagnosis?
A.
B.
C.
D.
E.

Erotomania
Kleptomania
Obsessive compulsive disorder
Sadomasochism
Voyeurism.

14. A 30-year-old woman always believes that she is socially inept and fears of negative
evaluation by other people. Her sisters describe her as timid and insecure. She is
single and stays with two elder sisters. She was admitted to the psychiatric ward due
to nervous breakdown after meeting new colleagues in her workplace. She refuses
to attend family meeting with her sisters because she feels that her sisters do not
like her although they appear to be very concerned about her condition. She is
isolated and has one best friend in her work place. Which of the following personality
best describes this patient?
A. Avoidant personality disorder
B. Borderline personality disorder
C. Histrionic personality disorder
D. Paranoid personality disorder
E. Schizoid personality disorder
15. Which of the following is not a common feature of serotonin syndrome?
A.
B.
C.
D.
E.

Acidosis
Diaphoresis
Hyperreflexia
Hypothermia
Myoclonus.

!
!
209

16. You are a resident working in the accident and emergency department
(AED). A single mother was sent to the AED with her 2-year-old daughter.
She was arrested by police because she tried to assault her daughter and
stopped by passer-by. According to the informant, her partner refuses to
marry her due to her drug habit. She does not have past forensic history. She
appears to be agitated and restless. At one moment, she wants to stay in the
AED. At another moment, she wants to walk out of the AED. Physical
examination reveals nystagmus. Which of the following diagnosis is most
relevant?
A. Antisocial personality disorder
B. Cannabis intoxication
C. Delusion of jealousy
D. Paranoid schizophrenia
E. Phencyclidine intoxication
Old age psychiatry
17. You are about to start a selective serotonin reuptake inhibitor (SSRI) for an 80-yearold man who suffers from depression. Which of the following pharmacokinetic
changes has the least effect on drug therapy in the elderly?
A.
B.
C.
D.
E.

Absorption
Distribution
Excretion
Metabolism
Protein binding.

18. A 79-year-old man complains of extreme drowsiness when he goes to toilet at


midnight and he almost fell down on several occasions. He wants to avoid fall at
night. The following is the list of his medications. Which of the following medications
should be stopped?
A.
B.
C.
D.
E.

Melatonin
Mirtazapine
Risperidone
Rivastigmine
Sodium valproate.

19. A 70-year-old woman suffering from bipolar disorder and her daughter came to see
you today. Her daughter read about the symptoms of bipolar disorder and found her
mother different from younger adults suffering from the same condition. Which of the
following clinical features is more likely to be found in this 70-year-old woman?
A.
B.
C.
D.

Demonstrates less paranoid ideas as compared to adult patients


Demonstrates more flight of ideas as compared to adult patients
More likely to be hyperactive as compared to adult patients
More likely to demonstrate mixed (depressive/manic) clinical presentations as

!
!
210

compared to adult patients


E. More spending as compared to adult patients.
20. Which of the following is most important risk factor for vascular dementia?
A.
B.
C.
D.
E.

History of depression
History of poor academic performance
History of hypertension
High level of physical activity
High level of high density lipoproteins.

21. You are the visiting physician of a nursing home. The nursing home staffs feel very
helpless when handling aggression in dementia patients and they need to seek your
advice. Which of the following psychotropic medication have the most evidence in
managing aggression in patients suffering from dementia?
A.
B.
C.
D.
E.

Antipsychotic drug
Beta-blocker
Benzodiazapine
Lithium
Stimulant.

Child and adolescent psychiatry


22. In children and adolescents, high stimulation-seeking behaviour is associated with
which of the following conditions?
A.
B.
C.
D.
E.

Adolescent-onset schizophrenia
Autism
Conduct disorder
Obsessive-compulsive disorder
School refusal.

23. Children with autism are most comfortable in which of the following situations?
A. Attending a party where there are a lot of surprises and have a chance to meet
different people.
B. Attending drama classes and imagines oneself playing different roles in the drama.
C. Looking for toys in a multi-storey department store from one level to another level.
D. Travelling in a bus when it stops at every bus stop until it reaches the bus terminal.
!
!
211

E. Going to school where there are multiple classes to attend in a day and teaching
content varies from subject to subject.
24. Which of the following statements is most correct regarding the impact of divorce on
children?
A.
B.
C.
D.
E.

Boys aged two to four are not significantly affected.


Children do best if they have no contact with the father for two years.
Most children have no significant problems after three to six months.
Socio-economic status is not associated with the degree of impairment.
To evaluate the impact, one must take into consideration the age and the
developmental level of the child.

25. A 15-year-old obese female is brought by her parents for an evaluation as part of the
admission process for a weight management program. The adolescent agrees with
the fact that she is overweight and she is keen to lose weight. Which of the following
is most indicative that she will be successful in the program?
A. Her current BMI is between 26 and 30.
B. Mother suffered from obesity and successfully lost weight
C. No family history of eating disorders
D. Motivation to take antidepressant
E. Willingness to change her eating habits.
Substance abuse
26. Methylene dimethylamphteamine (street name: ecstasy) usually cause people with
which of the following symptom?
a. Amotivation
b. Closeness to others
c. Depression
d. Introversion
e. Phobia.

27. A 30-year-old adult claims to suffer from adult onset attention deficit and
hyperactivity disorder (ADHD). He has been seeing different doctors to obtain
methylphenidate. He claims that he lost all methylphenidate in the bus few days ago
and his wife has noticed that he looks very tired. Which of the following is most
appropriate explanation for his tiredness?
A. He develops depression and psychomotor retardation
B. He exhibits secondary gain and seeking attention from his wife
C. He suffers from comorbid somatisation disorder in addition to ADHD
D. He suffers from psychological withdrawal symptoms from stimulant misuse
E. He suffers from stimulant intoxication.
28. The most important objective of psychological intervention in treating people
suffering from alcohol misuse is:

!
!
212

A.
B.
C.
D.
E.

To conduct in a group therapy format


To apply cognitive behaviour therapy (CBT) as the first approach
To combine psychological intervention and pharmacotherapy (e.g. disulfiram)
To focus on achieving and maintaining abstinence or reducing alcohol use
Explore unresolved psychodynamic conflicts.

Psychopharmacology
29. Which of the following medications is not associated with weight gain?
A.
B.
C.
D.
E.

Clozapine
Mirtazapine
Olanzapine
Sodium valproate
Topiramate.

30. A 60-year-old man is started on an antidepressant for treatment of major depressive


disorder. Three weeks later, he complains of blurred vision, dry mouth, constipation,
urinary retention and tachycardia. Which of the following medications is most likely
to have caused this presentation?
A.
B.
C.
D.
E.

Agomelatine
Duloxetine
Fluoxetine
Imipramine
Mirtazapine

31. Which of the following medications is most likely to be associated with polycystic
ovarian syndrome in female patients?
A.
B.
C.
D.
E.

Carbamazapine
Lamotrigine
Lithium
Sodium valproate
Topiramate.

32. Regarding an antidepressant called bupropion, which of the following statements is


false?
A.
B.
C.
D.

It causes insomnia.
It has a half-life of approximately 12 hours.
It is safe for patients suffering from bulimia nervosa.
It is an antidepressant used for depressed patients with cardiovascular disease and

!
!
213

habit of smoking.
E. It does not cause weight gain.
33. Mirtazapine is associated with lesser sexual side effects. Which of the following
pharmacodynamics actions explains this phenomenon?
A.
B.
C.
D.
E.

Serotonin receptor 5HT1A agonism


Serotonin receptor 5HT2A antagonism
Serotonin receptor 5HT2C antagonism
Serotonin receptor 5HT3 antagonism
Histamine receptor antagonism.

34. Which of the following side effects is least likely to occur in patients taking
quetiapine?
A.
B.
C.
D.
E.

Antihistamine effects
Orthostatic hypotension
Increase in liver transaminase
Increase in prolactin
Weight gain.

35. Which of the following is the best treatment option for severe depressive episode
with psychotic features?
A.
B.
C.
D.
E.

Amitriptyline
Cognitive behaviour therapy
Lamotrigine
Methylphenidate
Electroconvulsive therapy.

36. A 35-year-old man suffers from schizophrenia symptoms and his GP has started him
a medication to treat his symptoms. He develops jaundices after taking the
medication. Which of the following medications is most likely to cause jaundice?
A.
B.
C.
D.
E.

Clozapine
Chlorpromazine
Haloperidol
Risperidone
Sulpiride.

37. A 60-year-old man suffers from depressive disorder and you have increased the
!
!
214

dose of antidepressant. He is seeing you today. His blood pressure is 180/90. Which
of the following medications should be stopped?
A.
B.
C.
D.
E.

Diazepam
Quetiapine
Fluvoxamine
Hydroxyzine
Venlafaxine.

38. A 25-year-old woman with bipolar disorder is pregnant for the first time. Throughout
the pregnancy, she has been maintained on a mood stabilizer. At the time of birth,
the baby is noted to have Ebsteins anamoly. Which of the following medications is
most responsible for this teratogenic effect?
A. Carbamazepine
B. Lamotrigine
C. Lithium
D. Sodium valproate
E. Topiramate
Psychotherapy and defence mechanisms
39. A 28-year-old woman comes to consult you because she is very upset after being
scolded by her manager. The chief executive officer (CEO) of her company has
raised the standard for annual performance. Her manager is very upset this
arrangement and scolds her for no reason. Which of the following is the defence
mechanism exhibits by the manager?
A.
B.
C.
D.
E.

Altruism
Denial
Displacement
Repression
Reaction formation.

40. You are a doctor working in polyclinic. A 25-year-old man bought his 55-year-old
mother to see you. He is concerned that his mother suffers from depression but she
refuses to take antidepressant. You realise that the mother and son are enmeshed
as the son is unmarried and not able to work. He claims that he needs to look after
his mother although she has good past health. The mother and son hold very
negative view against the father who is the sole breadwinner of family. There is a
psychologist working in your polyclinic. Which of the following forms of psychological
treatment is most appropriate?
A.
B.
C.
D.

Couple therapy
Dialectical behaviour therapy
Problem solving psychotherapy
Supportive psychotherapy

!
!
215

E. Family therapy.
41. A national serviceman is seeing a male psychologist because he suffers from
adjustment disorder and he finds his army supervisor uncaring. Furthermore, he is
very angry with his own father. He recalls an incident in camp about getting angry
with his male supervisor. The psychologist asks a factual question about the
circumstance and rage in the patient. Then the patient complains that the
psychologist is uncaring and trying to find fault in him. The psychologist says, I
wonder if what youve feeling right now is just like the feeling you have in camp,
when you attributed the same uncaring attitude toward your father and supervisor in
camp. These repetitive patterns of misunderstanding seem to make you very upset
in different situations and affect your life. Which of the following most accurately
describes the psychotherapeutic technique?
A.
B.
C.
D.
E.

Affirmation
Clarification
Empathic validation
Observation
Interpretation.

42. A medical student used to be very keen to become an orthopaedic surgeon. He has
assisted an orthopaedic surgeon to conduct research since the second year of
medical school. During the residency application, he tells other classmates that he
does not bother whether he is accepted by the orthopaedic residency. He also
encourages other classmates not to apply for orthopaedic surgery as this specialty is
not a good career choice. On the other hand, he is secretly asking his orthopaedic
mentor to prepare him for the orthopaedic residency intake interview. What is his
defence mechanism?
A. Altruism
B. Denial
C. Displacement
D. Repression
E. Reaction formation.
43. Which of the following is the most effective approach for a 30-year-old woman with
obsession about contamination and washes her hands every time she touches
something which she considers dirty?
A. Having the patient place her hands in a container of worms to create extreme phobia
and ability to overcome obsession
B. Having the patient snap her wrist with a rubber band when she thinks about
contamination and about to wash her hands
C. Having the patient touch a dirty object, then not allowing her to wash her hands for
several hours
D. Highlighting to the patient that her obsession is a cognitive error.
E. Providing the patient with message cards to remind herself that the hand washing is
unnecessary.
44. A 30-year-old army officer presents for treatment of post-traumatic stress disorder
after surviving an accident which his tank fell into the river from a bridge. He was
!
!
216

driving a tank on a bridge when it suddenly collapsed. Several army officers were
seriously injured in this accident. As part of the treatment, the psychologist asks the
patient to imagine that he is safely driving his tank over a bridge. Which of the
following best describes this therapeutic intervention?
A.
B.
C.
D.
E.

Aversion therapy
Cognitive therapy
Exposure therapy
Interpersonal therapy
Supportive therapy.

Ethics, management dilemma and laws


45. You are a paediatric resident. A 14-year-old girl has stayed in the paediatric ward for
1 week after a suicide attempt by drug overdose. She is due for discharge tomorrow.
At 6 pm, she suddenly claims that her elder sister had physically abused her prior to
admission. Her medical record reveals that she suffers from conduct disorder. She
also made similar reports of physical abuse in the past. The police officer and social
worker found her claims inconclusive and did not investigate further. She is
ambivalent to make a report about the alleged abuse and she is not keen for
discharge. You try to contact the consultant in charge of this case but the consultant
is not contactable. Which of the following is most appropriate action?
A. Banning her parents from visiting her.
B. Call 995 to inform the police immediately
C. Inform the on-call team and request the on-call doctors to inform the police before
midnight
D. Refer the patient to gynaecologist for further assessment
E. Discuss with the consultant-in-charge and the social worker on the next day.
46. A 35-year-old man was hospitalised for treatment of severe depressive episode in
September. The inpatient team prescribed two antidepressants for the patient and
then referred the patient to polyclinic. The patient came to the polyclinic for review in
October and November respectively. The patient had a relapse of depression and
committed suicide in December. The computerised record shows that the polyclinic
doctors had forgotten to prescribe antidepressants since the first visit. Which of the
following best describes the above situation based on the Common Law?
A.
B.
C.
D.
E.

Battery
Diminished responsibility
Intentional torts
Negligence
Tarasoffs rule

Liaison psychiatry and neuropsychiatry


47. You are a resident in obstetrics. Which of the following is the safest medication to be
!
!
217

given to a pregnant woman who develops psychosis during pregnancy?


A. Aripiprazole
B. Haloperidol
C. Olanzapine
D. Quetiapine
E. Risperidone
48. A 25-year-old man expresses concerns that he got infected with HIV after visiting
commercial sex workers last week. According to him, he only received hand-genital
stimulation and there was no vaginal intercourse. He also mentions that the
commercial sex worker kissed his body. He has sought his girl-friends forgiveness.
Today, he requests HIV test urgently. He consulted you one year ago for similar
concern after he kissed another lady without informing his girl-friend. What is the
most likely explanation of his behaviour?
A. Underlying depression
B. Underlying guilt
C. Underlying obsession
D. Underlying psychosis
E. Underlying somatic concern
49. You are orthopaedic resident. A 25-year-old construction worker had a fall, fractured
his spine and injured the brain. He develops pain in multiple sites. He has one
episode of epilepsy after the injury. He is depressed but does not have suicidal
thought. Electrocardiogram is normal. Which of the following medication is most
helpful in this situation?
A. Amitriptyline
B. Bupropion
C. Chlorpromazine
D. Duloxetine
D. Moclobemide

50. You are a medical resident and rotated to rheumatology. A 24-year-old


woman suffers from systemic lupus erythematosus (SLE). She suffers from
lupus nephritis and requires dialysis three times a week. She was very
concerned about her illness and felt hopeless. She took an overdose of her
lupus medications and admitted to the medical ward. You need to take a
history from her. Which of the following is not part of the neuropsychiatric
symptoms of SLE experienced by this patient?
A. Anxiety
B. Binge eating
C. Cognitive impairment
!
!
218

D. Epilepsy
E. Headache.
!
MCQ exam 2014/2015 Rotation 4

History, mental state exam, psychopathology,


diagnosis
51. You are a resident doctor working at the Institute
of Mental Health (IMH). The police officers
bought a 39-year-old man charged with
possessing a scheduled weapon and was sent to
the IMH for a psychiatric evaluation. Although he
speaks English, you cannot make out what he is
saying because his speech becomes diffuse and
unfocused. Which of the following best describes
his psychopathology?
F.
G.
H.
I.
J.

Circumstantiality
Loosening of association
Thought insertion
Thought withdrawal
Tangentiality.

The answer is B.

Explanation: Loosening of
association is defined as a
speech where topics seem to be
disconnected and it is hard for
others to establish a logical link
between topics. The speech
becomes diffuse and unfocused.

Year: 2014.

52. You are a resident working at the Accident and


Emergency Department (AED). A 35-year-old
man was sent to the AED for psychiatric
assessment by an ambulance. He was involved
in a road traffic accident. He was a passenger in
a taxi. He told the taxi driver that he was God and
threatened the driver to believe that he was the
saviour of the world. The driver found him very
irritable and he kept hitting the head of the driver.
The driver stopped the taxi and tried to call the
police. The patient drove the taxi at high speed
and crashed the taxi into a lamp post. He was not
under influence of alcohol at the time of accident.
You are required to enter a diagnosis into the
AED computerised system. Based on the
information provided, which of the following
psychiatric diagnosis is most relevant?
F. Antisocial personality disorder
G. Bipolar disorder
H. Conversion disorder
!
!
219

The answer is B

Explanation: This man suffers


from bipolar disorder, manic
episode as evidence based by
grandiose delusion, irritability,
reckless and dangerous
behaviour.

Year: 2015.

I. Dissociative disorder
J. Schizophrenia.
Cognitive assessment
53. A 30-year-old woman has received 6 sessions of
electroconvulsive therapy (ECT). After the ECT,
she complains of cognitive impairment and
attributes the following symptoms as side effects
of ECT. Which of the following is most likely
caused by ECT?
F. Cannot follow the instructions given by her
supervisor after she returns to work
G. Cannot recall the password of her email account
H. Cannot remember details of her ex-marriage
I. Difficulty to perform two tasks at one time
J. Unable to calculate.

The answer is B.

Explanation: Young patients


usually develop retrograde
amnesia of recent events but not
anterograde amnesia (option A)
or life events (option C). ECT
does not affect multi-tasking and
calculation.

Year: 2015.

54. You are administering the Serial 7 test to a patient.


His answers are listed as follows: 100 7 = 90; 90 7
= 77; 77 7 = 70; 70 7 = 63; 63 7 = 50. What is his
score?
F.
G.
H.
I.
J.

The!answer!is!C.!
!
Explanation:!100!!7!=!90!(0!
point)!

0 out of 5
1 out of 5
2 out of 5
3 out of 5
4 out of 5.

90N7!=!77!(0!point)!
77N7!=!70!(1!point)!
70N7!=63!(1!point)!
63N7!=!50!(0!point)!
Total:!2!points!out!of!5!
!
Year:!2014.!

Psychiatric epidemiology
55. Which of the following is the most common
psychiatric co-morbidity of panic disorder?

!
!
220

The answer is B.

F.
G.
H.
I.
J.

Explanation: Agoraphobia (40%)


> alcohol dependence (30%).
The other options are less likely
to be associated with panic
disorder.

Alcohol misuse
Agoraphobia
Avoidant personality disorder
Benzodiazapine misuse
Generalised anxiety disorder.

Year: 2014.

Psychiatric aetiology, diagnosis and classification


The answer is C.
56. The prevalence of borderline personality
disorder in Singapore and worldwide has
increased in the past three decades. The most
likely explanation for this observation is:
F.
G.
H.
I.
J.

Changes in diagnostic criteria


Increase in competition in schools
Increase in divorce rate
Increase in immigration and migration
Increasing urbanization of the population.

Explanation: increase in divorce


rate is associated with trauma,
neglect and splitting in family.
These are important
predisposing factors for
borderline personality disorder.
There is no major change in
diagnostic criteria. Some of the
factors may occur too late to
affect personality development
(e.g. migration). As Singapore is
a city state, the effect of
urbanization is minimal.

Year: 2014
57. Which of the following is least likely to be a risk
factor for delusional disorder?

The answer is A.

P.
Q.
R.
S.
T.

Explanation: Having a religion is


not a risk factor for developing
delusional disorder. The risk of
delusional disorder increases
with age. Hearing loss is a risk
factor. Low socioeconomic status
and severe stress are also risk

Having a religion
Increased age
Immigration
Sensory impairment
Social isolation.

!
!
221

factors.

Year: 2014

58. The genetic polymorphisms of which of the


following neurotransmitter transporter genes
affect the risk of developing depressive disorder?

The answer is E.

F.
G.
H.
I.
J.

Explanation: Individuals with


homozygous short alleles at the
serotonin transporter gene are
more likely to develop
depression.

Dopamine
-Aminobutyric acid (GABA)
Glutamate
Norephinephrine
Serotonin.

Year: 2014

59. A 40-year-old patient with schizophrenia presents


to the emergency department with confusion and
agitation following an overdose of haloperidol.
The patient has a temperature of 41 degree
Celsius, blood pressure of 160/100 mm Hg, and
pulse of 120 beats per minute. Physical
examination reveals rigidity, hot, wet skin, and
decreased bowel sounds. This patients
symptoms, are most likely caused by blocking of
which of the following receptors?
F. Alpha1-adrenergic
G. Dopaminergic
H. Histaminergic
I. Muscurinic cholinergic
J. Nicotinic cholinergic.
General adult psychiatry

The answer is B.

Explanation: This patient


develops neuroleptic malignant
syndrome due to blockage of
dopamine receptors by
haloperidol.

Year: 2014

60. A 30-year-old man suffers from schizophrenia.


He first exhibited first rank symptoms at the age
of 16 and the onset was insidious. He exhibits
anhedonia although his hallucinations are under
control by antipsychotic drug. His uncle suffers
from depressive disorder. All of the following are
poor prognostic factors except?

!
!
222

The answer is C.

Explanation: Poor prognostic


factors include young onset, no
precipitating factors, insidious
onset, poor premorbid

F.
G.
H.
I.
J.

Anhedonia
Exhibition of first rank symptoms at the age of 16
Family history of depressive disorder
Male gender
Insidious onset.

functioning, family history of


schizophrenia, negative
symptoms, poor support,
neurological symptoms and poor
compliance.

Year: 2014
61. You are the resident working in the Accident and
Emergency Department. A schizophrenia patient
informs you that he has homicidal thought. Which
of the following sign or symptom is least
important in predicting homicide?
F.
G.
H.
I.
J.

Command hallucinations
Delusion of reference
History of previous violence
Irresistible urge to attack
Need to defend oneself as are result of
persecutory delusion.

The answer is B.

Explanation: Delusion of
reference is not directly related
to violence and carries lower risk
of homicide as compared to
other signs and symptoms.

Year: 2014
The answer is C.
62. All of the following are poor prognostic signs for
obsessive compulsive disorder except:
F.
G.
H.
I.
J.

Bizarre compulsions
Childhood onset
Episodic course
Comorbid major depression
Succumbing to obsessions and compulsions.

Explanation:
Poor prognosis:
1. yielding to compulsions
(not resisting)
2. childhood onset
3. bizarre compulsions
4. need for hospitalization
5. presence of overvalued
ideas (i.e. acceptance)
6. personality disorders
(especially schizotypal)
7. co-existing delusional
beliefs
8. co-existent major
depressive disorder
Year: 2014

63. You are a general practitioner. A 35-year-old


hotel butler came to see you because his lawyer
!
!
223

The answer is E.

wants him to be seen by a psychiatrist. He was


recently arrested for installing pinhole camera in
the hotel guestrooms. He secretly filmed couples
having sex in the hotel guestrooms. He has
filmed more than 100 videos and enjoyed
watching them over and over again. He claims
that he has the compulsion to collect those
videos and he feels relieved after the arrest.
Which of the following is the most likely
diagnosis?

Explanation: Voyeurism is
defined as recurrent and intense
sexual arousal from observing
other people being naked or
engaging sexual activity.

Year: 2014
F.
G.
H.
I.
J.

Erotomania
Kleptomania
Obsessive compulsive disorder
Sadomasochism
Voyeurism.

64. A 30-year-old woman always believes that she is


socially inept and fears of negative evaluation by
other people. Her sisters describe her as timid
and insecure. She is single and stays with two
elder sisters. She was admitted to the psychiatric
ward due to nervous breakdown after meeting
new colleagues in her workplace. She refuses to
attend family meeting with her sisters because
she feels that her sisters do not like her although
they appear to be very concerned about her
condition. She is isolated and has one best friend
in her work place. Which of the following
personality best describes this patient?
F.
G.
H.
I.
J.

Avoidant personality disorder


Borderline personality disorder
Histrionic personality disorder
Paranoid personality disorder
Schizoid personality disorder

The answer is A.

Explanation: She suffers from


avoidance personality disorder
as evidenced by pervasive
tension (e.g. nervous
breakdown), unwilling to be
involved unless being liked (e.g.
family meeting with her sister),
fear of criticism, showing
restraint in intimate relationship
(e.g. being single) and inhibition
of new interpersonal situations
(e.g. meeting new colleagues)
and belief of socially inept.

Year: 2014
65. Which of the following is not a common feature of
serotonin syndrome?
F. Acidosis
G. Diaphoresis
H. Hyperreflexia

Answer: D

Explanation: Serotonin syndrome


actually causes fever. Metabolic

!
!
224

I. Hypothermia
J. Myoclonus.

acidosis in seen in 9% of
patients.

Year: 2014

66. You are a resident working in the accident


and emergency department (AED). A single
mother was sent to the AED with her 2-yearold daughter. She was arrested by police
because she tried to assault her daughter
and stopped by passer-by. According to the
informant, her partner refuses to marry her
due to her drug habit. She does not have
past forensic history. She appears to be
agitated and restless. At one moment, she
wants to stay in the AED. At another
moment, she wants to walk out of the AED.
Physical examination reveals nystagmus.
Which of the following diagnosis is most
relevant?
F. Antisocial personality disorder
G. Cannabis intoxication
H. Delusion of jealousy
I. Paranoid schizophrenia
J. Phencyclidine intoxication

The answer is E.
Explanation: The immediate
diagnosis is PCP intoxication
as evidenced by
unpredictability (staying in
AED versus walking out of
AED), assaultativeness on her
daughter, agitation and
nystagmus. Antisocial
personality disorder requires
further evidence as forensic
history is negative and cannot
explain nystagmus.

Year: 2014

Old age psychiatry


67. You are about to start a selective serotonin
reuptake inhibitor (SSRI) for an 80-year-old man
who suffers from depression. Which of the
following pharmacokinetic changes has the least
effect on drug therapy in the elderly?
F.
G.
H.
I.
J.

Absorption
Distribution
Excretion
Metabolism
Protein binding.

!
!
225

The answer is A.

Explanation:
There
is
no
consistent
age
effect
on
absorption of antidepressant in
elderly. For distribution, the
elderly have more fat stores;
therefore they will have lower
plasma concentrations of lipid
soluble drugs, but will have
larger distribution volumes of
these drugs and therefore longer
half-lives. This may cause
problems for elderly taking
fluoxetine.

For metabolism, all psychotropic


medications metabolized in the
liver have their elimination halflife increased two or three-fold in
the elderly.

For excretion, decreased GFR


and renal blood flow lead to
decreased clearance of lithium
and antidepressant metabolites.

For protein binding, the elderly


have lower levels of plasma
albumin due to malnutrition, and
therefore have an increased free
fraction of protein bound drugs.

Year: 2014

This is an advanced level


question.

68. A 79-year-old man complains of extreme


drowsiness when he goes to toilet at midnight
and he almost fell down on several occasions. He
wants to avoid fall at night. The following is the
list of his medications. Which of the following
medications should be stopped?
F.
G.
H.
I.
J.

Melatonin
Mirtazapine
Risperidone
Rivastigmine
Sodium valproate.

The answer is B.

Explanation: Mirtazapine is the


most sedative medication among
all the options.

Year: 2014

The answer is D.

!
!
226

69. A 70-year-old woman suffering from bipolar


disorder and her daughter came to see you
today. Her daughter read about the symptoms of
bipolar disorder and found her mother different
from younger adults suffering from the same
condition. Which of the following clinical features
is more likely to be found in this 70-year-old
woman?
F. Demonstrates less paranoid ideas as compared
to adult patients
G. Demonstrates more flight of ideas as compared
to adult patients
H. More likely to be hyperactive as compared to
adult patients
I. More likely to demonstrate mixed
(depressive/manic) clinical presentations as
compared to adult patients
J. More spending as compared to adult patients.

Explanation: Elderly with bipolar


disorder are more likely to be
paranoid and present with mixed
clinical presentations. Elderly are
less likely to develop flight of
ideas, hyperactivity and
overspending.

Year: 2014

70. Which of the following is most important risk


factor for vascular dementia?

The answer is C.

F.
G.
H.
I.
J.

Explanation: Among all the


options, hypertension is the most
significant risk factor for vascular
dementia.

History of depression
History of poor academic performance
History of hypertension
High level of physical activity
High level of high density lipoproteins.

Year: 2014

71. You are the visiting physician of a nursing home.


The nursing home staffs feel very helpless when
handling aggression in dementia patients and
they need to seek your advice. Which of the
following psychotropic medication have the most
evidence in managing aggression in patients
suffering from dementia?
F.
G.
H.
I.
J.

Antipsychotic drug
Beta-blocker
Benzodiazapine
Lithium
Stimulant.

The answer is A.

Explanation: Antipsychotic drug


(e.g. risperidone) has the most
evidence in improving
aggression in dementia patients
among all options.

Year: 2014

!
!
227

Child and adolescent psychiatry


72. In children and adolescents, high stimulationseeking behaviour is associated with which of the
following conditions?
F.
G.
H.
I.
J.

The answer is C.

Explanation: Young people with


conduct disorder have to be
involved in delinquent behaviour
to get high stimulus.

Adolescent-onset schizophrenia
Autism
Conduct disorder
Obsessive-compulsive disorder
School refusal.

Year: 2014
73. Children with autism are most comfortable in
which of the following situations?

The answer is D.

F. Attending a party where there are a lot of


surprises and have a chance to meet different
people.
G. Attending drama classes and imagines oneself
playing different roles in the drama.
H. Looking for toys in a multi-storey department
store from one level to another level.
I. Travelling in a bus when it stops at every bus
stop until it reaches the bus terminal.
J. Going to school where there are multiple classes
to attend in a day and teaching content varies
from subject to subject.
74. Which of the following statements is most correct
regarding the impact of divorce on children?

Explanation: Travelling in a bus


because it has a predictable
route (i.e. the bus stops at every
bus stop and there is
uncertainty).

F. Boys aged two to four are not significantly


affected.
G. Children do best if they have no contact with the
father for two years.
H. Most children have no significant problems after
three to six months.
I. Socio-economic status is not associated with the
degree of impairment.
J. To evaluate the impact, one must take into
consideration the age and the developmental
level of the child.
75. A 15-year-old obese female is brought by her
parents for an evaluation as part of the admission
!
!
228

Year: 2014

The answer is E.

Explanation: Options A and B


have no evidence base. For
option C, most children take 3-5
years to recover. Socioeconomic status affects the
outcome.

Year: 2014
The answer is E.

process for a weight management program. The


adolescent agrees with the fact that she is
overweight and she is keen to lose weight.
Which of the following is most indicative that she
will be successful in the program?
F. Her current BMI is between 26 and 30.
G. Mother suffered from obesity and successfully
lost weight
H. No family history of eating disorders
I. Motivation to take antidepressant
J. Willingness to change her eating habits.

Explanation: The patient must


agree to use conservative
methods including changing diet
habits, seeing a dietician and
going for exercise to lose weight.

Year: 2014

Substance abuse
76. Methylene dimethylamphteamine (street name:
ecstasy) usually cause people with which of the
following symptom?
f. Amotivation
g. Closeness to others
h. Depression
i. Introversion
j. Phobia.

The answer is B.

Explanation: Ecstasy is also


known as an empathogen and
makes the patient to feel close to
others.

Year: 2014
77. A 30-year-old adult claims to suffer from adult
onset attention deficit and hyperactivity disorder
(ADHD). He has been seeing different doctors to
obtain methylphenidate. He claims that he lost all
methylphenidate in the bus few days ago and his
wife has noticed that he looks very tired. Which
of the following is most appropriate explanation
for his tiredness?
F. He develops depression and psychomotor
retardation
G. He exhibits secondary gain and seeking
attention from his wife
H. He suffers from comorbid somatisation
disorder in addition to ADHD
I. He suffers from psychological withdrawal
symptoms from stimulant misuse
J. He suffers from stimulant intoxication.
78. The most important objective of
psychological intervention in treating people
suffering from alcohol misuse is:
!
!
229

The answer is D.

Explanation: This patient is


doctor shopping and tries to
obtain methylphenidate because
he is dependent on this
medication. He lost the
medication few days ago and
developed psychological
withdrawal symptoms including
depression, tiredness,
oversleeping and overeating.

Year: 2014
The answer is D.

F. To conduct in a group therapy format


G. To apply cognitive behaviour therapy (CBT) as
the first approach
H. To combine psychological intervention and
pharmacotherapy (e.g. disulfiram)
I. To focus on achieving and maintaining
abstinence or reducing alcohol use
J. Explore unresolved psychodynamic conflicts.

Explanation: Achieving
abstinence or reduction of
alcohol use is most important
psychotherapeutic objective.

Year: 2014.

Psychopharmacology
79. Which of the following medications is not
associated with weight gain?

The answer is E.

F.
G.
H.
I.
J.

Explanation: Topiramate is
associated with weight loss and
not weight gain.

Clozapine
Mirtazapine
Olanzapine
Sodium valproate
Topiramate.

Year: 2014
80. A 60-year-old man is started on an
antidepressant for treatment of major depressive
disorder. Three weeks later, he complains of
blurred vision, dry mouth, constipation, urinary
retention and tachycardia. Which of the following
medications is most likely to have caused this
presentation?
F.
G.
H.
I.
J.

Agomelatine
Duloxetine
Fluoxetine
Imipramine
Mirtazapine

The answer is D.

Explanation: Imipramine is a
tricyclic antidepressant and
associated with anticholinergic
side effect.

Year: 2014

81. Which of the following medications is most likely


to be associated with polycystic ovarian
syndrome in female patients?

The answer is D.

F.
G.
H.
I.
J.

Explanation: Sodium valproate is


associated with polycystic ovary
syndrome in female patients.

Carbamazapine
Lamotrigine
Lithium
Sodium valproate
Topiramate.

!
!
230

Year: 2014
82. Regarding an antidepressant called bupropion,
which of the following statements is false?

The answer is C.

F. It causes insomnia.
G. It has a half-life of approximately 12 hours.
H. It is safe for patients suffering from bulimia
nervosa.
I. It is an antidepressant used for depressed
patients with cardiovascular disease and habit of
smoking.
J. It does not cause weight gain.

Explanation: bupoprion works on


noradrenaline and dopamine but
not serotonin. It is not
recommended for eating disorder
because of the risk of seizure.

Year: 2014
83. Mirtazapine is associated with lesser sexual side
effects. Which of the following
pharmacodynamics actions explains this
phenomenon?
F.
G.
H.
I.
J.

Serotonin receptor 5HT1A agonism


Serotonin receptor 5HT2A antagonism
Serotonin receptor 5HT2C antagonism
Serotonin receptor 5HT3 antagonism
Histamine receptor antagonism.

The answer is B.

Explanation: Mirtazapine
antagonises 5HT2A receptors and
leads to anxiolytic effects,
restores sleep and less sexual
side effects.

Year: 2014
84. Which of the following side effects is least likely
to occur in patients taking quetiapine?

The answer is D.

F.
G.
H.
I.
J.

Explanation: Quetiapine is
associated with antihistamine
effects, orthostatic hypotension,
increase in liver transaminase
and weight gain.

Antihistamine effects
Orthostatic hypotension
Increase in liver transaminase
Increase in prolactin
Weight gain.

Year: 2014
85. Which of the following is the best treatment
option for severe depressive episode with
psychotic features?

Answer is E.

Explanation: ECT has the most


evidence in treating psychotic

F. Amitriptyline
!
!
231

G.
H.
I.
J.

Cognitive behaviour therapy


Lamotrigine
Methylphenidate
Electroconvulsive therapy.

depression as compared to other


treatment options.

Year: 2014
86. A 35-year-old man suffers from schizophrenia
symptoms and his GP has started him a
medication to treat his symptoms. He develops
jaundices after taking the medication. Which of
the following medications is most likely to cause
jaundice?
F.
G.
H.
I.
J.

Clozapine
Chlorpromazine
Haloperidol
Risperidone
Sulpiride.

Diazepam
Quetiapine
Fluvoxamine
Hydroxyzine
Venlafaxine.

The answer is E.

Explanation: Venlafaxine is
associated with dose-dependent
hypertension.

Year: 2014

88. A 25-year-old woman with bipolar disorder is


pregnant for the first time. Throughout the
pregnancy, she has been maintained on a mood
stabilizer. At the time of birth, the baby is noted
to have Ebsteins anamoly. Which of the
following medications is most responsible for this
teratogenic effect?
F.
G.
H.
I.
J.

Explanation: Chlorpromazine is
associated with cholestatic
jaundice and is most likely to
cause jaundice among all
options.
Year: 2014

87. A 60-year-old man suffers from depressive


disorder and you have increased the dose of
antidepressant. He is seeing you today. His blood
pressure is 180/90. Which of the following
medications should be stopped?
F.
G.
H.
I.
J.

The answer is B.

Carbamazepine
Lamotrigine
Lithium
Sodium valproate
Topiramate

The answer is C

Explanation: Ebsteins anomaly


is associated with lithium.

Year: 2014

!
!
232

Psychotherapy and defence mechanisms


89. A 28-year-old woman comes to consult you
because she is very upset after being scolded by
her manager. The chief executive officer (CEO)
of her company has raised the standard for
annual performance. Her manager is very upset
this arrangement and scolds her for no reason.
Which of the following is the defence mechanism
exhibits by the manager?
F.
G.
H.
I.
J.

Altruism
Denial
Displacement
Repression
Reaction formation.

!
Explanation:!The!manager!
transferred!his!anger!to!his!
subordinate!which!carries!less!
emotional!risk!as!compared!to!the!
CEO.!This!is!known!as!
displacement.!
!
Year: 2014!

90. You are a doctor working in polyclinic. A 25-yearold man bought his 55-year-old mother to see
you. He is concerned that his mother suffers from
depression
but
she
refuses
to
take
antidepressant. You realise that the mother and
son are enmeshed as the son is unmarried and
not able to work. He claims that he needs to look
after his mother although she has good past
health. The mother and son hold very negative
view against the father who is the sole
breadwinner of family. There is a psychologist
working in your polyclinic. Which of the following
forms of psychological treatment is most
appropriate?
F.
G.
H.
I.
J.

The!answer!is!C.!

The answer is E.

Explanation: Family therapy such


as structural family therapy
focusing on boundary is helpful
in this situation.

Year: 2014

Couple therapy
Dialectical behaviour therapy
Problem solving psychotherapy
Supportive psychotherapy
Family therapy.

91. A national serviceman is seeing a male


psychologist because he suffers from adjustment
disorder and he finds his army supervisor
uncaring. Furthermore, he is very angry with his
own father. He recalls an incident in camp about
getting angry with his male supervisor. The
psychologist asks a factual question about the
circumstance and rage in the patient. Then the
patient complains that the psychologist is
!
!
233

The answer is E.

Explanation: Interpretation is a
statement made by the
psychologist to enhance patients
understanding of his or her own
thoughts or behaviours.

uncaring and trying to find fault in him. The


psychologist says, I wonder if what youve
feeling right now is just like the feeling you have
in camp, when you attributed the same uncaring
attitude toward your father and supervisor in
camp.
These
repetitive
patterns
of
misunderstanding seem to make you very upset
in different situations and affect your life. Which
of the following most accurately describes the
psychotherapeutic technique?
F.
G.
H.
I.
J.

Affirmation
Clarification
Empathic validation
Observation
Interpretation.

Affirmation refers to confirmation


of validity of previous judgement
or behaviour in a patient.
Empathetic validation refers to
the situation when the therapist
tries to put himself into the
patients shoes and tries to
understand patients inner state.

Year: 2014

92. A medical student used to be very keen to


become an orthopaedic surgeon. He has
assisted an orthopaedic surgeon to conduct
research since the second year of medical
school. During the residency application, he tells
other classmates that he does not bother whether
he is accepted by the orthopaedic residency. He
also encourages other classmates not to apply
for orthopaedic surgery as this specialty is not a
good career choice. On the other hand, he is
secretly asking his orthopaedic mentor to prepare
him for the orthopaedic residency intake
interview. What is his defence mechanism?

The answer is E.

Explanation: His external attitude


and behaviours (not caring about
orthopaedic surgery) is the
opposite of the internal intention
(wanted to join orthopaedic
surgery).

Year: 2014
F. Altruism
G. Denial
H. Displacement
I. Repression
J. Reaction formation.
93. Which of the following is the most effective
approach for a 30-year-old woman with
obsession about contamination and washes her
hands every time she touches something which
she considers dirty?
F. Having the patient place her hands in a container
of worms to create extreme phobia and ability to
overcome obsession
G. Having the patient snap her wrist with a rubber
band when she thinks about contamination and
about to wash her hands
H. Having the patient touch a dirty object, then not
allowing her to wash her hands for several hours
!
!
234

The answer is C.

Explanation: Option C is known


as exposure and response
prevention (ERP) and is the most
effective approach among all
options. Option D is not good
enough because the
psychologist should guide the
patient to further challenge the
obsessions rather than just

I.

Highlighting to the patient that her obsession is a


cognitive error.
J. Providing the patient with message cards to
remind herself that the hand washing is
unnecessary.

highlighting the obsessions.


Option B is thought stopped and
is not as effective as ERP.

Year: 2014
94. A 30-year-old army officer presents for treatment
of post-traumatic stress disorder after surviving
an accident which his tank fell into the river from
a bridge. He was driving a tank on a bridge when
it suddenly collapsed. Several army officers were
seriously injured in this accident. As part of the
treatment, the psychologist asks the patient to
imagine that he is safely driving his tank over a
bridge. Which of the following best describes this
therapeutic intervention?
F.
G.
H.
I.
J.

Aversion therapy
Cognitive therapy
Exposure therapy
Interpersonal therapy
Supportive therapy.

The answer is C.

Explanation: The psychologist is


providing in-vivo exposure by
imaging the accident scene to
overcome fear and anxiety
associated with flashback.

Year: 2014

Ethics, management dilemma and laws


95. You are a paediatric resident. A 14-year-old girl
has stayed in the paediatric ward for 1 week after
a suicide attempt by drug overdose. She is due
for discharge tomorrow. At 6 pm, she suddenly
claims that her elder sister had physically abused
her prior to admission. Her medical record
reveals that she suffers from conduct disorder.
She also made similar reports of physical abuse
in the past. The police officer and social worker
found her claims inconclusive and did not
investigate further. She is ambivalent to make a
report about the alleged abuse and she is not
keen for discharge. You try to contact the
consultant in charge of this case but the
consultant is not contactable. Which of the
following is most appropriate action?
F. Banning her parents from visiting her.
G. Call 995 to inform the police immediately
H. Inform the on-call team and request the on-call
doctors to inform the police before midnight
I. Refer the patient to gynaecologist for further
assessment
!
!
235

The answer is E.

Explanation: As this female


adolescent suffers from conduct
disorder, she has the tendency
to tell lies to avoid discharge. As
the patient is currently in the
ward and she is safe, there is no
need to inform the police
immediately without further
clarification from the treatment
team. Furthermore, the patient is
ambivalent to make a report. The
best approach is to discuss with
the treatment team and they can
clarify with her parents whether
the elder sister did physically
abuse her.

J. Discuss with the consultant-in-charge and the


social worker on the next day.

96. A 35-year-old man was hospitalised for treatment


of severe depressive episode in September. The
inpatient team prescribed two antidepressants for
the patient and then referred the patient to
polyclinic. The patient came to the polyclinic for
review in October and November respectively.
The patient had a relapse of depression and
committed
suicide
in
December.
The
computerised record shows that the polyclinic
doctors
had
forgotten
to
prescribe
antidepressants since the first visit. Which of the
following best describes the above situation
based on the Common Law?
F.
G.
H.
I.
J.

Battery
Diminished responsibility
Intentional torts
Negligence
Tarasoffs rule

Year: 2014

The answer is D.

Explanation: The polyclinic


doctors forgot to prescribe
antidepressants and resulted in
relapse and suicide. Their act is
considered to be negligence /
unintentional torts. Option A is
intentional torts and not relevant
for this case. Diminished
responsibility only applies in
murder cases. Tarasoffs rule
involves duty to warn when a
patient expresses his/her wish to
harm another person.

Year: 2014

Liaison psychiatry and neuropsychiatry


97. You are a resident in obstetrics. Which of the
following is the safest medication to be given to a
pregnant woman who develops psychosis during
pregnancy?
F.
G.
H.
I.
J.

Aripiprazole
Haloperidol
Olanzapine
Quetiapine
Risperidone

The answer is B.

Explanation: Haloperidol is the


oldest medication and is the
safest among all options.

Year: 2014

98. A 25-year-old man expresses concerns that he


got infected with HIV after visiting commercial
sex workers last week. According to him, he only
received hand-genital stimulation and there was
no vaginal intercourse. He also mentions that the
commercial sex worker kissed his body. He has
!
!
236

The answer is B.

Explanation: It is common in
clinical practice for patients

sought his girl-friends forgiveness. Today, he


requests HIV test urgently. He consulted you one
year ago for similar concern after he kissed
another lady without informing his girl-friend.
What is the most likely explanation of his
behaviour?
F.
G.
H.
I.
J.

Underlying depression
Underlying guilt
Underlying obsession
Underlying psychosis
Underlying somatic concern

asking for urgent HIV test after


they engaged in activities which
did not transmit HIV. The
patients request for such test as
a result of guilt. It is unlikely to be
caused by obsession as the
patient requests for the HIV test
after sexual activity but not a
thought. The patient does not
have any somatic concern.

Year: 2014
99. You are orthopaedic resident. A 25-year-old
construction worker had a fall, fractured his spine
and injured the brain. He develops pain in
multiple sites. He has one episode of epilepsy
after the injury. He is depressed but does not
have suicidal thought. Electrocardiogram is
normal. Which of the following medication is most
helpful in this situation?
A. Amitriptyline
B. Bupropion

The answer is D.

Explanation: Duloxetine is an
antidepressant indicated for both
depression and pain. Bupropion,
amitriptyline and chlorpromazine
increases the risk of seizure.
There is no indication to
prescribe moclobemide in this
case.

C. Chlorpromazine
D. Duloxetine

Year: 2014

D. Moclobemide

100.
You are a medical resident and
rotated to rheumatology. A 24-year-old
woman suffers from systemic lupus
erythematosus (SLE). She suffers from lupus
nephritis and requires dialysis three times a
week. She was very concerned about her
illness and felt hopeless. She took an
overdose of her lupus medications and
admitted to the medical ward. You need to
take a history from her. Which of the
following is not part of the neuropsychiatric
symptoms of SLE experienced by this
patient?
!
!
237

The!answer!is!B.!
!
Explanation:!!Binge!eating!is!not!a!
recognised!neuropsychiaric!
symptom!of!SLE.!
!
Year: 2014!

F.
G.
H.
I.
J.

Anxiety
Binge eating
Cognitive impairment
Epilepsy
Headache.

!
!
!

SAQ 1
Question 1
A 50-year-old man is admitted to the ward. He complains that there are a lot of bugs on
his skin and he needs to wash hands excessively. His wife wants to consult you how to
differentiate an obsession from a delusion.
1.

Define an obsession (2 marks).

Examiners use only:

2.

Define a delusion (2 marks).

3.

List TWO (2) features that differentiate an obsession from a delusion (4 marks).

/ 2 marks

Examiners use only:

Examiners use only:


4.

/ 2 marks

/ 4 marks

List TWO (2) types of evidence-based and commonly used treatment for obsessive compulsive
disorder (2 marks).
Examiners use only:

Administrators use only (Total): Administrator 1:

/ 10 marks

Administrator 2:

/ 2 marks

/10 marks

Question 3
A 3-year-old boy has delayed language development. He is of normal height and weight, and has no
obvious physical abnormalities.
1.

List THREE (3) main clinical features which you would look for to support the diagnosis of
autistic disorder (3 marks)
Examiners use only:

2.

/ 3 marks

List TWO other causes for his poor language skills besides autism (2 marks).
Examiners use only:

!
!
238

/ 2 marks

3.

Name THREE (3) investigations which you order to establish the underlying cause for language
delay. (3 marks)
Examiners use only:

4.

/ 3 marks

Name TWO (2) non-pharmacological interventions which are useful for autism. (2 marks)
Examiners use only:

Administrators use only (Total): Administrator 1:

/ 10 marks

Administrator 2:

/ 2 marks
/10 marks

Question 4
You are a medical resident. Your hospital does not have a psychiatric ward and a 40-year-old woman
presenting with catatonia was admitted last night. No investigation and treatment was ordered.
1.

List THREE (3) characteristic features of catatonia (3 marks)


Examiners use only:

2.

Name TWO (2) psychiatric disorder commonly associated with catatonia.(2 marks)
Examiners use only:

3.

/ 2 marks

At NAME TWO (2) medical disorders that are known to be associated with catatonia. (2 marks)
Examiners use only:

4.

/ 3 marks

/ 2 marks

List ONE (1) most important investigation which you would order to identify the underlying
neurological cause for catatonia.(1 mark)
Examiners use only:

5.

List ONE (1) pharmacological treatment for this patient. (1 mark)

6.

Examiners use only:


Name ONE (1) non-pharmacological management order for this patient (1 mark)
Examiners use only:

Administrators use only (Total): Administrator 1:

!
!
239

/ 10 marks

Administrator 2:

/ 1 mark

/ 1 mark
/ 1 mark

/10 marks

Question 1
A 50-year-old man is admitted to the ward. He complains that there are a lot of bugs on
his skin and he needs to wash hands excessively. His wife wants to consult you how to
differentiate an obsession from a delusion.
5.

Define an obsession (2 marks).

Examiners use only:

/ 2 marks

Answer may vary. The examiner is advised to use his or her discretion to decide whether marks should be
awarded.
Obsession is a
recurrent and persistent thoughts, impulses, or images that are experienced as intrusive and
inappropriate
they cause anxiety and distress
they are not simple excessive worries about real-life problems
the person attempts to ignore or suppress the thoughts, impulses, or images, or to neutralize them
with some other thought or action
the person recognizes them as products of his/her own mind.
6.

Define a delusion (2 marks).

Examiners use only:

/ 2 marks

Answer may vary. The examiner is advised to use his or her discretion to decide whether marks should be
awarded.
Delusion is a
a false belief, based on an incorrect inference about external reality.
the belief cannot be corrected by reasoning.
the patient firmly believes that the false belief is true (close to 100% true).

!
!
240

7.

List TWO (2) features that differentiate an obsession from a delusion (4 marks).
Examiners use only:

/ 4 marks

Answer may vary. The examiner is advised to use his or her discretion to decide whether marks should be
awarded.
Differences between obsessions and delusions:
obsessions are ego-dystonic (disharmony with ones own ego) while delusions are ego-syntonic (in
harmony with ones own ego) 2 marks or
obsessions involve intact reality testing, whereas in delusions reality testing is impaired (2 marks)
or
obsessions cause anxiety and distress, whereas patients with delusion are usually calm (2marks)
or
obsessions involve compulsive actions to neutralize them, whereas delusions do not (2 marks)
obsessions are not held with absolute certainty whereas delusions are (2 marks).
This is an advanced level question.
8.

List TWO (2) types of evidence-based and commonly used treatment for obsessive compulsive
disorder (2 marks).
Examiners use only:

/ 2 marks

Selective!serotonin!reuptake!inhibitors!!or!name!of!SSRI!e.g.!fluoxetine!(1!mark);!antidepressant!
(0.5!mark);!other!(0!mark)!
Cognitive!behaviour!therapy!(1!mark);!Exposure!and!response!prevention!(0.5!mark);!Other!(0!
mark)!

Administrators use only (Total): Administrator 1:

/ 10 marks

Administrator 2:

/10 marks

Question 3
A 3-year-old boy has delayed language development. He is of normal height and weight, and has no
obvious physical abnormalities.
5.

List THREE (3) main clinical features which you would look for to support the diagnosis of
autistic disorder (3 marks)
Examiners use only:

/ 3 marks

Answer may vary. The examiner is advised to use his or her discretion to decide whether marks should be
awarded.
1. Restricted interests or hobbies or repetitive behaiour.
2. Communication difficulties (both verbal language delay and non-vebal lack of eye contact, failure
to understand emotional cues)
3. Stereotyped movements or social interaction difficulty.
6.

List TWO other causes for his poor language skills besides autism (2 marks).
Examiners use only:

/ 2 marks

Answer may vary. The examiner is advised to use his or her discretion to decide whether marks should be
awarded. Any two of the following:
1. Maltreatment or abuse
2. Mental retardation

!
!
241

3.
4.
5.
6.

Neurological injury
Hearing impairment
Specific language impairment (such as expressive language disorder)
Genetic disorders (Fragile X syndrome)
7.

Name THREE (3) investigations which you order to establish the underlying cause for language
delay. (3 marks)
Examiners use only:

/ 3 marks

Answer may vary. The examiner is advised to use his or her discretion to decide whether marks should be
awarded. Any two of the following
1.
2.
3.
4.
5.
6.

Intelligence quotient (IQ) testing


Diagnostic interview for Autism
Electroencephalogram (EEG)
Hearing assessment
Speech assessment
Genetic testing (e.g. look for fragile X site or non tandem repeats in X chromosome.

8.

Name TWO (2) non-pharmacological interventions which are useful for autism. (2 marks)
Examiners use only:

/ 2 marks

Answer may vary. The examiner is advised to use his or her discretion to decide whether marks should be
awarded. Any two of the following
1. Behaviour therapy (e.g. intensive behaviour intervention)
2. Training to use alternative communication
3. Visual strategies or visual support for communication
4. Social skill training
5. Special education
6. Speech therapy

Administrators use only (Total): Administrator 1:

/ 10 marks

Administrator 2:

/10 marks

Question 4
You are a medical resident. Your hospital does not have a psychiatric ward and a 40-year-old woman
presenting with catatonia was admitted last night. No investigation and treatment was ordered.
7.

List THREE (3) characteristic features of catatonia (3 marks)


Examiners use only:

/ 3 marks

Any three of the following:


!
!
!
!
!
!
!
!

Motoric immobility as evidence by cataplesy (or waxy flexibility) or stupor


Excessive Motoric Activity (purposeless activity, not influenced by external stimuli)
Ambitendency
Automatic obedience
Negativism (as evidenced by motiveless resistance to all instructions or maintenance of a rigid posture
against attempts at being moved)
Mutisn
Stereotyped movements, prominent mannerisms or prominent grimacing)
Echolalia
Echopraxia.

!
!
242

8.

Name TWO (2) psychiatric disorder commonly associated with catatonia.(2 marks)
Examiners use only:

/ 2 marks

Any of the following:

9.

Schizophrenia
Bipolar disorder
Depressive disorder
Substance abuse with acute intoxication of recreation drugs.

At NAME TWO (2) medical disorders that are known to be associated with catatonia. (2 marks)
Examiners use only:

/ 2 marks

Any of the following:

Central nervous system infection: meningitis and encephalitis


Central nervous system tumour
Cerebrovascular accident
Lethal catatonia
Acute poisoning

10. List ONE (1) most important investigation which you would order to identify the underlying
neurological cause for catatonia.(1 mark)
Examiners use only:

/ 1 mark

Magnetic Resonance Imaging (MRI) brain scan (1 mark); Computerized Tomography (CT) brain scan
(1 mark); other answer (0 mark)
11. List ONE (1) pharmacological treatment for this patient. (1 mark)
Examiners use only:

/ 1 mark

Benzodiazepine or lorazepam (1 mark)


12. Name ONE (1) non-pharmacological management order for this patient (1 mark)
Examiners use only:

/ 1 mark

Any of the following:


1. Hydration
2. Nutrition
3. Early mobilization when condition is stable
4. Close observation and frequent monitoring for vital signs.
5. Transfer to ICU if patient deteriorates.
ECT = 0 mark as it involves general anaesthesia (pharmacological)
Administrators use only (Total): Administrator 1:

!
!
243

/ 10 marks

Administrator 2:

/10 marks

!
!
!
!
!
!
!
!
!
!
!
!
SAQ (Paper 2)
Question 1
You are a General Practitioner. A 30-year-old woman comes to your clinic and she complains of low
mood. You suspect that she suffers from depressive disorder. She read a lot of information on the internet
and has her personal views on treatment. She would like to discuss with you on various treatment options.
1.

List Four (4) pieces of information which you would inquire from the history to support the
decision for immediately prescribing an antidepressant to this patient (4 marks).

2.

The patient would like to know more about psychotherapy. Name TWO (2) types of
psychotherapy with MOST evidence to demonstrate effectiveness for her condition (2 marks).

!
!
244

1. Based on your answer in question 2, select ONE (1) type of psychotherapy and explain
how it works. (4 marks)

Question!2!
You are a general practitioner and looks after a 30-year-old woman who is 14-week pregnant.
She has been drinking alcohol and has poor compliance to antenatal follow-up. Besides alcohol,
she does not use other drug or substance. She feels that it is completely harmless to drink
alcohol during pregnancy. You need to offer psychoeducation to her.
1. What is the name of the syndrome which is of MOST concerned to her foetus? (2 mark)
2. List THREE (3) facial features associated with the syndrome described in Question 1. (3
marks)

3. If her baby is born with the syndrome described in question 1, list THREE (3)
psychiatric impairments which may occur during the childhood of her offspring (3
marks)
4. State TWO (2) specialities which you would refer this patient to consult for her current
problems. Please indicate the reasons for referral. (2 marks)
Specialities

Reasons for referral

Question!3!
You!are!a!general!practitioner!(GP)!and!seeing!a!65TyearTold!man!who!complains!of!
memory!loss.!He!also!suffers!from!depression.!!His!daughter!has!read!information!from!
the!internet!that!he!is!at!risk!of!developing!Alzheimers!disease!due!to!his!age.!She!is!not!
certain!about!the!impact!of!depression!on!his!memory.!!
1.!Name!FOUR!(4)!differences!between!dementia!and!pseudodementia!(4!marks)!
!
!
245

Dementia!

Pseudodementia!

Example:!

Caused!by!Alzheimers!disease!

Caused!by!depression!

Aetiology!
!
1.!

!
!
2.!

!
!

3.!

!
!
!
4.!

!
!
!
!
!
!
3. You!have!assessed!the!patient!and!he!suffers!from!depression.!You!have!decided!
to!start!fluoxetine.!Name!ONE!(1)!common!electrolyte!abnormality!found!in!old!
people!taking!SSRI.!(2!marks)!

!
!
246

!
4. Four!months!later,!this!patient!has!tried!fluoxetine!and!it!does!not!work.!He!took!
benzodiazepine!from!another!GP!for!insomnia.!He!develops!suicidal!thought.!You!
have!referred!this!case!to!a!geriatric!psychiatrist.!The!geriatric!psychiatrist!
recommends!electroconvulsive!therapy!(ECT).!Based!on!the!clinical!information!
provided,!name!TWO!(2)!factors!which!may!increase!the!seizure!threshold!for!this!
patient!(2!marks)!during!the!ECT.!
!
!
1. His!daughter!is!concerned!about!memory!impairment!after!ECT.!State!TWO!(2)!
adjustments!which!can!apply!during!the!administration!of!ECT!to!reduce!the!risk!
of!cognitive!impairment.!
!
!
Question!4!
You are a specialist working in renal medicine. A 60-year-old woman suffers from chronic
renal failure and she requires life-long hemodialysis. She wants to discontinue dialysis. It is a
long weekend and the psychiatrist is not available in your hospital. You need to determine her
capacity to make a decision to discontinue dialysis.
1. List FOUR (4) questions which you would ask this patient to assess her capacity to
make a decision to discontinue dialysis (4 marks).
2. You are reviewing her past psychiatric record. State FOUR (4) psychiatric disorders
that may affect her capacity to give consent.(4 marks)
3. If this patient is free from psychiatric illness and has the capacity to make decision. You respect
her right to make a decision to discontinue dialysis. State the MOST RELEVANT ethical
principle which support your decision. (2 marks)

!
!
!
!
!
SAQ (Paper 2)
!
!
247

Question 1
You are a General Practitioner. A 30-year-old woman comes to your clinic and she complains of low
mood. You suspect that she suffers from depressive disorder. She read a lot of information on the internet
and has her personal views on treatment. She would like to discuss with you on various treatment options.
5.

List Four (4) pieces of information which you would inquire from the history to support the
decision for immediately prescribing an antidepressant to this patient (4 marks).

Answer may vary. The examiner is advised to use his or her discretion to decide whether marks should be
awarded.
1. The severity of depression: her symptoms are severe (e.g. with the presence of suicidal thought and a
lot of biological symptoms).
2. Lack of precipitant in her depression: her depression seems to be endogeneous and it is not a reactive
depression.
3. Presence of family history of depression: suggest biological cause of depression.
4. Previous response to antidepressant or previous relapse after stopping antidepressant.
5.
6.
7.
8.

Severe impairment in functioning.


Failure of psychotherapy.
Past history of depression.
Patients preference

6.

The patient would like to know more about psychotherapy. Name TWO (2) types of
psychotherapy with MOST evidence to demonstrate effectiveness for her condition (2 marks).

2. Cognitive behaviour therapy


3. Interpersonal therapy
Other types of psychotherapies (0 mark)
4. Based on your answer in question 2, select ONE (1) type of psychotherapy and explain
how it works. (4 marks)
Answer may vary. The examiner is advised to use his or her discretion to decide whether marks should be
awarded.
For students who chose to explain CBT:
The frequency of Cognitive behaviour therapy (CBT) is usually weekly or fortnightly. It requires 12 to 16
sessions. The cognitive therapy involves identifying negative automatic thoughts and use dysfunctional thought
diary to identify pattern between the time, events, negative thoughts and resulted emotions and behaviours. The
psychologist will read the diary and help patients to question the negative automatic thoughts. Behaviour
therapy involves activity scheduling (for those depressed patients with psychomotor retardation), relaxation
techniques (for those patients with mixed anxiety and depression).

For students who chose to explain IPT:


Interpersonal therapy (IPT) is held weekly or fortnightly. It involves 12 to 20 sessions. IPT is indicated for
depressed patients whom precipitating factor is interpersonal problems. The psychologist closely examines
interpersonal relationship and relates depression to the interpersonal event. The therapist helps the patient to
work on one of the interpersonal problem areas: 1) grief; 2) interpersonal role dispute; 3) role transition; 4)
interpersonal deficits. IPT helps the patient to develop new skills to deal with people and life situations
associated with depressive symptpms. IPT works with the patient to look at interpersonal relationship from
another angle to minimise impact on the mood. The therapist also uses role-play to improve communication
skills.

!
!
248

Question!2!
You are a general practitioner and looks after a 30-year-old woman who is 14-week pregnant.
She has been drinking alcohol and has poor compliance to antenatal follow-up. Besides alcohol,
she does not use other drug or substance. She feels that it is completely harmless to drink
alcohol during pregnancy. You need to offer psychoeducation to her.
5. What is the name of the syndrome which is of MOST concerned to her foetus? (2 mark)
Foetal alcohol syndrome
6. List THREE (3) facial features associated with the syndrome described in Question 1. (3
marks)
Answer may vary. The examiner is advised to use his or her discretion to decide whether marks should be
awarded.

Any three of the following:

Small head
Small eyes
Short nose
Ear lobe abnormality
Short palpebral fissures (measured from inner to outer canthus; reflect underlying brain growth)
Thin upper lip
Long and flat philtrum

7. If her baby is born with the syndrome described in question 1, list THREE (3)
psychiatric impairments which may occur during the childhood of her offspring (3
marks)
Answer may vary. The examiner is advised to use his or her discretion to decide whether marks should be
awarded.

Any three of the following:

intellectual impairment
attention deficit or hyperactivity
difficulty with reasoning
depression
anxiety
oppositional defiant or conduct disorder

8. State TWO (2) specialities which you would refer this patient to consult for her current
problems. Please indicate the reasons for referral. (2 marks)
Specialities

Reasons for referral

!
!
249

Specialities

Reasons for referral

Obstetrics and gynaecology (0.5 mark)

Frequent ultrasound monitoring for foetal


abnormalities (0.5 mark)

Psychiatrist or addiction specialist (0.5 mark)

For detoxification and treatment of alcohol


dependence (0.5 mark)

Other specialist: no mark.


For the reasons of referral, the answer may vary. The examiner is advised to use his or her discretion to decide
whether marks should be awarded.
This is an advanced-level question.

!
!
!
!
!
!
!
!
!
!
!
!
!
250

!
!
!
!
!
!
!
Question!3!
You!are!a!general!practitioner!(GP)!and!seeing!a!65TyearTold!man!who!complains!of!
memory!loss.!He!also!suffers!from!depression.!!His!daughter!has!read!information!from!
the!internet!that!he!is!at!risk!of!developing!Alzheimers!disease!due!to!his!age.!She!is!not!
certain!about!the!impact!of!depression!on!his!memory.!!
1.!Name!FOUR!(4)!differences!between!dementia!and!pseudodementia!(4!marks)!
!

Dementia!

Pseudodementia!

Example:!

Caused!by!Alzheimers!disease!

Caused!by!depression!

Aetiology!
!
1.!

!
!
2.!

!
!

3.!

!
!
251

!
!
!
4.!

!
!
!
!
!
Answer may vary. The examiner is advised to use his or her discretion to decide whether marks should be
awarded.

!
Any 3 of the following categories:
Dementia

Pseudo-dementia

Onset

Insidious onset (1 mark)

Acute onset (1 mark)

Progression

Slow progression (1 mark)

Rapid progression (1 mark)

Disability

Minimize disability (1 mark)

Emphasize on disability (1 mark)

Extent of memory
loss

Recent memory loss more severe.


(1 mark)

Equal loss of recent and remote events.


(1 mark)

Consistency

Consistent pattern of memory loss


(1 mark)

Variable pattern of memory loss (1


mark)

Insight/ performance
during Mini Mental
State Examination

Deny memory loss or confabulate


to fill up memory gap

Focus on cognitive loss(1 mark) or

!
!
252

Dont know answer (1 mark)

Minimize disability (1 mark)

Variation within a
day

Sun-downer effect (worse in the


evening) (1 mark)

Diurnal variation (worse in the


morning) (1 mark)

Personal history or
family history of
depression

No or less likely (1 mark)

Yes or more likely (1 mark)

!
7. You!have!assessed!the!patient!and!he!suffers!from!depression.!You!have!decided!
to!start!fluoxetine.!Name!ONE!(1)!common!electrolyte!abnormality!found!in!old!
people!taking!SSRI.!(2!marks)!
Hyponatremia.!
!
8. Four!months!later,!this!patient!has!tried!fluoxetine!and!it!does!not!work.!He!took!
benzodiazepine!from!another!GP!for!insomnia.!He!develops!suicidal!thought.!You!
have!referred!this!case!to!a!geriatric!psychiatrist.!The!geriatric!psychiatrist!
recommends!electroconvulsive!therapy!(ECT).!Based!on!the!clinical!information!
provided,!name!TWO!(2)!factors!which!may!increase!the!seizure!threshold!for!this!
patient!(2!marks)!during!the!ECT.!
Any!2!of!the!following:!
2.
3.
4.
5.

Old!age!
Male!gender!
Baldness!
Usage!of!benzodiazepine.!

This!is!an!advancedNlevel!question.!
6. His!daughter!is!concerned!about!memory!impairment!after!ECT.!State!TWO!(2)!
adjustments!which!can!apply!during!the!administration!of!ECT!to!reduce!the!risk!
of!cognitive!impairment.!
Any!2!of!the!following:!
1. Unilateral!ECT!
2. Use!lower!energy!level!
3. Reduce!frequency!of!ECT!(two!times!per!week!instead!of!three!times!per!week)!
This!is!an!advancedNlevel!question.!
!
!
253

!
!
!
Question!4!
You are a specialist working in renal medicine. A 60-year-old woman suffers from chronic
renal failure and she requires life-long hemodialysis. She wants to discontinue dialysis. It is a
long weekend and the psychiatrist is not available in your hospital. You need to determine her
capacity to make a decision to discontinue dialysis.
4. List FOUR (4) questions which you would ask this patient to assess her capacity to
make a decision to discontinue dialysis (4 marks).
Answer may vary. The examiner is advised to use his or her discretion to decide whether marks should be
awarded. Any three of the following:
1) What is the nature of your medical condition? / What has the treatment team told you about your condition?/
What do you believe is wrong with your health?
2) What is the purpose of the dialysis? / What have you been told about the recommended treatment?
3) Can you tell me the benefits of dialysis? / What have you been told about benefits of dialysis?/ What is the
treatment likely to do for you?
4) Can you tell me the risks/side effects of dialysis? / What have you been told about the risks or discomforts
associated with dialysis?
5) What happens if you do not get dialysis?
6) Are there alternative to dialysis? / What have you been told about alternatives to dialysis?
7) Why do you want to discontinue dialysis? / What have you been told about risks and benefits of no
treatment?/ How did you decide to discontinue dialysis?
5. You are reviewing her past psychiatric record. State FOUR (4) psychiatric disorders
that may affect her capacity to give consent.(4 marks)
Answer may vary. The examiner is advised to use his or her discretion to decide whether marks should be
awarded.
Any three of the following:
1.
2.
3.
4.
5.
6.
7.

Dementia (any form: Alzheimers disease, vascular dementia)


Delirium or acute confusional state
Schizophrenia
Severe depressive disorder with psychotic features (1 mark); depression (0.5mark)
Acute psychosis
Delusional disorder
Bipolar disorder with psychotic features (1 mark); bipolar disorder/mania/hypomania (0.5 mark)

!
!
254

8.

Mental retardation

6. If this patient is free from psychiatric illness and has the capacity to make decision. You respect
her right to make a decision to discontinue dialysis. State the MOST RELEVANT ethical
principle which support your decision. (2 marks)
Respect patients autonomy

!
!
!
!
!
!
SAQ!(Paper!3)!!
Question!1!(Total!=!10!marks)!
You are a general practitioner (GP). You are asked to see a 36-year-old woman for the treatment of major depression.
A trial of fluoxetine 40mg every morning by another GP was unsuccessful.
She was very upset with the remarks made by her colleagues. As a result, she quitted her job 6 months ago and
became a housewife. She feels bored as a housewife and has conflicts with her in-laws. She has become more
depressed in the past 6 months. Her husband is supportive and there is no marital conflict. She has a 10-year-old son
and he has good relationship with her. She does not need to worry about his studies as he has done well in school.
She has other chronic medical illnesses and she takes other medications. She forgot to bring the medication and
cannot remember the name of those medications.
She has no history of substance abuse or dependence. Her ECG is normal.
1.

State THREE (2) MOST RELEVANT points of the history that you would like to clarify regarding previous trial
of fluoxetine? (2 marks)
/ 2 marks

2.

State TWO (2) medications which are used to treat chronic medical illnesses and known to be associated
with depression? (2 marks)
/ 2 marks

3.

Based on the patients background, state THREE (3) common medical conditions from different body
systems that is known to be associated with depression. (3 marks)
/ 3 marks

4.

Based on the patients history, state ONE (1) MOST appropriate and evidence-based psychotherapy that is
suitable for this patient. (1 mark)
/ 1 mark

5.

Based on the patients history, state how the psychotherapy recommended by you (in Question 4) works for
her. (2 marks)
/2 marks

Question!2!(Total!=!10!marks)!

!
!
255

You are a general practitioner. A 25-year-old man suffering from schizoaffective disorder is bought to the clinic by her mother. He
was diagnosed to suffer from schizoaffective disorder 6 months ago. Her mother is concerned about his recent weight gain of 10 kg
in 3 months. According to her mother, his psychiatrist from a restructured hospital started a new antipsychotic and mood stabilizer
to control his psychiatric symptoms.
1.

State TWO (2) antipsychotics which are MOST likely to cause weight gain in this patient. (2 marks)

/2 marks

2.

State TWO (2) mood stabilizers which are MOST likely to cause weight gain in this patient. (2 marks)

/2 marks

3.

State THREE (3) MOST relevant blood tests which you would like to order and abnormalities you would like to rule out for
this patient. (3 marks)
/ 3 marks

Investigations

Abnormalities
/0.5 mark

/0.5 mark

/ 0.5 mark

/ 0.5 mark

/ 0.5 mark

/ 0.5 mark

4.

State ONE (1) MOST RELEVANT AND SPECIFIC symptom in schizophrenia which may contribute to obesity (2 marks). !

5.

State ONE (1) antipsychotic that has the LOWEST RISK to cause weight gain. (1 mark)

Question 3
You are a paediatric resident working in the Child and Accident Emergency Department. You
are seeing an 11-year-old boy who was brought in by his foster parents. According to his
foster parents, he has a lot of behavioural problems at home and school. He cannot sit still to
do his homework at home. He also breaks school rules. Tonight, he had a fight with his foster
parents. This boy is a victim of child abuse.
1. State TWO (2) MOST RELEVANT diagnoses (2 marks) that this boy may suffer from and
indicate THREE (3) additional clinical features (6 marks) which you will look for to
support your diagnosis. (total 8 marks)
Diagnosis

3 additional clinical features per diagnosis

1.

1.

!
!
256

2.

3.

/1 mark

2.

/3 marks

1.

2.

3.

/1 mark

/ 3 marks

2. If you have a chance to speak to the child protection officer, state TWO (2) MOST
relevant psychiatric disorders that his parents may suffer from. (2 marks)
/ 2 marks

Any two of the following:

!
!
Question!4!
You!are!a!geriatric!resident.!A!70NyearNold!man!presents!with!dementia.!His!son!wants!to!find!
out!more!about!the!treatment!for!dementia.!
1. State the name of TWO (2) acetylcholinesterase inhibitors available in Singapore. (2 marks)!
/ 2 marks

!
!
257

2.

List TWO (2) types of dementia for which acetylcholinesterase inhibitors are indicated (2
marks)
/ 2 marks

3.

List TWO (2) COMMON side effects of acetylcholinesterase inhibitors (2 marks)


/ 2 marks

4.

List TWO (2) SPECIFIC contraindications to the use of acetylcholinesterase inhibitors (2


marks)
/ 2 marks

5.

His son wants to know potential benefits that can be expected from treatment with
acetylcholinesterase inhibitors. List TWO (2) potential benefits (2 marks).
/ 2 marks

!
!
!
!
!
!
!
!
!
SAQ!(Paper!3)!!
Question!1!(Total!=!10!marks)!
You are a general practitioner (GP). You are asked to see a 36-year-old woman for the treatment of major depression.
A trial of fluoxetine 40mg every morning by another GP was unsuccessful.
She was very upset with the remarks made by her colleagues. As a result, she quitted her job 6 months ago and
became a housewife. She feels bored as a housewife and has conflicts with her in-laws. She has become more
depressed in the past 6 months. Her husband is supportive and there is no marital conflict. She has a 10-year-old son
and he has good relationship with her. She does not need to worry about his studies as he has done well in school.
She has other chronic medical illnesses and she takes other medications. She forgot to bring the medication and
cannot remember the name of those medications.
She has no history of substance abuse or dependence. Her ECG is normal.
6.

State THREE (2) MOST RELEVANT points of the history that you would like to clarify regarding previous trial
of fluoxetine? (2 marks)
/ 2 marks

Any two of the following:


1)
2)
3)

Her previous adherence or compliance to fluoxetine (1 mark) or


Side effects associated with fluoxetine (1 mark) or
The duration of taking fluoxetine (1 mark).

!
!
258

7.

State TWO (2) medications which are used to treat chronic medical illnesses and known to be associated
with depression? (2 marks)
/ 2 marks

Any two of the following:

1)
2)
3)
4)
5)
6)

Corticosteroids!/!steroids/!prednisolone!(1!mark)!or!!
BetaNblockers!/!atenolol!/!propranolol!(1!mark)!or!!
Clonidine!(1!mark)!or!!
Metoclopramide!(1!mark)!or!!
Theophylline!(1!mark)!or!!
Calicium!channel!blockers:!Nifedipine!(1!mark).!

8.

Based on the patients background, state THREE (3) common medical conditions from different body
systems that is known to be associated with depression. (3 marks)
/ 3 marks

Answers may vary. Examiners are advised to use his or her own discretion to decide whether marks should be
awarded. Any three of the following:
1) Cardiovascular disorders: hypertension (1 mark)
2) Endocrine disorders: diabetes, hypothyroidism or Cushing syndrome or Addisons disease (1 mark)
3) Neurological disorders: epilepsy (1 mark)
4) Gastrointestinal system: peptic ulcer or irritable bowel syndrome or inflammatory bowel diseases (1 mark)
5) Immune system: systemic lupus erythematosus (1 mark)
Medical disorders which are more common in older adults or elderly e.g. Parkinsons disease, cerebrovascular accidents,
chronic obstructive pulmonary disease = 0 mark. Myocardial infarction = 0 mark as ECG is normal. If all answers are from one
single system = 1 mark.
9.

Based on the patients history, state ONE (1) MOST appropriate and evidence-based psychotherapy that is
suitable for this patient. (1 mark)
/ 1 mark
Interpersonal psychotherapy
Other psychotherapies = 0 mark.

10. Based on the patients history, state how the psychotherapy recommended by you (in Question 4) works for
her. (2 marks)
/2 marks
Answers may vary. Examiners are advised to use his or her own discretion to decide whether marks should be
awarded. For students who did not state interpersonal psychotherapy in Q.4, they should only get a maximum of 1
mark for question 5.
Intepersonal psychotherapy can help the patient to analyze previous interpersonal problems in her work place.
Interpersonal psychotherapy involves role play and patient can improve her communication with her in-laws. The role
play can identify previous maladaptive communication patterns. Interpersonal psychotherapy can help her to deal
with the role transition from an administrative person to become a housewife.

Question!2!(Total!=!10!marks)!
You are a general practitioner. A 25-year-old man suffering from schizoaffective disorder is bought to the clinic by her mother. He
was diagnosed to suffer from schizoaffective disorder 6 months ago. Her mother is concerned about his recent weight gain of 10 kg
in 3 months. According to her mother, his psychiatrist from a restructured hospital started a new antipsychotic and mood stabilizer
to control his psychiatric symptoms.
6.

State TWO (2) antipsychotics which are MOST likely to cause weight gain in this patient. (2 marks)

/2 marks

Any two of the following:

Olanzapine (1 mark) or

Clozapine (1 mark) or

Quetiapine (1 mark) or

Risperidone (0.5 mark)

Other antipsychotics = 0 mark


7.

State TWO (2) mood stabilizers which are MOST likely to cause weight gain in this patient. (2 marks)

Any two of the following:

Lithium (1 mark) or

!
!
259

/2 marks

8.

Sodium valproate (1 mark) or


Lamotrigine (1 mark) or
Other mood stabilizers: no mark

State THREE (3) MOST relevant blood tests which you would like to order and abnormalities you would like to rule out for
this patient. (3 marks)
/ 3 marks

Investigations

Abnormalities
/0.5 mark

/0.5 mark

/ 0.5 mark

/ 0.5 mark

/ 0.5 mark

/ 0.5 mark

Investigations

Abnormalities

Fasting glucose (0.5 mark)

Rule out diabetes (0.5 mark)

Fasting lipid / cholesterol / HDL /


LDL (0.5 mark)

Rule out hyperlipidaemia (0.5 mark)

Liver function tests (0.5 mark)

Rule out raised AST and ALT / fatty liver / liver impairment
(0.5 mark)

Thyroid function test

Rule out hypothyroidism and associated weight gain

9.

State ONE (1) MOST RELEVANT AND SPECIFIC symptom in schizophrenia which may contribute to obesity (2 marks). !
1. Avolition or ahehdonia or lack of interest to do exercise (2 marks)
2. Negative symptom (1 mark)
3. Other negative symptom e.g. apathy, alogia, attentional deficit (0 mark)

10. State ONE (1) antipsychotic that has the LOWEST RISK to cause weight gain. (1 mark)
Aripiprazole/ haloperidol (1 mark). Other antipsychotic = 0 mark

!
!
260

Question 3
You are a paediatric resident working in the Child and Accident Emergency Department. You
are seeing an 11-year-old boy who was brought in by his foster parents. According to his
foster parents, he has a lot of behavioural problems at home and school. He cannot sit still to
do his homework at home. He also breaks school rules. Tonight, he had a fight with his foster
parents. This boy is a victim of child abuse.
3. State TWO (2) MOST RELEVANT diagnoses (2 marks) that this boy may suffer from and
indicate THREE (3) additional clinical features (6 marks) which you will look for to
support your diagnosis. (total 8 marks)
Diagnosis

3 additional clinical features per diagnosis

1.

1.

2.

3.

/1 mark

2.

/3 marks

1.

2.

3.

/1 mark

/ 3 marks

!
!
261

Diagnosis
1.

Attention deficit
hyperactivity disorder (1
mark)
Abbreviation = 0 mark

3 additional clinical features per diagnosis


Any THREE of the following:

o
o
o
o
o

2.

Conduct disorder

Any THREE of the following:

/1 mark

Oppositional defiant disorder = 0 mark

Starts tasks or activities but not able to follow through and


finish.
Organization of tasks or activities is impaired.
Loses things necessary for tasks and activities such as
school assignments or stationary.
Instructions are not followed.
Distraction by external stimuli.
Careless mistakes
Forgetfulness in daily activities
Waiting for in lines or await turns in game cause
frustration.
On the move most of the time such as running and
climbing.
Restlessness and jitteriness.
Squirms on seat.
Impulsivness
Talk excessively without appropriate response to social
constraints.
Fidgets with hands and feet.
Answers are blurted out before questions.
Interruption of other peoples conversations.
Loud noise in playing.

Repetitive and persistent pattern of behaviour in which


either the basic rights of the others or major age
appropriate societal rules are violated
Often telling lies and breaking promises
Refusing adults requests or defying rules
Staying out after dark against parental prohibition
Blaming others for his mistakes
Initiating fights with the others
Using weapons to harm the others
Exhibiting physical cruelty (also to animals)
Confronting victims during a crime
Forcing another person into sexual activity
Frequently bulling the others.
Truancy
Involvement in gang.
/ 3 marks

!
!
262

4. If you have a chance to speak to the child protection officer, state TWO (2) MOST
relevant psychiatric disorders that his parents may suffer from. (2 marks)
/ 2 marks

Any two of the following:


1) Antisocial personality disorder (1 mark)
2) Substance abuse (1 mark)
Adult ADHD = 0 mark (It is unlikely MFS needs to foster out the child if parent has adult ADHD).

!
!
!
!
!
!
!
!
!
Question!4!
You!are!a!geriatric!resident.!A!70NyearNold!man!presents!with!dementia.!His!son!wants!to!find!
out!more!about!the!treatment!for!dementia.!
6. State the name of TWO (2) acetylcholinesterase inhibitors available in Singapore. (2 marks)!
/ 2 marks
Any two of the following:
Donepezil (1 mark) or
Rivastigmine (1 mark) or
Galantamine (1 mark).!
7.

List TWO (2) types of dementia for which acetylcholinesterase inhibitors are indicated (2
marks)
/ 2 marks
Any two of the following:
Alzheimers disease (1 mark)
Lewy body dementia (1 mark)

8.

List TWO (2) COMMON side effects of acetylcholinesterase inhibitors (2 marks)


/ 2 marks

!
!
263

Any!two!of!the!following:!
Diarrhoea!(excessive!cholinergic!effects)!1!mark!or!!
Difficulty!in!sleeping!(excessive!cholinergic!effects)!1!mark!or!!
Dizziness!1!mark!or!
Feeling!agitated!1!mark!or!
Headache!1!mark!or!
Loss!of!appetite!1!mark!or!
Muscle!cramps!(excessive!cholinergic!effects)!1!mark!or!
Tiredness!1!mark.!
Uncommon!side!effects!like!gastrointestinal!bleeding!or!bradycardia!(0!mark)!
9.

List TWO (2) SPECIFIC contraindications to the use of acetylcholinesterase inhibitors (2


marks)
/ 2 marks
Any two of the following:
Severe dementia or Score of Mini Mental State Examination < 10 or 12 out of 30 (1 mark)
Bradycardia or taking beta blockers (1 mark)
Gastrointestinal bleeding or peptic ulcer (1 mark)
Allergy to acetylcholinesterase inhibitors: not specific enough (0 mark)
10. His son wants to know potential benefits that can be expected from treatment with
acetylcholinesterase inhibitors. List TWO (2) potential benefits (2 marks).
/ 2 marks
Answers may vary. Examiners are requested to use his or her own discretion to decide whether
marks should be awarded.
1. Reduce the rate of deterioration of memory loss or improve cognition (1 mark)
2. Delay nursing home placement (1 mark)
3. Improve behavioural and psychiatric disorders associated with dementia (1 mark)

!
!
!
!
!
!
!
!
!
!
!
!
264

SAQ (Paper 4)
Question 1
You are the resident working in the Accident and Emergency Medicine Department. A 50-yearold man is admitted as a result of paracetamol overdose. He was treated in the Institute of
Mental Health for alcohol dependence 7 years ago.
1. State Four (4) clinical features which you would elicit in this man during history taking
to establish the diagnosis of alcohol dependence. (4 marks)
Examiner:
/4 marks
2. State TWO (2) medical or surgical disorders from two different body systems which will
result from prolonged alcohol usage. (2 marks) Examiner:
/2 marks
Body system

Specific medical or surgical disorder (not sign/symptom)


/1 mark

/1 mark

3. State TWO (2) physical signs which you will elicit from two different body systems to
support the diagnosis of alcohol dependence. (2 marks) Examiner:
/2 marks
Body system

Specific sign
/1 mark

/1 mark

4. Please explain the following laboratory finding (2 marks).


AST: 600 IU/L
ALT: 150 IU/L

!
!
265

Question 3
You are a general practitioner. A 33-year-old woman delivered a boy two
months ago. She is single and she broke up with her boyfriend recently. This is her first
pregnancy and it was unplanned. The infant was delivered by normal vaginal delivery. She is
unemployed and receives social assistance ($300 per month). She is not close to her family
members and stays alone in a 1-room HDB flat. She was seen by a psychiatrist at the Institute
of Mental Health 3 years ago and she did not return for further appointment. She complains of
low mood, poor sleep, poor appetite, poor energy and hopelessness for the past 6 weeks. You
suspect that she suffers from postnatal depression.
1. From her history, list FOUR (4) specific risk factors for postpartum depression. (4 marks)
Examiner:
/4 marks
2. State TWO (2) MOST important psychotic features which you would elicit as part of the risk
assessment (2 marks)
Examiner:
/2 marks
1. She has an appointment to see a psychiatrist in one month. She cannot wait and wants to
seek your opinion. She is very keen to continue breastfeeding. If she wants to take an
antidepressant such as a selective serotonin reuptake inhibitor (SSRI) and continues
breastfeeding, state TWO (2) advices to minimize SSRI exposure to the infant. (2 marks)
Examiner:
/ 2 marks
4. If the patient does not want to take antidepressant, please recommend TWO (2) alternative
treatment options. (2 marks)
Examiner:
/ 2 marks

Question 4
You are a general practitioner. A mother brings her 13-year-old girl to see you. She complains
of low mood, poor sleep, poor appetite and recurrent self-harm. She often cuts her wrist with a
pen knife. She has difficulty to cope with her study in Secondary 1. She often brings a pen
knife to school and cuts herself in the toilet. Her mother breaks into tear in the middle of the
interview because her husband is dependent on alcohol and has history of violence. The
patient is very close to her mother. As a result, the patient and her mother have no
communication with her father.
Question 1: Her mother worries about her personality development. State ONE (1) personality
disorder which she will MOST likely develop if the above pattern continues without any
intervention? (2 marks) Examiner: / 2 marks
Question 2: Based on your answer in Question 1, state TWO (2) MOST relevant comorbid
psychiatric disorders. (2 marks)
Examiner: / 2 marks

!
!
266

Question 3 Her mother hopes that you can prescribe a psychotropic medication to
stabilize her. State ONE (1) psychotropic medication which is most suitable to her. (2 marks)
Examiner: / 2 marks
Class of medication

Specific name of medication

/1 mark

/1 mark

Question 4
Besides cognitive behaviour therapy, state TWO (2) MOST relevant
psychotherapy which will benefit this patient. (2 marks)
Examiner:
/2 marks
Question 5 Her mother is concerned about her safety in the next few weeks while waiting for
the effect of the medication. List TWO (2) MOST relevant strategies to manage safety issues in
this patient. (2 marks)
Examiner:
/2 marks

!
!
267

SAQ (Paper 4)
Question 1
You are the resident working in the Accident and Emergency Medicine Department. A 50-yearold man is admitted as a result of paracetamol overdose. He was treated in the Institute of
Mental Health for alcohol dependence 7 years ago.
5. State Four (4) clinical features which you would elicit in this man during history taking
to establish the diagnosis of alcohol dependence. (4 marks)
Examiner:
/4 marks
Any 4 of the following criteria for substance dependence: WASTED/ CAGE
o Withdrawal
o Amounts of alcohol intake (large amount for long periods)
o Social activities replaced by time spent acquiring substance
o Tolerance: higher amount of alcohol to achieve the same effect
o Efforts to cut down unsuccessful
o Despite adverse consequences, keeps drinking alcohol
o
o
o

Feel Annoyed by criticisms of other people on drinking


Feel Guilty about drinking
Eye opening: drinking alcohol after wake-up in the morning

6. State TWO (2) medical or surgical disorders from two different body systems which will
result from prolonged alcohol usage. (2 marks) Examiner:
/2 marks
Body system

Specific medical or surgical disorder (not sign/symptom)


/1 mark

/1 mark

Any 2 of the following (The list is not completed and examiner will use his or her discretion to decide
whether marks should be awarded).

CNS complications: withdrawal fits/epilepsy, head injury as a result of fall, cerebellar damage
Gastrointestinal system: cancer in the oral cavity (e.g. tongue, throat), oesophagitis, gastritis,
gastric cancer.
Hepatobiliary system: pancreatic cancer, liver cirrhosis, alcohol hepatitis.
Peripheral nervous system: peripheral neuropathy
Rheumatology: gouty arthritis
Reproductive system: erectile dysfunction.

!
!
268

7. State TWO (2) physical signs which you will elicit from two different body systems to
support the diagnosis of alcohol dependence. (2 marks) Examiner:
/2 marks
Body system

Specific sign
/1 mark

/1 mark

Any 2 of the following (The list is not completed and examiner will use his or her discretion to decide
whether marks should be awarded).
Endocrine system: gynaecomastia
Hepatobiliary system: spider angiomata/navei; ascites; signs of liver cirrhosis: hard liver
Central nervous system: cerebellar ataxia (unsteady and wide-based gait)
Rheumatology: painful big toe
Peripheral nervous system: less of peripheral sensation to different stimuli (e.g. pin or cotton wool)
8. Please explain the following laboratory finding (2 marks).
AST: 600 IU/L
ALT: 150 IU/L
This patient suffers from alcohol hepatitis due to AST: ALT ratio > 2.

!
!
269

Question 3
You are a general practitioner. A 33-year-old woman delivered a boy two
months ago. She is single and she broke up with her boyfriend recently. This is her first
pregnancy and it was unplanned. The infant was delivered by normal vaginal delivery. She is
unemployed and receives social assistance ($300 per month). She is not close to her family
members and stays alone in a 1-room HDB flat. She was seen by a psychiatrist at the Institute
of Mental Health 3 years ago and she did not return for further appointment. She complains of
low mood, poor sleep, poor appetite, poor energy and hopelessness for the past 6 weeks. You
suspect that she suffers from postnatal depression.
3. From her history, list FOUR (4) specific risk factors for postpartum depression. (4 marks)
Examiner:
/4 marks
Any three of the following:
1. Single mother (the mother is not that old in this case).
2. Unplanned pregnancy
3. Recent break up/Unemployment / financial difficulties / low socio-economic status.
4. Poor support from family
5. History of depression
6. Negative confinement
7. Past psychiatric history
4. State TWO (2) MOST important psychotic features which you would elicit as part of the risk
assessment (2 marks)
Examiner:
/2 marks
Answer may vary. Examiner will use his or her discretion to decide whether marks should be
awarded.
2. Command hallucination which asks her to harm herself and her infant.
3. Delusion which suggests that the infant is malformed/abnormal or possessed by demons and
she need to attack the infant.
Other mood congruent delusions e.g. delusion of guilt or nihilistic delusion 0.5 marks
4. She has an appointment to see a psychiatrist in one month. She cannot wait and wants to
seek your opinion. She is very keen to continue breastfeeding. If she wants to take an
antidepressant such as a selective serotonin reuptake inhibitor (SSRI) and continues

!
!
270

breastfeeding, state TWO (2) advices to minimize SSRI exposure to the infant. (2 marks)
Examiner:
/ 2 marks
Answer may vary. Examiner will use his or her discretion to decide whether marks should be
awarded. Any two of the following:
1.
2.
3.
4.
5.
6.
7.

Take the lowest effective dose of a SSRI


Take SSRI once per day instead of twice per day.
Breastfeed or pump out breast milk immediately after or just before taking the SSRI
Discard the first round of breast milk which may contain SSRI.
Take SSRI just before the longest period of sleep of the infant.
Combination of breast milk and formula milk.
Use SSRI with short half live.

Other answers:
Name specific SSRI without reasoning (0 mark)
SSRIs are harmful to the infant and she must not take SSRI (0 mark)
She must give up breastfeeding (0 mark)
4. If the patient does not want to take antidepressant, please recommend TWO (2) alternative
treatment options. (2 marks)
Examiner:
/ 2 marks
1. Psychotherapy e.g. cognitive behaviour therapy or other forms of psychotherapy.
2. Electro-convulsive therapy
Suggest other medications: 0 mark

Question 4
You are a general practitioner. A mother brings her 13-year-old girl to see you. She complains
of low mood, poor sleep, poor appetite and recurrent self-harm. She often cuts her wrist with a
pen knife. She has difficulty to cope with her study in Secondary 1. She often brings a pen
knife to school and cuts herself in the toilet. Her mother breaks into tear in the middle of the
interview because her husband is dependent on alcohol and has history of violence. The
patient is very close to her mother. As a result, the patient and her mother have no
communication with her father.
Question 1: Her mother worries about her personality development. State ONE (1) personality
disorder which she will MOST likely develop if the above pattern continues without any
intervention? (2 marks) Examiner: / 2 marks
Answer: Borderline personality disorder (2 marks). Other diagnosis: no mark.
Question 2: Based on your answer in Question 1, state TWO (2) MOST relevant comorbid
psychiatric disorders. (2 marks)
Examiner: / 2 marks
Any two of the following:
1. Bulimia nervosa
2. Depressive disorder
3. Post-traumatic stress disorder

!
!
271

4. Substance abuse or dependence


Question 3 Her mother hopes that you can prescribe a psychotropic medication to
stabilize her. State ONE (1) psychotropic medication which is most suitable to her. (2 marks)
Examiner: / 2 marks
Class of medication

Specific name of medication

/1 mark

/1 mark

Class of medication

Specific name of medication

Selective serotonin reuptake inhibitor

Fluoxetine

/1 mark

/1 mark

Other answer: 0 mark.


Question 4
Besides cognitive behaviour therapy, state TWO (2) MOST relevant
psychotherapy which will benefit this patient. (2 marks)
Examiner:
/2 marks
Any two of the following:
1. Dialectical behaviour therapy
2. Metalization based therapy
3. Interpersonal psychotherapy
4. Family therapy.
Question 5 Her mother is concerned about her safety in the next few weeks while waiting for
the effect of the medication. List TWO (2) MOST relevant strategies to manage safety issues in
this patient. (2 marks)
Examiner:
/2 marks
Examiner has to use his or her own discretion to decide whether marks should be awarded. Any two
of the following:
1. Discuss need for her mother and teacher to monitor for changes in behaviour or self-harm at
home and in school.
2. Remove sharp objects, weapons or pen knives or unnecessary medications from home and
school.
3. Provide information and access to crisis services in the emergency department.
4. Her mother needs to supervise and administer antidepressant to patient.
5. Frequent and regular review by doctors to monitor response and side effects

!
!
272

!
!
!
!
!
!
!
!
!
!
!
!
!
SAQ 2014 EOPT First rotation
Question 1 (10 marks)

A 24-year-old female executive consults her family physician complaining that she has
faced problems at work for the past year. It started after a presentation when her slides
were mixed-up and she was at a loss of words. Since then, she feels tremulous and
stammers at presentations. She is increasingly fearful of doing presentations, worrying
that she may faint or vomit in front of her colleagues. Her fears have grown to the
extent that she has not gone to work for 2 weeks.
1. What is the most likely diagnosis? (1 mark)

2. List TWO (2) medications from different classes that can treat her symptoms? (4
marks)
Class of medications

Name of the medication

!
!
273

/1 mark)

/1 mark)

/1 mark)

/1 mark)

3. She wants to receive psychotherapy. Please state the most evidence based
psychotherapy based on your diagnosis. (1 mark)

4. She wants to know more how psychotherapy works. Based on your answer in
Question 3 and her clinical history, list TWO (2) specific strategies which can
help her. (4 marks)
!
!
!

Question 2 (10 marks)

A 40-year-old woman with a bipolar disorder was stable on sodium valproate till a
year ago when she suffered three episodes of relapses. Her psychiatrist recommends
switching to lithium. She uses recreational drugs, sometimes forgets to take her
medication when she is clubbing till 5 am and has had unprotected sex many times in
the past three months. A friend asks her to borrow $10,000 to invest in a food and
beverage business.
Question 1
She wants to know the systemic side effects of lithium when the dose is within
therapeutic range. State FOUR (4) body systems affected by lithium and Name ONE (1) specific
side effect of lithium under each system. (4 marks)
Systems

Specific side effects

/0.5 mark)

!
!
274

/0.5 mark)

/0.5 mark)

/0.5 mark)

/0.5 mark)

/0.5 mark)

/0.5 mark)

/0.5 mark)

Question 2
Based on her history and the possibility of starting lithium, List THREE (3) most
relevant laboratory investigations (3 marks)

(
Question 3
marks)

/ 3 marks)

Based on her current lifestyle, offer THREE (3) MOST relevant advices. (3

Question 3
You are an Emergency Room Resident assessing a 16-year-old girl brought by
ambulance after she was found lying in a corridor in a mall. The brief history given is
that she had left home 3 days ago after a break-up with her boyfriend. She had
overdosed with 50 tablets of paracetamol at the mall. She was found by passer-by and
then sent to the Accident and Emergency Department by ambulance.
!
!
275

Question 1 During your assessment, state FOUR (4) most relevant risk factors
suggesting high suicide risk which you would explore.
(

/ 4 marks)

Question 2 During your assessment, state TWO (2) most relevant protective factors
against suicide which you would explore.

/ 2 marks)

Question 3 State FOUR (4) most appropriate management orders for this patient in
the Accident and Emergency Department.

/ 4 marks)

Question 4 (10 marks)


A 70-year-old man was bought in by his family because he claims he has been seeing
ghosts for the past few months and is now hearing voices asking him to jump from his
9th floor flat. He has hypertension which is well-controlled and has been in good health.
Question 1 During the assessment, state THREE (3) risk factors for late onset
psychosis. (3 marks)

Question 2

State TWO (2) differences between psychotic features found in an old

!
!
276

person and psychotic features found in a young person suffering from schizophrenia (
/ 2 marks)

Question 3
(3 marks)

Based on his history, state THREE (3) most likely psychiatric diagnoses

Question 4 The family informs you that a family doctor started risperidone and he
developed severe rigidity and tremor. State ONE (1) antipsychotic drug which is MOST
suitable for him. (2 marks)
!
!
!
!
!
!
!
!
!
!
!
!
!
!
SAQ 2014 EOPT First rotation
Question 1 (10 marks)

A 24-year-old female executive consults her family physician complaining that she has
faced problems at work for the past year. It started after a presentation when her slides
were mixed-up and she was at a loss of words. Since then, she feels tremulous and
stammers at presentations. She is increasingly fearful of doing presentations, worrying
that she may faint or vomit in front of her colleagues. Her fears have grown to the
extent that she has not gone to work for 2 weeks.
!
!
277

5. What is the most likely diagnosis? (1 mark)


Social phobia (1 mark) or diagnosis: 0 mark including agoraphobia. (

/1 mark)

6. List TWO (2) medications from different classes that can treat her symptoms? (4
marks)
Class of medications

Name of the medication

/1 mark)

/1 mark)

Class of medications
Selective serotonin reuptake inhibitor
(SSRI) (1 mark)

/1 mark)

/1 mark)

Name of the medication


1. Paroxetine(1 mark) or
2. Fluvoxamine (1 mark) or
3. Fluoxetine (1 mark) or
4. Escitaloporam (1 mark) or
5. Sertraline (1 mark)
1. Propranolol (1 mark)

Beta-blocker (1 mark)
Benzodiazepine (1 mark)

1. Alprazolam (1 mark)
2. Clonazepam (1 mark)
3. Lorazepam (1 mark)

7. She wants to receive psychotherapy. Please state the most evidence based
psychotherapy based on your diagnosis. (1 mark)
Cognitive behaviour therapy; other psychotherapy (0 mark)

!
!
278

/1 mark)

8. She wants to know more how psychotherapy works. Based on your answer in
Question 3 and her clinical history, list TWO (2) specific strategies which can
help her. (4 marks)
!

Other logical answers are acceptable. Examiner is advised to use his or her discretion to
decide whether marks should be awarded.
Any two of the following:
1. Provide cognitive explanation for social phobia (2 marks) or
2. Exposure to simulated situations e.g. presentation with the psychologist (2 marks) or
3. Cognitive restructuring to teach control of maladaptive thoughts (e.g. humiliation,
vomiting in front of others). (2 marks) or
4. Social skill training (2 marks)

(
!
!
!

Question 2 (10 marks)

!
!
279

/ 4 marks)

A 40-year-old woman with a bipolar disorder was stable on sodium valproate till a
year ago when she suffered three episodes of relapses. Her psychiatrist recommends
switching to lithium. She uses recreational drugs, sometimes forgets to take her
medication when she is clubbing till 5 am and has had unprotected sex many times in
the past three months. A friend asks her to borrow $10,000 to invest in a food and
beverage business.
Question 1
She wants to know the systemic side effects of lithium when the dose is within
therapeutic range. State FOUR (4) body systems affected by lithium and Name ONE (1) specific
side effect of lithium under each system. (4 marks)
Systems

Specific side effects

/0.5 mark)

/0.5 mark)

/0.5 mark)

/0.5 mark)

/0.5 mark)

/0.5 mark)

/0.5 mark)

/0.5 mark)

Other logical answers are acceptable. Examiner is advised to use his or her discretion to
decide whether marks should be awarded. Signs of lithium toxicity e.g. ataxia, coarse tremor,
cardiac arrest, respiratory difficulty does not count.
Systems

Specific side effects

!
!
280

Endocrine system

Hypothyroidism

/ 0.5 mark)

Renal system

/1 mark)

/1 mark)

/1 mark)

/1 mark)

/1 mark)

Renal failure or
Polyuria

/0.5 mark)

Dermatological system

Psoriasis

/0.5 mark)

Gastrointestinal system

Metallic taste or nausea

/0.5 mark)

Nervous system

Tremor

/0.5 mark)

Question 2
Based on her history and the possibility of starting lithium, List THREE (3) most
relevant laboratory investigations (3 marks)

!
!
281

/ 3 marks)

Any three of the following:


1. Renal function tests (1 mark) or
2. Thyroid function test (1 mark) or
3. Urine drug screen (1 mark) or
4. Urine pregnancy test (1 mark) or
5. HIV test / screen for sexually transmitted diseases (1 mark) or
6. Valproate level (1 mark)
Question 3
marks)

Based on her current lifestyle, offer THREE (3) MOST relevant advices. (3

/ 3 marks)

Any three of the following:


1. Regular sleep time and wake-up time (1 mark) or
2. Avoid unprotected sex (1 mark) or
3. Avoid overspending or borrowing money or foolhardy investment (1mark) or
4. Avoid alcohol or recreational drug (1 mark) or
5. Ensure compliance to mood stabilizer (1 mark)

!
!
!

Question 3
You are an Emergency Room Resident assessing a 16-year-old girl brought by
ambulance after she was found lying in a corridor in a mall. The brief history given is
that she had left home 3 days ago after a break-up with her boyfriend. She had
overdosed with 50 tablets of paracetamol at the mall. She was found by passer-by and
then sent to the Accident and Emergency Department by ambulance.

!
!
282

Question 1 During your assessment, state FOUR (4) most relevant risk factors
suggesting high suicide risk which you would explore.
Other logical answers are acceptable. Examiner is advised to use his or her discretion to
decide whether marks should be awarded.
Poor relationship with parents and family members (1 mark) or
Previous suicidal ideation or attempts (1 mark) or
Belief about lethality of paracetamol overdose (1 mark) or
Circumstances of suicide (e.g. suicide notes, isolation, avoidance of discover) (1 mark) or
Current plans for further suicidality (1 mark) or
Strong preoccupation with the relationship/ suicide as a mean to salvage the relationship
(1 mark) or
7. Presence or absence of alcohol intoxication (1 mark) or
8. Family history of psychiatric illness (1 mark) or
9. Current or past history of major depression or other psychiatric illness/or chronic medical
illness (1 mark).
1.
2.
3.
4.
5.
6.

/ 4 marks)

Question 2 During your assessment, state TWO (2) most relevant protective factors
against suicide which you would explore.
Other logical answers are acceptable. Examiner is advised to use his or her discretion to
decide whether marks should be awarded.
1. Religion (1 mark) or
2. Seeing a counsellor or psychologist (1 mark) or
3. Regret over suicide attempt (1 mark) or
4. Positive plan in the near future (e.g. return to study) (1 mark).
(

/ 2 marks)

Question 3 State FOUR (4) most appropriate management orders for this patient in
the Accident and Emergency Department.
Other logical answers are acceptable. Examiner is advised to use his or her discretion to
decide whether marks should be awarded.
1. Order investigations including Full blood count, liver function tests, renal function
tests, paracetamol levels, urine drug screen, electrocardiogram etc (1 mark) or
2. Activate charcoal to reduce absorption (1 mark) or
3. Suicide precaution (1 mark) or
4. Start N-acetylcystine (NAC) (1 mark) or
5. Inform family about her suicide attempt or to obtain collateral information (1 mark) or
!
!
283

6. Inform on-call psychiatrist for assessment (1 mark) or


7. Admit to the medical ward for further management (1 mark).
(

!
!
284

/ 4 marks)

Question 4 (10 marks)


A 70-year-old man was bought in by his family because he claims he has been seeing
ghosts for the past few months and is now hearing voices asking him to jump from his
9th floor flat. He has hypertension which is well-controlled and has been in good health.
Question 1 During the assessment, state THREE (3) risk factors for late onset
psychosis. (3 marks)
Other logical answers are acceptable. Examiner is advised to use his or her discretion to
decide whether marks should be awarded.
1. Sensory impairment (e.g. deafness) (1 mark) or
2. Social isolation (1 mark) or
3. Paranoid or schizoid or schizotypal personality trait (1 mark) or
4. Past psychiatric illness (e.g. history of schizophrenia/ severe depression/ bipolar
disorder) (1 mark) or
5. Past medical illness or underlying medical illness (e.g. stroke, head injury, brain
tumour) or medication which causes psychosis steroid induced psychosis/
Parkinsons disease (1 mark)
(

/ 3 marks)

Question 2 State TWO (2) differences between psychotic features found in an old
person and psychotic features found in a young person suffering from schizophrenia (
/ 2 marks)
Other logical answers are acceptable. Examiner is advised to use his or her discretion to
decide whether marks should be awarded.
1. Elderly exhibits less first rank symptoms / the content of psychotic symptoms (e.g.
auditory hallucination) is often less elaborated in elderly (1 mark) or
2. Elderly exhibits less negative symptoms as compared to young people suffering from
chronic schizophrenia (1 mark) or
3. Elderly are more likely to exhibit more paranoid ideas or delusions compared to
young people (1 mark).
Question 3
(3 marks)

Based on his history, state THREE (3) most likely psychiatric diagnoses

Any 3 of the following:


1. Late onset schizophrenia/ schizophrenia (1 mark) or
2. Late onset depressive disorder with psychotic features/severe depressive disorder with
psychotic features (1 mark) or
!
!
285

3. Late onset bipolar disorder / bipolar disorder with psychotic features (1 mark) or
4. Dementia with psychotic features (1 mark) or
5. Delirium or acute confusional state (1 mark)
Question 4 The family informs you that a family doctor started risperidone and he
developed severe rigidity and tremor. State ONE (1) antipsychotic drug which is MOST
suitable for him. (2 marks)
Any 1 of the following
Quetiapine (2 marks) or
Olanzapine (1 mark) or
Aripiprazole (2 marks)
(

/ 2 marks)

!
!
SAQ!!2014!Rotation!2!
Question 1
You are a general practitioner. Depression is a common mental health problem. You are about
to start a patient on an antidepressant.
1. List FOUR (4) factors related to the patients which you would consider in choosing an
antidepressant for a patient suffering from depression (4 marks).
(
/4 marks)
2. Besides selective serotonin reuptake inhibitors (SSRI), name two other
antidepressants available in Singapore and fill the following table. (6 marks)
(
Name of antidepressants

Other therapeutic effects (besides


antidepressant effect and anxiety
reduction effect)

/6 marks)

Specific side effects associated with


antidepressant (besides gastric
discomfort, headache, giddiness)

/1 mark)

/1 mark)

/1 mark)

/1 mark)

/1 mark)

/1 mark)

!
!
286

Question!2!
You!are!a!general!practitioner.!A!25NyearNold!man!presents!with!disorganized!thinking!and!
speech.!You!have!a!few!psychiatric!differential!diagnoses.!
1. State!and!define!ONE!(1)!common!type!of!disorganised!thinking!that!are!found!in!people!
without!any!mental!illness.(4!marks)!!!!!!!!!!!!!!!!!!!!!!!!!!!!!!!!!!!!!!!!!!!!!!!!!!!!!!!!!!!!!!!!!!!!!!(!!!!!!!!!!!/4!
marks)!
!
!
!
2. State!and!define!TWO(2)!common!types!of!disorganised!thinking!that!are!found!in!
people!suffering!from!schizophrenia.!(4!marks)!!!!!!!!!!!!!!!!!!!!!!!!!!!!!!!!!!!!!!!!!!!!!!!!!!!!!!!!!!!!!!!!!!(!!!!!!!!!!!
/4!marks)!
!
Type!of!disorganised!thinking!
Definition!
!
!

!
!
(!!!!!!!/1!mark)!

!
!

(!!!!!!!/1!mark)!
!

(!!!!!!!/1!mark)! !

!
(!!!!!!!/1!mark)!

!
!
3. State!and!define!ONE!(1)!common!type!of!disorganised!thinking!that!is!found!in!people!
suffering!from!bipolar!disorder.!(2!marks)!!!!!!!!!!!!!!!!!!!!!!!!!!!!!!!!!!!!!!!!!!!!!!!!!!!!!!!!!!!!!!(!!!!!!!!!!!/2!
marks)!
!
Type!of!disorganised!thinking!
Definition!
!
!

!
!
(!!!!!!!/1!mark)!

(!!!!!!!/1!mark)!

!
!
!
Question 3

!
!
287

You are a paediatric resident. You are seeing an 11-year-old boy with repetitive eye blinking
and this has caused social dysfunction. He exhibits facial tics and gets frequent facial tics
when he is anxious. He sometimes cannot control his hand movements which affect his
writing.

1. You suspect that he suffers from Tourettes syndrome. State the most APPROPRIATE
question which you would ask to establish the diagnosis of Tourettes syndrome (2
marks)
(
/ 2 marks)

2. What distinguishes Tourettes disorder from a chronic motor or vocal tic disorder? (2
marks)
(
/2 marks)

3. State ONE (1) psychiatric condition often comorbid with Tourettes syndrome. (2
marks)
(
/2 marks)

4. State ONE (1) psychiatric medication that can be used to treat Tourettes syndrome. (2
marks)
(
/2 marks)

5. State Two (2) psychological interventions that can be used to treat Tourettes
syndrome. (2 marks)
(
/2 marks)

!
Question!4:!
You! are! a! general! practitioner! (GP).! ! The! son! of! a! 60NyearNold! man! is! concerned! because! his!
father!is!taking!a!medication!called,!Flunitrazepam!(Rohypnol)!for!2!years.!The!patient!obtained!
Rohypnol! from! another! GP! who! had! retired! 1! month! ago.! Rophypnol! is! a! very! potent!
benzodiazepine!and!is!a!derivative!of!nitrazepam.!Rophypnol!is!highly!addictive!and!is!listed!as!
a! controlled! drug! by! the! Central! Narcotics! Bureau.! ! His! son! has! noted! the! following!
symptoms:! anxiety,! aggression,! getting! high,! insomnia,! memory! loss,! slurred! speech,!
tremor!in!his!father.!The!patient!was!admitted!to!the!hospital!due!to!fits.!!
!
1. Based! on! the! symptoms! reported! by! his! son! (underlined),! classify! the! symptoms! as!
intoxication!or!withdrawal!symptoms!associated!with!benzodiazepine!(4!marks)!
(!!!!!!!!!!!!!!/!4!marks)!
Intoxication!symptoms!

Withdrawal!symptoms!

1.!

1.!

!
!
288

!
(!!!!!!!!!/!0.5!marks)!

(!!!!!!!!!/!0.5!marks)!

2.!

2.!

!
(!!!!!!!!!/!0.5!marks)!

(!!!!!!!!!/!0.5!marks)!

3.!

3.!

!
(!!!!!!!!!/!0.5!marks)!

(!!!!!!!!!/!0.5!marks)!

4.!

4.!

!
(!!!!!!!!!/!0.5!marks)!

(!!!!!!!!!/!0.5!marks)!

!
!
!
!
2. State! the! MOST! APPROPRIATE! psychotropic! medication! to! treat! his! withdrawals!
(2!marks).!
!
3. After! stabilisation,! his! son! would! like! you! to! prescribe! a! nonTbenzodiazapine!
medication!to!help!his!father!to!sleep.!Name!ONE!sedative!antidepressant!and!ONE!
other!sedative!medication.!4!marks!!
!
!
!
!
!
!(!!!!!!!!!!
/4!marks)!
Sedative!antidepressant

(!!!!!!!!!!/2!marks)
Other!sedative!medication!(nonT

!
!
289

benzodiazapine)

!!!!!!!!!!!!!!!!!!!!!!!!!!!!!!!!!!!!!!!!!!!!!!!!!!!!!!!!!!!!!!!!!!!!(!!!!!!!!!!/2!marks)
!

!
!
!
!
!
!
!
!
!
!
!
!
SAQ!!2014!Rotation!2!
Question 1
You are a general practitioner. Depression is a common mental health problem. You are about
to start a patient on an antidepressant.
3. List FOUR (4) factors related to the patients which you would consider in choosing an
antidepressant for a patient suffering from depression (4 marks).
(
/4 marks)
Any four of the following:

Previous trials of antidepressants (1 mark) or


Drug-to-drug interaction (1 mark)
Previous response to antidepressants / efficacy(1 mark) or
Side effect profile/ tolerability/ safety profile of antidepressant (1 mark)
Psychiatric comorbidity (1 mark) or e.g. sleep difficulties, comorbid anxiety
Risk of suicide or overdose (1 mark) or

!
!
290

Underlying medical conditions (1 mark) or


Concurrent medications (1 mark) or
Symptom profile of depression (1 mark) or
Patients preference (1 mark) or
Age of patient (1 mark) or
Pregnancy of patient / breast feeding (1 mark) or
Frequency of dose (only fluoxetine can be given every other day) (1mark) or
Financial status or affordability by patients (1 mark).
Other answers from students: Liver function (more important than kidney function),
current body weight and BMI, occupation (with explanation e.g. drowsiness when driving)

4. Besides selective serotonin reuptake inhibitors (SSRI), name two other


antidepressants available in Singapore and fill the following table. (6 marks)
(
Name of antidepressants

Other therapeutic effects (besides


antidepressant effect and anxiety
reduction effect)

/6 marks)

Specific side effects associated with


antidepressant (besides gastric
discomfort, headache, giddiness)

/1 mark)

/1 mark)

/1 mark)

/1 mark)

/1 mark)

/1 mark)

Examples: mirtazapine, venlafaxine, trazodone, agomelatine, amitriptyline etc. The following is an


example of acceptable answer. Examiner has to use his or her own discretion to decide whether
marks should be awarded. Spelling error of the name of the drug (deduct 0.5 marks).
Name of antidepressants

Other therapeutic effects (besides


antidepressant effect and anxiety
reduction effect)

Specific side effects associated with


antidepressant (besides gastric
discomfort, headache, giddiness)

Mirtazapine

Useful for sedation and insomnia

Weight gain

/1 mark)

Bupropion

/1 mark)

Neuropathic pain or enuresis

Amitriptyline or
imipramine
(

/1 mark)

Seizure

!
!
291

/1 mark)

/1 mark)

Constipation

/1 mark)
Smoking cessation

Question!2!
You!are!a!general!practitioner.!A!25NyearNold!man!presents!with!disorganized!thinking!and!
speech.!You!have!a!few!psychiatric!differential!diagnoses.!
4. State!and!define!ONE!(1)!common!type!of!disorganised!thinking!that!are!found!in!people!
without!any!mental!illness.(4!marks)!!!!!!!!!!!!!!!!!!!!!!!!!!!!!!!!!!!!!!!!!!!!!!!!!!!!!!!!!!!!!!!!!!!!!!(!!!!!!!!!!!/4!
marks)!
!
Type!of!disorganised!thinking!
Definition!
!
!

!
!
(!!!!!!!/2!marks)!

(!!!!!!!/2!marks)!

!
!
5. State!and!define!TWO(2)!common!types!of!disorganised!thinking!that!are!found!in!
people!suffering!from!schizophrenia.!(4!marks)!!!!!!!!!!!!!!!!!!!!!!!!!!!!!!!!!!!!!!!!!!!!!!!!!!!!!!!!!!!!!!!!!!(!!!!!!!!!!!
/4!marks)!
!
Type!of!disorganised!thinking!
Definition!
!
!

!
!
(!!!!!!!/1!mark)!

!
!

(!!!!!!!/1!mark)!
!

(!!!!!!!/1!mark)! !

!
(!!!!!!!/1!mark)!

!
!
6. State!and!define!ONE!(1)!common!type!of!disorganised!thinking!that!is!found!in!people!
suffering!from!bipolar!disorder.!(2!marks)!!!!!!!!!!!!!!!!!!!!!!!!!!!!!!!!!!!!!!!!!!!!!!!!!!!!!!!!!!!!!!(!!!!!!!!!!!/2!
marks)!
!
Type!of!disorganised!thinking!
Definition!
!
!

!
!

!
!
292

(!!!!!!!/1!mark)!

(!!!!!!!/1!mark)!

!
!
!
!
!
!
!
!
Answers!to!question!2!
1. State!and!define!ONE!(1)!common!types!of!disorganised!thinking!that!are!found!in!
people!without!any!mental!illness.(4!marks)!!!!!!!!!!!!!!!!!!!!!!!!!!!!!!!!!!!!!!!!!!!!!!!!!!!!!!!!!!!!!!!!!!!!!!(!!!!!!!!!!!
/4!marks)!
!
Type of disorganised thinking

Definition (Other definitions are acceptable. Examiners


will decide on the marks)

Circumstantiality

Unwarranted detail, elaborate and irrelevant, but returns


to the original topic after a long time
(2 marks)
(2 marks)

!
2. State!and!define!THREE!(3)!specific!types!of!disorganised!thinking!that!are!found!in!
people!suffering!from!schizophrenia.!(6!marks)!!!!!!!!!!!!!!!!!!!!!!!!!!!!!!!!!!!!!!!!!!!!!!!!!!!!!!!!!!!!!!!!!!(!!!!!!!!!!!
/6!marks)!
!
Any!three!of!the!following:!
Type of disorganised thinking

Definition

Thought Blocking (1 mark)

Cessation or complete interruption in the flow of the stream


of thought (1 mark)

Loosening of association or
knights move thinking (1 mark)
Derailment (1 mark)

Loss of logical connections between ideas (1 mark)


Insertion of novel or irrelevant idea in mid-sentence (1

!
!
293

mark)
Thought insertion (1 mark)

External thoughts are inserted into the patients mind which


does not belong to the patient. (1 mark)

Thought withdrawal (1 mark)

Patients thoughts are being taken away by the others (1


mark)

Thought broadcasting (1 mark)

Patients own thoughts are known to the others through


broadcasting like a radio or TV station. (1 mark)

!
3. State!and!define!ONE!(1)!specific!type!of!disorganised!thinking!that!is!found!in!people!
suffering!from!bipolar!disorder.!(2!marks)!!!!!!!!!!!!!!!!!!!!!!!!!!!!!!!!!!!!!!!!!!!!!!!!!!!!!!!!!!!!!!(!!!!!!!!!!!/2!
marks)!
!
Type!of!disorganised!thinking!
Definition!
!
Flight!of!ideas!

!
A continuous speech which topics jump rapidly from one
(1!mark)! to another and there is a logical link between ideas.

(!1!mark)!

!
!
!
!
!
!
Question 3
You are a paediatric resident. You are seeing an 11-year-old boy with repetitive eye blinking
and this has caused social dysfunction. He exhibits facial tics and gets frequent facial tics
when he is anxious. He sometimes cannot control his hand movements which affect his
writing.

6. You suspect that he suffers from Tourettes syndrome. State the most APPROPRIATE
question which you would ask to establish the diagnosis of Tourettes syndrome (2
marks)
(
/ 2 marks)
Answer: Does he have vocal tics? (2 marks)

!
!
294

7. What distinguishes Tourettes disorder from a chronic motor or vocal tic disorder? (2
marks)
(
/2 marks)
Answer: Patients suffering from Tourettes disorder present with both motor and vocal tics.(2
marks)

8. State ONE (1) psychiatric condition often comorbid with Tourettes syndrome. (2
marks)
(
/2 marks)
Answer: Obsessive compulsive disorder (2 marks)

9. State ONE (1) psychiatric medication that can be used to treat Tourettes syndrome. (2
marks)
(
/2 marks)
Answer: Risperidone (2 marks), sulpiride (2 marks), haloperidol (2 marks)
Clonidine (0 mark) not psychiatric medication
10. State Two (2) psychological interventions that can be used to treat Tourettes
syndrome. (2 marks)
(
/2 marks)

Any!two!of!the!following:!

Behaviour!therapy!or!behaviour!modification!(1!mark)!!
Relaxation!exercise!(1!mark)!
Social!skill!training!(1!mark)!
SelfNesteem!building!(1!mark)!
Remedial!academic!help!(1!mark)!
Mass!practice!(1!mark)!
Habit!rehearsal!(1!mark)!

!
Question!4:!
You! are! a! general! practitioner! (GP).! ! The! son! of! a! 60NyearNold! man! is! concerned! because! his!
father!is!taking!a!medication!called,!Flunitrazepam!(Rohypnol)!for!2!years.!The!patient!obtained!
Rohypnol! from! another! GP! who! had! retired! 1! month! ago.! Rophypnol! is! a! very! potent!
benzodiazepine!and!is!a!derivative!of!nitrazepam.!Rophypnol!is!highly!addictive!and!is!listed!as!
a! controlled! drug! by! the! Central! Narcotics! Bureau.! ! His! son! has! noted! the! following!
symptoms:! anxiety,! aggression,! getting! high,! insomnia,! memory! loss,! slurred! speech,!
tremor!in!his!father.!The!patient!was!admitted!to!the!hospital!due!to!fits.!!
!
4. Based! on! the! symptoms! reported! by! his! son! (underlined),! classify! the! symptoms! as!
intoxication!or!withdrawal!symptoms!associated!with!benzodiazepine!(4!marks)!
(!!!!!!!!!!!!!!/!4!marks)!
!
!
295

Intoxication!symptoms!

Withdrawal!symptoms!

1.!

1.!

!
(!!!!!!!!!/!0.5!marks)!

(!!!!!!!!!/!0.5!marks)!

2.!

2.!

!
(!!!!!!!!!/!0.5!marks)!

(!!!!!!!!!/!0.5!marks)!

3.!

3.!

!
(!!!!!!!!!/!0.5!marks)!

(!!!!!!!!!/!0.5!marks)!

4.!

4.!

!
(!!!!!!!!!/!0.5!marks)!

(!!!!!!!!!/!0.5!marks)!

!
Answers:!
Intoxication!symptoms!

Withdrawal!symptoms!

1.!Aggression!(0.5!marks)!

1.!Anxiety!(0.5!marks)!

!
2.!Getting!high!(euphoria)!(0.5!marks)!

!
2.!Insomnia!(0.5!marks)!
!

3.!Memory!loss!(amnesia)!(0.5!marks)!

3.!Tremor!(0.5!marks)!
!

4.!Slurred!speech!(0.5!marks)!

4.!Fits!(withdrawal!fits)!(0.5!marks)!
!

!
!

!
!
296

!
5. State! the! MOST! APPROPRIATE! psychotropic! medication! to! treat! his! withdrawals!
(2!marks).!
(!!!!!!!!!!!!!!/!2!marks)!
Diazepam!(Valium)!is!the!only!acceptable!answer.!(2!marks)!
!
6. After! stabilisation,! his! son! would! like! you! to! prescribe! a! nonTbenzodiazapine!
medication!to!help!his!father!to!sleep.!Name!ONE!sedative!antidepressant!and!ONE!
other!sedative!medication.!4!marks!!
!
!
!
!
!
!(!!!!!!!!!!
/4!marks)!
Sedative!antidepressant

(!!!!!!!!!!/2!marks)
Other!sedative!medication!(nonT
benzodiazapine)

!!!!!!!!!!!!!!!!!!!!!!!!!!!!!!!!!!!!!!!!!!!!!!!!!!!!!!!!!!!!!!!!!!!!(!!!!!!!!!!/2!marks)
!
Sedative!antidepressant

Any one of the following:


Mirtazapine (2 marks)
Agomelatine (2 marks)
Amitriptyline (2 marks)
Fluvoxamine (2 marks)
Trazodone (2 marks)
Paroxetine (2 marks)
Sertraline (1 mark)
O t h e r a n t id e p r e s s a n t i f a p p r o p r i a t e : 1 m a r k

!
!
297

Fluoxetine/ bupoprion: 0 mark.

Other!sedative!medication!(nonT
benzodiazapine)

Any one of the following:


Hydroxyzine (2 marks)
Anti-histamine (1 mark)
Melantonin (2 marks)
Antipsychotics e.g. quetiapine (2 marks),
c h l o r p r o m a z i n e !!(2!marks),!!!o l a n z a p i n e ( 2
marks)
Non-benzodiazapine hypnotics: Zopiclone (2
marks) / Zolpidem (2 marks)

!
!
!
!
!
!
!
!
!
!
!
!
!
!
!
!
!
298

!
!
!
!
!

EOPT Rotation 3
Question 1
You are a general practitioner. A 30-year-old man suffers from major
depressive disorder. You are about to prescribe a selective serotonin reuptake
inhibitor (SSRI). He read about the SSRI on internet. You need to address the
following concerns.
1. SSRI may cause erectile dysfunction. Can you name ONE (1) antidepressant that
has relatively less sexual side effect as compared to SSRI? (2 marks)

2. If he wants to continue SSRI, state TWO (2) management strategies to improve his
erectile function if he develops SSRI induced erectile dysfunction. (2 marks)
3) He understands that the combination of SSRI and other medication may lead to
high fever. What is the name of such condition? (1 mark)
4) State ONE (1) medication that may cause serotonin syndrome if he takes together
with SSRI (1 mark)
5) State FOUR (4) signs or symptoms associated with serotonin syndrome (4 marks)

Question 2
You are a resident working in the Accident and Emergency Department (AED).
A 25-year-old man is brought in by his mother to see you after he reveals to
her about his great plan to fly to Europe by changing the magnetic pole of the
earth. He is very talkative and appears to be very excited. He disturbs
other patients in the AED.
1) State THREE (3) MOST LIKELY psychiatry diagnoses that you would
consider. (3 marks)

!
!
299

2) You are unable to perform any blood test as he is very agitated. Name TWO
(2) MOST relevant psychotropic medications to sedate this patient at the
AED. (2 marks)
3) List TWO (2) most important laboratory investigations you would do before
beginning lithium (2 marks)
!

4) State THREE (3) common side effects associated with lithium. (3 marks)
!
Question!3!
You!are!a!resident!working!in!surgery.!A!45TyearTold!man!with!a!dual!diagnosis!of!
schizophrenia!and!alcohol!dependence!is!admitted!to!the!ward!due!to!upper!
gastrointestinal!bleeding.!The!onTcall!consultant!recommends!urgent!
oesophagogastroduodenoscopy!(OGD).!!
1. You!need!to!take!an!informed!consent!from!this!patient.!State!THREE!(3)!important!
pieces!information!you!must!inform!this!patient!before!seeking!his!consent.!(3!
marks)!
!
2. State!THREE!(3)!criteria!which!you!would!like!to!assess!in!order!to!certify!he!has!the!
capacity!to!take!consent.!(3!marks)!
3) After the OGD is done, his wife hopes that he can quit drinking. List Four (4) techniques of
motivational interviewing which you could use to maximize his motivation to quit drinking (4
marks)

!
!
300

EOPT Rotation 3
Question 1
You are a general practitioner. A 30-year-old man suffers from major
depressive disorder. You are about to prescribe a selective serotonin reuptake
inhibitor (SSRI). He read about the SSRI on internet. You need to address the
following concerns.
3. SSRI may cause erectile dysfunction. Can you name ONE (1) antidepressant that
has relatively less sexual side effect as compared to SSRI? (2 marks)
Mirtazapine (2 marks) or Bupropion (2 marks) or agomelatine (2 marks)

4. If he wants to continue SSRI, state TWO (2) management strategies to improve his
erectile function if he develops SSRI induced erectile dysfunction. (2 marks)
Any two of the following:
1) Reduce the dose or prescribe the lowest dose of SSRI (1 mark)
2) Drug holiday or stop SSRI during weekend or take a long acting SSRI on alternate
day (1 mark)
3) Add sildenafil or PDE5 inhibitor (1 mark)
4) Refer patient to sensate focus or sex therapy / vacuum pump (1 mark)
Not grip method for premature ejaculation
3) He understands that the combination of SSRI and other medication may lead to
high fever. What is the name of such condition? (1 mark)
Serotonin syndrome (1 mark) Not Neuroleptic malignant syndrome

!
!
301

4) State ONE (1) medication that may cause serotonin syndrome if he takes together
with SSRI (1 mark)
Monoamine oxidase inhibitor or irreversible monoamine oxidase inhibitor (1 mark),
moclobemide (0.5 mark) MAOI (0.5 mark) (Isocaroxazid, phenelzine irreversible MAOI
1 mark)
5) State FOUR (4) signs or symptoms associated with serotonin syndrome (4 marks)
Any 4 of the following:

Rigidity, myoclonus, uncontrollable shivering


Sweating or tremor or hyperthermia or coma
Hyperkinesia or bradykinesia
High blood pressure, labile blood pressure
hyper-reflexia, headache
dysphagia, dilated pupils
cardiovascular collapse
confusion (delirium)
decreased level of consciousness/mutism
disorientation
status epilepticus
ataxia

Question 2
You are a resident working in the Accident and Emergency Department (AED).
A 25-year-old man is brought in by his mother to see you after he reveals to
her about his great plan to fly to Europe by changing the magnetic pole of the
earth. He is very talkative and appears to be very excited. He disturbs
other patients in the AED.
5) State THREE (3) MOST LIKELY psychiatry diagnoses that you would
consider. (3 marks)
Bipolar disorder / Mania (1 mark)
Schizoaffective disorder (1 mark)
Substance abuse stimulant (1 mark)
6) You are unable to perform any blood test as he is very agitated. Name TWO
(2) MOST relevant psychotropic medications to sedate this patient at the
AED. (2 marks)
Antipsychotics: risperidone, olanzapine, haloperidol (1 mark)
Benzodiazapine: diazepam, lorazepam (1 mark)
7) List TWO (2) most important laboratory investigations you would do before
beginning lithium (2 marks)

!
!
302

Renal function test (1 mark)


Thyroid function test (1 mark)
!

8) State THREE (3) common side effects associated with lithium. (3 marks)
!
Any!one!of!the!following:!
Polyuria!(1!mark)!or!Nephreogenic!diabetes!insipidus!
Polydipsia!(1!mark)!or!
Weight!gain!(1!mark)!or!
Cognitive!impairment!(1!mark)!or!!
Fine!tremor!(1!mark)!or!
Metallic!taste!(1!mark)!or!
Hypothyroidism!(1!mark)!or!
Sexual!dysfunction!(1!mark)!or!!
Drowsiness!(1!mark)!or!!
Alopecia!(1!mark)!
!
!
!
Question!3!
You!are!a!resident!working!in!surgery.!A!45TyearTold!man!with!a!dual!diagnosis!of!
schizophrenia!and!alcohol!dependence!is!admitted!to!the!ward!due!to!upper!
gastrointestinal!bleeding.!The!onTcall!consultant!recommends!urgent!
oesophagogastroduodenoscopy!(OGD).!!
3. You!need!to!take!an!informed!consent!from!this!patient.!State!THREE!(3)!important!
pieces!information!you!must!inform!this!patient!before!seeking!his!consent.!(3!
marks)!
!
1. Explain the potential diagnosis of Mallory-Weiss syndrome which is associated with alcohol
misuse (1 mark) or
2. Explain purpose and nature of the diagnostic procedure about OGD (e.g. passing an
endoscope to examine the stomach and identify bleeding site, may take some tissues) (1
mark) or
3. Explain risks and benefits of OGD (e.g. small risk of perforation of feeding tube; benefit: can
assist diagnosis) (1 mark) or
4. Alternative investigation (e.g. monitor haemoglobin levels but bleeding may continue) ( 1
mark) or
5. Prognosis if without this investigation (e.g. continue bleeding) (1 mark)

!
!
303

4. State!THREE!(3)!criteria!which!you!would!like!to!assess!in!order!to!certify!he!has!the!
capacity!to!take!consent.!(3!marks)!
!
Any!3!of!the!following:!
1.
2.
3.
4.
5.

He has a factual understanding of the information provided. ( 1 mark) or


He can appreciate the nature of the condition (1 mark)
He can appreciate consequences of accepting OGD (1 mark)
He can appreciate consequences of refusing OGD (1 mark)
He can manipulate information provided in a rational manner to reach a decision (1 mark).

!
3) After the OGD is done, his wife hopes that he can quit drinking. List Four (4) techniques of
motivational interviewing which you could use to maximize his motivation to quit drinking (4
marks)
4 Elements to the process of Motivational interviewing:
!

!
!
!

Listen empathically (1 mark for any of the following)


1. Understand the patients unique perspective and experience
2. To put you and patient on equal ground
3. Recognition and justification and very important parts of the process
Have patients voice their own reasons for change (1 mark for any of the following)
1. Articulate reasons for or not for changing
2. Look at what they find challenging
Roll with resistance (1 mark)
1. Never put yourself in position of arguing, or of trying to persuade patient to take a difficult
position, or of telling the patient what must be done immediately
Support Self-Efficacy (1 mark for any of the following)
1. Give patient hope, optimism, and provide tools to help patient succeed
2. Not to blame for failures

Short!answer!questions!(2014/2015!rotation!4)!
Question!1!(10!marks)!
You!are!a!general!practitioner.!A!65NyearNold!woman!has!taken!haloperidol!for!30!years!is!
concerned!about!a!side!effect!called!tardive!dyskinesia!(TD).!
1. State!FOUR!(4)!risk!factors!for!TD!(4!marks).!!!!!!!!!!!!!!!!!!!!!!!!!!!!!!!!!!!!!!!!!!!!!!!!!!!!!(!!!!!!!!!!!!/!4!
marks)!
!
!
2. You!are!about!to!perform!a!physical!examination.!State!TWO!(2)!anatomical!structures!
and!ONE!(1)!clinical!feature!under!each!anatomical!structure!which!you!would!look!for!
to!confirm!the!presence!of!TD!(4!marks).!!!!!!!!!!!!!!!!!!!!!!!!!!!!!!!!!!!!!!!!!!!!!!!!!!!!!!!!!!!!!!!!!(!!!!!!!!!!!!/!4!
marks)!
!
Anatomical!structure!
Clinical!feature!

!
!
304

!
!
!!!!!!!!!!!!!!!!!!!!!!!!!!!!!!!!!!!!!!!!
!
!!!!!!!!!!!!!!!!!!!!!!!!!!!!!!!!!!!!!!(!!!!!!!!!/!1!mark)!

!
!
!
!

!
!
!
!

!
!
!
!

(!!!!!!!!!/!1!mark)!

(!!!!!!!!!/!1!mark)!

(!!!!!!!!!/!1!mark)!

!!

3. If she indeed suffers from TD, state TWO (2) management strategies (2
marks) (
/ 2 marks)

Question 2 (10 marks)


A 68-year-old woman presents with poor sleep and decreased appetite for two months.
She has stopped reading because she cannot concentrate, and she is no longer seeing
friends that she is close to because she does not have the energy. She tells you that her
symptoms started after her son moved out of the house two months ago. Prior to
that, she was an energetic person who took care of her affairs and enjoyed life.
1. Besides major depressive disorder, state ONE (1) other psychiatric diagnosis based
on the above information (1 mark)
(
/ 1 mark)

!
!
305

2. Based on your diagnosis stated in Q.1, state THREE (3) non-pharmacological


interventions (3 marks)
(
/3 marks)

3. State the term which describes the above phenomenon. (2 marks) (

/2 marks)

4. With the additional information, you suspect the patient may suffer from abnormal
grief reaction although she claims she has coped well with the death of her husband.
State FOUR (4) clinical features which are found in abnormal grief but not normal
grief (4 marks)
(
/4 marks)
!

Question 3 (10 marks)


The public relation department of your hospital has received queries from a local newspaper. A
reporter wants to write about an article about alcohol misuse. You are invited to address the following
questions raised by the reporter.
1. How does long-term alcohol misuse damage our body systems? Please state FOUR (4)
physiological systems and name ONE (1) complication under each system (4 marks)
(
/ 4 marks)
Physiological system

Complication

/ 0.5 mark)

/0.5 mark)

/ 0.5 mark)

/0.5 mark)

/ 0.5 mark)

/0.5 mark)

/ 0.5 mark)

/0.5 mark)

!
!
306

2. If an alcoholic stops drinking for 1 day, what would happen to him/her? State TWO (2) common
symptoms. (2 marks)
(
/ 2 marks)
3. If an alcoholic wants to join peer support group, name ONE (1) organisation which can support
the person (2 marks)
(
/2
marks)

4. If an alcoholic wants to be admitted to a hospital to detoxify from alcohol, what are the
indications. State TWO (2) indications. (2 marks)
(
/ 2 marks)
Question 4
You are a medical resident. The accident and emergency department has sent a 55-year-old man to the
medical ward for further management due to change in mental state. When you assess him, he appears
to be manic.
1. State FIVE (5) neurological disorders which may cause mania. (5 marks)

/ 5marks)

2. State!TWO!(2)!medications!(not!illicit!drugs)!which!may!cause!mania!(2!marks)!!(!!!!!!!/2!
marks)!

3. When!you!are!clerking!the!patient,!he!suddenly!becomes!very!aggressive!and!tries!to!run!
away!from!the!ward.!You!have!tried!verbal!deNescalation!but!fails.!You!need!to!apply!
physical!restraint.!State!THREE!(3)!specific!instructions!to!nursing!staff!to!ensure!the!
physical!restraint!will!be!applied!successfully.!(3!marks)!!!!!!!!!!!!!!!!!!!!!!!!!!!!!!!!!!(!!!!!!!!!!!!!!/!3!
marks)!
!
!
!
!

Short!answer!questions!(2014/2015!rotation!4)!
Question!1!(10!marks)!
You!are!a!general!practitioner.!A!65NyearNold!woman!has!taken!haloperidol!for!30!years!is!
concerned!about!a!side!effect!called!tardive!dyskinesia!(TD).!

!
!
307

4. State!FOUR!(4)!risk!factors!for!TD!(4!marks).!!!!!!!!!!!!!!!!!!!!!!!!!!!!!!!!!!!!!!!!!!!!!!!!!!!!!(!!!!!!!!!!!!/!4!
marks)!
!
Any!3!of!the!following:!
> Old!age!(1!mark)!or!
> Female!gender!(1!mark)!or!
> Comorbid!affective!disorder!/!alcohol!abuse!(1!mark)!or!
> Comorbid!organic!brain!disorder!(1!mark)!or!
> Long!exposure!to!first!generation!antipsychotic!drug!/!potent!dopamine!receptor!
blockade!(1!mark)!or!!
> Precipitation!by!anticholinergic!drug!(1!mark)!or!
> High!antipsychotic!dose!/!rapid!increment!in!dose!(1!mark)!or!
> Previous!extra!pyramidal!side!effect!/!history!of!tremor!or!Parkinsons!disease!(1!
mark)!or!
> Genetic!predisposition!due!to!hypersensitive!dopamine!receptor!/!Family!history!of!
TD!(1!mark)!or!
> AfroNCaribbean!race!(!1!mark)!
!
5. You!are!about!to!perform!a!physical!examination.!State!TWO!(2)!anatomical!structures!
and!ONE!(1)!clinical!feature!under!each!anatomical!structure!which!you!would!look!for!
to!confirm!the!presence!of!TD!(4!marks).!!!!!!!!!!!!!!!!!!!!!!!!!!!!!!!!!!!!!!!!!!!!!!!!!!!!!!!!!!!!!!!!!(!!!!!!!!!!!!/!4!
marks)!
!
Anatomical!structure!
Clinical!feature!
!
!
!!!!!!!!!!!!!!!!!!!!!!!!!!!!!!!!!!!!!!!!
!
!!!!!!!!!!!!!!!!!!!!!!!!!!!!!!!!!!!!!!(!!!!!!!!!/!1!mark)!

!
!
!
!

!
!
!
!

!
!
!
!

(!!!!!!!!!/!1!mark)!

(!!!!!!!!!/!1!mark)!

(!!!!!!!!!/!1!mark)!

!!
Any!two!of!the!following:!
Anatomical!structure! Clinical!feature!
Mouth!!!!!!!!!!!!!!!!!!!!!!!!

Mouth!opening!

Eye!brow!

Elevation!or!depression!

!
!
308

Eye!lids!

Blepharospasm/sustained,!forced,!involuntary!closing!of!the!
eyelids!

Lips!

pursuing/retraction!to!lateral!angle/!lip!smacking!

Tongue!

Fly!catching,!tongue!protrusion!

Jaw!

Abnormal!masticationNlike!movement!/!protrusion!and!
deviation!

Neck!

Torticollis!

Upper!limb/!lower!
limb!

Hypertonia/wrist!flexion/!pilling!rolling!tremor!

Hand!

Choreiform!hand!pill!rolling!movements!

6. If she indeed suffers from TD, state TWO (2) management strategies (2
marks) (
/ 2 marks)
Any two of the following:
1.
2.
3.
4.

Change!to!second!generation!antipsychotics!(1!mark)!or!
Add!vitamin!E!or!vitamin!B!(1!mark)!or!
Benzodiazapine!e.g.!clonazapine!(1!mark)!or!
Reduce!the!dose!of!haloperidol!to!minimum!effective!dose!in!long!run!(1!mark).!

Wrong!answers:!
There!is!not!enough!evidence!to!suggest!anticholinergic!as!good!treatment!for!TD.!
Drug!holiday!may!cause!relapse!in!schizophrenia.!

!
!
309

Question 2 (10 marks)


A 68-year-old woman presents with poor sleep and decreased appetite for two months.
She has stopped reading because she cannot concentrate, and she is no longer seeing
friends that she is close to because she does not have the energy. She tells you that her
symptoms started after her son moved out of the house two months ago. Prior to
that, she was an energetic person who took care of her affairs and enjoyed life.
5. Besides major depressive disorder, state ONE (1) other psychiatric diagnosis based
on the above information (1 mark)
(
/ 1 mark)
Adjustment disorder (1 mark), Other diagnosis: 0 mark

6. Based on your diagnosis stated in Q.1, state THREE (3) non-pharmacological


interventions (3 marks)
(
/3 marks)
Any three of the following:
Supportive psychotherapy (1 mark) or
!
!
310

Problem solving therapy (1 mark) or


Cognitive behaviour therapy (1 mark) or
Interpersonal therapy (1 mark) or
Attending a day care centre (1 mark) or
Home visit by community psychiatric team (1 mark)
In the later part of interview, she mentions about the death of her husband three years ago.
She admits that she has kept everything in her late husbands room exactly the same since
his death.
7. State the term which describes the above phenomenon. (2 marks) (

/2 marks)

Mummification (2 marks), other term = 0 mark


8. With the additional information, you suspect the patient may suffer from abnormal
grief reaction although she claims she has coped well with the death of her husband.
State FOUR (4) clinical features which are found in abnormal grief but not normal
grief (4 marks)
(
/4 marks)
Any 4 of the following:
>
Prolonged!grief!with!duration!longer!than!6!months!(1!mark)!or!
>
Inhibited!grief!at!the!time!of!death!(1!mark)!or!
>
Delayed!grief!!(i.e.!no!grief!reaction!at!the!time!of!death!but!appeared!some!time!later)!(1!
mark)!!
>
Strong!suicidal!ideation!to!join!the!deceased!(1!mark)!or!!
>
Psychotic!experience!other!than!deceased!(1!mark)!or!
>
Severe!psychomotor!retardation!(1!mark)!or!
>
Severe!feeling!of!worthlessness!or!hopelessness!(1!mark)!
!

Question 3 (10 marks)


The public relation department of your hospital has received queries from a local newspaper. A
reporter wants to write about an article about alcohol misuse. You are invited to address the following
questions raised by the reporter.
5. How does long-term alcohol misuse damage our body systems? Please state FOUR (4)
physiological systems and name ONE (1) complication under each system (4 marks)
(
/ 4 marks)
Physiological system

Complication

/ 0.5 mark)

!
!
311

/0.5 mark)

/ 0.5 mark)

/0.5 mark)

/ 0.5 mark)

/0.5 mark)

/ 0.5 mark)

/0.5 mark)

Physiological system

Complication (The candidate needs to state one complication under


each physiological system)

Nervous system

Blackouts/head injury, withdrawal fits/epilepsy, acute confusional


state, subdural haematoma, degeneration of cerebellum/ataxia,
peripheral neuropathy/loss of pain sensation, alcohol dementia,
Wernicke encephalopathy, Korsakoff psychosis, central pontine
myelinolysis, Marchiafava-Bignami syndrome, depression, anxiety,
morbid jealousy

Opthalmological
system

Optic atrophy, retrobulbar neuropathy, loss of central vision,


bilateral central scotoma

Gastrointestinal
system

Cancer of mouth, larynx, pharynx, oesophagus, upper


gastrointestinal bleeding, gastritis, malabsorption syndrome

Cardiovascular
system
Respiratory system

Hypertension, cardiomyopathy, atrial fibrillation

Hepatobiliary system

Cirrhosis, liver cancer, alcohol hepatitis, fatty liver, pancreatitis,


hepatic encephalopathy

Urogenital system

Erectile dysfunction, sexual dysfunction, infertility, loss of


secondary sexual characteristics

Musculoskeletal
system
Endocrine

Myopathy, gouty arthritis, palmar erythema.

Increase in risk in respiratory infection (e.g. pneumonia,


tuberculosis)

Cushing syndrome

!
!
312

Haematological
system

Macrocytic anaemia

6. If an alcoholic stops drinking for 1 day, what would happen to him/her? State TWO (2) common
symptoms. (2 marks)
(
/ 2 marks)
Any two of the following:
> Tremor (1 mark) or
> Anxiety (1 mark) or
> Agitation (1 mark) or
> Muscle pain (1 mark) or
> Sweating (1 mark) or
> Nausea (1 mark)
> Sleep disturbance (1 mark)
> Withdrawal fits (1 mark)
7. If an alcoholic wants to join peer support group, name ONE (1) organisation which can support
the person (2 marks)
(
/2
marks)
Alcoholic anonymous (2 marks)
12-step programme (1 mark)
Other answer: 0 mark
8. If an alcoholic wants to be admitted to a hospital to detoxify from alcohol, what are the
indications. State TWO (2) indications. (2 marks)
(
/ 2 marks)
Any two of the following:
> History of delirium tremens or withdrawal fits (1 mark) or
> Current moderate to severe symptoms of alcohol withdrawal (1 mark) or
> Significant risk of self-harm, suicide or harming other people (1 mark) or
> Unable to tolerate oral medication (e.g. naltrexone) (1 mark) or
> Multiple past inpatient detoxification (1 mark) or
> Acute psychosis (1 mark) or
> Severe cognitive impairment (e.g. Wernicke encephalopathy) 1 mark or
> Poor social support (1 mark) or
> Pregnancy (1 mark) or
> Comorbid severe mental illness (1 mark)

!
!
313

Question 4
You are a medical resident. The accident and emergency department has sent a 55-year-old man to the
medical ward for further management due to change in mental state. When you assess him, he appears
to be manic.
4. State FIVE (5) neurological disorders which may cause mania. (5 marks)

/ 5marks)

Any!5!of!the!following:!
o
o
o
o
o
o
o
o
o
o
o

Multiple sclerosis (1 mark) or


Subcortical dementia (e.g. Lewy body dementia) (1 mark) or
Frontal lobe dementia (1 mark) or
Temporal lobe epilepsy (1 mark) or
Cerebrovascular accidents (1 mark) or
Head injury (1 mark) or
Encephalitis (virus e.g. HSV, HIV or parasite) (1 mark) or
Huntingtons disease (1 mark) or
Pseudobulbar palsy (1 mark) or
Brain tumour (1 mark) or
Neurosyphilis(1 mark)

5. State!TWO!(2)!medications!(not!illicit!drugs)!which!may!cause!mania!(2!marks)!!(!!!!!!!/2!
marks)!
Any!two!of!the!following:!
o
o
o
o
o

Steroid / prednisolone
Antidepressant
Anticholinergic drug
Levodopa / bomocrptine
Bronchodilator

6. When!you!are!clerking!the!patient,!he!suddenly!becomes!very!aggressive!and!tries!to!run!
away!from!the!ward.!You!have!tried!verbal!deNescalation!but!fails.!You!need!to!apply!
physical!restraint.!State!THREE!(3)!specific!instructions!to!nursing!staff!to!ensure!the!
physical!restraint!will!be!applied!successfully.!(3!marks)!!!!!!!!!!!!!!!!!!!!!!!!!!!!!!!!!!(!!!!!!!!!!!!!!/!3!
marks)!
Any!three!of!the!following:!
1. Call!for!additional!manpower!(e.g.!call!security!guards!for!help,!at!least!4!additional!
staff)!(1!mark)!or!
2. Obtain!adequate!physical!restraints!(e.g.!1!body!jacket,!1!restraint!per!limb)!(1!mark)!or!
!
!
314

3. Standby!psychotropic!medication!(e.g.!rapid!dissolvable!antipsychotics!or!intravenous!
lorazepam!/!haloperidol)!(1!mark)!or!
4. The!doctor!will!ensure!the!airway!is!not!blocked.!Other!staffs!will!focus!on!the!body!and!
four!limbs.!(1!mark)!!
!
!
!
!
!
!
!
Sample!OSCE!

Introduction
OSCE Title: Oppositional Defiant Disorder
Construct: This is a station designed to test the candidates ability to take a history
of oppositional defiant disorder.
Objectives of the OSCE station:
To observe and assess candidates ability in:
1. Eliciting symptoms of oppositional defiant disorder
2. Can distinguish oppositional defiant disorder and conduct disorder
3. Expressing empathy and demonstrating good interview skills

Time:

8 minutes

Instructions to Candidates
Parents name: Ms. Tan or Mr. Tan
!
!
315

You are a general practitioner. Mr/Mrs Tan comes to see you. His/her 11-year-old
son exhibits difficult behaviour such as being disrespectful, spiteful and
argumentative at home and school. His behaviour starts to concern his parents and
teachers.

Tasks:
1. Take a history from the parent to establish a diagnosis.
2. Perform a risk assessment
3. At the end of your interview with the parent, please write down your diagnosis
and pass it to the examiner.

Time:

8 minutes

!
!
316

Answer sheet
Please write down the diagnosis of this patient and pass it to your examiner.
___________________________________________________________________

Instructions to Simulated Patients


Background
Your name is Ms. Tan. You are married and has a single child, an 11-year-old
son called Jason. You are currently staying in a 3-room HDB flat. You worked as
an administrative officer in a company and your spouse also works as an
administrative staff in another company.
You have a 11-month-old son (Jason). When yourself and your spouse go to
work, he is looking after by a domestic helper at home. He goes to school by
school bus. However, there were no problems in the pregnancy and giving birth.
He has normal development (e.g. normal in speaking, hearing and walking)
Jason has average results from Primary 1 to Primary 4. He did not have
behavioural problems in the past.
He is your first child. Yourself and your spouse cannot afford to have a second
child.
Yourself and your spouse have no past or family history of mental illness.
Yourself and your spouse do not have forensic history.
Symptoms
Main symptoms
Jason currently studies in Primary 5 and he finds the subjects in Primary 5 are
difficult to study. In the past one year, he has been disrespectful to you, your spouse,
domestic helper and his teachers. He is noted to be defiant in almost any situation
(at home, going out with family and school). He finds his temper is bad and worse
than other children of his age. He has been argumentative, spiteful and challenges
you and the teacher the reasons to do homework. He wanted to go for a buffet
dinner last weekend but the family wants to save money and ask him to wait until
next school holiday. He was very anger and throws things at home.
Your spouse is very exhausted and gives in and tells your son that, Yes, you do not
need to do your homework. Jason then argues, There is no need to study. This
type of behaviour, over and over, leaves you and your spouse feeling overwhelmed
and exhausted. Your domestic helper finds Jason very annoying and blames her for
his mistakes.
!
!
317

You met the teacher last week. In this academic year, the teacher finds Jason
struggling in school and keeping his old friends. Jason deliberately annoys his
classmates. The teacher finds him refuse to take responsibility and blame his
classmates for his mistakes. His academic performance was above average last
year and he got a borderline pass this year. His results are similar across subjects.
He does not have a favoured subject. He has no attention problems in school and he
can sit still to do his work. He have not broken any school rule and never being
suspended before.
He never abuses animals.
His mood is normal. He sleeps and eats well. No self-harm and no suicide.
He is not nervous. He always attends school and never skips school.
There is no history of truancy.
He does not have other unusual experiences such as hearing voices or seeing
things that are not there.
He does not use recreational drugs. He does not sniff glue.
His IQ should be normal. He studies in a normal neighbourhood school.
He does not have specific learning problem (e.g. he can read, write, spell and do
mathematics)
He has normal attention and can sit still to study at home and in school.
He has normal development and there is no delay in walking and speaking.
He likes to watch TV and plays computer games. He is not addicted to internet.
Your views about your parenting

You have poor relationship with your spouse.


You agree that your parenting is ineffective because yourself and your spouse
are inconsistent. You are very harsh but your spouse is always very lenient.
Both yourself and your spouse never punish Jason physically. Jason was never
abused.
You agree that Jason may be lack of supervision when you and your spouse are
working. The domestic helper spends a lot of time on cleaning the house and no
one supervises him.

Risks
Although his behaviour is difficult to manage, there are some good things
about Jason. Jason does not violate more serious rules like running away
from home or breaking school rules. He has never been physically aggressive
toward other people (parents, classmates) and animals. Jason never initiates
a fight. He does not bully other children. He lies about minor things but never
tells a big lie. He never steals and never damages others properties. He has
no trouble with the police.
He does not harm himself
He does not harm other people.
!
!
318

He seldom has accident. E.g. falling from height due to excessive climbing or
running too fast.

Other points:
You have not consulted any doctor or counsellor on this matter.
You want to seek advice from the GP.
Time:

8 minutes

!
!
319

Instructions to Examiners
Candidate is not expected to interact with the examiners in any way.!
Please keep to the exact time.
Things not to do: use/switch on mobile phone, yawn, teach or prompt.
If observers sit in, marking is not discussed with them. Observers record their own
assessment on separate sheet.
Marks are given depending on completeness or competency in each section
respectively. The global rating should be based on the total scores of
subsections. The total mark is 25. A candidate fails the OSCE station if he or
she scores less than 12.5.

!
!
320

EXAMINERS REPORT
Students Name: _________________ Matriculation No.: _____________ Examiners Name: ______________
Objectives

very well
(excellen
t)

Appropri
ate

1. Opens interview appropriately (greet, introduce, seek


permission, explains purpose)
Closes interview (summarizes, allows patient to clarify + to ask
questions, refers to future arrangements if any)

2. Creates safe environment, develops and maintains rapport,


controls interview appropriately (open ended questions,
respect, concern, sensitivity, empathy, responsive, normalizing
statements, avoids jargon)

patchy
needs to
improve

clearly
inadeq
uate

not
done

1.5

0.5

1.5

0.5

3. Enquires about symptoms of Oppositional Defiant Disorder


a. Duration (1 year in this case)
b. Annoying and argue with other people
c. Blame others for mistake
d. Compliance refusal (e.g. refuse to study)
e. Defiant or Spiteful and Angry
f. Temper tantrums

4. Assess for symptoms for Conduct Disorder


a. Violates the rights of other people/animals: e.g.
bullying, fighting and torturing animals.
b. Vandalism and destroy properties/fire setting
c. Break rules at home or school
d. Forensic history: shoplifting, use of weapon, trouble
with police.
e. Truancy / Substance abuse / other relevant symptoms

5. Assess for risk factors and impact of for ODD


a. Inconsistent parenting
b. Lack of supervision
c. Impact on school results
d. Impact on interpersonal relationship
e. Parental relationship

6. Assess for risk issues


a. Harm to self accident, fall from height, self-harm.
b. Harm to others aggression to other people/animals

0.5

7. Other relevant information (e.g. Comorbidity: ADHD, IQ,


learning difficulty, family history of psychiatric illness/ forensic
history of parents, developmental history etc)

0.5

8. Correct diagnosis: Oppositional defiant disorder (3 marks); Any


other diagnoses (0 marks)

Subtotal score (For administrator to enter). Examiner can leave it


blank.
DECISION (circle one):

Pass

Fail

(For failure, the total score must be less than 12.5).

Signature of Examiner: ____________________________

Date: ______________________

!
!
321

Total score: Administrator 1:______________________

Administrator 2: _________________

EXAMINERS REPORT
Students Name: _________________ Matriculation No.: _____________ Examiners Name: ______________
Explain if student fails or is discussed in the examination board for borderline performance:
__________________________________________________________________________________________
__________________________________________________________________________________________
__________________________________________________________________________________________
__________________________________________________________________________________________
__________________________________________________________________________________________

!
!
!

!
!
!
!
!
!
!
!
!
!
!
322

!
!
!

Introduction
OSCE Title: Suicide risk assessment
Construct: This is a station designed to test the candidates ability to perform a
suicide risk assessment.
Objectives of the OSCE station:
To observe and assess candidates ability in:
4. To take a history of recent suicide attempt and assess suicide risk.
5. To assess depressive symptoms and other relevant history.
6. Recommend a management plan for suicide attempt.

Time:

8 minutes
Instructions to Candidates

Patients name: Ms. Gigi Lim


You are a resident working in the Accident and Emergency Department (AED). Ms.
Lim was bought to the AED by police because she tried to attempt suicide by
jumping from height. Your hospital is new and does not have a psychiatric ward.

Tasks:
4. Take a history from Ms Lim to assess her suicide attempt and current suicide
risk.
5. Assess depressive symptoms and obtain further relevant information.

!
!
323

6. At the end of your interview with Ms Lim, please select the MOST
APPROPRIATE management strategy and pass to the examiner.

Time:

8 minutes

Students sticker

Answer sheet
Please CIRCLE the MOST appropriate management strategy.

1. Discharge against medical advice.

2. Admit to the medical ward of your hospital.

3. Transfer the patient to the Institute of Mental Health.


!
!

Instructions to Simulated Patients


Background:
You are Ms. Gigi Lim, a 35-year-old woman tried to jump down from the roof
of her HDB flat tonight.
You are bought in by the police because someone spotted that you wanted to
jump down from a building and called the police who stopped you.

!
!
324

You are an unemployed, recently tested to be pregnant in urine pregnancy


test 2 weeks ago. This is an unplanned pregnancy. You do not use
contraception.
You are not married with your boyfriend. He is doing odd-job with unstable
income.

Suicide attempt
Why did you want to jump down tonight?
After you had argued with your boyfriend tonight, you wanted to jump from
height. You have thought about jumping for 2 weeks since you know the urine
result is positive.
Your relationship with him is not good in the past 1month. You have thought
of jumping down to end it all because he is not good to you and both of you
have no savings.
You want to get married with him as you are two-week pregnant. You
discovered the urine pregnancy test was positive 2 weeks ago. You have
known this boyfriend for 1 year but relationship has been unstable. He is not
keen to get married. He works as an odd job worker with unstable income.
You feel hopeless because there will not be enough money to support the
baby after the baby is born.
After you had argued with him, your boyfriend left the HDB flat.
As your boyfriend does not look after you and will leave you alone while you
are pregnant, you think there is no way out but to die.
The incident of jumping from height
You went up to the roof alone. You firmly believe that you would die by
jumping from the rooftop. You went there for a few times and planned for your
suicide if your boyfriend argues with you. Tonight, you are determined to die.
You went up alone and thought that no one would discover.
You drank two cans of beer to relax yourself so that it would be easier to jump
from height.
You did not do other things to harm yourself (e.g. overdose of Panadol or cut
yourself).
You sent a good bye sms message to your boyfriend and inform him that you
are going to die. You did not write a suicide letter.
You stayed at the rooftop for 30 minutes. When you are about to jump, two
police officers came up and pulled you down.
You were sent to the Accident and Emergency Department immediately.
Your current view of suicide:
You still want to die as your boyfriend did not respond to your sms. I do not
think he cares about me.
You do not feel sorry for the attempt.
You want to die with the foetus in your body.

!
!
325

If you are released from the AED, you will jump from height again and die with
your foetus to make your boy friend guilty.
You do not want to be admitted to any hospital and insist to go back on your
own.

Your mood and other relevant information:


In the past one month, due to frequent argument with your boy friend,
Your mood is low (Throughout the day, no variation in the day).
You lost your interest (e.g. shopping)/ not enjoy shopping nowadays
Your sleep is poor (Cannot fall asleep, wake up 5 times at night, wake up at 5
am cannot fall asleep).
Your appetite is poor.
Your energy level is low.
You have low sexual drive.
You cannot concentrate when you watch TV.
You feel hopeless.
You feel guilty towards your foetus because you have no money to raise your
future baby.
You feel very negative (hopeless) about the relationship with your boy friend
because he is an irresponsible person.
You have no one to talk to when you are depressed.
You have poor relationship with your family. You stay in your boy friends
HDB.
Your boy friend gives you $50 per week.
Say No to the following:
No past history of suicide
You did not overdose with medication tonight.
No stocking of medication or sharp objects at home.
You did not cut yourself with pen knife.
You have no religion.
You have not attempted suicide before.
You do not hear voices when stressed. No other false belief. (e.g. you
deserve severe punishment for minor mistake).
No family history of suicide.

You have not seen psychiatrist before.


No admission to psychiatric ward in the past.
No past medical problems (e.g. no thyroid problem, no chronic pain)
No family history of mental illness
No past history of mental illness
Not seen by psychiatrist before
Not on psychiatric medication
No history of alcohol or sleeping pill use.
Not seen by gynaecologist or obstetrician yet.

!
!
326

No other relationship in the past.


No past trauma
No emptiness
No fear of abandonment
No identity disturbance
No voices
No friend to talk to
Not in contact with family

Behaviour
Depressed; Still suicidal
If doctor says that he needs to transfer you to IMH, you do not look too happy
but you do not violently object.
Time:

8 minutes

Instructions to Examiners
Candidate is not expected to interact with the examiners in any way.!
!
!
327

Please keep to the exact time.


Things not to do: use/switch on mobile phone, yawn, teach or prompt.
If observers sit in, marking is not discussed with them. Observers record their own
assessment on separate sheet.
Marks are given depending on completeness or competency in each section
respectively. The global rating should be based on the total scores of
subsections. The total mark is 20. A candidate fails the OSCE station if he or
she scores less than 10.

!
!
328

EXAMINERS REPORT
Students Name: _________________ Matriculation No.: _____________ Examiners Name: ______________
Objectives

very well
(excellen
t)

Appropri
ate

9. Opens interview appropriately (greet, introduce, seek


permission, explains purpose)
Closes interview (summarizes, allows patient to clarify + to ask
questions, refers to future arrangements if any)

10. Creates safe environment, develops and maintains rapport,


controls interview appropriately (open ended questions,
respect, concern, sensitivity, empathy, responsive, normalizing
statements, avoids jargon)

patchy
needs to
improve

clearly
inadeq
uate

not
done

1.5

0.5

1.5

0.5

11. Enquires about the suicide attempt


a. Precipitant (Why the patient attempted suicide tonight,
what is the participant?)
b. Preparation of suicide (Plan, avoid discovery, sms
suicide message)
c. The suicide method (e.g. jumping, other methods e.g.
overdose, cutting and use of alcohol during suicide)
d. The consequences of suicide attempt (e.g. how was
she discovered?)

12. Assess for symptoms of depression and other relevant history


a. Mood symptoms (e.g. low mood, loss of interest)
b. Biological symptoms of depression (e.g. sleep,
appetite)
c. Cognitive & other symptoms of depression (e.g. guilty,
psychosis)
d. Other relevant history (past history of
depression/suicide, family history of
depression/suicide).

13. Assess current suicide risk at the AED


a. Her views towards the suicide attempt? (Any remorse)
b. Her current suicide plan / view on treatment (any plan
for further attempt)

1.5

0.5

14. Asses for the cause of current suicide attempt/ongoing stress


a. Poor relationship with boy friend and financial problem
b. 2-week pregnancy

1.5

0.5

15. Other relevant information (e.g. medical history: chronic pain,


family history of depression, substance use (benzodiazepine
use, alcohol), religion, social support, antenatal care.

1.5

0.5

16. Correct management: Send to IMH (2 marks); Other options (0


mark)

Subtotal score (For administrator to enter). Examiner can leave it


blank.
DECISION (circle one):

Pass

Fail

(For failure, the total score must be less than 10).

Signature of Examiner: ____________________________

Date: ______________________

!
!
329

Total score: Administrator 1:______________________

Administrator 2: _________________

EXAMINERS REPORT
Students Name: _________________ Matriculation No.: _____________ Examiners Name: ______________
Explain if student fails or is discussed in the examination board for borderline performance:
__________________________________________________________________________________________
__________________________________________________________________________________________
__________________________________________________________________________________________
__________________________________________________________________________________________
__________________________________________________________________________________________
!
!
!
!
!
!
!
!
!
!
!
!
!
!
!
!
!
!
!
330

!
!
!

Introduction
OSCE Title: Cognitive Assessment
Construct: This is a station designed to test the candidates ability to perform a
cognitive assessment and elicit signs of post-concussion syndrome

Objectives of the OSCE station:


To observe and assess candidates ability in:
7. Interview and communication skills
8. Carry out a cognitive assessment, in particular frontal lobe assessment and Minimental state examination (MMSE)

Time:

8 minutes

!
!
331

Instructions to Candidates
Patients Name: Mr. Tan
You are a General Practitioner. A 45-year-old man comes to see you today because
he injured his head after he was assaulted by another man in a pub.

Tasks:
1. Obtain the answer sheet (with your name label on it).
2. Take a brief history (less than 4 minutes) from the patient to assess postconcussion syndrome, risk assessment and sequelae after head injury.
3. Perform cognitive tasks listed on the left hand side of the table found on your
answer sheet.
4. Fill the right hand side of the table and provide the scores for the patient and
indicate maximum score for each task.
You are NOT required to perform a physical examination.
Time:

8 minutes

!
!
332

Instructions to Simulated Patients


Background of patient

Your name is Mr. Tan, a 45-year-old married person, staying with your spouse and son.
You work as an account manager. You have university education.
You have NO personal or family history of mental illness or dementia.
You have good past health.
You are a non-smoker and a social drinker. You go to drink in a pub once a while.
One month ago, you went to a pub and drank with your friends. You had an argument
with another man in the pub. He used a baseball bat to hit the front part and both sides
of your head.
You had a black-out for 30 minutes and your scalp was bleeding from superficial wound.
Your friends sent you to the hospital immediately. You can remember events before and
after the fight.
You were treated at the Accident and Emergency Department. The doctor said the head
injury was mild. You were not required to stay in the hospital. The brain scan did not
show major internal bleeding in the head. No surgery was required. No bleeding from
ear, nose and throat. No bruising from eyes.
The man who attacked you left the pub and police could not catch him. Insurance
company thinks the injury is mild and refuses to compensate.

Problems after head injury

After head injury, you find your brain function not very good. You cannot concentrate at
accounting work and often make mistakes. Your supervisor asks you to take no pay
leave to consult a doctor. You cannot plan or make decision.
In your head, you experience headache over the frontal part of your head and giddiness.
You lost your balance a few times after getting up from bed and almost fell down.
For your eyes and ears, you are very sensitive and become irritable to light and noise.
You hear wee sound in your eyes (Tinnitus).
For your mental functions, you are slow in mental speed and cannot think as fast as the
past. You have noticed increased difficulty in finding the right words to express yourself.
You are also unable to play crossword puzzles which you used to enjoy as you are
unable to come up with the right words.
Your memory is failing you: misplacing things (wallet), forgetting recent conversations
you had with your family, forgetting what you just ate for meals.
For your mood, you are sad all the time but sometimes becomes angry for no reason,
poor appetite, poor sleep (4 hours at night, cannot fall asleep), low energy level, loss of
interest (e.g. tennis), loss of sexual drive, poor concentration and attention, feels
hopeless after the head injury. Your mood swings from normal mood to anger.
You do not have suicidal thought. You do not hear voices or see things that are not
there.
You feel nervous most of the time and easily startled. You worry that the head injury
caused a lot of problems in your head. You cannot sit still, need to pace around in the

!
!
333

house. No sweating, no diarrhea, no difficulty in breathing, no chest pain and no panic


attack.
You forget to switch fire and almost caught the whole house on fire. Luckily, no flooding
at home so far.
You can handle money and ATM machine.
Your wife and son are concerned as you become more irritable and angry. This affect
your relationship with them. They want you to see a doctor. You are so irritable and
argued with your friends. Now, you do not want to go out to meet people. You mainly
stay at home most of the time.
No nightmare, no flashbacks, no avoidance of pub, no reliving of experiences
No difficulty in breathing, no shortness of breath, no panic attack
No fear of losing control
No fear of dying/going to die
No fear to centre of attention, no fear of talking to other people.
No repetitive checking or hand-washing
No repetitive thoughts
No suicidal thought
Not hearing voices
Not an anxious or angry person In the past
You do not have a driving license and you do not drive.

No other treatment/ no other problems.


No past medical problems (e.g. no stroke), no surgery done, no head injury
No family history of mental illness
No past history of mental illness, no memory problems
Not seeing other doctors at this moment.
Not seen by psychiatrist or psychologist before
Not on psychiatric medication nor other medications.
No past hospitalisation
During memory testing

Be cooperative and helpful to the candidate.


You are able to repeat three words (out of 3 words) named by student.
e.g. Student told you apple, newspaper and pencil. You say apple, newspaper and
pencil.
Subtracting 7 from 100. You do this test fairly quickly. Able to say 93, but get all the
remaining 4 wrong. For example, 100-7 = 93; 93-7 = 90: 90-7 = 85; 85 7 = 80; 80 7 =
75.
If students asks you to spell the word WORLD backwards, say D and get all other
letters wrong.
The student ask you to say English words after an alphabet. For example, letter F. Say a
3 words Friend, Fly, face and then repeat the 3 words again slowly.
Unable to recall the three items which were told (e.g. apple, newspaper and pencil) and
you come up with another 3 items (orange, train and tea).

!
!
334

Time:

8 minutes

!
!
335

Instructions to Examiners
Candidate is not expected to interact with the examiners in any way.
Please keep to the exact time.
Things not to do: use/switch on mobile phone, yawn, teach or prompt.
If observers sit in, marking is not discussed with them. Observers record their own
assessment on separate sheet.

The global rating should be based on the total scores of subsections. The total
mark is 20. A candidate fails the OSCE station if he or she scores less than 10.

!
!
336

Students sticker

Answer sheet
1. Take a history (4 minutes) to assess post-concussion symptoms, sequelae after
head injury and risk assessments
2. Please perform the following cognitive assessments and fill the right side of the
table
Mental tasks

Please indicate patients scores and


maximum scores for each task

Assess registration
Patients score:
Maximum score:
Assess attention and
concentration

Patients score:
Maximum score:

Assess verbal fluency


with one alphabet in
one minute

Number of words mentioned by patient:

Average number of words mention by a normal


person in one minute:

Assess short term


Patients score:
!
!
337

recall
Maximum score:

3. Please pass this form to the examiner.

EXAMINERS REPORT
Students are not allowed access to this document
Students Name: _________________ Matriculation No.: _____________ Examiners Name: ______________
Objectives

very well
(excellen
t)

Appropri
ate

18. Take an appropriate history to assess background of head


injury (e.g. type of injury, treatment, duration of blackout,
amnesia before and after injury, duration of impairment.)

patchy
needs to
improve

clearly
inadeq
uate

not
done

1.5

0.5

1.5

0.5

19. Assess neurological symptoms (e.g. headache, giddiness,


tinnitus)

1.5

0.5

20. Assess psychiatric symptoms (e.g. anxiety, depression,


irritability)

1.5

0.5

21. Assess cognitive symptoms (e.g. slow motor speed, poor


concentration, attention, memory loss )

1.5

0.5

22. Risk assessment (e.g. fall, forget to turn off stove/fire, flooding
at home) or background history: alcohol abuse, past medical
history/

1.5

0.5

23. Competency to assess registration of 3 items and correct


scores:
Patients scores = 3 Maximum scores 3

1.5

0.5

24. Competency to perform serial 7 or WORLD backwards


Patients scores = 1 Maximum scores 3

0.5

25. Verbal fluency


Patients scores = 3 Average number of a normal person: 10-15

1.5

0.5

Basic Communication Skills


17. Opens interview appropriately (greet, introduce, seek
permission, explains purpose), Creates safe environment
Closes interview (summarizes, allows patient to clarify + to ask
questions, refers to future arrangements if any)
Post-concussion symptoms

Brief cognitive assessment

!
!
338

26. Competency to assess recall of 3 items


Patients scores = 0 ; Maximum scores 0

Subtotal score (For administrator to enter). Examiner can leave it


blank.
DECISION (circle one):

Pass

Fail

(For failure, the total score must be less than 12.5).

Signature of Examiner: ____________________________


Total score: Administrator 1:______________________

Date: _____________________
Administrator 2: _________________

EXAMINERS REPORT
Students are not allowed access to this document
Students Name: _________________ Matriculation No.: _____________ Examiners Name: ______________
Explain if student fails or is discussed in the examination board for borderline performance:
__________________________________________________________________________________________
__________________________________________________________________________________________
__________________________________________________________________________________________
__________________________________________________________________________________________
__________________________________________________________________________________________

!
!
!
!
!
!
!
!

!
!
339

!
!
!
!
!
!

Introduction
OSCE Title: Borderline personality disorder
Construct: This is a station designed to test the candidates ability to take a history
of borderline personality disorder.
Objectives of the OSCE station:
To observe and assess candidates ability in:
9. Eliciting symptoms of borderline personality disorder
10. Assess common psychiatric comorbidity associated with borderline personality
disorder
11. Establishing a diagnosis
12. Expressing empathy and demonstrating good interview skills

Time:

8 minutes

!
!
340

Instructions to Candidates
You are a paediatrician working in a private medical group. A mother bought her 15year-old daughter, Angela to see your paediatric colleague because of low body
weight and lethargy. Angela used to be a high academic achiever in school but she
was not able to sit for examination in November last year. Your colleague has
ordered a set of investigations but he needs to go off to attend some urgent matter.
He has asked you to review the laboratory results and electrocardiogram (ECG) on
his behalf.

Tasks:

After entering the room, please review your colleagues record, laboratory results
and ECG and identify abnormalities.

PLEASE DO NOT WRITE OR LEAVE ANY MARKS ON THE LABORATORY


RESULTS AND ECG. There is a calculator in the room. Please clear your
answer from the calculator after usage.

Please answer the questions stated on the answer sheet.

Please double check your name is correct before answering the questions.

Please pass your answer sheet to the circuit administrator at the end of the station.

You are required to stay in the room until the end of the 8th minute. You cannot
leave the room if you finish early.
Time:

8 minutes

!
!
341

Outpatient record
(To be placed on the desk of the candidates room)

Miss Angela Chan


S1234567X 15 Y 11 M Female

Visit date: 8 January, 2015

Doctor: Dr. Albert Tan


History:

Angela was bought in by her mother due to low body weight


and lethargy.
Used to be a high achiever in exam.
Weight: 40 kg
Height: 170 cm
PE:
Her physical examination showed marked malnutrition
and paleness.
Plan:
Order full blood count, liver function test, renal function test,
thyroid function test and electrocardiogram.
To review results; then speak to patient and mother.

!
!
342

Laboratory results:
(To be placed on the desk of the candidates room)
Full Blood Count
Test

Results

Unit

Reference interval

White Blood Cell

4.83

X 10 9/L

3.40 9.60

Red Blood Cell

2.60

X 10 9/L

4.30 5.70

Haemoglobin

10.0

g/dL

12.9 to 17.0

Mean corpuscular
volume (MCV)

113.5

fL

80.0 95.0

Mean corpsular
haemoglobin (MCH)

38.5

Pg

27.0 33.0

Platelets

140

X 10 9/L

132 - 372

Liver Function Test


Test

Results

Unit

Reference interval

Albumin

29

g/L

38 48

Bilirubin, Total

20

umol/L

5 30

Bilirubin, Conj

Umol/L

05

Bilirubin, Unconj

Umol/L

5 25

Aspartate aminotransferase (AST)

30

U/L

10 50

Alanine aminotransferase (ALT)

35

U/L

10 70

Renal Function Test

!
!
343

Test

Results

Unit

Reference interval

Sodium

143

mmol/L

135 145

Potassium

3.1

mmol/L

3.5 5.0

Urea

4.8

mmol/L

2.5 7.5

Creatinine

92

mmol/L

65 125

Test

Results

Unit

Reference interval

Free thyroxine

15

pmol/L

10 23

Thyroid stimulating hormone (TSH)

3.50

mIU/L

0.45 4.50

Thyroid Function Test

Insert Students Sticker


1. Based on her height and weight, calculate her body mass index (BMI). (2 marks)
(Examiners Use only: /2 marks)

2. Please review the laboratory results and electrocardiogram. Identify THREE (3)
abnormalities. (6 marks)
(Examiners Use only:

!
!
344

/6 marks)

3. Based on the above information, state the MOST LIKELY psychiatric diagnosis. (2 marks)
(Examiners Use only: /2 marks)

(PLEASE TURN OVER)


4. Based on your psychiatric diagnosis, state FOUR (4) additional questions which you would
ask the patient to confirm your diagnosis (Examiners Use only: /4 marks)

5. Based on your psychiatric diagnosis, state FOUR (4) additional clinical signs which you
would look for during physical examination to confirm your diagnosis. (Examiners Use only:
/4 marks)

!
!
345

6. Based on your psychiatric diagnosis, state TWO (2) additional investigations which your
colleague forgot to order but relevant to this condition (2 marks)
(Examiners Use only:
/2 marks)

Total marks: Marker: ______________

Verification by administrator: _____________

!
!
346

Instructions to Candidates
Patients name: Amy
You are a resident working at the Accident and Emergency Department (AED). Amy,
a 35-year-old woman comes to see you because she has cut her thighs. Your
colleague has put dressing on her thighs and the wounds are superficial.

Tasks:
7. Take a history from Amy to assess her personality and establish a diagnosis.
8. Assess common psychiatric comorbidity associated with her diagnosis.
9. At the end of your interview with Amy, please write down the most likely
psychiatric diagnosis and pass it to the examiner.

You are NOT required to examine the wound.

Time:

8 minutes

347!
!

Students sticker

Answer sheet
Please write down the most likely psychiatric diagnosis of this patient and
pass it to your examiner.
___________________________________________________________________

Instructions to Simulated Patients


348!
!

Background:
Your name is Amy, a 35-year-old woman. You live alone in a one-room HDB flat.
You are estranged from your family, being an only child whose father died of
cancer when you were 6 years old, and who has a very difficult relationship with
her mother. You have a history of physical abuse in childhood and her mother
was the perpetuator and she caned you very often for no reason. You are not in
contact with your mother. There is no history of sexual abuse.
You had a number of relatively short-lived jobs in the past as a caregiver for the
elderly in nursing home. You resigned from your last job after an argument with
the manager and have been unemployed for several years.
You consult the accident and emergency department today because you are very
stressed and cut your thigh.
You have no known medical illness. No known family history of mental illness.
Symptoms
Main symptoms:

The doctor may ask you a question how to describe yourself. You tell the doctor
that you dont know how to describe yourself. You are confused about who you
are. You are lost about yourself.

The doctor may ask you how your friends describe you. You tell the doctor that
you have a small number of close female friends, but find that people often let
you down so you do not have a wide social network. You feel that people in
general do not like you. It is very hard to maintain relationship with others.

The reason for cutting yourself is to re-experience the pain. The pain inflicted
by your mother during the childhood abuse. You are not suicidal at this moment.
You always cut your thighs with pen knife. You have no intention to kill yourself.

Your experience the following symptoms since young:


o Feel very empty inside yourself since young, cutting yourself since
adolescence
o Fear of being abandoned (see details below)
o Impulsive, like to cut your thighs, binge eating (i.e. eat a lot of cookies)
when stressed but does not vomit out the food. You dont regret about
cutting.
o For your mood, hard to say whether you are depression You have mood
swing or fluctuation: normal mood can become sudden anger and
irritability for no reason.
o You are confused about your own identity (e.g. you do not know who you
are).
o You had suicidal thoughts in the past (see details below).
o You saw angels when you were stressed. The angels appeared in the
corner of your flat. They just stood there but did not say anything. When
349!

doctors ask you to elaborate further about the angels. You cant. You do
not hear voices when nobody is around. You are not paranoid (i.e. does
not believe any one wants to harm you)
o You do tend to classify people as either all bad or all good. For example, in
your previous work place, the manager was the bad guy and other coworkers were all good. No one was in between.
Relationship: You have a history of brief unstable relationships. You had 10 boy
friends in the past who were physically abusive. You have not been in a close
relationship for several years and no recent sexual activities..
Past suicide attempts: You are well apart from the lacerations, with no significant
medical history. You have scars on your thighs from past superficial lacerations. You
have had several past psychiatric admissions for self-harm or suicidality by drug
overdose across the last few years.
You use emergency A&E services at times. You have recently been admitted with
overdoses twice in the last month. Im just fed up and I cant go on.
Abandonment by your psychiatrist and psychologist: You are supposed to be
having follow-up from the psychiatrist at a general hospital. You are currently saying
that the psychiatrist has abandoned her and dont care. You are not able to talk to a
staff member at the psychiatric department who knew you, as the receptionist of the
clinic said that both your doctor and psychologist were away during Christmas.
Other problems:
You have nightmare, the images of childhood abuse popping up into your mind.
You have binge eating when under stress but no self-induced vomiting.
For your mood, very unpredictable and always have mood swings.
Your sleep, appetite and energy are normal.
You are not in any relationship at this moment and no sexual activity recently.
Say No to the following:
No body image problem. You never put yourself on diet or do excessive exercise.
You are not special when compared to other people.
You do not like to be centre of attention.
No past history of stealing, no problem with police.
You do not drink alcohol, smoke cigarettes or use sleeping pills or illicit drugs.
You are not on any psychotropic medications currently.
You have no memory problem. No past wandering from home.
You do not have other unusual experiences e.g. being controlled by other people
or your thoughts being interfered.
You do not feel low in mood. You do not have feelings of
hopelessness/worthlessness. You do not think that life is meaningless and have
no suicidal thoughts. In other words, your mood is unpredictable, your appetite is
ok (no problem), your sleep is ok (no problems).
You do not have much interest throughout your life.
There is no previous history of accident.

350!
!

No other medical problem. When you were admitted, the blood tests show normal
results.
No difficulty in breathing, no shortness of breath, no panic attack
No fear of losing control
No fear of dying/going to die
No fear of going out of the house. No fear of using MRT or bus.
No fear of going to a shopping mall or crowded place/enclosed space
No fear to centre of attention, no fear of talking to other people.
No fear of blood, height or spider.
No repetitive checking or hand-washing
No repetitive thoughts of contamination
Not hearing voices

No other treatment/ no other problems.


No past medical problems (e.g. no thyroid problem)
No family history of mental illness
Not seeing other doctors at this moment.
Not on psychiatric medication nor other medications.
Behaviour
You are cooperative during the interview.

Time:

8 minutes

Instructions to Examiners
Candidate is not expected to interact with the examiners in any way.!
Please keep to the exact time.
351!
!

Things not to do: use/switch on mobile phone, yawn, teach or prompt.


If observers sit in, marking is not discussed with them. Observers record their own
assessment on separate sheet.
Please use the calculator to add up the marks and write down the score at the
bottom of marking form.
Marks are given depending on completeness or competency in each section
respectively. The global rating should be based on the total scores of
subsections. The total mark is 20. A candidate fails the OSCE station if he or
she scores less than 10.

352!
!

EXAMINERS REPORT
Students Name: _________________ Matriculation No.: _____________ Examiners Name: ______________
Objectives

very well
(excellen
t)

Appropri
ate

27. Opens interview appropriately (greet, introduce, seek


permission, explains purpose)
Closes interview (summarizes, allows patient to clarify + to ask
questions, refers to future arrangements if any)

28. Open questions to allow patient to comment on her own


personality (1 mark) (e.g. How would you describe yourself as
a person?) and explore reason for cutting (1 mark)

patchy
needs to
improve

clearly
inadeq
uate

not
done

1.5

0.5

1.5

0.5

29. Chronic feeling of emptiness (1 mark) and fear of abandonment


(1 mark)

1.5

0.5

30. Unstable emotion (explore what constitutes mood swing,


normal mood to anger and irritability

1.5

0.5

31. Impulsive behaviour (e.g. past self harm-1 mark , binge eating /
stealing 1 mark)

1.5

0.5

32. Suicide risk assessment (e.g. current and past intention to die1 mark, explore past hospitalisation due to suicide 1 mark)

1.5

0.5

33. Other borderline symptoms e.g. past unstable relationship /


explore transient psychotic feature / identity disturbance
defence mechanism idealisation and devaluation or splitting

1.5

0.5

34. Assess for common comorbidity e.g. PTSD, depression,


bulimia, substance abuse, stealing)

1.5

0.5

35. Other relevant information (e.g. past trauma, childhood abuse,


past psychiatric history, family history social support etc) and
general performance of candidate

1.5

0.5

36. Correct diagnosis: Borderline personality disorder (2 marks for


full term and correct spelling); BPD abbreviation = 0 mark Any
other diagnoses: (0 mark)

Enquires about symptoms of borderline personality disorder

DECISION (circle one):

Pass

Fail

(For failure, the total score must be less than 10).

Signature of Examiner: ____________________________


Total score: Examiners score:______________________

Date: ______________________
Verification by administrator : _________________

353!
!

EXAMINERS REPORT
Students Name: _________________ Matriculation No.: _____________ Examiners Name: ______________
Explain if student fails or is discussed in the examination board for borderline performance:
__________________________________________________________________________________________
__________________________________________________________________________________________
__________________________________________________________________________________________
__________________________________________________________________________________________
__________________________________________________________________________________________

354!
!

Mastering Psychiatry
A Core Textbook for Undergraduates
- Now with a companion guide to help you master your undergraduate exams!
The comprehensive textbook covers common psychiatric conditions
encountered in adults, children, adolescents and old people. The main textbook
provides core information you need for undergraduate examination and
future clinical practices.
This companion guide includes past exam papers to help you practice for the following
components in the actual exam:

Mul$ple'choice'ques$ons'

Short'Answer'ques$ons'

OSCE'sta$ons'
'
This'book'is'a'joint'eort'between'authors'from'Singapore'&'Authors'from'the'University'of'
Toronto.''

Melvyn'WB'Zhang'MBBS'(Singapore),'Diploma'(Cl'Psychiatry)''(RCP&S'Ireland),'MRPCPsych(UK)'

Resident'(Psychiatry),'Na$onal'Healthcare'Group'
'

Cyrus'SH'Ho'MBBS'(Singapore),'Diploma'(Cl'Psychiatry)''(RCP&S'Ireland),'MRCPsych'(UK)'

Registrar,'Na$onal'University'Healthcare'System'(NUHS)'

'

Roger'CM'Ho'MBBS'(Hong'Kong),'DPM'(Psych)'(Ireland)'Diploma'(Cl'Psychiatry)''(RCP&S'Ireland),'DIP''
(Psychotherapy)'(NUS),''MMed'(Psych)'(Singapore),'MRCPsych'(UK),'FRCPC'

Assistant'Professor'and'Consultant'Psychiatrist,'Department'of'Psychological'Medicine,'
Yong'Loo'Lin'School'of'Medicine,'Na$onal'University'of'Singapore'
'

'

'
Sanjeev'Sockalingam'MD,'FRCPC'
Deputy'Psychiatrist'ion'Chief'(TGH'&'PMH),'Director,'Con$nuing'Professional'and'Prac$ce'
Development'
Assistant'Professor,'Department'of'Psychiatry,'University'of'Toronto'
'

Raed'Hawa'MD,'FRCPC'

Associate'Professor,'Department'of'Psychiatry,'University'of'Toronto'

'
'
'
'
'
'
Our'online'companion:'www.masteringpsychiatry.com'
['Website'QR]''''''''''''''[Apple'Store]'''''''''''''[Android'Store]'''''''
'
'

ISBN'(Book):'978E981E07E0163E5'
ISBN'(DVD):''978E981E07E0164E2'
ISBN'(Ebook/App):''978-981-07-1493-2'
'

You might also like